АдрСс: 105678, Π³. Москва, ШоссС Энтузиастов, Π΄. 55 (ΠšΠ°Ρ€Ρ‚Π° ΠΏΡ€ΠΎΠ΅Π·Π΄Π°)
ВрСмя Ρ€Π°Π±ΠΎΡ‚Ρ‹: ПН-ПВ: с 9.00 Π΄ΠΎ 18.00, Π‘Π‘: с 9.00 Π΄ΠΎ 14.00

Как ΠΈΠ·ΠΌΠ΅Ρ€ΠΈΡ‚ΡŒ Π΅ΠΌΠΊΠΎΡΡ‚ΡŒ кондСнсатора ΠΌΡƒΠ»ΡŒΡ‚ΠΈΠΌΠ΅Ρ‚Ρ€ΠΎΠΌ: Как ΠΎΠΏΡ€Π΅Π΄Π΅Π»ΠΈΡ‚ΡŒ Π΅ΠΌΠΊΠΎΡΡ‚ΡŒ кондСнсатора ΠΌΡƒΠ»ΡŒΡ‚ΠΈΠΌΠ΅Ρ‚Ρ€ΠΎΠΌ

Π‘ΠΎΠ΄Π΅Ρ€ΠΆΠ°Π½ΠΈΠ΅

Как ΠΎΠΏΡ€Π΅Π΄Π΅Π»ΠΈΡ‚ΡŒ Π΅ΠΌΠΊΠΎΡΡ‚ΡŒ кондСнсатора ΠΌΡƒΠ»ΡŒΡ‚ΠΈΠΌΠ΅Ρ‚Ρ€ΠΎΠΌ

Иногда Π½Π° кондСнсаторС Π½Π΅ указываСтся Π΅Π³ΠΎ ΠΌΠ°Ρ€ΠΊΠΈΡ€ΠΎΠ²ΠΊΠ°. Как ΡƒΠ·Π½Π°Ρ‚ΡŒ Ρ‚ΠΎΠ³Π΄Π° Ρ€Π΅Π°Π»ΡŒΠ½ΡƒΡŽ Π΅Π³ΠΎ Π΅ΠΌΠΊΠΎΡΡ‚ΡŒ, Ссли ΡΠΏΠ΅Ρ†ΠΈΠ°Π»ΡŒΠ½ΠΎΠ³ΠΎ оборудования ΠΏΠΎΠ΄ Ρ€ΡƒΠΊΠΎΠΉ Π½Π΅Ρ‚, Π° устройство Π±Π΅Π· ΠΎΠ±ΠΎΠ·Π½Π°Ρ‡Π΅Π½ΠΈΠΉ? Π’ΠΎΠ³Π΄Π° Π½Π° ΠΏΠΎΠΌΠΎΡ‰ΡŒ приходят Ρ€Π°Π·Π»ΠΈΡ‡Π½Ρ‹Π΅ ΠΏΠΎΠ΄Ρ€ΡƒΡ‡Π½Ρ‹Π΅ срСдства ΠΈ Ρ„ΠΎΡ€ΠΌΡƒΠ»Ρ‹. ΠŸΡ€Π΅ΠΆΠ΄Π΅ Ρ‡Π΅ΠΌ ΠΏΡ€ΠΈΡΡ‚ΡƒΠΏΠ°Ρ‚ΡŒ ΠΊ Ρ€Π°Π±ΠΎΡ‚Π΅, Π½Π΅ΠΎΠ±Ρ…ΠΎΠ΄ΠΈΠΌΠΎ ΠΏΠΎΠΌΠ½ΠΈΡ‚ΡŒ ΠΎ Ρ‚ΠΎΠΌ, Ρ‡Ρ‚ΠΎ кондСнсатор ΠΏΠ΅Ρ€Π΅Π΄ ΠΏΡ€ΠΎΠ²Π΅Ρ€ΠΊΠΎΠΉ Π΄ΠΎΠ»ΠΆΠ΅Π½ Π±Ρ‹Ρ‚ΡŒ разряТСн (слСдуСт Ρ€Π°Π·Ρ€ΡΠ΄ΠΈΡ‚ΡŒ Π΅Π³ΠΎ ΠΊΠΎΠ½Ρ‚Π°ΠΊΡ‚Ρ‹). Для этого ΠΌΠΎΠΆΠ½ΠΎ ΠΈΡΠΏΠΎΠ»ΡŒΠ·ΠΎΠ²Π°Ρ‚ΡŒ ΠΎΠ±Ρ‹Ρ‡Π½ΡƒΡŽ ΠΎΡ‚Π²Π΅Ρ€Ρ‚ΠΊΡƒ с ΠΈΠ·ΠΎΠ»ΠΈΡ€ΠΎΠ²Π°Π½Π½ΠΎΠΉ Ρ€ΡƒΡ‡ΠΊΠΎΠΉ. Π”Π΅Ρ€ΠΆΠ°ΡΡŒ Π·Π° Ρ€ΡƒΡ‡ΠΊΡƒ ΠΎΡ‚Π²Π΅Ρ€Ρ‚ΠΊΠΎΠΉ ΠΊΠΎΡΠ½ΡƒΡ‚ΡŒΡΡ ΠΊΠΎΠ½Ρ‚Π°ΠΊΡ‚ΠΎΠ², Ρ‚Π°ΠΊΠΈΠΌ ΠΎΠ±Ρ€Π°Π·ΠΎΠΌ ΠΈΡ… замыкая. Π”Π°Π»Π΅Π΅ ΠΌΡ‹ ΠΏΠΎΠ΄Ρ€ΠΎΠ±Π½ΠΎ расскаТСм, ΠΊΠ°ΠΊ ΠΎΠΏΡ€Π΅Π΄Π΅Π»ΠΈΡ‚ΡŒ Π΅ΠΌΠΊΠΎΡΡ‚ΡŒ кондСнсатора ΠΌΡƒΠ»ΡŒΡ‚ΠΈΠΌΠ΅Ρ‚Ρ€ΠΎΠΌ, прСдоставив ΠΈΠ½ΡΡ‚Ρ€ΡƒΠΊΡ†ΠΈΡŽ с Π²ΠΈΠ΄Π΅ΠΎ ΠΏΡ€ΠΈΠΌΠ΅Ρ€ΠΎΠΌ.

ИспользованиС Ρ€Π΅ΠΆΠΈΠΌΠ° Β«CxΒ»

ПослС Ρ‚ΠΎΠ³ΠΎ, ΠΊΠ°ΠΊ ΠΊΠΎΠ½Ρ‚Π°ΠΊΡ‚Ρ‹ Π·Π°ΠΊΠΎΡ€ΠΎΡ‚ΠΈΠ»ΠΈ, ΠΌΠΎΠΆΠ½ΠΎ ΠΎΡΡƒΡ‰Π΅ΡΡ‚Π²Π»ΡΡ‚ΡŒ ΠΎΠΏΡ€Π΅Π΄Π΅Π»Π΅Π½ΠΈΠ΅ сопротивлСния. Если элСмСнт исправлСн, Ρ‚ΠΎ сразу послС ΠΏΠΎΠ΄ΠΊΠ»ΡŽΡ‡Π΅Π½ΠΈΡ ΠΎΠ½ Π½Π°Ρ‡Π½Π΅Ρ‚ Π·Π°Ρ€ΡΠΆΠ°Ρ‚ΡŒΡΡ постоянным Ρ‚ΠΎΠΊΠΎΠΌ. Π’ этом случаС сопротивлСниС ΠΎΡ‚ΠΎΠ±Ρ€Π°Π·ΠΈΡ‚ΡŒΡΡΒ ΠΌΠΈΠ½ΠΈΠΌΠ°Π»ΡŒΠ½ΠΎΠ΅ ΠΈ Π±ΡƒΠ΄Π΅Ρ‚ ΠΏΡ€ΠΎΠ΄ΠΎΠ»ΠΆΠ°Ρ‚ΡŒ расти.

Π’ случаС Ссли кондСнсатор нСисправСн, Ρ‚ΠΎ ΠΌΡƒΠ»ΡŒΡ‚ΠΈΠΌΠ΅Ρ‚Ρ€ Π±ΡƒΠ΄Π΅Ρ‚ сразу ΡƒΠΊΠ°Π·Ρ‹Π²Π°Ρ‚ΡŒ Π±Π΅ΡΠΊΠΎΠ½Π΅Ρ‡Π½ΠΎΡΡ‚ΡŒ ΠΈΠ»ΠΈ Π±ΡƒΠ΄Π΅Ρ‚ ΡƒΠΊΠ°Π·Ρ‹Π²Π°Ρ‚ΡŒ Π½ΡƒΠ»Π΅Π²ΠΎΠ΅ сопротивлСниС ΠΈ ΠΏΡ€ΠΈ этом ΠΏΠΈΡ‰Π°Ρ‚ΡŒ.

Вакая ΠΏΡ€ΠΎΠ²Π΅Ρ€ΠΊΠ° осущСствляСтся, Ссли конструкция полярная.

Для Ρ‚ΠΎΠ³ΠΎ Ρ‡Ρ‚ΠΎΠ±Ρ‹ ΡƒΠ·Π½Π°Ρ‚ΡŒ Π΅ΠΌΠΊΠΎΡΡ‚ΡŒ Π½Π΅ΠΎΠ±Ρ…ΠΎΠ΄ΠΈΠΌΠΎ ΠΈΠΌΠ΅Ρ‚ΡŒ ΠΌΡƒΠ»ΡŒΡ‚ΠΈΠΌΠ΅Ρ‚Ρ€ с Ρ„ΡƒΠ½ΠΊΡ†ΠΈΠ΅ΠΉ измСрСния ΠΏΠ°Ρ€Π°ΠΌΠ΅Ρ‚Ρ€Π° Β«Π‘Ρ…Β».

ΠžΠΏΡ€Π΅Π΄Π΅Π»ΠΈΡ‚ΡŒ Π΅ΠΌΠΊΠΎΡΡ‚ΡŒ с ΠΏΠΎΠΌΠΎΡ‰ΡŒΡŽ Ρ‚Π°ΠΊΠΎΠ³ΠΎ ΠΌΡƒΠ»ΡŒΡ‚ΠΈΠΌΠ΅Ρ‚Ρ€Π° просто: ΡƒΡΡ‚Π°Π½ΠΎΠ²ΠΈΡ‚ΡŒ Π΅Π³ΠΎ Π² Ρ€Π΅ΠΆΠΈΠΌ Β«Π‘Ρ…Β» ΠΈ ΡƒΠΊΠ°Π·Π°Ρ‚ΡŒ ΠΌΠΈΠ½ΠΈΠΌΠ°Π»ΡŒΠ½Ρ‹ΠΉ ΠΏΡ€Π΅Π΄Π΅Π» измСрСния, ΠΊΠΎΡ‚ΠΎΡ€Ρ‹ΠΌ Π΄ΠΎΠ»ΠΆΠ΅Π½ ΠΎΠ±Π»Π°Π΄Π°Ρ‚ΡŒ Π΄Π°Π½Π½Ρ‹ΠΉ кондСнсатор. Π’ Ρ‚Π°ΠΊΠΈΡ… ΠΌΡƒΠ»ΡŒΡ‚ΠΈΠΌΠ΅Ρ‚Ρ€Π°Ρ… Π΅ΡΡ‚ΡŒ ΡΠΏΠ΅Ρ†ΠΈΠ°Π»ΡŒΠ½Ρ‹Π΅ Π³Π½Π΅Π·Π΄Π° с ΠΎΠΏΡ€Π΅Π΄Π΅Π»Π΅Π½Π½Ρ‹ΠΌΠΈ прСдСлами измСрСния. Π’ эти Π³Π½Π΅Π·Π΄Π° вставляСтся кондСнсатор согласно Π΅Π³ΠΎ ΠΏΡ€Π΅Π΄Π΅Π»Ρƒ измСрСния ΠΈ происходит ΠΎΠΏΡ€Π΅Π΄Π΅Π»Π΅Π½ΠΈΠ΅ Π΅Π³ΠΎ ΠΏΠ°Ρ€Π°ΠΌΠ΅Ρ‚Ρ€ΠΎΠ².

Если Π² тСстСрС таких Π³Π½Π΅Π·Π΄ Π½Π΅Ρ‚, Ρ‚ΠΎ ΠΎΠΏΡ€Π΅Π΄Π΅Π»ΠΈΡ‚ΡŒ Π΅ΠΌΠΊΠΎΡΡ‚ΡŒΒ ΠΌΠΎΠΆΠ½ΠΎ с ΠΏΠΎΠΌΠΎΡ‰ΡŒΡŽ ΠΈΠ·ΠΌΠ΅Ρ€ΠΈΡ‚Π΅Π»ΡŒΠ½Ρ‹Ρ… Ρ‰ΡƒΠΏΠΎΠ², ΠΊΠ°ΠΊ ΠΏΠΎΠΊΠ°Π·Π°Π½ΠΎ Π½Π° Ρ„ΠΎΡ‚ΠΎ Π½ΠΈΠΆΠ΅:

Π’Π°ΠΆΠ½ΠΎ! Π’ ΠΎΡ‚Π΄Π΅Π»ΡŒΠ½ΠΎΠΉ ΡΡ‚Π°Ρ‚ΡŒΠ΅ ΠΌΡ‹ рассказывали ΠΎ Ρ‚ΠΎΠΌ, ΠΊΠ°ΠΊ ΠΏΡ€ΠΎΠ²Π΅Ρ€ΠΈΡ‚ΡŒ ΠΈΡΠΏΡ€Π°Π²Π½ΠΎΡΡ‚ΡŒ кондСнсатора. Π Π΅ΠΊΠΎΠΌΠ΅Π½Π΄ΡƒΠ΅ΠΌ Ρ‚Π°ΠΊΠΆΠ΅ ΠΎΠ·Π½Π°ΠΊΠΎΠΌΠΈΡ‚ΡŒΡΡ с этим ΠΌΠ°Ρ‚Π΅Ρ€ΠΈΠ°Π»ΠΎΠΌ!

ΠŸΡ€ΠΈΠΌΠ΅Π½Π΅Π½ΠΈΠ΅ Ρ„ΠΎΡ€ΠΌΡƒΠ»

Π§Ρ‚ΠΎ Π΄Π΅Π»Π°Ρ‚ΡŒ, Ссли ΠΏΠΎΠ΄ Ρ€ΡƒΠΊΠΎΠΉ Π½Π΅Ρ‚ Ρ‚Π°ΠΊΠΎΠ³ΠΎ ΠΌΡƒΠ»ΡŒΡ‚ΠΈΠΌΠ΅Ρ‚Ρ€Π° с Π³Π½Π΅Π·Π΄Π°ΠΌΠΈ измСрСния, Π° Π΅ΡΡ‚ΡŒ Ρ‚ΠΎΠ»ΡŒΠΊΠΎ ΠΎΠ±Ρ‹Ρ‡Π½Ρ‹ΠΉ Π±Ρ‹Ρ‚ΠΎΠ²ΠΎΠΉ ΠΏΡ€ΠΈΠ±ΠΎΡ€? Π’ Ρ‚Π°ΠΊΠΎΠΌ случаС Π½Π΅ΠΎΠ±Ρ…ΠΎΠ΄ΠΈΠΌΠΎ Π²ΡΠΏΠΎΠΌΠ½ΠΈΡ‚ΡŒ Π·Π°ΠΊΠΎΠ½Ρ‹ Ρ„ΠΈΠ·ΠΈΠΊΠΈ, ΠΊΠΎΡ‚ΠΎΡ€Ρ‹Π΅ ΠΏΠΎΠΌΠΎΠ³ΡƒΡ‚ ΠΎΠΏΡ€Π΅Π΄Π΅Π»ΠΈΡ‚ΡŒ Π΅ΠΌΠΊΠΎΡΡ‚ΡŒ.

Для Π½Π°Ρ‡Π°Π»Π° вспомним, Ρ‡Ρ‚ΠΎ Π² случаС, ΠΊΠΎΠ³Π΄Π° кондСнсатор заряТаСтся ΠΎΡ‚ источника Π½Π΅ΠΈΠ·ΠΌΠ΅Π½Π½ΠΎΠ³ΠΎ напряТСния Ρ‡Π΅Ρ€Π΅Π· рСзистор, Ρ‚ΠΎ сущСствуСт Π·Π°ΠΊΠΎΠ½ΠΎΠΌΠ΅Ρ€Π½ΠΎΡΡ‚ΡŒ, согласно ΠΊΠΎΡ‚ΠΎΡ€ΠΎΠΉ напряТСниС Π½Π° устройствС Π±ΡƒΠ΄Π΅Ρ‚ ΠΏΠΎΠ΄Ρ…ΠΎΠ΄ΠΈΡ‚ΡŒ ΠΊ Π½Π°ΠΏΡ€ΡΠΆΠ΅Π½ΠΈΡŽ источника ΠΈ Π² ΠΊΠΎΠ½Π΅Ρ‡Π½ΠΎΠΌ ΠΈΡ‚ΠΎΠ³Π΅ сравняСтся с Π½ΠΈΠΌ.

Но для Ρ‚ΠΎΠ³ΠΎ Ρ‡Ρ‚ΠΎΠ±Ρ‹ этого Π½Π΅ ΠΎΠΆΠΈΠ΄Π°Ρ‚ΡŒ, ΠΌΠΎΠΆΠ½ΠΎ процСсс ΡƒΠΏΡ€ΠΎΡΡ‚ΠΈΡ‚ΡŒ. НапримСр, Π·Π° ΠΎΠΏΡ€Π΅Π΄Π΅Π»Π΅Π½Π½ΠΎΠ΅ врСмя, ΠΊΠΎΡ‚ΠΎΡ€ΠΎΠ΅ равняСтся 3*RC, Π²ΠΎ врСмя заряТСния элСмСнт достигаСт напряТСния 95% ΠΏΡ€ΠΈΠΌΠ΅Π½Π΅Π½Π½ΠΎΠ³ΠΎ ΠΊ RC Ρ†Π΅ΠΏΠΈ. Π’Π°ΠΊΠΈΠΌ ΠΎΠ±Ρ€Π°Π·ΠΎΠΌ, ΠΏΠΎ Ρ‚ΠΎΠΊΡƒ ΠΈ Π½Π°ΠΏΡ€ΡΠΆΠ΅Π½ΠΈΡŽ ΠΌΠΎΠΆΠ½ΠΎ ΠΎΠΏΡ€Π΅Π΄Π΅Π»ΠΈΡ‚ΡŒ константу Π²Ρ€Π΅ΠΌΠ΅Π½ΠΈ. А ΠΏΡ€Π°Π²ΠΈΠ»ΡŒΠ½Π΅Π΅, Ссли Π·Π½Π°Ρ‚ΡŒ Π²ΠΎΠ»ΡŒΡ‚Π°ΠΆΒ Π² Π±Π»ΠΎΠΊΠ΅ питания, Π½ΠΎΠΌΠΈΠ½Π°Π» самого рСзистора, происходит ΠΎΠΏΡ€Π΅Π΄Π΅Π»Π΅Π½ΠΈΠ΅ постоянной Π²Ρ€Π΅ΠΌΠ΅Π½ΠΈ, Π° Π·Π°Ρ‚Π΅ΠΌ ΠΈ Смкости устройства.

НапримСр, Π΅ΡΡ‚ΡŒ элСктролитичСский кондСнсатор, ΡƒΠ·Π½Π°Ρ‚ΡŒ Π΅ΠΌΠΊΠΎΡΡ‚ΡŒ ΠΊΠΎΡ‚ΠΎΡ€ΠΎΠ³ΠΎ ΠΌΠΎΠΆΠ½ΠΎ ΠΏΠΎ ΠΌΠ°Ρ€ΠΊΠΈΡ€ΠΎΠ²ΠΊΠ΅, Π³Π΄Π΅ прописываСтся 6800Β ΠΌΠΊΡ„ 50Π². Но Ρ‡Ρ‚ΠΎ Ссли устройство Π΄Π°Π²Π½ΠΎ Π»Π΅ΠΆΠ°Π»ΠΎ Π±Π΅Π· Π΄Π΅Π»Π°, Π° ΠΏΠΎ надписи слоТно ΠΎΠΏΡ€Π΅Π΄Π΅Π»ΠΈΡ‚ΡŒ Π΅Π³ΠΎ Ρ€Π°Π±ΠΎΡ‡Π΅Π΅ состояниС? Π’ этом случаС Π»ΡƒΡ‡ΡˆΠ΅ ΠΏΡ€ΠΎΠ²Π΅Ρ€ΠΈΡ‚ΡŒ Π΅Π³ΠΎ Π΅ΠΌΠΊΠΎΡΡ‚ΡŒ, Ρ‡Ρ‚ΠΎΠ±Ρ‹ Π·Π½Π°Ρ‚ΡŒ навСрняка.

Для этого Π½Π΅ΠΎΠ±Ρ…ΠΎΠ΄ΠΈΠΌΠΎ Π²Ρ‹ΠΏΠΎΠ»Π½ΠΈΡ‚ΡŒ ΡΠ»Π΅Π΄ΡƒΡŽΡ‰Π΅Π΅:

  1. Π‘ ΠΏΠΎΠΌΠΎΡ‰ΡŒΡŽ ΠΌΡƒΠ»ΡŒΡ‚ΠΈΠΌΠ΅Ρ‚Ρ€Π° ΠΈΠ·ΠΌΠ΅Ρ€ΠΈΡ‚ΡŒ сопротивлСниС рСзистора Π² 10 кОм. НапримСр, ΠΎΠ½ΠΎ ΠΏΠΎΠ»ΡƒΡ‡ΠΈΠ»ΠΎΡΡŒ Ρ€Π°Π²Π½ΠΎ 9880 Ом.
  2. ΠŸΠΎΠ΄ΠΊΠ»ΡŽΡ‡Π°Π΅ΠΌ Π±Π»ΠΎΠΊ питания. ΠœΡƒΠ»ΡŒΡ‚ΠΈΠΌΠ΅Ρ‚Ρ€ ΠΏΠ΅Ρ€Π΅Π²ΠΎΠ΄ΠΈΠΌ Π² Ρ€Π΅ΠΆΠΈΠΌ Π·Π°ΠΌΠ΅Ρ€Π° постоянного напряТСния. Π—Π°Ρ‚Π΅ΠΌ ΠΏΠΎΠ΄ΠΊΠ»ΡŽΡ‡Π°Π΅ΠΌ Π΅Π³ΠΎ ΠΊ Π±Π»ΠΎΠΊΡƒ питания (Ρ‡Π΅Ρ€Π΅Π· Π΅Π³ΠΎ Π²Ρ‹Π²ΠΎΠ΄Ρ‹). ПослС этого Π² Π±Π»ΠΎΠΊΠ΅ устанавливаСтся 12 Π²ΠΎΠ»ΡŒΡ‚ (Π½Π° ΠΌΡƒΠ»ΡŒΡ‚ΠΈΠΌΠ΅Ρ‚Ρ€Π΅ Π΄ΠΎΠ»ΠΆΠ½Π° ΠΏΠΎΡΠ²ΠΈΡ‚ΡŒΡΡ Ρ†ΠΈΡ„Ρ€Π° 12,00 Π’). Если ΠΆΠ΅ Π½Π΅ ΡƒΠ΄Π°Π»ΠΎΡΡŒ ΠΎΡ‚Ρ€Π΅Π³ΡƒΠ»ΠΈΡ€ΠΎΠ²Π°Ρ‚ΡŒ напряТСниС Π² Π±Π»ΠΎΠΊΠ΅ ΠΏΠΈΡ‚Π°Π½ΠΈΠ΅, Ρ‚ΠΎ Ρ‚ΠΎΠ³Π΄Π° записываСм Ρ‚Π΅ Ρ€Π΅Π·ΡƒΠ»ΡŒΡ‚Π°Ρ‚Ρ‹, ΠΊΠΎΡ‚ΠΎΡ€Ρ‹Π΅ ΠΏΠΎΠ»ΡƒΡ‡ΠΈΠ»ΠΈΡΡŒ.
  3. Π‘ ΠΏΠΎΠΌΠΎΡ‰ΡŒΡŽ кондСнсатора ΠΈ рСзистора собираСм ΡΠ»Π΅ΠΊΡ‚Ρ€ΠΈΡ‡Π΅ΡΠΊΡƒΡŽ RC-Ρ†Π΅ΠΏΡŒ. На схСмС Π½ΠΈΠΆΠ΅ ΡƒΠΊΠ°Π·Π°Π½Π° простая RC-Ρ†Π΅ΠΏΠΎΡ‡ΠΊΠ°:
  4. Π—Π°ΠΊΠΎΡ€ΠΎΡ‚ΠΈΡ‚ΡŒ кондСнсатор ΠΈ ΠΏΠΎΠ΄ΠΊΠ»ΡŽΡ‡ΠΈΡ‚ΡŒ Ρ†Π΅ΠΏΡŒ ΠΊ ΠΏΠΈΡ‚Π°Π½ΠΈΡŽ. Π‘ ΠΏΠΎΠΌΠΎΡ‰ΡŒΡŽ ΠΏΡ€ΠΈΠ±ΠΎΡ€Π° Π΅Ρ‰Π΅ Ρ€Π°Π· ΠΎΠΏΡ€Π΅Π΄Π΅Π»ΠΈΡ‚ΡŒ напряТСниС, ΠΊΠΎΡ‚ΠΎΡ€ΠΎΠ΅ подаСтся Π½Π° Ρ†Π΅ΠΏΡŒ, ΠΈ Π·Π°ΠΏΠΈΡΠ°Ρ‚ΡŒ это Π·Π½Π°Ρ‡Π΅Π½ΠΈΠ΅.
  5. Π—Π°Ρ‚Π΅ΠΌ Π½Π΅ΠΎΠ±Ρ…ΠΎΠ΄ΠΈΠΌΠΎ Π²Ρ‹ΡΡ‡ΠΈΡ‚Π°Ρ‚ΡŒ 95% ΠΎΡ‚ ΠΏΠΎΠ»ΡƒΡ‡Π΅Π½Π½ΠΎΠ³ΠΎ значСния. К ΠΏΡ€ΠΈΠΌΠ΅Ρ€Ρƒ, Ссли это 12Β Π’ΠΎΠ»ΡŒΡ‚, Ρ‚ΠΎ это Π±ΡƒΠ΄Π΅Ρ‚ 11,4Β Π’. Π’ΠΎ Π΅ΡΡ‚ΡŒ, Π·Π° ΠΎΠΏΡ€Π΅Π΄Π΅Π»Π΅Π½Π½ΠΎΠ΅ врСмя, ΠΊΠΎΡ‚ΠΎΡ€ΠΎΠ΅ равняСтся 3*RC, кондСнсатор ΠΏΠΎΠ»ΡƒΡ‡ΠΈΡ‚ напряТСниС Π² 11,4 Π’. Π€ΠΎΡ€ΠΌΡƒΠ»Π° выглядит ΡΠ»Π΅Π΄ΡƒΡŽΡ‰ΠΈΠΌ ΠΎΠ±Ρ€Π°Π·ΠΎΠΌ:
  6. ΠžΡΡ‚Π°Π»ΠΎΡΡŒ ΠΎΠΏΡ€Π΅Π΄Π΅Π»ΠΈΡ‚ΡŒ врСмя. Для этого устройство раскорачиваСм ΠΈ с ΠΏΠΎΠΌΠΎΡ‰ΡŒΡŽ сСкундомСра ΠΏΡ€ΠΎΠΈΠ·Π²ΠΎΠ΄ΠΈΠΌ отсчСт. ΠžΠΏΡ€Π΅Π΄Π΅Π»Π΅Π½ΠΈΠ΅ 3*RC Π±ΡƒΠ΄Π΅Ρ‚ Π²Ρ‹Ρ‡ΠΈΡΠ»ΡΡ‚ΡŒΡΡ Ρ‚Π°ΠΊΠΈΠΌ ΠΎΠ±Ρ€Π°Π·ΠΎΠΌ: ΠΊΠ°ΠΊ Ρ‚ΠΎΠ»ΡŒΠΊΠΎ напряТСниС Π½Π° устройствС Π±ΡƒΠ΄Π΅Ρ‚ Ρ€Π°Π²Π½ΠΎ 11,4Β Π’, Ρ‚ΠΎ это ΠΈ Π±ΡƒΠ΄Π΅Ρ‚ ΠΎΠ·Π½Π°Ρ‡Π°Ρ‚ΡŒ Π½ΡƒΠΆΠ½ΠΎΠ΅ врСмя.
  7. ΠŸΡ€ΠΎΠΈΠ·Π²ΠΎΠ΄ΠΈΠΌ ΠΎΠΏΡ€Π΅Π΄Π΅Π»Π΅Π½ΠΈΠ΅. Для этого ΠΏΠΎΠ»ΡƒΡ‡Π΅Π½Π½ΠΎΠ΅ врСмя (Π² сСкундах) Π΄Π΅Π»ΠΈΠΌ Π½Π° сопротивлСниС Π² рСзисторС ΠΈ Π½Π° Ρ‚Ρ€ΠΈ. НапримСр, ΠΏΠΎΠ»ΡƒΡ‡ΠΈΠ»ΠΎΡΡŒ 210 сСкунд. Π­Ρ‚Ρƒ Ρ†ΠΈΡ„Ρ€Ρƒ Π΄Π΅Π»ΠΈΠΌ Π½Π° 9880 ΠΈ Π½Π° 3. ΠŸΠΎΠ»ΡƒΡ‡ΠΈΠ»ΠΎΡΡŒ Π·Π½Π°Ρ‡Π΅Π½ΠΈΠ΅ 0,007085. Π­Ρ‚ΠΎ Π²Π΅Π»ΠΈΡ‡ΠΈΠ½Π° указываСтся Π² Ρ„Π°Ρ€Π°Π΄Π°Ρ…, ΠΈΠ»ΠΈ 7085Β ΠΌΠΊΡ„. ДопустимоС ΠΎΡ‚ΠΊΠ»ΠΎΠ½Π΅Π½ΠΈΠ΅ ΠΌΠΎΠΆΠ΅Ρ‚ Π±Ρ‹Ρ‚ΡŒ Π½Π΅ Π±ΠΎΠ»Π΅Π΅ 20%. Если ΡƒΡ‡ΠΈΡ‚Ρ‹Π²Π°Ρ‚ΡŒ, Ρ‡Ρ‚ΠΎ Π½Π° ΠΈΠ·Π΄Π΅Π»ΠΈΠΈ ΡƒΠΊΠ°Π·Π°Π½ΠΎ 6800 ΠΌΠΊΡ„, наши расчСты ΠΏΠΎΠ΄Ρ‚Π²Π΅Ρ€ΠΆΠ΄Π°ΡŽΡ‚ΡΡ ΠΈ ΡƒΠΊΠ»Π°Π΄Ρ‹Π²Π°ΡŽΡ‚ΡΡ Π² Π½ΠΎΡ€ΠΌΠ°Ρ‚ΠΈΠ².

А ΠΊΠ°ΠΊ ΠΎΠΏΡ€Π΅Π΄Π΅Π»ΠΈΡ‚ΡŒ Π΅ΠΌΠΊΠΎΡΡ‚ΡŒ кСрамичСского кондСнсатора? Π’ этом случаС ΠΌΠΎΠΆΠ½ΠΎ ΡΠ΄Π΅Π»Π°Ρ‚ΡŒ ΠΎΠΏΡ€Π΅Π΄Π΅Π»Π΅Π½ΠΈΠ΅ с ΠΏΠΎΠΌΠΎΡ‰ΡŒΡŽ сСтСвого трансформатора. Для этого RC-Ρ†Π΅ΠΏΠΎΡ‡ΠΊΡƒ подсоСдиняСм ΠΊΠΎ Π²Ρ‚ΠΎΡ€ΠΈΡ‡Π½ΠΎΠΉ ΠΎΠ±ΠΌΠΎΡ‚ΠΊΠ΅ трансформатора, ΠΈ Π΅Π³ΠΎ ΠΏΠΎΠ΄ΡΠΎΠ΅Π΄ΠΈΠ½ΡΡŽΡ‚ Π² ΡΠ΅Ρ‚ΡŒ. Π”Π°Π»Π΅Π΅ с ΠΏΠΎΠΌΠΎΡ‰ΡŒΡŽ ΠΌΡƒΠ»ΡŒΡ‚ΠΈΠΌΠ΅Ρ‚Ρ€Π° осущСствляСтся Π·Π°ΠΌΠ΅Ρ€ напряТСния Π½Π° кондСнсаторС ΠΈ Π½Π° рСзисторС. ПослС этого Π½Π΅ΠΎΠ±Ρ…ΠΎΠ΄ΠΈΠΌΠΎ ΡΠ΄Π΅Π»Π°Ρ‚ΡŒ подсчСты: высчитываСтся Ρ‚ΠΎΠΊ, Ρ‡Ρ‚ΠΎ ΠΏΡ€ΠΎΡ…ΠΎΠ΄ΠΈΡ‚ Ρ‡Π΅Ρ€Π΅Π· рСзистор, Π·Π°Ρ‚Π΅ΠΌ Π΅Π³ΠΎ напряТСниС дСлится Π½Π° сопротивлСниС. ΠŸΠΎΠ»ΡƒΡ‡Π°Π΅Ρ‚ΡΡ СмкостноС сопротивлСниС Π₯с.

Если Π΅ΡΡ‚ΡŒ частота Ρ‚ΠΎΠΊΠ° ΠΈ Π₯с, ΠΌΠΎΠΆΠ½ΠΎ ΠΎΠΏΡ€Π΅Π΄Π΅Π»ΠΈΡ‚ΡŒ Π΅ΠΌΠΊΠΎΡΡ‚ΡŒ ΠΏΠΎ Ρ„ΠΎΡ€ΠΌΡƒΠ»Π΅:


Π”Ρ€ΡƒΠ³ΠΈΠ΅ ΠΌΠ΅Ρ‚ΠΎΠ΄ΠΈΠΊΠΈ

Π’Π°ΠΊΠΆΠ΅Β Π΅ΠΌΠΊΠΎΡΡ‚ΡŒ ΠΌΠΎΠΆΠ½ΠΎ ΠΎΠΏΡ€Π΅Π΄Π΅Π»ΠΈΡ‚ΡŒ ΠΈ с ΠΏΠΎΠΌΠΎΡ‰ΡŒΡŽ баллистичСского Π³Π°Π»ΡŒΠ²Π°Π½ΠΎΠΌΠ΅Ρ‚Ρ€Π°. Для этого ΠΈΡΠΏΠΎΠ»ΡŒΠ·ΡƒΠ΅Ρ‚ΡΡ Ρ„ΠΎΡ€ΠΌΡƒΠ»Π°:

Π³Π΄Π΅:

  • Cq β€” баллистичСская постоянная Π³Π°Π»ΡŒΠ²Π°Π½ΠΎΠΌΠ΅Ρ‚Ρ€Π°;
  • U2 β€” показания Π²ΠΎΠ»ΡŒΡ‚ΠΌΠ΅Ρ‚Ρ€Π°;
  • a2 β€” ΡƒΠ³ΠΎΠ» отклонСния Π³Π°Π»ΡŒΠ²Π°Π½ΠΎΠΌΠ΅Ρ‚Ρ€Π°.

ΠžΠΏΡ€Π΅Π΄Π΅Π»Π΅Π½ΠΈΠ΅ значСния ΠΌΠ΅Ρ‚ΠΎΠ΄ΠΎΠΌ Π°ΠΌΠΏΠ΅Ρ€ΠΌΠ΅Ρ‚Ρ€Π° Π²ΠΎΠ»ΡŒΡ‚ΠΌΠ΅Ρ‚Ρ€Π° осущСствляСтся ΡΠ»Π΅Π΄ΡƒΡŽΡ‰ΠΈΠΌ ΠΎΠ±Ρ€Π°Π·ΠΎΠΌ: измСряСтся напряТСниС ΠΈ Ρ‚ΠΎΠΊ Π² Ρ†Π΅ΠΏΠΈ, послС Ρ‡Π΅Π³ΠΎ Π·Π½Π°Ρ‡Π΅Π½ΠΈΠ΅ Смкости опрСдСляСтся ΠΏΠΎ Ρ„ΠΎΡ€ΠΌΡƒΠ»Π΅:

НапряТСниС ΠΏΡ€ΠΈ Ρ‚Π°ΠΊΠΎΠΌ ΠΌΠ΅Ρ‚ΠΎΠ΄Π΅ опрСдСлСния долТно Π±Ρ‹Ρ‚ΡŒ ΡΠΈΠ½ΡƒΡΠΎΠΈΠ΄Π°Π»ΡŒΠ½Ρ‹ΠΌ.

Π˜Π·ΠΌΠ΅Ρ€Π΅Π½ΠΈΠ΅ значСния Π²ΠΎΠ·ΠΌΠΎΠΆΠ½ΠΎ ΠΈ ΠΏΡ€ΠΈ ΠΏΠΎΠΌΠΎΡ‰ΠΈ мостиковой схСмы. Π’ этом случаС схСма моста ΠΏΠ΅Ρ€Π΅ΠΌΠ΅Π½Π½ΠΎΠ³ΠΎ Ρ‚ΠΎΠΊΠ° указываСтся Π½ΠΈΠΆΠ΅:

Π—Π΄Π΅ΡΡŒ ΠΎΠ΄Π½ΠΎ ΠΏΠ»Π΅Ρ‡ΠΎ моста образуСтся Π·Π° счСт элСмСнта, ΠΊΠΎΡ‚ΠΎΡ€Ρ‹ΠΉ Π½Π΅ΠΎΠ±Ρ…ΠΎΠ΄ΠΈΠΌΠΎ ΠΈΠ·ΠΌΠ΅Ρ€ΠΈΡ‚ΡŒ (Cx).

Π‘Π»Π΅Π΄ΡƒΡŽΡ‰Π΅Π΅ ΠΏΠ»Π΅Ρ‡ΠΎ состоит ΠΈΠ· кондСнсатора Π±Π΅Π· ΠΏΠΎΡ‚Π΅Ρ€ΡŒ ΠΈ ΠΌΠ°Π³Π°Π·ΠΈΠ½Π° сопротивлСний. ΠžΡΡ‚Π°Π²ΡˆΠΈΠ΅ΡΡ Π΄Π²Π° ΠΏΠ»Π΅Ρ‡Π° состоят ΠΈΠ· ΠΌΠ°Π³Π°Π·ΠΈΠ½ΠΎΠ² сопротивлСний. ΠŸΠΎΠ΄ΠΊΠ»ΡŽΡ‡Π°Π΅ΠΌ Π² ΠΎΠ΄Π½Ρƒ диагональ источник питания, Π² Π΄Ρ€ΡƒΠ³ΡƒΡŽ – Π½ΡƒΠ»Π΅Π²ΠΎΠΉ ΠΈΠ½Π΄ΠΈΠΊΠ°Ρ‚ΠΎΡ€. И рассчитываСм Π·Π½Π°Ρ‡Π΅Π½ΠΈΠ΅ ΠΏΠΎ Ρ„ΠΎΡ€ΠΌΡƒΠ»Π΅:

НапослСдок Ρ€Π΅ΠΊΠΎΠΌΠ΅Π½Π΄ΡƒΠ΅ΠΌ ΠΏΡ€ΠΎΡΠΌΠΎΡ‚Ρ€Π΅Ρ‚ΡŒ ΠΏΠΎΠ»Π΅Π·Π½ΠΎΠ΅ Π²ΠΈΠ΄Π΅ΠΎ ΠΏΠΎ Ρ‚Π΅ΠΌΠ΅:

Π­Ρ‚ΠΎ всС, Ρ‡Ρ‚ΠΎ ΠΌΡ‹ Ρ…ΠΎΡ‚Π΅Π»ΠΈ Ρ€Π°ΡΡΠΊΠ°Π·Π°Ρ‚ΡŒ Π²Π°ΠΌ ΠΎ Ρ‚ΠΎΠΌ, ΠΊΠ°ΠΊ ΠΎΠΏΡ€Π΅Π΄Π΅Π»ΠΈΡ‚ΡŒ Π΅ΠΌΠΊΠΎΡΡ‚ΡŒ кондСнсатора ΠΌΡƒΠ»ΡŒΡ‚ΠΈΠΌΠ΅Ρ‚Ρ€ΠΎΠΌ. НадССмся, прСдоставлСнная информация Π±Ρ‹Π»Π° для вас ΠΏΠΎΠ»Π΅Π·Π½ΠΎΠΉ ΠΈ интСрСсной!

НавСрняка Π²Ρ‹ Π½Π΅ Π·Π½Π°Π΅Ρ‚Π΅:

Как ΠΏΡ€ΠΎΠ²Π΅Ρ€ΠΈΡ‚ΡŒ кондСнсатор ΠΌΡƒΠ»ΡŒΡ‚ΠΈΠΌΠ΅Ρ‚Ρ€ΠΎΠΌ Π½Π° Ρ€Π°Π±ΠΎΡ‚ΠΎΡΠΏΠΎΡΠΎΠ±Π½ΠΎΡΡ‚ΡŒ

По сути Ρ€Π΅ΠΌΠΎΠ½Ρ‚ любой радиоэлСктронной Π°ΠΏΠΏΠ°Ρ€Π°Ρ‚ΡƒΡ€Ρ‹ сводится ΠΊ поиску ΠΈ Π·Π°ΠΌΠ΅Π½Π΅ нСисправных Π΄Π΅Ρ‚Π°Π»Π΅ΠΉ. И, Π²ΠΎΠ·ΠΌΠΎΠΆΠ½ΠΎ, Π²Ρ‹ ΡƒΠ΄ΠΈΠ²ΠΈΡ‚Π΅ΡΡŒ Ρ‚ΠΎΠΌΡƒ, насколько часто выходят ΠΈΠ· строя Ρ‚Π°ΠΊΠΈΠ΅, казалось Π±Ρ‹, простыС ΠΊΠΎΠΌΠΏΠΎΠ½Π΅Π½Ρ‚Ρ‹ ΠΊΠ°ΠΊ кондСнсаторы. Π’ Ρ‚ΠΎ врСмя ΠΊΠ°ΠΊ Π½Π΅ΠΆΠ½Ρ‹Π΅ Π΄ΠΈΠΎΠ΄Ρ‹, Ρ‡ΡƒΠ²ΡΡ‚Π²ΠΈΡ‚Π΅Π»ΡŒΠ½Ρ‹Π΅ транзисторы ΠΈ слоТныС микросхСмы ΠΎΡΡ‚Π°ΡŽΡ‚ΡΡ Ρ†Π΅Π»Ρ‹ΠΌΠΈ ΠΈ Π½Π΅Π²Ρ€Π΅Π΄ΠΈΠΌΡ‹ΠΌΠΈ.

Π’ΠΈΠΏΠΈΡ‡Π½Ρ‹Π΅ нСисправности кондСнсаторов:

  • ΠšΠ— ΠΌΠ΅ΠΆΠ΄Ρƒ ΠΎΠ±ΠΊΠ»Π°Π΄ΠΊΠ°ΠΌΠΈ. Как ΠΏΡ€Π°Π²ΠΈΠ»ΠΎ, это слСдствиС мСханичСского поврСТдСния, ΠΏΠ΅Ρ€Π΅Π³Ρ€Π΅Π²Π° ΠΈΠ»ΠΈ ΠΏΡ€Π΅Π²Ρ‹ΡˆΠ΅Π½ΠΈΡ Ρ€Π°Π±ΠΎΡ‡Π΅Π³ΠΎ напряТСния (ΠΏΡ€ΠΎΠ±ΠΎΠΉ). Π‘Π°ΠΌΡ‹ΠΉ простой случай, Ρ‚.ΠΊ. Π»Π΅Π³ΠΊΠΎ выявляСтся Π»ΡŽΠ±Ρ‹ΠΌ ΠΌΡƒΠ»ΡŒΡ‚ΠΈΠΌΠ΅Ρ‚Ρ€ΠΎΠΌ Π² Ρ€Π΅ΠΆΠΈΠΌΠ΅ ΠΏΡ€ΠΎΠ·Π²ΠΎΠ½ΠΊΠΈ;
  • Π²Π½ΡƒΡ‚Ρ€Π΅Π½Π½ΠΈΠΉ ΠΎΠ±Ρ€Ρ‹Π² с ΠΏΠΎΠ»Π½ΠΎΠΉ ΠΏΠΎΡ‚Π΅Ρ€Π΅ΠΉ Смкости (Π²ΠΎΡ‚ ΠΏΠΎΡ‡Π΅ΠΌΡƒ нСльзя ΠΊΠΎΡ€ΠΎΡ‚ΠΈΡ‚ΡŒ ΠΎΡ‚Π²Π΅Ρ€Ρ‚ΠΊΠ°ΠΌΠΈ). Π’ случаС с кондСнсаторами большой Смкости этот Π΄Π΅Ρ„Π΅ΠΊΡ‚ достаточно просто диагностируСтся. ВыявлСниС ΠΎΠ±Ρ€Ρ‹Π²Π° Ρƒ ΠΌΠ΅Π»ΠΊΠΈΡ… ΠΊΠΎΠ½Π΄Π΅Ρ€ΠΎΠ² (ΠΌΠ΅Π½Π΅Π΅ 500 ΠΏΠ€) являСтся довольно Ρ‚Ρ€ΡƒΠ΄ΠΎΠ΅ΠΌΠΊΠΎΠΉ Π·Π°Π΄Π°Ρ‡Π΅ΠΉ ΠΈ осущСствляСтся Ρ‚ΠΎΠ»ΡŒΠΊΠΎ ΠΏΡ€ΠΈ ΠΏΠΎΠΌΠΎΡ‰ΠΈ спСц. ΠΏΡ€ΠΈΠ±ΠΎΡ€ΠΎΠ²;
  • частичная потСря Смкости. Для элСктролитичСских кондСнсаторов потСря Смкости с Π³ΠΎΠ΄Π°ΠΌΠΈ практичСски Π½Π΅ΠΈΠ·Π±Π΅ΠΆΠ½Π°, ΠΎΠ΄Π½Π°ΠΊΠΎ это Π½Π΅ всСгда ΠΏΡ€ΠΈΠ²ΠΎΠ΄ΠΈΡ‚ ΠΊ нСисправности устройства (Π½ΠΎ ΠΌΠΎΠΆΠ΅Ρ‚ ΡƒΡ…ΡƒΠ΄ΡˆΠ°Ρ‚ΡŒ Π΅Π³ΠΎ характСристики). ΠšΠ΅Ρ€Π°ΠΌΠΈΡ‡Π΅ΡΠΊΠΈΠ΅, ΠΏΠ»Π΅Π½ΠΎΡ‡Π½Ρ‹Π΅ ΠΈ ΠΏΡ€ΠΎΡ‡ΠΈΠ΅ с Ρ‚Π²Π΅Ρ€Π΄Ρ‹ΠΌ диэлСктриком, ΠΊΠ°ΠΊ ΠΏΡ€Π°Π²ΠΈΠ»ΠΎ, Π±ΠΎΠ»Π΅Π΅ ΡΡ‚Π°Π±ΠΈΠ»ΡŒΠ½Ρ‹, Π½ΠΎ ΠΌΠΎΠ³ΡƒΡ‚ ΠΏΠΎΡ‚Π΅Ρ€ΡΡ‚ΡŒ Π΅ΠΌΠΊΠΎΡΡ‚ΡŒ Π² Ρ€Π΅Π·ΡƒΠ»ΡŒΡ‚Π°Ρ‚Π΅ мСханичСского поврСТдСния;
  • слишком Π½ΠΈΠ·ΠΊΠΎΠ΅ сопротивлСниС ΡƒΡ‚Π΅Ρ‡ΠΊΠΈ (кондСнсатор «Π½Π΅ Π΄Π΅Ρ€ΠΆΠΈΡ‚» заряд). Π’ основном это свойствСнно элСктролитичСским кондСнсаторам. Π₯отя Ρ‚Π°Π½Ρ‚Π°Π»ΠΎΠ²Ρ‹Π΅ Π² этом ΠΏΠ»Π°Π½Π΅ ΠΎΡ‡Π΅Π½ΡŒ Ρ…ΠΎΡ€ΠΎΡˆΠΈ;
  • слишком большоС эквивалСнтноС ΠΏΠΎΡΠ»Π΅Π΄ΠΎΠ²Π°Ρ‚Π΅Π»ΡŒΠ½ΠΎΠ΅ сопротивлСниС (Π•ΠŸΠ‘ ΠΈΠ»ΠΈ ESR). ΠŸΡ€ΠΎΠ±Π»Π΅ΠΌΠ° ΠΏΠΎ большСй части касаСтся «ΡΠ»Π΅ΠΊΡ‚Ρ€ΠΎΠ»ΠΈΡ‚ΠΎΠ²» ΠΈ проявляСтся Ρ‚ΠΎΠ»ΡŒΠΊΠΎ ΠΏΡ€ΠΈ Ρ€Π°Π±ΠΎΡ‚Π΅ с высокочастотными ΠΈΠ»ΠΈ ΠΈΠΌΠΏΡƒΠ»ΡŒΡΠ½Ρ‹ΠΌΠΈ Ρ‚ΠΎΠΊΠ°ΠΌΠΈ.

БущСствуСт масса способов ΠΊΠ°ΠΊ ΠΏΡ€ΠΎΠ²Π΅Ρ€ΠΈΡ‚ΡŒ кондСнсатор ΠΌΡƒΠ»ΡŒΡ‚ΠΈΠΌΠ΅Ρ‚Ρ€ΠΎΠΌ Π½Π° Ρ€Π°Π±ΠΎΡ‚ΠΎΡΠΏΠΎΡΠΎΠ±Π½ΠΎΡΡ‚ΡŒ. ПойдСм ΠΏΠΎ-порядку.

Π‘ΠΎΠ΄Π΅Ρ€ΠΆΠ°Π½ΠΈΠ΅ ΡΡ‚Π°Ρ‚ΡŒΠΈ:

Π’Π½Π΅ΡˆΠ½ΠΈΠΉ осмотр

Иногда достаточно ΠΎΠ΄Π½ΠΎΠ³ΠΎ взгляда, Ρ‡Ρ‚ΠΎΠ±Ρ‹ ΠΎΠΏΡ€Π΅Π΄Π΅Π»ΠΈΡ‚ΡŒ нСисправный кондСнсатор Π½Π° ΠΏΠ»Π°Ρ‚Π΅. Π’ Ρ‚Π°ΠΊΠΈΡ… случаях Π½Π΅Ρ‚ смысла ΠΏΡ€ΠΎΠ²Π΅Ρ€ΡΡ‚ΡŒ Π΅Π³ΠΎ ΠΊΠ°ΠΊΠΈΠΌΠΈ-Π»ΠΈΠ±ΠΎ ΠΏΡ€ΠΈΠ±ΠΎΡ€Π°ΠΌΠΈ.ΠšΠΎΠ½Π΄Π΅Π½ΡΠ°Ρ‚ΠΎΡ€ ΠΏΠΎΠ΄Π»Π΅ΠΆΠΈΡ‚ Π·Π°ΠΌΠ΅Π½Π΅, Ссли Π²ΠΈΠ·ΡƒΠ°Π»ΡŒΠ½Ρ‹ΠΉ осмотр ΠΏΠΎΠΊΠ°Π·Π°Π» Π½Π°Π»ΠΈΡ‡ΠΈΠ΅:

  • Π΄Π°ΠΆΠ΅ Π½Π΅Π·Π½Π°Ρ‡ΠΈΡ‚Π΅Π»ΡŒΠ½ΠΎΠ³ΠΎ вздутия, слСдов ΠΏΠΎΠ΄Ρ‚Π΅ΠΊΠΎΠ²;
  • мСханичСских ΠΏΠΎΠ²Ρ€Π΅ΠΆΠ΄Π΅Π½ΠΈΠΉ, вмятин;
  • Ρ‚Ρ€Π΅Ρ‰ΠΈΠ½, сколов (Π°ΠΊΡ‚ΡƒΠ°Π»ΡŒΠ½ΠΎ для ΠΊΠ΅Ρ€Π°ΠΌΠΈΠΊΠΈ).

ΠšΠΎΠ½Π΄Π΅Π½ΡΠ°Ρ‚ΠΎΡ€Ρ‹, ΠΈΠΌΠ΅ΡŽΡ‰ΠΈΠ΅ любой ΠΈΠ· ΡƒΠΊΠ°Π·Π°Π½Π½Ρ‹Ρ… ΠΏΡ€ΠΈΠ·Π½Π°ΠΊΠΎΠ², ΡΠΊΡΠΏΠ»ΡƒΠ°Ρ‚ΠΈΡ€ΠΎΠ²Π°Ρ‚ΡŒ НЕЛЬЗЯ.

Π˜Π·ΠΌΠ΅Ρ€Π΅Π½ΠΈΠ΅ Смкости кондСнсатора ΠΌΡƒΠ»ΡŒΡ‚ΠΈΠΌΠ΅Ρ‚Ρ€ΠΎΠΌ ΠΈ ΡΠΏΠ΅Ρ†ΠΈΠ°Π»ΡŒΠ½Ρ‹ΠΌΠΈ ΠΏΡ€ΠΈΠ±ΠΎΡ€Π°ΠΌΠΈ

НСкоторыС ΠΌΡƒΠ»ΡŒΡ‚ΠΈΠΌΠ΅Ρ‚Ρ€Ρ‹ ΠΈΠΌΠ΅ΡŽΡ‚ Ρ„ΡƒΠ½ΠΊΡ†ΠΈΡŽ измСрСния Смкости. Π’Π·ΡΡ‚ΡŒ хотя Π±Ρ‹ эти распространСнныС ΠΌΠΎΠ΄Π΅Π»ΠΈ: M890D, AM-1083, DT9205A, UT139C ΠΈ Ρ‚.Π΄.Π’Π°ΠΊΠΆΠ΅ Π² ΠΏΡ€ΠΎΠ΄Π°ΠΆΠ΅ Π΅ΡΡ‚ΡŒ Ρ†ΠΈΡ„Ρ€ΠΎΠ²Ρ‹Π΅ ΠΈΠ·ΠΌΠ΅Ρ€ΠΈΡ‚Π΅Π»ΠΈ Смкости, Π½Π°ΠΏΡ€ΠΈΠΌΠ΅Ρ€, XC6013L ΠΈΠ»ΠΈ A6013L.

Π‘ ΠΏΠΎΠΌΠΎΡ‰ΡŒΡŽ любого ΠΈΠ· этих ΠΏΡ€ΠΈΠ±ΠΎΡ€ΠΎΠ² ΠΌΠΎΠΆΠ½ΠΎ Π½Π΅ Ρ‚ΠΎΠ»ΡŒΠΊΠΎ ΡƒΠ·Π½Π°Ρ‚ΡŒ Ρ‚ΠΎΡ‡Π½ΡƒΡŽ Π΅ΠΌΠΊΠΎΡΡ‚ΡŒ кондСнсатора, Π½ΠΎ ΠΈ ΡƒΠ±Π΅Π΄ΠΈΡ‚ΡŒΡΡ Π² отсутствии ΠΊΠΎΡ€ΠΎΡ‚ΠΊΠΎΠ³ΠΎ замыкания ΠΌΠ΅ΠΆΠ΄Ρƒ ΠΎΠ±ΠΊΠ»Π°Π΄ΠΊΠ°ΠΌΠΈ ΠΈΠ»ΠΈ Π²Π½ΡƒΡ‚Ρ€Π΅Π½Π½Π΅Π³ΠΎ ΠΎΠ±Ρ€Ρ‹Π²Π° ΠΎΠ΄Π½ΠΎΠ³ΠΎ ΠΈΠ· Π²Ρ‹Π²ΠΎΠ΄ΠΎΠ².

НСкоторыС ΠΏΡ€ΠΎΠΈΠ·Π²ΠΎΠ΄ΠΈΡ‚Π΅Π»ΠΈ Π΄Π°ΠΆΠ΅ ΡƒΠ²Π΅Ρ€ΡΡŽΡ‚, Ρ‡Ρ‚ΠΎ ΠΈΡ… ΠΌΡƒΠ»ΡŒΡ‚ΠΈΠΌΠ΅Ρ‚Ρ€Ρ‹ способны ΠΏΡ€ΠΎΠ²Π΅Ρ€ΠΈΡ‚ΡŒ Π΅ΠΌΠΊΠΎΡΡ‚ΡŒ кондСнсатора Π½Π΅ выпаивая Π΅Π³ΠΎ с ΠΏΠ»Π°Ρ‚Ρ‹. Π§Ρ‚ΠΎ, ΠΊΠΎΠ½Π΅Ρ‡Π½ΠΎ ΠΆΠ΅, ΠΏΡ€ΠΎΡ‚ΠΈΠ²ΠΎΡ€Π΅Ρ‡ΠΈΡ‚ Π·Π΄Ρ€Π°Π²ΠΎΠΌΡƒ смыслу.

К соТалСнию, ΠΏΡ€ΠΎΠ²Π΅Ρ€ΠΊΠ° кондСнсатора ΠΌΡƒΠ»ΡŒΡ‚ΠΈΠΌΠ΅Ρ‚Ρ€ΠΎΠΌ Π½Π΅ ΠΏΠΎΠΌΠΎΠΆΠ΅Ρ‚ ΠΎΠΏΡ€Π΅Π΄Π΅Π»ΠΈΡ‚ΡŒ Ρ‚Π°ΠΊΠΈΠ΅ наиваТнСйшиС ΠΏΠ°Ρ€Π°ΠΌΠ΅Ρ‚Ρ€Ρ‹, ΠΊΠ°ΠΊ Ρ‚ΠΎΠΊ ΡƒΡ‚Π΅Ρ‡ΠΊΠΈ ΠΈ эквивалСнтноС ΠΏΠΎΡΠ»Π΅Π΄ΠΎΠ²Π°Ρ‚Π΅Π»ΡŒΠ½ΠΎΠ΅ сопротивлСниС (ESR). Π˜Ρ… ΠΈΠ·ΠΌΠ΅Ρ€ΠΈΡ‚ΡŒ Ρ‚ΠΎΠ»ΡŒΠΊΠΎ с ΠΏΠΎΠΌΠΎΡ‰ΡŒΡŽ спСциализированных тСстСров. НапримСр, с ΠΏΠΎΠΌΠΎΡ‰ΡŒΡŽ вСсьма Π½Π΅Π΄ΠΎΡ€ΠΎΠ³ΠΎΠ³ΠΎ LC-ΠΌΠ΅Ρ‚Ρ€Π°.

ΠŸΡ€ΠΎΠ²Π΅Ρ€ΠΊΠ° Π½Π° ΠΊΠΎΡ€ΠΎΡ‚ΠΊΠΎΠ΅ Π·Π°ΠΌΡ‹ΠΊΠ°Π½ΠΈΠ΅

Бпособ β„–1: ΠΎΠΏΡ€Π΅Π΄Π΅Π»Π΅Π½ΠΈΠ΅ ΠšΠ— Π² Ρ€Π΅ΠΆΠΈΠΌΠ΅ ΠΏΡ€ΠΎΠ·Π²ΠΎΠ½ΠΊΠΈ

Как ΠΏΡ€ΠΎΠ·Π²Π°Π½ΠΈΠ²Π°Ρ‚ΡŒ кондСнсаторы ΠΌΡƒΠ»ΡŒΡ‚ΠΈΠΌΠ΅Ρ‚Ρ€ΠΎΠΌ? НуТно Π²ΠΊΠ»ΡŽΡ‡ΠΈΡ‚ΡŒ ΠΌΡƒΠ»ΡŒΡ‚ΠΈΠΌΠ΅Ρ‚Ρ€ Π² Ρ€Π΅ΠΆΠΈΠΌ ΠΏΡ€ΠΎΠ·Π²ΠΎΠ½ΠΊΠΈ ΠΈΠ»ΠΈ измСрСния сопротивлСния ΠΈ ΠΏΡ€ΠΈΠ»ΠΎΠΆΠΈΡ‚ΡŒ Ρ‰ΡƒΠΏΡ‹ ΠΊ Π²Ρ‹Π²ΠΎΠ΄Π°ΠΌ кондСнсатора.

Π’ зависимости ΠΎΡ‚ Смкости ΠΌΡƒΠ»ΡŒΡ‚ΠΈΠΌΠ΅Ρ‚Ρ€ Π»ΠΈΠ±ΠΎ сразу ΠΆΠ΅ ΠΏΠΎΠΊΠ°ΠΆΠ΅Ρ‚ бСсконСчноС сопротивлСниС, Π»ΠΈΠ±ΠΎ Ρ‡Π΅Ρ€Π΅Π· ΠΊΠ°ΠΊΠΎΠ΅-Ρ‚ΠΎ врСмя (ΠΎΡ‚ Π½Π΅ΡΠΊΠΎΠ»ΡŒΠΊΠΈΡ… сСкунд Π΄ΠΎ дСсятков сСкунд).

Если ΠΆΠ΅ ΠΏΡ€ΠΈΠ±ΠΎΡ€ постоянно ΠΏΠΈΡ‰ΠΈΡ‚ Π² Ρ€Π΅ΠΆΠΈΠΌΠ΅ ΠΏΡ€ΠΎΠ·Π²ΠΎΠ½ΠΊΠΈ (ΠΈΠ»ΠΈ ΠΏΠΎΠΊΠ°Π·Ρ‹Π²Π°Π΅Ρ‚ ΠΎΡ‡Π΅Π½ΡŒ Π½ΠΈΠ·ΠΊΠΎΠ΅ сопротивлСниС Π² Ρ€Π΅ΠΆΠΈΠΌΠ΅ измСрСния сопротивлСния), Ρ‚ΠΎ кондСнсатор ΠΌΠΎΠΆΠ½ΠΎ смСло Π²Ρ‹ΠΊΠΈΠ΄Ρ‹Π²Π°Ρ‚ΡŒ.

Бпособ β„–2: ΠΎΠΏΡ€Π΅Π΄Π΅Π»Π΅Π½ΠΈΠ΅ ΠšΠ— кондСнсатора с ΠΏΠΎΠΌΠΎΡ‰ΡŒΡŽ свСтодиода ΠΈ Π±Π°Ρ‚Π°Ρ€Π΅ΠΉΠΊΠΈ

Если Π½Π΅Ρ‚ ΠΌΡƒΠ»ΡŒΡ‚ΠΈΠΌΠ΅Ρ‚Ρ€Π° (ΠΈ Π΄Π°ΠΆΠ΅ старой совСтской «Ρ†Π΅ΡˆΠΊΠΈ» Π½Π΅Ρ‚Ρƒ), Ρ‚ΠΎ ΠΌΠΎΠΆΠ½ΠΎ ΠΏΠΎΠΏΡ€ΠΎΠ±ΠΎΠ²Π°Ρ‚ΡŒ ΠΏΠΎΠ΄ΠΊΠ»ΡŽΡ‡ΠΈΡ‚ΡŒ свСтодиод ΠΈΠ»ΠΈ Π»Π°ΠΌΠΏΠΎΡ‡ΠΊΡƒ ΠΊ Π±Π°Ρ‚Π°Ρ€Π΅ΠΉΠΊΠ΅ Ρ‡Π΅Ρ€Π΅Π· исслСдуСмый кондСнсатор.

Π’.ΠΊ. исправный кондСнсатор ΠΈΠΌΠ΅Π΅Ρ‚ ΠΎΠΎΠΎΡ‡Π΅Π½ΡŒ большоС сопротивлСниС постоянному Ρ‚ΠΎΠΊΡƒ, Π»Π°ΠΌΠΏΠΎΡ‡ΠΊΠ° Π³ΠΎΡ€Π΅Ρ‚ΡŒ Π½Π΅ Π΄ΠΎΠ»ΠΆΠ½Π°. Π₯отя, Ссли Π΅ΠΌΠΊΠΎΡΡ‚ΡŒ кондСнсатора достаточно большая, Π»Π°ΠΌΠΏΠΎΡ‡ΠΊΠ° ΠΌΠΎΠΆΠ΅Ρ‚ Π²ΡΠΏΡ‹Ρ…Π½ΡƒΡ‚ΡŒ Π½Π° ΠΊΠΎΡ€ΠΎΡ‚ΠΊΠΎΠ΅ врСмя (ΠΏΠΎΠΊΠ° кондСнсатор Π½Π΅ зарядится).

Если ΠΆΠ΅ свСтодиод Π³ΠΎΡ€ΠΈΡ‚ постоянно, кондСнсатор 100% нСисправСн.

Если ΠΏΡ€ΠΈ ΠΏΡ€ΠΎΠ²Π΅Ρ€ΠΊΠ΅ кондСнсатора Π½Π°Π±Π»ΡŽΠ΄Π°Π΅Ρ‚ΡΡ эффСкт постСпСнного роста сопротивлСния Π²ΠΏΠ»ΠΎΡ‚ΡŒ Π΄ΠΎ бСсконСчности (Π½Ρƒ ΠΈΠ»ΠΈ свСтодиод Π½Π° ΠΊΠ°ΠΊΠΎΠ΅-Ρ‚ΠΎ врСмя вспыхиваСт ΠΈ гаснСт) Ρ‚ΠΎ кондСнсатор ΡΠΎΠ²Π΅Ρ€ΡˆΠ΅Π½Π½ΠΎ Ρ‚ΠΎΡ‡Π½ΠΎ ΠΈΠΌΠ΅Π΅Ρ‚ ΠΊΠ°ΠΊΡƒΡŽ-Ρ‚ΠΎ Π΅ΠΌΠΊΠΎΡΡ‚ΡŒ. Π‘Π»Π΅Π΄ΠΎΠ²Π°Ρ‚Π΅Π»ΡŒΠ½ΠΎ, ΠΏΡ€ΠΎΠ²Π΅Ρ€ΠΊΡƒ Π½Π° ΠΎΠ±Ρ€Ρ‹Π² ΠΌΠΎΠΆΠ½ΠΎ Π½Π΅ Π΄Π΅Π»Π°Ρ‚ΡŒ.

Бпособ β„–3: ΠΏΡ€ΠΎΠ²Π΅Ρ€ΠΊΠ° кондСнсатора Π»Π°ΠΌΠΏΠΎΡ‡ΠΊΠΎΠΉ Π½Π° 220Π’

ΠŸΠΎΠ΄Ρ…ΠΎΠ΄ΠΈΡ‚ для Π²Ρ‹ΡΠΎΠΊΠΎΠ²ΠΎΠ»ΡŒΡ‚Π½Ρ‹Ρ… нСполярных кондСнсаторов (Π½Π°ΠΏΡ€ΠΈΠΌΠ΅Ρ€, пусковыС кондСнсаторы ΠΈΠ· ΡΡ‚ΠΈΡ€Π°Π»ΡŒΠ½Ρ‹Ρ… машин, насосов, Ρ€Π°Π·Π»ΠΈΡ‡Π½Ρ‹Ρ… станков ΠΈ Ρ‚.ΠΏ.).

ВсС Ρ‡Ρ‚ΠΎ Π½ΡƒΠΆΠ½ΠΎ ΡΠ΄Π΅Π»Π°Ρ‚ΡŒ — просто ΠΏΠΎΠ΄ΠΊΠ»ΡŽΡ‡ΠΈΡ‚ΡŒ Π»Π°ΠΌΠΏΡƒ накаливания нСбольшой мощности (25-40 Π’Ρ‚) Ρ‡Π΅Ρ€Π΅Π· кондСнсатор. ΠŸΠΎΠ»ΡΡ€Π½ΠΎΡΡ‚ΡŒ кондСнсатора Π½Π΅ ΠΈΠΌΠ΅Π΅Ρ‚ значСния:

Бпособ позволяСт ΠΎΠ΄Π½ΠΈΠΌ выстрСлом ΡƒΠ±ΠΈΡ‚ΡŒ Π΄Π²ΡƒΡ… Π·Π°ΠΉΡ†Π΅Π²: ΠΎΠ±Π½Π°Ρ€ΡƒΠΆΠΈΡ‚ΡŒ ΠšΠ—, Ссли ΠΎΠ½ΠΎ Π΅ΡΡ‚ΡŒ, ΠΈ ΡƒΠ±Π΅Π΄ΠΈΡ‚ΡŒΡΡ Π² Ρ‚ΠΎΠΌ, Ρ‡Ρ‚ΠΎ кондСнсатор ΠΈΠΌΠ΅Π΅Ρ‚ Π½Π΅Π½ΡƒΠ»Π΅Π²ΡƒΡŽ Π΅ΠΌΠΊΠΎΡΡ‚ΡŒ (Π½Π΅ находится Π² ΠΎΠ±Ρ€Ρ‹Π²Π΅).

ΠŸΡ€ΠΈ исправном кондСнсаторС Π»Π°ΠΌΠΏΠΎΡ‡ΠΊΠ° Π±ΡƒΠ΄Π΅Ρ‚ Π³ΠΎΡ€Π΅Ρ‚ΡŒ Π² ΠΏΠΎΠ»Π½Π°ΠΊΠ°Π»Π°. Π§Π΅ΠΌ мСньшС Π΅ΠΌΠΊΠΎΡΡ‚ΡŒ — Ρ‚Π΅ΠΌ тусклСС Π±ΡƒΠ΄Π΅Ρ‚ Π³ΠΎΡ€Π΅Ρ‚ΡŒ Π»Π°ΠΌΠΏΠΎΡ‡ΠΊΠ°.

Если Π»Π°ΠΌΠΏΠ° Π³ΠΎΡ€ΠΈΡ‚ Π² ΠΏΠΎΠ»Π½ΡƒΡŽ ΠΌΠΎΡ‰Π½ΠΎΡΡ‚ΡŒ (Ρ‚ΠΎΡ‡Π½ΠΎ Ρ‚Π°ΠΊΠΆΠ΅ ΠΊΠ°ΠΊ ΠΈ Π±Π΅Π· кондСнсатора), Π·Π½Π°Ρ‡ΠΈΡ‚ кондСнсатор «ΠΏΡ€ΠΎΠ±ΠΈΡ‚» ΠΈ ΠΏΠΎΠ΄Π»Π΅ΠΆΠΈΡ‚ Π·Π°ΠΌΠ΅Π½Π΅. Если Π»Π°ΠΌΠΏΠΎΡ‡ΠΊΠ° совсСм Π½Π΅ свСтится — Π²Π½ΡƒΡ‚Ρ€ΠΈ кондСнсатора ΠΎΠ±Ρ€Ρ‹Π².

Бпособ β„–3 ΠΎΡ‡Π΅Π½ΡŒ наглядно продСмонстрирован Π² этом Π²ΠΈΠ΄Π΅ΠΎ:

ΠŸΡ€ΠΎΠ²Π΅Ρ€ΠΊΠ° Π½Π° отсутствиС Π²Π½ΡƒΡ‚Ρ€Π΅Π½Π½Π΅Π³ΠΎ ΠΎΠ±Ρ€Ρ‹Π²Π°

ΠžΠ±Ρ€Ρ‹Π² — распространСнный Π΄Π΅Ρ„Π΅ΠΊΡ‚ кондСнсатора, ΠΏΡ€ΠΈ ΠΊΠΎΡ‚ΠΎΡ€ΠΎΠΌ ΠΎΠ΄ΠΈΠ½ ΠΈΠ· Π΅Π³ΠΎ элСктродов тСряСт элСктричСскоС соСдинСниС с ΠΎΠ±ΠΊΠ»Π°Π΄ΠΊΠΎΠΉ ΠΈ фактичСски прСвращаСтся Π² ΠΊΠΎΡ€ΠΎΡ‚ΠΊΠΈΠΉ, Π½ΠΈ с Ρ‡Π΅ΠΌ Π½Π΅ соСдинСнный (висящий Π² Π²ΠΎΠ·Π΄ΡƒΡ…Π΅), ΠΏΡ€ΠΎΠ²ΠΎΠ΄Π½ΠΈΠΊ.

Π§Π°Ρ‰Π΅ всСго ΠΎΠ±Ρ€Ρ‹Π² происходит ΠΈΠ·-Π·Π° ΠΏΡ€Π΅Π²Ρ‹ΡˆΠ΅Π½ΠΈΡ Ρ€Π°Π±ΠΎΡ‡Π΅Π³ΠΎ напряТСния кондСнсатора. Π­Ρ‚ΠΈΠΌ Π³Ρ€Π΅ΡˆΠ°Ρ‚ Π½Π΅ Ρ‚ΠΎΠ»ΡŒΠΊΠΎ элСктролитичСскиС кондСнсаторы, Π½ΠΎ ΠΈ ΡΠΏΠ΅Ρ†ΠΈΠ°Π»ΡŒΠ½Ρ‹Π΅ ΠΏΠΎΠΌΠ΅Ρ…ΠΎΠΏΠΎΠ΄Π°Π²Π»ΡΡŽΡ‰ΠΈΠ΅ кондСнсаторы Ρ‚ΠΈΠΏΠ° Y (ΠΎΠ½ΠΈ, кстати говоря, ΡΠΏΠ΅Ρ†ΠΈΠ°Π»ΡŒΠ½ΠΎ Ρ‚Π°ΠΊ спроСктированы, Ρ‡Ρ‚ΠΎΠ±Ρ‹ ΡƒΡ…ΠΎΠ΄ΠΈΡ‚ΡŒ Π² ΠΎΡ‚Ρ€Ρ‹Π², Π° Π½Π΅ Π² ΠšΠ—).

ΠšΠΎΠ½Π΄Π΅Π½ΡΠ°Ρ‚ΠΎΡ€ с Π²Π½ΡƒΡ‚Ρ€Π΅Π½Π½ΠΈΠΌ ΠΎΠ±Ρ€Ρ‹Π²ΠΎΠΌ внСшнС Π½ΠΈΡ‡Π΅ΠΌ Π½Π΅ отличаСтся ΠΎΡ‚ исправного, ΠΊΡ€ΠΎΠΌΠ΅ случаСв, ΠΊΠΎΠ³Π΄Π° Π½ΠΎΠΆΠΊΡƒ физичСски ΠΎΡ‚ΠΎΡ€Π²Π°Π»ΠΈ ΠΎΡ‚ корпуса πŸ™‚

РазумССтся, Π² случаС ΠΎΡ‚Ρ€Ρ‹Π²Π° ΠΎΠ΄Π½ΠΎΠ³ΠΎ ΠΈΠ· Π²Ρ‹Π²ΠΎΠ΄ΠΎΠ² ΠΎΡ‚ ΠΎΠ±ΠΊΠ»Π°Π΄ΠΊΠΈ кондСнсатора, Π΅ΠΌΠΊΠΎΡΡ‚ΡŒ Ρ‚Π°ΠΊΠΎΠ³ΠΎ кондСнсатора становится Ρ€Π°Π²Π½ΠΎΠΉ Π½ΡƒΠ»ΡŽ. ΠŸΠΎΡΡ‚ΠΎΠΌΡƒ ΡΡƒΡ‚ΡŒ ΠΏΡ€ΠΎΠ²Π΅Ρ€ΠΊΠΈ Π½Π° ΠΎΠ±Ρ€Ρ‹Π² состоит Π² Ρ‚ΠΎΠΌ, Ρ‡Ρ‚ΠΎΠ±Ρ‹ ΡƒΠ»ΠΎΠ²ΠΈΡ‚ΡŒ Ρ…ΠΎΡ‚ΡŒ малСйшиС ΠΏΡ€ΠΈΠ·Π½Π°ΠΊΠΈ наличия Смкости Ρƒ провСряСмого кондСнсатора.

Как это ΡΠ΄Π΅Π»Π°Ρ‚ΡŒ? Π•ΡΡ‚ΡŒ Ρ‚Ρ€ΠΈ способа.

Бпособ β„–1: ΠΈΡΠΊΠ»ΡŽΡ‡Π΅Π½ΠΈΠ΅ ΠΎΠ±Ρ€Ρ‹Π²Π° Ρ‡Π΅Ρ€Π΅Π· Π·Π²ΡƒΠΊΠΎΠ²ΠΎΠΉ сигнал Π² Ρ€Π΅ΠΆΠΈΠΌΠ΅ ΠΏΡ€ΠΎΠ·Π²ΠΎΠ½ΠΊΠΈ

Π’ΠΊΠ»ΡŽΡ‡ΠΈΡ‚ΡŒ ΠΌΡƒΠ»ΡŒΡ‚ΠΈΠΌΠ΅Ρ‚Ρ€ Π² Ρ€Π΅ΠΆΠΈΠΌ ΠΏΡ€ΠΎΠ·Π²ΠΎΠ½ΠΊΠΈ, ΠΏΡ€ΠΈΠΊΠΎΡΠ½ΡƒΡ‚ΡŒΡΡ Ρ‰ΡƒΠΏΠ°ΠΌΠΈ ΠΊ Π²Ρ‹Π²ΠΎΠ΄Π°ΠΌ кондСнсатора ΠΈ Π² этот ΠΌΠΎΠΌΠ΅Π½Ρ‚ ΠΌΡƒΠ»ΡŒΡ‚ΠΈΠΌΠ΅Ρ‚Ρ€ Π΄ΠΎΠ»ΠΆΠ΅Π½ ΠΈΠ·Π΄Π°Ρ‚ΡŒ Π½Π΅ΠΏΡ€ΠΎΠ΄ΠΎΠ»ΠΆΠΈΡ‚Π΅Π»ΡŒΠ½Ρ‹ΠΉ писк. Иногда Π·Π²ΡƒΠΊ Π½Π°ΡΡ‚ΠΎΠ»ΡŒΠΊΠΎ ΠΊΠΎΡ€ΠΎΡ‚ΠΊΠΈΠΉ (зависит ΠΎΡ‚ Смкости кондСнсатора), Ρ‡Ρ‚ΠΎ большС ΠΏΠΎΡ…ΠΎΠΆ Π½Π° Ρ‰Π΅Π»Ρ‡ΠΎΠΊ ΠΈ Π½ΡƒΠΆΠ½ΠΎ ΠΎΡ‡Π΅Π½ΡŒ ΠΏΠΎΡΡ‚Π°Ρ€Π°Ρ‚ΡŒΡΡ, Ρ‡Ρ‚ΠΎΠ±Ρ‹ Π΅Π³ΠΎ ΡƒΡΠ»Ρ‹ΡˆΠ°Ρ‚ΡŒ.

НСбольшой Π»Π°ΠΉΡ„Ρ…Π°ΠΊ: Ρ‡Ρ‚ΠΎΠ±Ρ‹ ΡƒΠ²Π΅Π»ΠΈΡ‡ΠΈΡ‚ΡŒ ΠΏΡ€ΠΎΠ΄ΠΎΠ»ΠΆΠΈΡ‚Π΅Π»ΡŒΠ½ΠΎΡΡ‚ΡŒ Π·Π²ΡƒΠΊΠΎΠ²ΠΎΠ³ΠΎ сигнала ΠΏΡ€ΠΈ ΠΏΡ€ΠΎΠ·Π²ΠΎΠ½ΠΊΠ΅ совсСм ΠΌΠ°Π»Π΅Π½ΡŒΠΊΠΈΡ… кондСнсаторов, Π½ΡƒΠΆΠ½ΠΎ ΠΏΡ€Π΅Π΄Π²Π°Ρ€ΠΈΡ‚Π΅Π»ΡŒΠ½ΠΎ Π·Π°Ρ€ΡΠ΄ΠΈΡ‚ΡŒ ΠΈΡ… ΠΎΡ‚Ρ€ΠΈΡ†Π°Ρ‚Π΅Π»ΡŒΠ½Ρ‹ΠΌ напряТСниСм, ΠΏΡ€ΠΈΠ»ΠΎΠΆΠΈΠ² Ρ‰ΡƒΠΏΡ‹ ΠΌΡƒΠ»ΡŒΡ‚ΠΈΠΌΠ΅Ρ‚Ρ€Π° Π² ΠΎΠ±Ρ€Π°Ρ‚Π½ΠΎΠΌ порядкС. Π’ΠΎΠ³Π΄Π° ΠΏΡ€ΠΈ ΠΏΠΎΡΠ»Π΅Π΄ΡƒΡŽΡ‰Π΅ΠΉ ΠΏΡ€ΠΎΠ·Π²ΠΎΠ½ΠΊΠ΅ ΠΌΡƒΠ»ΡŒΡ‚ΠΈΠΌΠ΅Ρ‚Ρ€Ρƒ сначала придСтся ΠΏΠ΅Ρ€Π΅Π·Π°Ρ€ΡΠ΄ΠΈΡ‚ΡŒ кондСнсатор ΠΎΡ‚ ΠΊΠ°ΠΊΠΎΠ³ΠΎ-Ρ‚ΠΎ ΠΎΡ‚Ρ€ΠΈΡ†Π°Ρ‚Π΅Π»ΡŒΠ½ΠΎΠ³ΠΎ напряТСния Π΄ΠΎ нуля, ΠΈ Ρ‚ΠΎΠ»ΡŒΠΊΠΎ ΠΏΠΎΡ‚ΠΎΠΌ — ΠΎΡ‚ нуля Π΄ΠΎ ΠΌΠΎΠΌΠ΅Π½Ρ‚Π° ΠΎΡ‚ΠΊΠ»ΡŽΡ‡Π΅Π½ΠΈΡ ΠΏΠΈΡ‰Π°Π»ΠΊΠΈ. На всС это ΡƒΠΉΠ΄Π΅Ρ‚ Π·Π½Π°Ρ‡ΠΈΡ‚Π΅Π»ΡŒΠ½ΠΎ большС Π²Ρ€Π΅ΠΌΠ΅Π½ΠΈ, Π° Π·Π½Π°Ρ‡ΠΈΡ‚ сигнал Π±ΡƒΠ΄Π΅Ρ‚ Π·Π²ΡƒΡ‡Π°Ρ‚ΡŒ дольшС ΠΈ Π΅Π³ΠΎ ΠΏΡ€ΠΎΡ‰Π΅ Π±ΡƒΠ΄Π΅Ρ‚ Ρ€Π°ΡΡΠ»Ρ‹ΡˆΠ°Ρ‚ΡŒ.

Π’ΠΎΡ‚ ΠΊΠ°ΠΊΠΎΠΉ-Ρ‚ΠΎ Ρ‡ΡƒΠ²Π°ΠΊ, сам Ρ‚ΠΎΠ³ΠΎ Π½Π΅ подозрСвая, примСняСт этот Π»Π°ΠΉΡ„Ρ…Π°ΠΊ Π½Π° Π²ΠΈΠ΄Π΅ΠΎ:

Из своСй ΠΏΡ€Π°ΠΊΡ‚ΠΈΠΊΠ΅ ΠΌΠΎΠ³Ρƒ ΡΠΊΠ°Π·Π°Ρ‚ΡŒ, Ρ‡Ρ‚ΠΎ с ΠΏΠΎΠΌΠΎΡ‰ΡŒΡŽ ΡƒΠ»ΠΎΠ²ΠΊΠΈ, описанной Π²Ρ‹ΡˆΠ΅, ΠΌΠ½Π΅ ΡƒΠ΄Π°Π²Π°Π»ΠΎΡΡŒ ΡƒΠ»ΠΎΠ²ΠΈΡ‚ΡŒ Ρ€Π΅Π°ΠΊΡ†ΠΈΡŽ ΠΌΡƒΠ»ΡŒΡ‚ΠΈΠΌΠ΅Ρ‚Ρ€Π° Π½Π° кондСнсатор Π΅ΠΌΠΊΠΎΡΡ‚ΡŒΡŽ всСго лишь 0.1 ΠΌΠΊΠ€ (ΠΈΠ»ΠΈ 100 Π½Π€)!

Бпособ β„–2: ΡƒΠ²Π΅Π»ΠΈΡ‡Π΅Π½ΠΈΠ΅ сопротивлСния постоянному Ρ‚ΠΎΠΊΡƒ ΠΊΠ°ΠΊ ΠΏΡ€ΠΈΠ·Π½Π°ΠΊ отсутствия ΠΎΠ±Ρ€Ρ‹Π²Π°

Если ΠΏΡ€Π΅Π΄Ρ‹Π΄ΡƒΡ‰ΠΈΠΉ способ Π½Π΅ ΠΏΠΎΠΌΠΎΠ³ ΠΈ Π²ΠΎΠΎΠ±Ρ‰Π΅ Π½Π΅ понятно, ΠΊΠ°ΠΊ ΠΏΡ€ΠΎΠ²Π΅Ρ€ΠΈΡ‚ΡŒ кондСнсатор тСстСром, Ρ‚ΠΎ Π²ΠΎΡ‚ Π²Π°ΠΌ Π±ΠΎΠ»Π΅Π΅ Ρ‡ΡƒΠ²ΡΡ‚Π²ΠΈΡ‚Π΅Π»ΡŒΠ½Ρ‹ΠΉ ΠΌΠ΅Ρ‚ΠΎΠ΄ ΠΏΡ€ΠΎΠ²Π΅Ρ€ΠΊΠΈ.

НСобходимо ΠΏΠ΅Ρ€Π΅ΠΊΠ»ΡŽΡ‡ΠΈΡ‚ΡŒ ΠΌΡƒΠ»ΡŒΡ‚ΠΈΠΌΠ΅Ρ‚Ρ€ Π² Ρ€Π΅ΠΆΠΈΠΌ измСрСния сопротивлСния. Π’Ρ‹Π±Ρ€Π°Ρ‚ΡŒ максимально доступный ΠΏΡ€Π΅Π΄Π΅Π» измСрСния (20 ΠΈΠ»ΠΈ Π»ΡƒΡ‡ΡˆΠ΅ 200 МОм). ΠŸΡ€ΠΈΠ»ΠΎΠΆΠΈΡ‚ΡŒ Ρ‰ΡƒΠΏΡ‹ ΠΊ Π²Ρ‹Π²ΠΎΠ΄Π°ΠΌ кондСнсатора ΠΈ Π½Π°Π±Π»ΡŽΠ΄Π°Ρ‚ΡŒ Π·Π° показаниями ΠΌΡƒΠ»ΡŒΡ‚ΠΈΠΌΠ΅Ρ‚Ρ€Π°.

По ΠΌΠ΅Ρ€Π΅ заряда кондСнсатора ΠΎΡ‚ Π²Π½ΡƒΡ‚Ρ€Π΅Π½Π½Π΅Π³ΠΎ источника ΠΌΡƒΠ»ΡŒΡ‚ΠΈΠΌΠ΅Ρ‚Ρ€Π°, Π΅Π³ΠΎ сопротивлСниС Π±ΡƒΠ΄Π΅Ρ‚ постоянно расти Π΄ΠΎ Ρ‚Π΅Ρ… ΠΏΠΎΡ€, ΠΏΠΎΠΊΠ° Π½Π΅ Π²Ρ‹ΠΉΠ΄Π΅Ρ‚ Π·Π° ΠΏΡ€Π΅Π΄Π΅Π»Ρ‹ Π΄ΠΈΠ°ΠΏΠ°Π·ΠΎΠ½Π° измСрСния. Если Ρ‚Π°ΠΊΠΎΠΉ эффСкт Π½Π°Π±Π»ΡŽΠ΄Π°Π΅Ρ‚ΡΡ, Π·Π½Π°Ρ‡ΠΈΡ‚ ΠΎΠ±Ρ€Ρ‹Π²Π° Π½Π΅Ρ‚.

ΠšΡΡ‚Π°Ρ‚ΠΈ говоря, ΠΌΠΎΠΆΠ΅Ρ‚ Ρ‚Π°ΠΊ ΠΎΠΊΠ°Π·Π°Ρ‚ΡŒΡΡ, Ρ‡Ρ‚ΠΎ рост сопротивлСния остановится Π½Π° Π·Π½Π°Ρ‡Π΅Π½ΠΈΠΈ ΠΎΡ‚ Π΅Π΄ΠΈΠ½ΠΈΡ† Π΄ΠΎ ΠΏΠ°Ρ€Ρ‹ дСсятков МОм — для кондСнсаторов с ΠΆΠΈΠ΄ΠΊΠΈΠΌ элСктролитом (ΠΊΡ€ΠΎΠΌΠ΅ Ρ‚Π°Π½Ρ‚Π°Π»ΠΎΠ²Ρ‹Ρ…) это Π°Π±ΡΠΎΠ»ΡŽΡ‚Π½ΠΎ Π½ΠΎΡ€ΠΌΠ°Π»ΡŒΠ½ΠΎ. Для ΠΎΡΡ‚Π°Π»ΡŒΠ½Ρ‹Ρ… кондСнсаторов сопротивлСниС ΡƒΡ‚Π΅Ρ‡ΠΊΠΈ Π΄ΠΎΠ»ΠΆΠ½ΠΎ Π±Ρ‹Ρ‚ΡŒ большС, ΠΊΠ°ΠΊ ΠΌΠΈΠ½ΠΈΠΌΡƒΠΌ, Π½Π° порядок.

ΠŸΡ€ΠΈ ΠΈΠ·ΠΌΠ΅Ρ€Π΅Π½ΠΈΠΈ Ρ‚Π°ΠΊΠΈΡ… высоких сопротивлСний Π½Π΅ΠΎΠ±Ρ…ΠΎΠ΄ΠΈΠΌΠΎ ΡΠ»Π΅Π΄ΠΈΡ‚ΡŒ Π·Π° Ρ‚Π΅ΠΌ, Ρ‡Ρ‚ΠΎΠ±Ρ‹ Π½Π΅ ΠΊΠ°ΡΠ°Ρ‚ΡŒΡΡ ΠΏΠ°Π»ΡŒΡ†Π°ΠΌΠΈ сразу ΠΎΠ±ΠΎΠΈΡ… ΠΈΠ·ΠΌΠ΅Ρ€ΠΈΡ‚Π΅Π»ΡŒΠ½Ρ‹Ρ… Ρ‰ΡƒΠΏΠΎΠ². Π˜Π½Π°Ρ‡Π΅ сопротивлСниС ΠΊΠΎΠΆΠΈ внСсСт свои ΠΊΠΎΡ€Ρ€Π΅ΠΊΡ‚ΠΈΠ²Ρ‹ ΠΈ исказит всС Ρ€Π΅Π·ΡƒΠ»ΡŒΡ‚Π°Ρ‚Ρ‹.

Π‘ ΠΏΠΎΠΌΠΎΡ‰ΡŒΡŽ измСрСния сопротивлСния Π½Π° ΠΏΡ€Π΅Π΄Π΅Π»Π΅ 200 МОм ΠΌΠ½Π΅ ΡƒΠ΄Π°Π²Π°Π»ΠΎΡΡŒ ΠΎΠ΄Π½ΠΎΠ·Π½Π°Ρ‡Π½ΠΎ ΠΎΠΏΡ€Π΅Π΄Π΅Π»ΠΈΡ‚ΡŒ отсутствиС ΠΎΠ±Ρ€Ρ‹Π²Π° Π² кондСнсаторах Π΅ΠΌΠΊΠΎΡΡ‚ΡŒΡŽ всСго 0.001 ΠΌΠΊΠ€ (ΠΈΠ»ΠΈ 1000 ΠΏΠ€).

Π’ΠΎΡ‚ Π²ΠΈΠ΄Π΅ΠΎ для наглядности:

Бпособ β„–3: ΠΈΠ·ΠΌΠ΅Ρ€Π΅Π½ΠΈΠ΅ остаточного напряТСния для ΠΈΡΠΊΠ»ΡŽΡ‡Π΅Π½ΠΈΡ Π²Π½ΡƒΡ‚Ρ€Π΅Π½Π½Π΅Π³ΠΎ ΠΎΠ±Ρ€Ρ‹Π²Π°

Π­Ρ‚ΠΎ самый Ρ‡ΡƒΠ²ΡΡ‚Π²ΠΈΡ‚Π΅Π»ΡŒΠ½Ρ‹ΠΉ способ, ΠΏΠΎΠ·Π²ΠΎΠ»ΡΡŽΡ‰ΠΈΠΉ ΡƒΠ±Π΅Π΄ΠΈΡ‚ΡŒΡΡ Π² отсутствии ΠΎΠ±Ρ€Ρ‹Π²Π° кондСнсатора Π΄Π°ΠΆΠ΅ Ρ‚ΠΎΠ³Π΄Π°, ΠΊΠΎΠ³Π΄Π° всС ΠΏΡ€Π΅Π΄Ρ‹Π΄ΡƒΡ‰ΠΈΠ΅ способы Π½Π΅ ΠΏΠΎΠΌΠΎΠ³Π»ΠΈ.

БСрСтся ΠΌΡƒΠ»ΡŒΡ‚ΠΈΠΌΠ΅Ρ‚Ρ€ Π² Ρ€Π΅ΠΆΠΈΠΌΠ΅ ΠΏΡ€ΠΎΠ·Π²ΠΎΠ½ΠΊΠΈ ΠΈΠ»ΠΈ Π² Ρ€Π΅ΠΆΠΈΠΌΠ΅ измСрСния сопротивлСния (Π½Π΅ Π²Π°ΠΆΠ½ΠΎ Π² ΠΊΠ°ΠΊΠΎΠΌ Π΄ΠΈΠ°ΠΏΠ°Π·ΠΎΠ½Π΅) ΠΈ Π½Π° ΠΏΠ°Ρ€Ρƒ сСкунд ΠΏΡ€ΠΈΠΊΠ»Π°Π΄Ρ‹Π²Π°Π΅ΠΌ Ρ‰ΡƒΠΏΡ‹ ΠΊ Π²Ρ‹Π²ΠΎΠ΄Π°ΠΌ испытуСмого кондСнсатора. Π’ этот ΠΌΠΎΠΌΠ΅Π½Ρ‚ кондСнсатор зарядится ΠΎΡ‚ ΠΌΡƒΠ»ΡŒΡ‚ΠΈΠΌΠ΅Ρ‚Ρ€Π° Π΄ΠΎ ΠΊΠ°ΠΊΠΎΠ³ΠΎ-Ρ‚ΠΎ нСбольшого напряТСния (ΠΎΠ±Ρ‹Ρ‡Π½ΠΎ 2.8 Π’).

Π—Π°Ρ‚Π΅ΠΌ ΠΌΡ‹ быстро ΠΏΠ΅Ρ€Π΅ΠΊΠ»ΡŽΡ‡Π°Π΅ΠΌ ΠΌΡƒΠ»ΡŒΡ‚ΠΈΠΌΠ΅Ρ‚Ρ€ Π² Ρ€Π΅ΠΆΠΈΠΌ измСрСния постоянного напряТСния Π½Π° самом Ρ‡ΡƒΠ²ΡΡ‚Π²ΠΈΡ‚Π΅Π»ΡŒΠ½ΠΎΠΌ Π΄ΠΈΠ°ΠΏΠ°Π·ΠΎΠ½Π΅ ΠΈ, Π½Π΅ мСшкая слишком Π΄ΠΎΠ»Π³ΠΎ, снова ΠΏΡ€ΠΈΠΊΠ»Π°Π΄Ρ‹Π²Π°Π΅ΠΌ Ρ‰ΡƒΠΏΡ‹ ΠΊ кондСнсатору, Ρ‡Ρ‚ΠΎΠ±Ρ‹ ΠΈΠ·ΠΌΠ΅Ρ€ΠΈΡ‚ΡŒ Π½Π° Π½Π΅ΠΌ напряТСниС. Если Ρƒ ΠΊΠΎΠ½Π΄Π΅Ρ€Π° Π΅ΡΡ‚ΡŒ Ρ…ΠΎΡ‚ΡŒ какая-Π½ΠΈΠ±ΡƒΠ΄ΡŒ Π²Ρ€Π°Π·ΡƒΠΌΠΈΡ‚Π΅Π»ΡŒΠ½Π°Ρ Π΅ΠΌΠΊΠΎΡΡ‚ΡŒ, Ρ‚ΠΎ ΠΌΡƒΠ»ΡŒΡ‚ΠΈΠΌΠ΅Ρ‚Ρ€ успССт ΠΏΠΎΠΊΠ°Π·Π°Ρ‚ΡŒ напряТСниС, Π΄ΠΎ ΠΊΠΎΡ‚ΠΎΡ€ΠΎΠ³ΠΎ Π±Ρ‹Π» заряТСн кондСнсатор.

Π­Ρ‚ΠΈΠΌ способом ΠΌΠ½Π΅ ΡƒΠ΄Π°Π²Π°Π»ΠΎΡΡŒ с ΠΏΠΎΠΌΠΎΡ‰ΡŒΡŽ ΠΎΠ±Ρ‹Ρ‡Π½ΠΎΠ³ΠΎ Ρ†ΠΈΡ„Ρ€ΠΎΠ²ΠΎΠ³ΠΎ ΠΌΡƒΠ»ΡŒΡ‚ΠΈΠΌΠ΅Ρ‚Ρ€Π° M890D ΠΎΡ‚Π»ΠΎΠ²ΠΈΡ‚ΡŒ Π΅ΠΌΠΊΠΎΡΡ‚ΡŒ Π²ΠΏΠ»ΠΎΡ‚ΡŒ Π΄ΠΎ 470 ΠΏΠ€ (0.00047 ΠΌΠΊΠ€)! А это ΠΎΡ‡Π΅Π½ΡŒ малСнькая Π΅ΠΌΠΊΠΎΡΡ‚ΡŒ.

Π’ΠΎΠΎΠ±Ρ‰Π΅ говоря, это Π½Π°ΠΈΠ±ΠΎΠ»Π΅Π΅ эффСктивный ΠΌΠ΅Ρ‚ΠΎΠ΄ ΠΏΡ€ΠΎΠ·Π²ΠΎΠ½ΠΊΠΈ кондСнсаторов. Π’Π°ΠΊΠΈΠΌ способ ΠΌΠΎΠΆΠ½ΠΎ ΠΏΡ€ΠΎΠ²Π΅Ρ€ΡΡ‚ΡŒ ΠΊΠΎΠ½Π΄Π΅Ρ€Ρ‹ любой Смкости — ΠΎΡ‚ ΠΌΠ°Π»ΡŽΡΠ΅Π½ΡŒΠΊΠΈΡ… Π΄ΠΎ самых Π±ΠΎΠ»ΡŒΡˆΠΈΡ…, Π° Ρ‚Π°ΠΊΠΆΠ΅ любого Ρ‚ΠΈΠΏΠ° — полярныС, нСполярныС, элСктролитичСскиС, ΠΏΠ»Π΅Π½ΠΎΡ‡Π½Ρ‹Π΅, кСрамичСскиС, оксидныС, Π²ΠΎΠ·Π΄ΡƒΡˆΠ½Ρ‹Π΅, ΠΌΠ΅Ρ‚Π°Π»Π»ΠΎ-Π±ΡƒΠΌΠ°ΠΆΠ½Ρ‹Π΅ ΠΈ Ρ‚.Π΄.

ΠŸΡ€Π°Π²Π΄Π°, Ссли кондСнсатор ΠΈΠΌΠ΅Π΅Ρ‚ совсСм ΠΌΠ°Π»Π΅Π½ΡŒΠΊΡƒΡŽ Π΅ΠΌΠΊΠΎΡΡ‚ΡŒ, Π΄ΠΎ 470 ΠΏΠ€, Ρ‚ΠΎ, ΡƒΠ²Ρ‹, ΠΏΡ€ΠΎΠ²Π΅Ρ€ΠΈΡ‚ΡŒ Π΅Π³ΠΎ Π½Π° ΠΎΠ±Ρ€Ρ‹Π² Π±Π΅Π· ΡΠΏΠ΅Ρ†ΠΈΠ°Π»ΡŒΠ½ΠΎΠ³ΠΎ ΠΏΡ€ΠΈΠ±ΠΎΡ€Π°, Π²Ρ€ΠΎΠ΄Π΅ упомянутого Ρ€Π°Π½Π΅Π΅ LC-ΠΌΠ΅Ρ‚Ρ€Π°, Π½ΠΈΠΊΠ°ΠΊ Π½Π΅ получится.

ΠžΠΏΡ€Π΅Π΄Π΅Π»Π΅Π½ΠΈΠ΅ Ρ€Π°Π±ΠΎΡ‡Π΅Π³ΠΎ напряТСния кондСнсатора

Π‘Ρ‚Ρ€ΠΎΠ³ΠΎ говоря, Ссли Π½Π° кондСнсаторС Π½Π΅Ρ‚ ΠΌΠ°Ρ€ΠΊΠΈΡ€ΠΎΠ²ΠΊΠΈ ΠΈ Π½Π΅ извСстна схСма, Π² ΠΊΠΎΡ‚ΠΎΡ€ΠΎΠΉ ΠΎΠ½ стоял, Ρ‚ΠΎ ΡƒΠ·Π½Π°Ρ‚ΡŒ Π΅Π³ΠΎ Ρ€Π°Π±ΠΎΡ‡Π΅Π΅ напряТСниС Π½Π΅Ρ€Π°Π·Ρ€ΡƒΡˆΠ°ΡŽΡ‰ΠΈΠΌΠΈ ΠΌΠ΅Ρ‚ΠΎΠ΄Π°ΠΌΠΈ ΠΠ•Π’ΠžΠ—ΠœΠžΠ–ΠΠž.

Однако, имСя Π½Π΅ΠΊΠΎΡ‚ΠΎΡ€Ρ‹ΠΉ ΠΎΠΏΡ‹Ρ‚, ΠΌΠΎΠΆΠ½ΠΎ ΠΎΠΎΠΎΠΎΡ‡Π΅Π½ΡŒ ΠΏΡ€ΠΈΠ±Π»ΠΈΠ·ΠΈΡ‚Π΅Π»ΡŒΠ½ΠΎ ΠΏΡ€ΠΈΠΊΠΈΠ½ΡƒΡ‚ΡŒ «Π½Π° Π³Π»Π°Π·ΠΎΠΊ» Ρ€Π°Π±ΠΎΡ‡Π΅Π΅ напряТСниС исходя ΠΈΠ· Π³Π°Π±Π°Ρ€ΠΈΡ‚ΠΎΠ² кондСнсатора. ЕстСствСнно, Ρ‡Π΅ΠΌ большС Ρ€Π°Π·ΠΌΠ΅Ρ€Ρ‹ кондСнсатора ΠΈ Ρ‡Π΅ΠΌ мСньшС ΠΏΡ€ΠΈ этом Π΅Π³ΠΎ Π΅ΠΌΠΊΠΎΡΡ‚ΡŒ, Ρ‚Π΅ΠΌ Π½Π° большСС напряТСниС ΠΎΠ½ расчитан.

Бпособ β„–1: ΠΎΠΏΡ€Π΅Π΄Π΅Π»Π΅Π½ΠΈΠ΅ Ρ€Π°Π±ΠΎΡ‡Π΅Π³ΠΎ напряТСния Ρ‡Π΅Ρ€Π΅Π· напряТСния пробоя

Если имССтся нСсколько ΠΎΠ΄ΠΈΠ½Π°ΠΊΠΎΠ²Ρ‹Ρ… кондСнсаторов ΠΈ ΠΎΠ΄Π½ΠΈΠΌ ΠΈΠ· Π½ΠΈΡ… Π½Π΅ ΠΆΠ°Π»ΠΊΠΎ ΠΏΠΎΠΆΠ΅Ρ€Ρ‚Π²ΠΎΠ²Π°Ρ‚ΡŒ, Ρ‚ΠΎ ΠΌΠΎΠΆΠ½ΠΎ ΠΎΠΏΡ€Π΅Π΄Π΅Π»ΠΈΡ‚ΡŒ напряТСниС пробоя, ΠΊΠΎΡ‚ΠΎΡ€ΠΎΠ΅ ΠΎΠ±Ρ‹Ρ‡Π½ΠΎ Ρ€Π°Π·Π° Π² 2-3 Π²Ρ‹ΡˆΠ΅ Ρ€Π°Π±ΠΎΡ‡Π΅Π³ΠΎ напряТСния.

НапряТСниС пробоя кондСнсатора измСряСтся ΡΠ»Π΅Π΄ΡƒΡŽΡ‰ΠΈΠΌ ΠΎΠ±Ρ€Π°Π·ΠΎΠΌ. ΠšΠΎΠ½Π΄Π΅Π½ΡΠ°Ρ‚ΠΎΡ€ ΠΏΠΎΠ΄ΠΊΠ»ΡŽΡ‡Π°Π΅Ρ‚ΡΡ Ρ‡Π΅Ρ€Π΅Π· Ρ‚ΠΎΠΊΠΎΠΎΠ³Ρ€Π°Π½ΠΈΡ‡ΠΈΡ‚Π΅Π»ΡŒΠ½Ρ‹ΠΉ рСзистор ΠΊ Ρ€Π΅Π³ΡƒΠ»ΠΈΡ€ΡƒΠ΅ΠΌΠΎΠΌΡƒ источнику напряТСния, способного Π²Ρ‹Π΄Π°Π²Π°Ρ‚ΡŒ Π·Π°Π²Π΅Π΄ΠΎΠΌΠΎ большС, Ρ‡Π΅ΠΌ напряТСниС пробоя. НапряТСниС Π½Π° кондСнсаторС контроллируСтся Π²ΠΎΠ»ΡŒΡ‚ΠΌΠ΅Ρ‚Ρ€ΠΎΠΌ.

Π—Π°Ρ‚Π΅ΠΌ напряТСниС ΠΏΠ»Π°Π²Π½ΠΎ ΠΏΠΎΠ²Ρ‹ΡˆΠ°ΡŽΡ‚ Π΄ΠΎ Ρ‚Π΅Ρ… ΠΏΠΎΡ€, ΠΏΠΎΠΊΠ° Π½Π΅ ΠΏΡ€ΠΎΠΈΠ·ΠΎΠΉΠ΄Π΅Ρ‚ ΠΏΡ€ΠΎΠ±ΠΎΠΉ (ΠΌΠΎΠΌΠ΅Π½Ρ‚, ΠΊΠΎΠ³Π΄Π° напряТСниС Π½Π° кондСнсаторС Ρ€Π΅Π·ΠΊΠΎ ΡƒΠΏΠ°Π΄Π΅Ρ‚ Π΄ΠΎ нуля).

Π—Π° Ρ€Π°Π±ΠΎΡ‡Π΅Π΅ напряТСниС ΠΌΠΎΠΆΠ½ΠΎ ΠΏΡ€ΠΈΠ½ΡΡ‚ΡŒ Π·Π½Π°Ρ‡Π΅Π½ΠΈΠ΅, Π² 2-3 Ρ€Π°Π·Π° мСньшС, Ρ‡Π΅ΠΌ напряТСниС пробоя. Но это Ρ‚Π°ΠΊΠΎΠ΅… Π’Ρ‹ ΠΌΠΎΠΆΠ΅Ρ‚Π΅ ΠΈΠΌΠ΅Ρ‚ΡŒ своС ΠΌΠ½Π΅Π½ΠΈΠ΅ Π½Π° этот счСт.

Π’Π½ΠΈΠΌΠ°Π½ΠΈΠ΅! ΠžΠ±ΡΠ·Π°Ρ‚Π΅Π»ΡŒΠ½ΠΎ ΡΠΎΠ±Π»ΡŽΠ΄Π°ΠΉΡ‚Π΅ всС ΠΌΠ΅Ρ€Ρ‹ прСдостороТности! ΠŸΡ€ΠΈ ΠΏΡ€ΠΎΠ²Π΅Ρ€ΠΊΠ΅ кондСнсатора Π½Π° ΠΏΡ€ΠΎΠ±ΠΎΠΉ Π½Π΅ΠΎΠ±Ρ…ΠΎΠ΄ΠΈΠΌΠΎ ΠΈΡΠΏΠΎΠ»ΡŒΠ·ΠΎΠ²Π°Ρ‚ΡŒ Π·Π°Ρ‰ΠΈΡ‰Π΅Π½Π½Ρ‹ΠΉ стСнд, Π° Ρ‚Π°ΠΊΠΆΠ΅ ΠΈΠ½Π΄ΠΈΠ²ΠΈΠ΄ΡƒΠ°Π»ΡŒΠ½Ρ‹Π΅ срСдства Π·Π°Ρ‰ΠΈΡ‚Ρ‹ зрСния.

Π­Π½Π΅Ρ€Π³ΠΈΠΈ заряТСнного кондСнсатора Π±Ρ‹Π²Π°Π΅Ρ‚ достаточно, Ρ‡Ρ‚ΠΎΠ±Ρ‹ ΡƒΡΡ‚Ρ€ΠΎΠΈΡ‚ΡŒ нСбольшой ядСрный Π²Π·Ρ€Ρ‹Π² прямо Π½Π° Ρ€Π°Π±ΠΎΡ‡Π΅ΠΌ столС. Π’ΠΎΡ‚, ΠΌΠΎΠΆΠ½ΠΎ ΠΏΠΎΡΠΌΠΎΡ‚Ρ€Π΅Ρ‚ΡŒ, ΠΊΠ°ΠΊ это Π±Ρ‹Π²Π°Π΅Ρ‚:

А Π½Π΅ΠΊΠΎΡ‚ΠΎΡ€Ρ‹Π΅ Ρ‚ΠΈΠΏΡ‹ кСрамичСских кондСнсаторов ΠΏΡ€ΠΈ элСктричСском ΠΏΡ€ΠΎΠ±ΠΎΠ΅ способны Ρ€Π°Π·Π»Π΅Ρ‚Π°Ρ‚ΡŒΡΡ Π½Π° ΠΎΡ‡Π΅Π½ΡŒ ΠΌΠ΅Π»ΠΊΠΈΠ΅, Π½ΠΎ Ρ‚Π²Π΅Ρ€Π΄Ρ‹Π΅ осколки, Π±Π΅Π· Ρ‚Ρ€ΡƒΠ΄Π° ΠΏΡ€ΠΎΠ±ΠΈΠ²Π°ΡŽΡ‰ΠΈΠ΅ ΠΊΠΎΠΆΡƒ (Π½Π΅ говоря ΡƒΠΆΠ΅ ΠΎ Π³Π»Π°Π·Π°Ρ…).

Бпособ β„–2: Π½Π°Ρ…ΠΎΠΆΠ΄Π΅Π½ΠΈΠ΅ Ρ€Π°Π±ΠΎΡ‡Π΅Π³ΠΎ напряТСния кондСнсатора Ρ‡Π΅Ρ€Π΅Π· Ρ‚ΠΎΠΊ ΡƒΡ‚Π΅Ρ‡ΠΊΠΈ

Π­Ρ‚ΠΎΡ‚ способ ΡƒΠ·Π½Π°Ρ‚ΡŒ Ρ€Π°Π±ΠΎΡ‡Π΅Π΅ напряТСниС кондСнсатора ΠΏΠΎΠ΄Ρ…ΠΎΠ΄ΠΈΡ‚ для Π°Π»ΡŽΠΌΠΈΠ½ΠΈΠ΅Π²Ρ‹Ρ… элСктролитичСских кондСнсаторов (полярных ΠΈ нСполярных). А Ρ‚Π°ΠΊΠΈΡ… кондСнсаторов Π±ΠΎΠ»ΡŒΡˆΠΈΠ½ΡΡ‚Π²ΠΎ.

Π‘ΡƒΡ‚ΡŒ Π·Π°ΠΊΠ»ΡŽΡ‡Π°Π΅Ρ‚ΡΡ Π² Ρ‚ΠΎΠΌ, Ρ‡Ρ‚ΠΎΠ±Ρ‹ ΠΎΡ‚Π»ΠΎΠ²ΠΈΡ‚ΡŒ ΠΌΠΎΠΌΠ΅Π½Ρ‚, ΠΏΡ€ΠΈ ΠΊΠΎΡ‚ΠΎΡ€ΠΎΠΌ Π΅Π³ΠΎ Ρ‚ΠΎΠΊ ΡƒΡ‚Π΅Ρ‡ΠΊΠΈ Π½Π°Ρ‡ΠΈΠ½Π°Π΅Ρ‚ Π½Π΅Π»ΠΈΠ½Π΅ΠΉΠ½ΠΎ Π²ΠΎΠ·Ρ€Π°ΡΡ‚Π°Ρ‚ΡŒ. Для этого собираСм ΠΏΡ€ΠΎΡΡ‚Π΅ΠΉΡˆΡƒΡŽ схСму:

ΠΈ Π΄Π΅Π»Π°Π΅ΠΌ Π·Π°ΠΌΠ΅Ρ€Ρ‹ Ρ‚ΠΎΠΊΠ° ΡƒΡ‚Π΅Ρ‡ΠΊΠΈ ΠΏΡ€ΠΈ Ρ€Π°Π·Π»ΠΈΡ‡Π½Ρ‹Ρ… значСниях ΠΏΡ€ΠΈΠ»ΠΎΠΆΠ΅Π½Π½ΠΎΠ³ΠΎ напряТСния (начиная с 5 Π²ΠΎΠ»ΡŒΡ‚ ΠΈ Π΄Π°Π»Π΅Π΅). НапряТСниС слСдуСт ΠΏΠΎΠ²Ρ‹ΡˆΠ°Ρ‚ΡŒ постСпСнно, ΠΎΠ΄ΠΈΠ½Π°ΠΊΠΎΠ²Ρ‹ΠΌΠΈ порциями, записывая показания Π²ΠΎΠ»ΡŒΡ‚ΠΌΠ΅Ρ‚Ρ€Π° ΠΈ ΠΌΠΈΠΊΡ€ΠΎΠ°ΠΌΠΏΠ΅Ρ€Π°ΠΌΠ΅Ρ‚Ρ€Π° Π² Ρ‚Π°Π±Π»ΠΈΡ†Ρƒ.

Π£ мСня ΠΏΠΎΠ»ΡƒΡ‡ΠΈΠ»Π°ΡΡŒ такая Ρ‚Π°Π±Π»ΠΈΡ‡ΠΊΠ° (моя Ρ‡ΡƒΠΉΠΊΠ° подсказала ΠΌΠ½Π΅, Ρ‡Ρ‚ΠΎ это довольно Π²Ρ‹ΡΠΎΠΊΠΎΠ²ΠΎΠ»ΡŒΡ‚Π½Ρ‹ΠΉ кондСнсатор, Ρ‚Π°ΠΊ Ρ‡Ρ‚ΠΎ я сразу Π½Π°Ρ‡Π°Π» ΠΏΡ€ΠΈΠ±Π°Π²Π»ΡΡ‚ΡŒ ΠΏΠΎ 10Π’):

НапряТСниС Π½Π°
кондСнсаторС, Π’
Π’ΠΎΠΊ ΡƒΡ‚Π΅Ρ‡ΠΊΠΈ,
мкА
ΠŸΡ€ΠΈΡ€ΠΎΡΡ‚ Ρ‚ΠΎΠΊΠ°,
мкА
101.11.1
202.21.1
303.31.1
404.51.2
505.81.3
607. 21.4
708.91.7
8011.02.1
9013.42.4
10016.02.6

Как Ρ‚ΠΎΠ»ΡŒΠΊΠΎ станСт Π·Π°ΠΌΠ΅Ρ‚Π½ΠΎ, Ρ‡Ρ‚ΠΎ ΠΎΠ΄ΠΈΠ½Π°ΠΊΠΎΠ²Ρ‹ΠΉ прирост напряТСния ΠΊΠ°ΠΆΠ΄Ρ‹ΠΉ Ρ€Π°Π· ΠΏΡ€ΠΈΠ²ΠΎΠ΄ΠΈΡ‚ ΠΊ Π½Π΅ΠΏΡ€ΠΎΠΏΠΎΡ€Ρ†ΠΈΠΎΠ½Π°Π»ΡŒΠ½ΠΎ Π±ΠžΠ»ΡŒΡˆΠ΅ΠΌΡƒ приросту Ρ‚ΠΎΠΊΠ° ΡƒΡ‚Π΅Ρ‡ΠΊΠΈ, экспСримСнт слСдуСт ΠΎΡΡ‚Π°Π½ΠΎΠ²ΠΈΡ‚ΡŒ, Ρ‚Π°ΠΊ ΠΊΠ°ΠΊ ΠΏΠ΅Ρ€Π΅Π΄ Π½Π°ΠΌΠΈ Π½Π΅ стоит Π·Π°Π΄Π°Ρ‡Π° довСсти кондСнсатор Π΄ΠΎ элСктричСского пробоя.

Если ΠΈΠ· ΠΏΠΎΠ»ΡƒΡ‡Π΅Π½Π½Ρ‹Ρ… Π·Π½Π°Ρ‡Π΅Π½ΠΈΠΉ ΠΏΠΎΡΡ‚Ρ€ΠΎΠΈΡ‚ΡŒ Π³Ρ€Π°Ρ„ΠΈΠΊ, Ρ‚ΠΎ ΠΎΠ½ Π±ΡƒΠ΄Π΅Ρ‚ ΠΈΠΌΠ΅Ρ‚ΡŒ ΡΠ»Π΅Π΄ΡƒΡŽΡ‰ΠΈΠΉ Π²ΠΈΠ΄:

Π’ΠΈΠ΄Π½ΠΎ, Ρ‡Ρ‚ΠΎ начиная с 50-60 Π²ΠΎΠ»ΡŒΡ‚, Π³Ρ€Π°Ρ„ΠΈΠΊ зависимости Ρ‚ΠΎΠΊΠ° ΡƒΡ‚Π΅Ρ‡ΠΊΠΈ ΠΎΡ‚ напряТСния ΠΎΠ±Ρ€Π΅Ρ‚Π°Π΅Ρ‚ явно Π²Ρ‹Ρ€Π°ΠΆΠ΅Π½Π½ΡƒΡŽ Π½Π΅Π»ΠΈΠ½Π΅ΠΉΠ½ΠΎΡΡ‚ΡŒ. А Ссли ΠΏΡ€ΠΈΠ½ΡΡ‚ΡŒ Π²ΠΎ Π²Π½ΠΈΠΌΠ°Π½ΠΈΠ΅ стандартный ряд напряТСний:

Π‘Ρ‚Π°Π½Π΄Π°Ρ€Ρ‚Π½Ρ‹ΠΉ ряд Π½ΠΎΠΌΠΈΠ½Π°Π»ΡŒΠ½Ρ‹Ρ… Ρ€Π°Π±ΠΎΡ‡ΠΈΡ… напряТСний кондСнсаторов, Π’
6.3101620253240506380100125160200250315350400450500

Ρ‚ΠΎ ΠΌΠΎΠΆΠ½ΠΎ ΠΏΡ€Π΅Π΄ΠΏΠΎΠ»ΠΎΠΆΠΈΡ‚ΡŒ, Ρ‡Ρ‚ΠΎ для Π΄Π°Π½Π½ΠΎΠ³ΠΎ кондСнсатора Ρ€Π°Π±ΠΎΡ‡Π΅Π΅ напряТСниС составляСт Π»ΠΈΠ±ΠΎ 50 Π»ΠΈΠ±ΠΎ 63 Π’.

БогласСн, ΠΌΠ΅Ρ‚ΠΎΠ΄ достаточно Ρ‚Ρ€ΡƒΠ΄ΠΎΠ΅ΠΌΠΊΠΈΠΉ, Π½ΠΎ Π½Π΅ ΡΠΊΠ°Π·Π°Ρ‚ΡŒ ΠΎ Π½Π΅ΠΌ Π±Ρ‹Π»ΠΎ Π±Ρ‹ ошибкой.

Как ΠΈΠ·ΠΌΠ΅Ρ€ΠΈΡ‚ΡŒ Ρ‚ΠΎΠΊ ΡƒΡ‚Π΅Ρ‡ΠΊΠΈ кондСнсатора?

Π§ΡƒΡ‚ΡŒ Π²Ρ‹ΡˆΠ΅ ΡƒΠΆΠ΅ Π±Ρ‹Π»Π° описана ΠΌΠ΅Ρ‚ΠΎΠ΄ΠΈΠΊΠ° измСрСния Ρ‚ΠΎΠΊΠ° ΡƒΡ‚Π΅Ρ‡ΠΊΠΈ. Π₯ΠΎΡ‚Π΅Π»ΠΎΡΡŒ Π±Ρ‹ Ρ‚ΠΎΠ»ΡŒΠΊΠΎ Π΄ΠΎΠ±Π°Π²ΠΈΡ‚ΡŒ, Ρ‡Ρ‚ΠΎ IΡƒΡ‚ измСряСтся Π»ΠΈΠ±ΠΎ ΠΏΡ€ΠΈ максимальном Ρ€Π°Π±ΠΎΡ‡Π΅ΠΌ напряТСнии кондСнсатора Π»ΠΈΠ±ΠΎ ΠΏΡ€ΠΈ Ρ‚Π°ΠΊΠΎΠΌ напряТСнии, ΠΏΡ€ΠΈ ΠΊΠΎΡ‚ΠΎΡ€ΠΎΠΌ кондСнсатор планируСтся ΠΈΡΠΏΠΎΠ»ΡŒΠ·ΠΎΠ²Π°Ρ‚ΡŒ.

Π’Π°ΠΊΠΆΠ΅ ΠΌΠΎΠΆΠ½ΠΎ Π²Ρ‹Ρ‡ΠΈΡΠ»ΠΈΡ‚ΡŒ Ρ‚ΠΎΠΊ ΡƒΡ‚Π΅Ρ‡ΠΊΠΈ кондСнсатора косвСнным ΠΌΠ΅Ρ‚ΠΎΠ΄ΠΎΠΌ — Ρ‡Π΅Ρ€Π΅Π· ΠΏΠ°Π΄Π΅Π½ΠΈΠ΅ напряТСния Π½Π° Π·Π°Ρ€Π°Π½Π΅Π΅ извСстном сопротивлСнии:

ΠŸΡ€ΠΈ ΠΏΡ€ΠΎΠ²Π΅Ρ€ΠΊΠ΅ полярных кондСнсаторов Π½Π° ΡƒΡ‚Π΅Ρ‡ΠΊΡƒ Π½Π΅ΠΎΠ±Ρ…ΠΎΠ΄ΠΈΠΌΠΎ ΡΠΎΠ±Π»ΡŽΠ΄Π°Ρ‚ΡŒ ΠΏΠΎΠ»ΡΡ€Π½ΠΎΡΡ‚ΡŒ ΠΈΡ… ΠΏΠΎΠ΄ΠΊΠ»ΡŽΡ‡Π΅Π½ΠΈΡ. Π’ ΠΏΡ€ΠΎΡ‚ΠΈΠ²Π½ΠΎΠΌ случаС Π±ΡƒΠ΄ΡƒΡ‚ ΠΏΠΎΠ»ΡƒΡ‡Π΅Π½Ρ‹ Π½Π΅ΠΊΠΎΡ€Ρ€Π΅ΠΊΡ‚Π½Ρ‹Π΅ Ρ€Π΅Π·ΡƒΠ»ΡŒΡ‚Π°Ρ‚Ρ‹.

ΠŸΡ€ΠΈ ΠΈΠ·ΠΌΠ΅Ρ€Π΅Π½ΠΈΠΈ Ρ‚ΠΎΠΊΠ° ΡƒΡ‚Π΅Ρ‡ΠΊΠΈ элСктролитичСских кондСнсаторов послС ΠΏΠΎΠ΄Π°Ρ‡ΠΈ напряТСния ΠΎΡ‡Π΅Π½ΡŒ Π²Π°ΠΆΠ½ΠΎ Π²Ρ‹ΠΆΠ΄Π°Ρ‚ΡŒ ΠΊΠ°ΠΊΠΎΠ΅-Ρ‚ΠΎ врСмя (ΠΌΠΈΠ½ΡƒΡ‚ 5-10) для Ρ‚ΠΎΠ³ΠΎ, Ρ‡Ρ‚ΠΎΠ±Ρ‹ всС элСктрохимичСскиС процСссы Π·Π°Π²Π΅Ρ€ΡˆΠΈΠ»ΠΈΡΡŒ. ОсобСнно это Π°ΠΊΡ‚ΡƒΠ°Π»ΡŒΠ½ΠΎ для кондСнсаторов, ΠΊΠΎΡ‚ΠΎΡ€Ρ‹Π΅ Π² Ρ‚Π΅Ρ‡Π΅Π½ΠΈΠ΅ Π΄Π»ΠΈΡ‚Π΅Π»ΡŒΠ½ΠΎΠ³ΠΎ Π²Ρ€Π΅ΠΌΠ΅Π½ΠΈ Π±Ρ‹Π»ΠΈ Π²Ρ‹Π²Π΅Π΄Π΅Π½Ρ‹ ΠΈΠ· эксплуатации.

Π’ΠΎΡ‚ Π²ΠΈΠ΄Π΅ΠΎ с наглядной дСмонстрациСй описанного ΠΌΠ΅Ρ‚ΠΎΠ΄Π° измСрСния Ρ‚ΠΎΠΊΠ° ΡƒΡ‚Π΅Ρ‡ΠΊΠΈ кондСнсатора:

ΠžΠΏΡ€Π΅Π΄Π΅Π»Π΅Π½ΠΈΠ΅ Смкости нСизвСстного кондСнсатора

Бпособ β„–1: ΠΈΠ·ΠΌΠ΅Ρ€Π΅Π½ΠΈΠ΅ Смкости ΡΠΏΠ΅Ρ†ΠΈΠ°Π»ΡŒΠ½Ρ‹ΠΌΠΈ ΠΏΡ€ΠΈΠ±ΠΎΡ€Π°ΠΌΠΈ

Π‘Π°ΠΌΡ‹ΠΉ просто способ — ΠΈΠ·ΠΌΠ΅Ρ€ΠΈΡ‚ΡŒ Π΅ΠΌΠΊΠΎΡΡ‚ΡŒ с ΠΏΠΎΠΌΠΎΡ‰ΡŒΡŽ ΠΏΡ€ΠΈΠ±ΠΎΡ€Π°, ΠΈΠΌΠ΅ΡŽΡ‰Π΅Π³ΠΎ Ρ„ΡƒΠ½ΠΊΡ†ΠΈΡŽ измСрСния СмкостСй. Π­Ρ‚ΠΎ ΠΈ Ρ‚Π°ΠΊ понятно, ΠΈ ΠΎΠ± этом ΡƒΠΆΠ΅ Π³ΠΎΠ²ΠΎΡ€ΠΈΠ»ΡΡŒ Π² Π½Π°Ρ‡Π°Π»Π΅ ΡΡ‚Π°Ρ‚ΡŒΠΈ ΠΈ Ρ‚ΡƒΡ‚ Π½Π΅Ρ‡Π΅Π³ΠΎ большС Π΄ΠΎΠ±Π°Π²ΠΈΡ‚ΡŒ.Если с ΠΏΡ€ΠΈΠ±ΠΎΡ€Π°ΠΌΠΈ совсСм Ρ‚ΡƒΠ³Π°Π½, ΠΌΠΎΠΆΠ½ΠΎ ΠΏΠΎΠΏΡ€ΠΎΠ±ΠΎΠ²Π°Ρ‚ΡŒ ΡΠΎΠ±Ρ€Π°Ρ‚ΡŒ ΠΏΡ€ΠΎΡΡ‚Π΅Π½ΡŒΠΊΠΈΠΉ ΡΠ°ΠΌΠΎΠ΄Π΅Π»ΡŒΠ½Ρ‹ΠΉ тСстСр. Π’ ΠΈΠ½Ρ‚Π΅Ρ€Π½Π΅Ρ‚Π΅ ΠΌΠΎΠΆΠ½ΠΎ Π½Π°ΠΉΡ‚ΠΈ Π½Π΅ΠΏΠ»ΠΎΡ…ΠΈΠ΅ схСмы (послоТнСС, ΠΏΠΎΠΏΡ€ΠΎΡ‰Π΅, совсСм простая).

Ну ΠΈΠ»ΠΈ Ρ€Π°ΡΠΊΠΎΡˆΠ΅Π»ΠΈΡ‚ΡŒΡΡ, Π½Π°ΠΊΠΎΠ½Π΅Ρ†, Π½Π° ΡƒΠ½ΠΈΠ²Π΅Ρ€ΡΠ°Π»ΡŒΠ½Ρ‹ΠΉ тСстСр, ΠΊΠΎΡ‚ΠΎΡ€Ρ‹ΠΉ измСряСт Π΅ΠΌΠΊΠΎΡΡ‚ΡŒ Π΄ΠΎ 100000 ΠΌΠΊΠ€, ESR, сопротивлСниС, ΠΈΠ½Π΄ΡƒΠΊΡ‚ΠΈΠ²Π½ΠΎΡΡ‚ΡŒ, позволяСт ΠΏΡ€ΠΎΠ²Π΅Ρ€ΡΡ‚ΡŒ Π΄ΠΈΠΎΠ΄Ρ‹ ΠΈ ΠΈΠ·ΠΌΠ΅Ρ€ΡΡ‚ΡŒ ΠΏΠ°Ρ€Π°ΠΌΠ΅Ρ‚Ρ€Ρ‹ транзисторов. Бколько Ρ€Π°Π· ΠΎΠ½ мСня Π²Ρ‹Ρ€ΡƒΡ‡Π°Π»!

Бпособ β„–2: ΠΈΠ·ΠΌΠ΅Ρ€Π΅Π½ΠΈΠ΅ Смкости Π΄Π²ΡƒΡ… ΠΏΠΎΡΠ»Π΅Π΄ΠΎΠ²Π°Ρ‚Π΅Π»ΡŒΠ½ΠΎ Π²ΠΊΠ»ΡŽΡ‡Π΅Π½Π½Ρ‹Ρ… кондСнсаторов

Иногда Π±Ρ‹Π²Π°Π΅Ρ‚ Ρ‚Π°ΠΊ, Ρ‡Ρ‚ΠΎ имССтся ΠΌΡƒΠ»ΡŒΡ‚ΠΈΠΌΠ΅Ρ‚Ρ€ с измСрялкой Смкости, Π½ΠΎ Π΅Π³ΠΎ ΠΏΡ€Π΅Π΄Π΅Π»Π° Π½Π΅ Ρ…Π²Π°Ρ‚Π°Π΅Ρ‚. ΠžΠ±Ρ‹Ρ‡Π½ΠΎ Π²Π΅Ρ€Ρ…Π½ΠΈΠΉ ΠΏΠΎΡ€ΠΎΠ³ ΠΌΡƒΠ»ΡŒΡ‚ΠΈΠΌΠ΅Ρ‚Ρ€ΠΎΠ² — это 20 ΠΈΠ»ΠΈ 200 ΠΌΠΊΠ€, Π° Π½Π°ΠΌ Π½ΡƒΠΆΠ½ΠΎ ΠΈΠ·ΠΌΠ΅Ρ€ΠΈΡ‚ΡŒ Π΅ΠΌΠΊΠΎΡΡ‚ΡŒ, Π½Π°ΠΏΡ€ΠΈΠΌΠ΅Ρ€, Π² 1200 ΠΌΠΊΠ€. Как Ρ‚ΠΎΠ³Π΄Π° Π±Ρ‹Ρ‚ΡŒ?

На ΠΏΠΎΠΌΠΎΡ‰ΡŒ ΠΏΡ€ΠΈΡ…ΠΎΠ΄ΠΈΡ‚ Ρ„ΠΎΡ€ΠΌΡƒΠ»Π° Смкости Π΄Π²ΡƒΡ… ΠΏΠΎΡΠ»Π΅Π΄ΠΎΠ²Π°Ρ‚Π΅Π»ΡŒΠ½ΠΎ соСдинСнных кондСнсаторов:Π‘ΡƒΡ‚ΡŒ Π² Ρ‚ΠΎΠΌ, Ρ‡Ρ‚ΠΎ Ρ€Π΅Π·ΡƒΠ»ΡŒΡ‚ΠΈΡ€ΡƒΡŽΡ‰Π°Ρ Π΅ΠΌΠΊΠΎΡΡ‚ΡŒ CΡ€Π΅Π· Π΄Π²ΡƒΡ… ΠΏΠΎΡΠ»Π΅Π΄ΠΎΠ²Π°Ρ‚Π΅Π»ΡŒΠ½Ρ‹Ρ… ΠΊΠΎΠ½Π΄Π΅Ρ€ΠΎΠ² Π±ΡƒΠ΄Π΅Ρ‚ всСгда мСньшС Смкости самого малСнького ΠΈΠ· этих кондСнсаторов. Π”Ρ€ΡƒΠ³ΠΈΠΌΠΈ словами, Ссли Π²Π·ΡΡ‚ΡŒ кондСнсатор Π½Π° 20 ΠΌΠΊΠ€, Ρ‚ΠΎ ΠΊΠ°ΠΊΠΎΠΉ Π±Ρ‹ большой Π΅ΠΌΠΊΠΎΡΡ‚ΡŒΡŽ Π½Π΅ ΠΎΠ±Π»Π°Π΄Π°Π» Π±Ρ‹ Π²Ρ‚ΠΎΡ€ΠΎΠΉ кондСнсатор, Ρ€Π΅Π·ΡƒΠ»ΡŒΡ‚ΠΈΡ€ΡƒΡŽΡ‰Π°Ρ Π΅ΠΌΠΊΠΎΡΡ‚ΡŒ всС Ρ€Π°Π²Π½ΠΎ Π±ΡƒΠ΄Π΅Ρ‚ мСньшС, Ρ‡Π΅ΠΌ 20 ΠΌΠΊΠ€.

Π’Π°ΠΊΠΈΠΌ ΠΎΠ±Ρ€Π°Π·ΠΎΠΌ, Ссли ΠΏΡ€Π΅Π΄Π΅Π» измСрСния нашСго ΠΌΡƒΠ»ΡŒΡ‚ΠΈΠΌΠ΅Ρ‚Ρ€Π° 20 ΠΌΠΊΠ€, Ρ‚ΠΎ нСизвСстный кондСнсатор Π½ΡƒΠΆΠ½ΠΎ ΠΏΠΎΡΠ»Π΅Π΄ΠΎΠ²Π°Ρ‚Π΅Π»ΡŒΠ½ΠΎ с кондСнсатором Π½Π΅ Π±ΠΎΠ»Π΅Π΅ 20 ΠΌΠΊΠ€.ΠžΡΡ‚Π°Π΅Ρ‚ΡΡ Ρ‚ΠΎΠ»ΡŒΠΊΠΎ ΠΈΠ·ΠΌΠ΅Ρ€ΠΈΡ‚ΡŒ ΠΎΠ±Ρ‰ΡƒΡŽ Π΅ΠΌΠΊΠΎΡΡ‚ΡŒ Ρ†Π΅ΠΏΠΎΡ‡ΠΊΠΈ ΠΈΠ· Π΄Π²ΡƒΡ… ΠΏΠΎΡΠ»Π΅Π΄ΠΎΠ²Π°Ρ‚Π΅Π»ΡŒΠ½ΠΎ Π²ΠΊΠ»ΡŽΡ‡Π΅Π½Π½Ρ‹Ρ… кондСнсаторов. Π•ΠΌΠΊΠΎΡΡ‚ΡŒ нСизвСстного кондСнсатора рассчитываСтся ΠΏΠΎ Ρ„ΠΎΡ€ΠΌΡƒΠ»Π΅:Π”Π°Π²Π°ΠΉΡ‚Π΅ для ΠΏΡ€ΠΈΠΌΠ΅Ρ€Π° рассчитаСм Π΅ΠΌΠΊΠΎΡΡ‚ΡŒ большого кондСнсатора Π‘Ρ… с Ρ„ΠΎΡ‚ΠΎΠ³Ρ€Π°Ρ„ΠΈΠΈ Π²Ρ‹ΡˆΠ΅. Для провСдСния измСрСния ΠΏΠΎΡΠ»Π΅Π΄ΠΎΠ²Π°Ρ‚Π΅Π»ΡŒΠ½ΠΎ с этим кондСнсатором Π²ΠΊΠ»ΡŽΡ‡Π΅Π½ кондСнсатор Π‘1 Π½Π° 10.06 ΠΌΠΊΠ€ (ΠΎΠ½ Π±Ρ‹Π» ΠΏΡ€Π΅Π΄Π²Π°Ρ€ΠΈΡ‚Π΅Π»ΡŒΠ½ΠΎ ΠΈΠ·ΠΌΠ΅Ρ€Π΅Π½). Π’ΠΈΠ΄Π½ΠΎ, Ρ‡Ρ‚ΠΎ Ρ€Π΅Π·ΡƒΠ»ΡŒΡ‚ΠΈΡ€ΡƒΡŽΡ‰Π°Ρ Π΅ΠΌΠΊΠΎΡΡ‚ΡŒ составила CΡ€Π΅Π· = 9. 97 ΠΌΠΊΠ€.

ΠŸΠΎΠ΄ΡΡ‚Π°Π²Π»ΡΠ΅ΠΌ эти Ρ†ΠΈΡ„Ρ€Ρ‹ Π² Ρ„ΠΎΡ€ΠΌΡƒΠ»Ρƒ ΠΈ ΠΏΠΎΠ»ΡƒΡ‡Π°Π΅ΠΌ:

Бпособ β„–3: ΠΈΠ·ΠΌΠ΅Ρ€Π΅Π½ΠΈΠ΅ Смкости Ρ‡Π΅Ρ€Π΅Π· ΠΏΠΎΡΡ‚ΠΎΡΠ½Π½ΡƒΡŽ Π²Ρ€Π΅ΠΌΠ΅Π½ΠΈ Ρ†Π΅ΠΏΠΈ

Как извСстно, постоянная Π²Ρ€Π΅ΠΌΠ΅Π½ΠΈ RC-Ρ†Π΅ΠΏΠΈ зависит ΠΎΡ‚ Π²Π΅Π»ΠΈΡ‡ΠΈΠ½Ρ‹ сопротивлСния R ΠΈ значСния Смкости CΡ…:ΠŸΠΎΡΡ‚ΠΎΡΠ½Π½Π°Ρ Π²Ρ€Π΅ΠΌΠ΅Π½ΠΈ — это врСмя, Π·Π° ΠΊΠΎΡ‚ΠΎΡ€ΠΎΠ΅ напряТСниС Π½Π° кондСнсаторС ΡƒΠΌΠ΅Π½ΡŒΡˆΠΈΡ‚ΡΡ Π² Π΅ Ρ€Π°Π· (Π³Π΄Π΅ Π΅ — это основаниС Π½Π°Ρ‚ΡƒΡ€Π°Π»ΡŒΠ½ΠΎΠ³ΠΎ Π»ΠΎΠ³Π°Ρ€ΠΈΡ„ΠΌΠ°, ΠΏΡ€ΠΈΠ±Π»ΠΈΠ·ΠΈΡ‚Π΅Π»ΡŒΠ½ΠΎ Ρ€Π°Π²Π½ΠΎΠ΅ 2,718).

Π’Π°ΠΊΠΈΠΌ ΠΎΠ±Ρ€Π°Π·ΠΎΠΌ, Ссли Π·Π°ΡΠ΅Ρ‡ΡŒ Π·Π° ΠΊΠ°ΠΊΠΎΠ΅ врСмя разрядится кондСнсатор Ρ‡Π΅Ρ€Π΅Π· извСстноС сопротивлСниС, Ρ€Π°ΡΡΡ‡ΠΈΡ‚Π°Ρ‚ΡŒ Π΅Π³ΠΎ Π΅ΠΌΠΊΠΎΡΡ‚ΡŒ Π½Π΅ составит Ρ‚Ρ€ΡƒΠ΄Π°.Для ΠΏΠΎΠ²Ρ‹ΡˆΠ΅Π½ΠΈΡ точности измСрСния Π½Π΅ΠΎΠ±Ρ…ΠΎΠ΄ΠΈΠΌΠΎ Π²Π·ΡΡ‚ΡŒ рСзистор с ΠΌΠΈΠ½ΠΈΠΌΠ°Π»ΡŒΠ½Ρ‹ΠΌ ΠΎΡ‚ΠΊΠ»ΠΎΠ½Π΅Π½ΠΈΠ΅ΠΌ сопротивлСния. Π”ΡƒΠΌΠ°ΡŽ, 0.005% Π±ΡƒΠ΄Π΅Ρ‚ Π½ΠΎΡ€ΠΌΠ°Π»ΡŒΠ½ΠΎ =)Π₯отя ΠΌΠΎΠΆΠ½ΠΎ Π²Π·ΡΡ‚ΡŒ ΠΎΠ±Ρ‹Ρ‡Π½Ρ‹ΠΉ рСзистор с 5-10%-ΠΎΠΉ ΠΏΠΎΠ³Ρ€Π΅ΡˆΠ½ΠΎΡΡ‚ΡŒΡŽ ΠΈ Ρ‚ΡƒΠΏΠΎ ΠΈΠ·ΠΌΠ΅Ρ€ΠΈΡ‚ΡŒ Π΅Π³ΠΎ Ρ€Π΅Π°Π»ΡŒΠ½ΠΎΠ΅ сопротивлСниС ΠΌΡƒΠ»ΡŒΡ‚ΠΈΠΌΠ΅Ρ‚Ρ€ΠΎΠΌ. РСзистор ΠΆΠ΅Π»Π°Ρ‚Π΅Π»ΡŒΠ½ΠΎ Π²Ρ‹Π±ΠΈΡ€Π°Ρ‚ΡŒ Ρ‚Π°ΠΊΠΎΠΉ, Ρ‡Ρ‚ΠΎΠ±Ρ‹ врСмя разряда кондСнсатора Π±Ρ‹Π»ΠΎ Π±ΠΎΠ»Π΅Π΅-ΠΌΠ΅Π½Π΅Π΅ вмСняСмым (сСкунд 10-30).

Π’ΠΎΡ‚ ΠΊΠ°ΠΊΠΎΠΉ-Ρ‚ΠΎ Ρ‡Π΅Π» ΠΎΡ‡Π΅Π½ΡŒ Ρ…ΠΎΡ€ΠΎΡˆΠΎ всС рассказал Π½Π° Π²ΠΈΠ΄Π΅ΠΎ:

Π”Ρ€ΡƒΠ³ΠΈΠ΅ способы измСрСния Смкости

Π’Π°ΠΊΠΆΠ΅ ΠΌΠΎΠΆΠ½ΠΎ ΠΎΡ‡Π΅Π½ΡŒ ΠΏΡ€ΠΈΠ±Π»ΠΈΠ·ΠΈΡ‚Π΅Π»ΡŒΠ½ΠΎ ΠΎΡ†Π΅Π½ΠΈΡ‚ΡŒ Π΅ΠΌΠΊΠΎΡΡ‚ΡŒ кондСнсатора Ρ‡Π΅Ρ€Π΅Π· ΡΠΊΠΎΡ€ΠΎΡΡ‚ΡŒ роста Π΅Π³ΠΎ сопротивлСния постоянному Ρ‚ΠΎΠΊΡƒ Π² Ρ€Π΅ΠΆΠΈΠΌΠ΅ ΠΏΡ€ΠΎΠ·Π²ΠΎΠ½ΠΊΠΈ. Об этом ΡƒΠΆΠ΅ ΡƒΠΏΠΎΠΌΠΈΠ½Π°Π»ΠΎΡΡŒ, ΠΊΠΎΠ³Π΄Π° шла Ρ€Π΅Ρ‡ΡŒ ΠΏΡ€ΠΎ ΠΏΡ€ΠΎΠ²Π΅Ρ€ΠΊΡƒ Π½Π° ΠΎΠ±Ρ€Ρ‹Π².

Π―Ρ€ΠΊΠΎΡΡ‚ΡŒ свСчСния Π»Π°ΠΌΠΏΠΎΡ‡ΠΊΠΈ (см. ΠΌΠ΅Ρ‚ΠΎΠ΄ поиска ΠšΠ—) Ρ‚Π°ΠΊΠΆΠ΅ Π΄Π°Π΅Ρ‚ вСсьма ΠΏΡ€ΠΈΠ±Π»ΠΈΠ·ΠΈΡ‚Π΅Π»ΡŒΠ½ΡƒΡŽ ΠΎΡ†Π΅Π½ΠΊΡƒ Смкости, Π½ΠΎ Ρ‚Π΅ΠΌ Π½Π΅ ΠΌΠ΅Π½Π΅Π΅ Ρ‚Π°ΠΊΠΎΠ΅ способ ΠΈΠΌΠ΅Π΅Ρ‚ ΠΏΡ€Π°Π²ΠΎ Π½Π° сущСствованиС.

БущСствуСт Ρ‚Π°ΠΊΠΆΠ΅ ΠΌΠ΅Ρ‚ΠΎΠ΄ измСрСния Смкости посрСдством измСрСния Π΅Π΅ сопротивлСния ΠΏΠ΅Ρ€Π΅ΠΌΠ΅Π½Π½ΠΎΠΌΡƒ Ρ‚ΠΎΠΊΡƒ. ΠŸΡ€ΠΈΠΌΠ΅Ρ€ΠΎΠΌ Ρ€Π΅Π°Π»ΠΈΠ·Π°Ρ†ΠΈΠΈ Π΄Π°Π½Π½ΠΎΠ³ΠΎ ΠΌΠ΅Ρ‚ΠΎΠ΄Π° слуТит ΠΏΡ€ΠΎΡΡ‚Π΅ΠΉΡˆΠ°Ρ мостовая схСма:Π’Ρ€Π°Ρ‰Π΅Π½ΠΈΠ΅ΠΌ Ρ€ΠΎΡ‚ΠΎΡ€Π° ΠΏΠ΅Ρ€Π΅ΠΌΠ΅Π½Π½ΠΎΠ³ΠΎ кондСнсатора Π‘2 Π΄ΠΎΠ±ΠΈΠ²Π°ΡŽΡ‚ΡΡ баланса моста (балансировка опрСдСляСтся ΠΏΠΎ ΠΌΠΈΠ½ΠΈΠΌΠ°Π»ΡŒΠ½Ρ‹ΠΌ показаниям Π²ΠΎΠ»ΡŒΡ‚ΠΌΠ΅Ρ‚Ρ€Π°). Π¨ΠΊΠ°Π»Π° Π·Π°Ρ€Π°Π½Π΅Π΅ ΠΏΡ€ΠΎΠ³Ρ€Π°Π΄ΡƒΠΈΡ€ΠΎΠ²Π°Π½Π° Π² значСниях Смкости измСряСмого кондСнсатора. ΠŸΠ΅Ρ€Π΅ΠΊΠ»ΡŽΡ‡Π°Ρ‚Π΅Π»ΡŒ SA1 слуТит для ΠΏΠ΅Ρ€Π΅ΠΊΠ»ΡŽΡ‡Π΅Π½ΠΈΡ Π΄ΠΈΠ°ΠΏΠ°Π·ΠΎΠ½Π° измСрСния. Π—Π°ΠΌΠΊΠ½ΡƒΡ‚ΠΎΠ΅ ΠΏΠΎΠ»ΠΎΠΆΠ΅Π½ΠΈΠ΅ соотвСтствуСт шкалС 40…85 ΠΏΠ€. ΠšΠΎΠ½Π΄Π΅Π½ΡΠ°Ρ‚ΠΎΡ€Ρ‹ Π‘3 ΠΈ Π‘4 ΠΌΠΎΠΆΠ½ΠΎ Π·Π°ΠΌΠ΅Π½ΠΈΡ‚ΡŒ ΠΎΠ΄ΠΈΠ½Π°ΠΊΠΎΠ²Ρ‹ΠΌΠΈ рСзисторами.

НСдостаток схСмы — Π½Π΅ΠΎΠ±Ρ…ΠΎΠ΄ΠΈΠΌ Π³Π΅Π½Π΅Ρ€Π°Ρ‚ΠΎΡ€ ΠΏΠ΅Ρ€Π΅ΠΌΠ΅Π½Π½ΠΎΠ³ΠΎ напряТСния, плюс трСбуСтся ΠΏΡ€Π΅Π΄Π²Π°Ρ€ΠΈΡ‚Π΅Π»ΡŒΠ½Π°Ρ ΠΊΠ°Π»ΠΈΠ±Ρ€ΠΎΠΊΠ°.

МоТно Π»ΠΈ ΠΏΡ€ΠΎΠ²Π΅Ρ€ΠΈΡ‚ΡŒ кондСнсатор ΠΌΡƒΠ»ΡŒΡ‚ΠΈΠΌΠ΅Ρ‚Ρ€ΠΎΠΌ Π½Π΅ выпаивая Π΅Π³ΠΎ с ΠΏΠ»Π°Ρ‚Ρ‹?

НС сущСствуСт ΠΎΠ΄Π½ΠΎΠ·Π½Π°Ρ‡Π½ΠΎΠ³ΠΎ ΠΎΡ‚Π²Π΅Ρ‚Π° Π½Π° вопрос ΠΊΠ°ΠΊ ΠΏΡ€ΠΎΠ²Π΅Ρ€ΠΈΡ‚ΡŒ кондСнсатор ΠΌΡƒΠ»ΡŒΡ‚ΠΈΠΌΠ΅Ρ‚Ρ€ΠΎΠΌ Π½Π΅ выпаивая: всС зависит ΠΎ схСмы, Π² ΠΊΠΎΡ‚ΠΎΡ€ΠΎΠΉ стоит кондСнсатор.

ВсС Π΄Π΅Π»ΠΎ Π² Ρ‚ΠΎΠΌ, Ρ‡Ρ‚ΠΎ ΠΏΡ€ΠΈΠ½Ρ†ΠΈΠΏΠΈΠ°Π»ΡŒΠ½Ρ‹Π΅ схСмы, ΠΊΠ°ΠΊ ΠΏΡ€Π°Π²ΠΈΠ»ΠΎ, состоят ΠΈΠ· мноТСства элСмСнтов, ΠΊΠΎΡ‚ΠΎΡ€Ρ‹Π΅ ΠΌΠΎΠ³ΡƒΡ‚ Π±Ρ‹Ρ‚ΡŒ соСдинСны с исслСдуСмым кондСнсатором самым замысловатым ΠΎΠ±Ρ€Π°Π·ΠΎΠΌ.

НапримСр, нСсколько кондСнсаторов ΠΌΠΎΠ³ΡƒΡ‚ Π±Ρ‹Ρ‚ΡŒ соСдинСны ΠΏΠ°Ρ€Π°Π»Π»Π΅Π»ΡŒΠ½ΠΎ ΠΈ Ρ‚ΠΎΠ³Π΄Π° ΠΏΡ€ΠΈΠ±ΠΎΡ€ ΠΏΠΎΠΊΠ°ΠΆΠ΅Ρ‚ ΠΈΡ… ΡΡƒΠΌΠΌΠ°Ρ€Π½ΡƒΡŽ Π΅ΠΌΠΊΠΎΡΡ‚ΡŒ. Если ΠΏΡ€ΠΈ этом ΠΎΠ΄ΠΈΠ½ ΠΈΠ· кондСнсаторов Π±ΡƒΠ΄Π΅Ρ‚ Π² ΠΎΠ±Ρ€Ρ‹Π²Π΅, Ρ‚ΠΎ это Π±ΡƒΠ΄Π΅Ρ‚ ΠΎΡ‡Π΅Π½ΡŒ слоТно Π·Π°ΠΌΠ΅Ρ‚ΠΈΡ‚ΡŒ.

Или, Π½Π°ΠΏΡ€ΠΈΠΌΠ΅Ρ€, довольно часто ΠΏΠ°Ρ€Π°Π»Π»Π΅Π»ΡŒΠ½ΠΎ элСктролитичСскому кондСнсатору ΡƒΡΡ‚Π°Π½Π°Π²Π»ΠΈΠ²Π°ΡŽΡ‚ кСрамичСский. Π’ этом случаС Π½Π΅Ρ‚ Π½ΠΈ малСйшСй возмоТности ΠΏΡ€ΠΎΠ·Π²ΠΎΠ½ΠΈΡ‚ΡŒ кондСнсатор ΠΌΡƒΠ»ΡŒΡ‚ΠΈΠΌΠ΅Ρ‚Ρ€ΠΎΠΌ Π½Π° ΠΏΠ»Π°Ρ‚Π΅ ΠΈ ΠΎΠΏΡ€Π΅Π΄Π΅Π»ΠΈΡ‚ΡŒ Π²Π½ΡƒΡ‚Ρ€Π΅Π½Π½ΠΈΠΉ ΠΎΠ±Ρ€Ρ‹Π².Π’ ΠΊΠΎΠ»Π΅Π±Π°Ρ‚Π΅Π»ΡŒΠ½Ρ‹Ρ… ΠΊΠΎΠ½Ρ‚ΡƒΡ€Π°Ρ…, Π²ΠΎΠΎΠ±Ρ‰Π΅, ΠΏΠ°Ρ€Π°Π»Π»Π΅Π»ΡŒΠ½ΠΎ ΠΊΠΎΠ½Π΄Π΅Ρ€Ρƒ ΠΌΠΎΠΆΠ΅Ρ‚ ΠΎΠΊΠ°Π·Π°Ρ‚ΡŒΡΡ ΠΊΠ°Ρ‚ΡƒΡˆΠΊΠ° индуктивности. Π’ΠΎΠ³Π΄Π° ΠΏΡ€ΠΎΠ·Π²ΠΎΠ½ΠΊΠ° кондСнсатора ΠΏΠΎΠΊΠ°ΠΆΠ΅Ρ‚ ΠΊΠΎΡ€ΠΎΡ‚ΠΊΠΎΠ΅ Π·Π°ΠΌΡ‹ΠΊΠ°Π½ΠΈΠ΅, хотя Π½Π° самом Π΄Π΅Π»Π΅ Π΅Π³ΠΎ Π½Π΅Ρ‚.

Π’ΠΎΡ‚ ΠΏΡ€ΠΈΠΌΠ΅Ρ€, ΠΊΠΎΠ³Π΄Π° всС ΠΏΡΡ‚ΡŒ кондСнсаторов ΠΏΠΎΠΊΠ°ΠΆΡƒΡ‚ Π»ΠΎΠΆΠ½ΠΎΠ΅ ΠšΠ—:

Π’Π°ΠΊΠΈΠΌ ΠΎΠ±Ρ€Π°Π·ΠΎΠΌ, ΠΏΡ€ΠΎΠ²Π΅Ρ€ΠΊΠ° кондСнсаторов ΠΌΡƒΠ»ΡŒΡ‚ΠΈΠΌΠ΅Ρ‚Ρ€ΠΎΠΌ Π±Π΅Π· выпаивания Π²ΠΎΠΎΠ±Ρ‰Π΅ Π½Π΅Π²ΠΎΠ·ΠΌΠΎΠΆΠ½Π°.

Π’ схСмах ΠΈΠΌΠΏΡƒΠ»ΡŒΡΠ½Ρ‹Ρ… Π±Π»ΠΎΠΊΠΎΠ² питания ΠΎΡ‡Π΅Π½ΡŒ часто Π²ΡΡ‚Ρ€Π΅Ρ‡Π°ΡŽΡ‚ΡΡ ΠΊΠΎΠ½Ρ‚ΡƒΡ€Π°, состоящиС ΠΈΠ· Π²Ρ‚ΠΎΡ€ΠΈΡ‡Π½ΠΎΠΉ ΠΎΠ±ΠΌΠΎΡ‚ΠΊΠΈ трансформатора, Π΄ΠΈΠΎΠ΄Π° ΠΈ Π²Ρ‹ΠΏΡ€ΡΠΌΠΈΡ‚Π΅Π»ΡŒΠ½ΠΎΠ³ΠΎ кондСнсатора. Π’Π°ΠΊ Π²ΠΎΡ‚ любая «ΠΏΡ€ΠΎΠ·Π²ΠΎΠ½ΠΊΠ°» кондСнсатора ΠΏΡ€ΠΈ ΠΏΡ€ΠΎΠ±ΠΈΡ‚ΠΎΠΌ Π΄ΠΈΠΎΠ΄Π΅ ΠΏΠΎΠΊΠ°ΠΆΠ΅Ρ‚ ΠšΠ—. А Π½Π° самом Π΄Π΅Π»Π΅ кондСнсатор ΠΌΠΎΠΆΠ΅Ρ‚ Π±Ρ‹Ρ‚ΡŒ Π²ΠΏΠΎΠ»Π½Π΅ исправСн.Π’ΠΎΠΎΠ±Ρ‰Π΅-Ρ‚ΠΎ, ΠΏΡ€ΠΎΠ²Π΅Ρ€ΠΈΡ‚ΡŒ элСктролитичСский кондСнсатор ΠΌΡƒΠ»ΡŒΡ‚ΠΈΠΌΠ΅Ρ‚Ρ€ΠΎΠΌ Π½Π΅ выпаивая ΠΌΠΎΠΆΠ½ΠΎ, Π½ΠΎ это Ρ‚ΠΎΠ»ΡŒΠΊΠΎ для ΠΊΠΎΠ½Π΄Π΅Ρ€ΠΎΠ² ΠΎΡ‰ΡƒΡ‚ΠΈΠΌΠΎΠΉ Смкости (>1 ΠΌΠΊΠ€) ΠΈ Ρ‚ΠΎΠ»ΡŒΠΊΠΎ ΠΏΡ€ΠΎΠ²Π΅Ρ€ΠΈΡ‚ΡŒ Π½Π°Π»ΠΈΡ‡ΠΈΠ΅ Смкости ΠΈ отсутствиС ΠΊΠΎΡ€ΠΎΡ‚Ρ‹ΡˆΠ°. Ни ΠΎ ΠΊΠ°ΠΊΠΎΠΌ ΠΈΠ·ΠΌΠ΅Ρ€Π΅Π½ΠΈΠΈ Смкости ΠΈ Ρ€Π΅Ρ‡ΠΈ Π±Ρ‹Ρ‚ΡŒ Π½Π΅ ΠΌΠΎΠΆΠ΅Ρ‚. К Ρ‚ΠΎΠΌΡƒ ΠΆΠ΅, Ссли ΠΏΡ€ΠΈΠ±ΠΎΡ€ ΠΏΠΎΠΊΠ°ΠΆΠ΅Ρ‚ ΠšΠ—, Ρ‚ΠΎ Π²Ρ‹ΠΏΠ°ΠΈΠ²Π°Ρ‚ΡŒ всС-Ρ‚Π°ΠΊΠΈ придСтся, Ρ‚Π°ΠΊ ΠΊΠ°ΠΊ ΠΊΠΎΡ€ΠΎΡ‚ΠΈΡ‚ΡŒ ΠΌΠΎΠΆΠ΅Ρ‚ Ρ‡Ρ‚ΠΎ ΡƒΠ³ΠΎΠ΄Π½ΠΎ Π½Π° ΠΏΠ»Π°Ρ‚Π΅.

МСлкиС ΠΊΠΎΠ½Π΄Π΅Ρ€Ρ‹ ΠΏΡ€ΠΎΠ²Π΅Ρ€ΡΡŽΡ‚ΡΡ Ρ‚ΠΎΠ»ΡŒΠΊΠΎ Π½Π° отсутствиС ΠšΠ—, ΠΎΠ±Ρ€Ρ‹Π² ΠΈ Π½ΡƒΠ»Π΅Π²ΡƒΡŽ Π΅ΠΌΠΊΠΎΡΡ‚ΡŒ Ρ‚Π°ΠΊΠΈΠΌ ΠΎΠ±Ρ€Π°Π·ΠΎΠΌ Π½Π΅ ΠΏΡ€ΠΎΠ²Π΅Ρ€ΠΈΡˆΡŒ.

Π’ΠΎΡ‚ ΠΎΡ‡Π΅Π½ΡŒ ΠΏΡ€Π°Π²ΠΈΠ»ΡŒΠ½Ρ‹ΠΉ ΠΈ понятный видос Π½Π° эту Ρ‚Π΅ΠΌΡƒ:

ΠŸΡ€ΠΈΠΌΠ΅Ρ€Ρ‹ Π²Ρ‹ΡˆΠ΅ (Π° Ρ‚Π°ΠΊΠΆΠ΅ Π΄ΠΎΡ…ΠΎΠ΄Ρ‡ΠΈΠ²ΠΎΠ΅ Π²ΠΈΠ΄Π΅ΠΎ) Π½Π΅ ΠΎΡΡ‚Π°Π²Π»ΡΡŽΡ‚ Π½ΠΈΠΊΠ°ΠΊΠΈΡ… сомнСний, Ρ‡Ρ‚ΠΎ ΠΏΡ€ΠΎΠ²Π΅Ρ€ΠΊΠ° кондСнсаторов Π½Π΅ выпаивая ΠΈΠ· схСмы — это фантастика.

Если ΠΊΠ°ΠΊΠΎΠΉ-Π»ΠΈΠ±ΠΎ кондСнсатор Π²Ρ‹Π·Ρ‹Π²Π°Π΅Ρ‚ сомнСния, Π»ΡƒΡ‡ΡˆΠ΅ сразу Π·Π°ΠΌΠ΅Π½ΠΈΡ‚ΡŒ Π΅Π³ΠΎ Π½Π° Π·Π°Π²Π΅Π΄ΠΎΠΌΠΎ исправный. Или хотя Π±Ρ‹ Π²Ρ€Π΅ΠΌΠ΅Π½Π½ΠΎ ΠΏΠΎΠ΄ΠΏΠ°ΡΡ‚ΡŒ Ρ…ΠΎΡ€ΠΎΡˆΠΈΠΉ кондСнсатор ΠΏΠ°Ρ€Π°Π»Π»Π΅Π»ΡŒΠ½ΠΎ ΡΠΎΠΌΠ½ΠΈΡ‚Π΅Π»ΡŒΠ½ΠΎΠΌΡƒ, Ρ‡Ρ‚ΠΎΠ±Ρ‹ ΠΏΠΎΠ΄Ρ‚Π²Π΅Ρ€Π΄ΠΈΡ‚ΡŒ ΠΈΠ»ΠΈ ΠΎΠΏΡ€ΠΎΠ²Π΅Ρ€Π³Π½ΡƒΡ‚ΡŒ подозрСния.

Как ΠΏΡ€ΠΎΠ²Π΅Ρ€ΠΈΡ‚ΡŒ кондСнсатор ΠΌΡƒΠ»ΡŒΡ‚ΠΈΠΌΠ΅Ρ‚Ρ€ΠΎΠΌ. ΠŸΡ€ΠΎΠ²Π΅Ρ€ΠΊΠ° кондСнсатора ΠΌΡƒΠ»ΡŒΡ‚ΠΈΠΌΠ΅Ρ‚Ρ€ΠΎΠΌ

ΠŸΡ€ΠΈΠ²Π΅Ρ‚ΡΡ‚Π²ΡƒΡŽ всСх Π΄Ρ€ΡƒΠ·ΡŒΡ ΠΈ Ρ‡ΠΈΡ‚Π°Ρ‚Π΅Π»ΠΈ сайта Β«Π­Π»Π΅ΠΊΡ‚Ρ€ΠΈΠΊ Π² Π΄ΠΎΠΌΠ΅Β». Π”ΡƒΠΌΠ°ΡŽ всСм извСстно, Ρ‡Ρ‚ΠΎ Ρ‚Π°ΠΊΠΎΠ΅ кондСнсатор. Если ΠΊΡ‚ΠΎ Π½Π΅ Π²ΠΈΠ΄Π΅Π» Π΄Π°Π½Π½Ρ‹ΠΉ элСмСнт микросхСм, Ρ‚ΠΎ Ρ‚ΠΎΡ‡Π½ΠΎ ΡΠ»ΡƒΡˆΠ°Π» ΠΎ Π½Π΅ΠΌ. Π‘Π°ΠΌΠΎΠΉ распространСнной ΠΏΡ€ΠΈΡ‡ΠΈΠ½ΠΎΠΉ нСисправности Π² радиоэлСктроникС являСтся ΠΏΠΎΠ²Ρ€Π΅ΠΆΠ΄Π΅Π½ΠΈΠ΅ ΠΈΠΌΠ΅Π½Π½ΠΎ этого элСмСнта. БоврСмСнная бытовая Ρ‚Π΅Ρ…Π½ΠΈΠΊΠ° Β«Π½Π°Ρ‡ΠΈΠ½Π΅Π½Π°Β» элСктроникой ΠΈ ΠΏΠΎΠ»ΠΎΠΌΠΊΠ° Ρ‚Π°ΠΊΠΎΠΉ ΠΊΡ€ΠΎΡ…ΠΎΡ‚Π½ΠΎΠΉ Π΄Π΅Ρ‚Π°Π»ΠΈ ΠΏΡ€ΠΈΠ²ΠΎΠ΄ΠΈΡ‚ ΠΊ ΠΏΠΎΡ‚Π΅Ρ€Π΅ Ρ„ΡƒΠ½ΠΊΡ†ΠΈΠΎΠ½Π°Π»ΡŒΠ½ΠΎΡΡ‚ΠΈ всСго ΠΌΠ΅Ρ…Π°Π½ΠΈΠ·ΠΌΠ° Π² Ρ†Π΅Π»ΠΎΠΌ.

Π§Ρ‚ΠΎΠ±Ρ‹ ΠΎΠΏΡ€Π΅Π΄Π΅Π»ΠΈΡ‚ΡŒ ΠΊΠ°ΠΊΠΎΠΉ ΠΈΠΌΠ΅Π½Π½ΠΎ кондСнсатор Π² схСмС Π²Ρ‹ΡˆΠ΅Π» ΠΈΠ· строя ΠΈΡ… Π½Π΅ΠΎΠ±Ρ…ΠΎΠ΄ΠΈΠΌΠΎ ΠΏΡ€ΠΎΠ²Π΅Ρ€ΠΈΡ‚ΡŒ Π½Π° Ρ€Π°Π±ΠΎΡ‚ΠΎΡΠΏΠΎΡΠΎΠ±Π½ΠΎΡΡ‚ΡŒ. И ΠΆΠ΅Π»Π°Ρ‚Π΅Π»ΡŒΠ½ΠΎ это Π΄Π΅Π»Π°Ρ‚ΡŒ с ΠΏΠΎΠΌΠΎΡ‰ΡŒΡŽ элСктронный ΠΏΡ€ΠΈΠ±ΠΎΡ€ΠΎΠ², Ρ‚Π° ΠΊΠ°ΠΊ Π²ΠΈΠ·ΡƒΠ°Π»ΡŒΠ½Ρ‹ΠΉ осмотр Π½Π΅ Π΄Π°Π΅Ρ‚ Π·Π°ΠΊΠ»ΡŽΡ‡Π΅Π½ΠΈΡ ΠΎ нСисправности.

Π”Π΅Π»Π°Ρ‚ΡŒ ΠΌΡ‹ это Π±ΡƒΠ΄Π΅ΠΌ с ΠΏΠΎΠΌΠΎΡ‰ΡŒΡŽ Π½Π΅Π΄ΠΎΡ€ΠΎΠ³ΠΎΠ³ΠΎ ΠΈ Ρ„ΡƒΠ½ΠΊΡ†ΠΈΠΎΠ½Π°Π»ΡŒΠ½ΠΎΠ³ΠΎ ΠΏΡ€ΠΈΠ±ΠΎΡ€Π° — ΠΌΡƒΠ»ΡŒΡ‚ΠΈΠΌΠ΅Ρ‚Ρ€Π°. Π’ ΠΏΡ€ΠΎΡˆΠ»ΠΎΠΉ ΡΡ‚Π°Ρ‚ΡŒΠ΅ я писал ΠΎ Ρ‚ΠΎΠΌ, ΠΊΠ°ΠΊ с Π΅Π³ΠΎ ΠΏΠΎΠΌΠΎΡ‰ΡŒΡŽ ΠΌΠΎΠΆΠ½ΠΎ Π²Ρ‹ΠΏΠΎΠ»Π½ΠΈΡ‚ΡŒ ΠΏΡ€ΠΎΠ²Π΅Ρ€ΠΊΡƒ сопротивлСния, Π° сСгодня рассмотрим ΠΌΠ΅Ρ‚ΠΎΠ΄ΠΈΠΊΡƒ, ΠΊΠ°ΠΊ ΠΏΡ€ΠΎΠ²Π΅Ρ€ΠΈΡ‚ΡŒ кондСнсатор ΠΌΡƒΠ»ΡŒΡ‚ΠΈΠΌΠ΅Ρ‚Ρ€ΠΎΠΌ.

ΠΠ°ΠΏΠΈΡΠ°Ρ‚ΡŒ Π΄Π°Π½Π½ΡƒΡŽ ΡΡ‚Π°Ρ‚ΡŒΡŽ мСня попросил ΠΎΠ΄ΠΈΠ½ ΠΈΠ· подписчиков. Π― ΠΊΠ°ΠΊ всСгда ΠΏΠΎΡΡ‚Π°Ρ€Π°ΡŽΡΡŒ ΠΈΠ·Π»ΠΎΠΆΠΈΡ‚ΡŒ ΠΌΠ°Ρ‚Π΅Ρ€ΠΈΠ°Π» доступным языком, Π½ΠΎ Ссли останутся вопросы, Π½Π΅ ΡΡ‚Π΅ΡΠ½ΡΠΉΡ‚Π΅ΡΡŒ Π·Π°Π΄Π°Π²Π°Ρ‚ΡŒ ΠΈΡ… Π² коммСнтариях.

ΠŸΡ€ΠΎΠ²Π΅Ρ€ΠΊΠ° кондСнсатора ΠΌΡƒΠ»ΡŒΡ‚ΠΈΠΌΠ΅Ρ‚Ρ€ΠΎΠΌ

Для Π½Π°Ρ‡Π°Π»Π° Π΄Π°Π²Π°ΠΉΡ‚Π΅ разбСрСмся, Ρ‡Ρ‚ΠΎ это Π·Π° устройство, ΠΈΠ· Ρ‡Π΅Π³ΠΎ ΠΎΠ½ состоит, ΠΈ ΠΊΠ°ΠΊΠΈΠ΅ Π²ΠΈΠ΄Ρ‹ кондСнсаторов ΡΡƒΡ‰Π΅ΡΡ‚Π²ΡƒΡŽΡ‚.

ΠšΠΎΠ½Π΄Π΅Π½ΡΠ°Ρ‚ΠΎΡ€ прСдставляСт собой устройство, ΠΊΠΎΡ‚ΠΎΡ€ΠΎΠ΅ способно Π½Π°ΠΊΠ°ΠΏΠ»ΠΈΠ²Π°Ρ‚ΡŒ элСктричСский заряд. Π’Π½ΡƒΡ‚Ρ€ΠΈ ΠΎΠ½ состоит ΠΈΠ· Π΄Π²ΡƒΡ… мСталличСских пластин ΠΏΠ°Ρ€Π°Π»Π»Π΅Π»ΡŒΠ½Ρ‹Ρ… ΠΌΠ΅ΠΆΠ΄Ρƒ собой. ΠœΠ΅ΠΆΠ΄Ρƒ пластинами располоТСн диэлСктрик (ΠΏΡ€ΠΎΠΊΠ»Π°Π΄ΠΊΠ°). Π§Π΅ΠΌ большС пластины, Ρ‚Π΅ΠΌ соотвСтствСнно больший заряд ΠΎΠ½ΠΈ ΠΌΠΎΠ³ΡƒΡ‚ Π½Π°ΠΊΠ°ΠΏΠ»ΠΈΠ²Π°Ρ‚ΡŒ.

БущСствуСт Π΄Π²Π° Π²ΠΈΠ΄Π° кондСнсаторов:

  1. 1) полярныС;
  2. 2) нСполярныС.

Как ΠΌΠΎΠΆΠ½ΠΎ Π΄ΠΎΠ³Π°Π΄Π°Ρ‚ΡŒΡΡ ΠΏΠΎ названию полярныС ΠΈΠΌΠ΅ΡŽΡ‚ ΠΏΠΎΠ»ΡΡ€Π½ΠΎΡΡ‚ΡŒ (плюс ΠΈ минус) ΠΈ ΠΏΠΎΠ΄ΠΊΠ»ΡŽΡ‡Π°ΡŽΡ‚ΡΡ ΠΊ элСктронным схСмам со строгим соблюдСниСм ΠΏΠΎΠ»ΡΡ€Π½ΠΎΡΡ‚ΡŒ: плюс ΠΊ ΠΏΠ»ΡŽΡΡƒ, минус ΠΊ минусу. Π’ ΠΏΡ€ΠΎΡ‚ΠΈΠ²Π½ΠΎΠΌ случаС кондСнсатор ΠΌΠΎΠΆΠ΅Ρ‚ Π²Ρ‹ΠΉΡ‚ΠΈ ΠΈΠ· строя.

ВсС полярныС кондСнсаторы – элСктролитичСскиС. Π‘Ρ‹Π²Π°ΡŽΡ‚ ΠΊΠ°ΠΊ с Ρ‚Π²Π΅Ρ€Π΄Ρ‹ΠΌ, Ρ‚Π°ΠΊ ΠΈ с ΠΆΠΈΠ΄ΠΊΠΈΠΌ элСктролитом. Π•ΠΌΠΊΠΎΡΡ‚ΡŒ колСблСтся Π² Π΄ΠΈΠ°ΠΏΠ°Π·ΠΎΠ½Π΅ 0.1 Γ· 100000 ΠΌΠΊΠ€.

НСполярныС кондСнсаторы Π±Π΅Π· Ρ€Π°Π·Π½ΠΈΡ†Ρ‹ ΠΊΠ°ΠΊ ΠΏΠΎΠ΄ΠΊΠ»ΡŽΡ‡Π°Ρ‚ΡŒ ΠΈΠ»ΠΈ Π²ΠΏΠ°ΠΈΠ²Π°Ρ‚ΡŒ Π² схСму, Ρƒ Π½ΠΈΡ… Π½Π΅Ρ‚ плюса ΠΈΠ»ΠΈ минуса. Π’ нСполярных ΠΊΠΎΠ½Π΄Π΅Ρ€Π°Ρ… диэлСктричСским ΠΌΠ°Ρ‚Π΅Ρ€ΠΈΠ°Π»ΠΎΠΌ являСтся Π±ΡƒΠΌΠ°Π³Π°, ΠΊΠ΅Ρ€Π°ΠΌΠΈΠΊΠ°, слюда, стСкло. Π˜Ρ… Π΅ΠΌΠΊΠΎΡΡ‚ΡŒ Π½Π΅ ΠΎΡ‡Π΅Π½ΡŒ большая колСблСтся Π² ΠΏΡ€ΠΈΠ΄Π΅Π»Π°Ρ… ΠΎΡ‚ нСсколько ΠΏΠ€ (ΠΏΠΈΠΊΠΎΡ„Π°Ρ€Π°Π΄) Π΄ΠΎ Π΅Π΄ΠΈΠ½ΠΈΡ† ΠΌΠΊΠ€ (ΠΌΠΈΠΊΡ€ΠΎΡ„Π°Ρ€Π°Π΄).

Π”Ρ€ΡƒΠ·ΡŒΡ Π½Π΅ΠΊΠΎΡ‚ΠΎΡ€Ρ‹Π΅ ΠΈΠ· Вас ΠΌΠΎΠ³ΡƒΡ‚ Π·Π°Π΄Π°Ρ‚ΡŒΡΡ вопросом, Π·Π°Ρ‡Π΅ΠΌ эта нСнуТная информация? Какая Ρ€Π°Π·Π½ΠΈΡ†Π° полярный-нСполярный? ВсС это влияСт Π½Π° ΠΌΠ΅Ρ‚ΠΎΠ΄ΠΈΠΊΡƒ ΠΈΠ·ΠΌΠ΅Ρ€Π΅Π½ΠΈΠΉ. И ΠΏΠ΅Ρ€Π΅Π΄ Ρ‚Π΅ΠΌ ΠΊΠ°ΠΊ ΠΏΡ€ΠΎΠ²Π΅Ρ€ΠΈΡ‚ΡŒ кондСнсатор ΠΌΡƒΠ»ΡŒΡ‚ΠΈΠΌΠ΅Ρ‚Ρ€ΠΎΠΌ Π½ΡƒΠΆΠ½ΠΎ ΠΏΠΎΠ½ΠΈΠΌΠ°Ρ‚ΡŒ, ΠΊΠ°ΠΊΠΎΠΉ ΠΈΠΌΠ΅Π½Π½ΠΎ Ρ‚ΠΈΠΏ устройства ΠΏΠ΅Ρ€Π΅Π΄ Π½Π°ΠΌΠΈ находится.

Как ΠΏΡ€ΠΎΠ²Π΅Ρ€ΠΈΡ‚ΡŒ кондСнсатор с ΠΏΠΎΠΌΠΎΡ‰ΡŒΡŽ ΠΏΡ€ΠΈΠ±ΠΎΡ€ΠΎΠ²

ΠŸΡ€Π΅ΠΆΠ΄Π΅ всСго, выполняСтся внСшний осмотр кондСнсатора Π½Π° ΠΏΡ€Π΅Π΄ΠΌΠ΅Ρ‚ Ρ‚Ρ€Π΅Ρ‰ΠΈΠ½ ΠΈ вздутия. НСрСдко ΠΏΡ€ΠΈΡ‡ΠΈΠ½ΠΎΠΉ нСисправности являСтся Π²Π½ΡƒΡ‚Ρ€Π΅Π½Π½ΠΈΠ΅ поврСТдСния элСктролитов, Ρ‡Ρ‚ΠΎ Π² свою ΠΎΡ‡Π΅Ρ€Π΅Π΄ΡŒ ΠΏΡ€ΠΈΠ²ΠΎΠ΄ΠΈΡ‚ ΠΊ ΡƒΠ²Π΅Π»ΠΈΡ‡Π΅Π½ΠΈΡŽ давлСния Π²Π½ΡƒΡ‚Ρ€ΠΈ корпуса, ΠΈ ΠΊΠ°ΠΊ слСдствиС Π²Π·Π΄ΡƒΡ‚ΠΈΠ΅ ΠΎΠ±ΠΎΠ»ΠΎΡ‡ΠΊΠΈ.

Если кондСнсатор с Π²ΠΈΠ΄Ρƒ Ρ†Π΅Π», Ρ‚ΠΎ Π±Π΅Π· ΡΠΏΠ΅Ρ†ΠΈΠ°Π»ΡŒΠ½Ρ‹Ρ… ΠΏΡ€ΠΈΠ±ΠΎΡ€ΠΎΠ² Ρ‚Ρ€ΡƒΠ΄Π½ΠΎ ΡΠΊΠ°Π·Π°Ρ‚ΡŒ работоспособный ΠΎΠ½ ΠΈΠ»ΠΈ Π½Π΅Ρ‚. ΠŸΠΎΡΡ‚ΠΎΠΌΡƒ Π² этом случаС выполняСтся ΠΏΡ€ΠΎΠ²Π΅Ρ€ΠΊΠ° кондСнсатора ΠΌΡƒΠ»ΡŒΡ‚ΠΈΠΌΠ΅Ρ‚Ρ€ΠΎΠΌ. Π­Ρ‚ΠΎΡ‚ простой ΠΏΡ€ΠΈΠ±ΠΎΡ€ ΠΏΠΎΠ·Π²ΠΎΠ»ΠΈΡ‚ Π½Π°ΠΌ ΠΎΠΏΡ€Π΅Π΄Π΅Π»ΠΈΡ‚ΡŒ Π΅ΠΌΠΊΠΎΡΡ‚ΡŒ кондСнсатора ΠΈ Π½Π°Π»ΠΈΡ‡ΠΈΠ΅ ΠΎΠ±Ρ€Ρ‹Π²ΠΎΠ² Π²Π½ΡƒΡ‚Ρ€ΠΈ.

ΠŸΠ΅Ρ€Π΅Π΄ Ρ‚Π΅ΠΌ, ΠΊΠ°ΠΊ ΠΏΡ€ΠΈΡΡ‚ΡƒΠΏΠΈΡ‚ΡŒ ΠΊ ΠΏΡ€ΠΎΠ²Π΅Ρ€ΠΊΠ΅, Π½ΡƒΠΆΠ½ΠΎ ΠΎΠΏΡ€Π΅Π΄Π΅Π»ΠΈΡ‚ΡŒΡΡ ΠΊΠ°ΠΊΠΎΠ³ΠΎ Ρ€ΠΎΠ΄Π° кондСнсатор находится ΠΏΠ΅Ρ€Π΅Π΄ Π²Π°ΠΌΠΈ: полярный ΠΈΠ»ΠΈ нСполярный. ΠŸΠΎΠΌΠ½ΠΈΡ‚Π΅, Π²Ρ‹ΡˆΠ΅ я писал, Ρ‡Ρ‚ΠΎ это Π±ΡƒΠ΄Π΅Ρ‚ Π²Π°ΠΆΠ½ΠΎ ΠΏΡ€ΠΈ измСрСниях.

Π’Π°ΠΊ Π²ΠΎΡ‚ ΠΏΡ€ΠΈ Π²Ρ‹ΠΏΠΎΠ»Π½Π΅Π½ΠΈΠΈ ΠΏΡ€ΠΎΠ²Π΅Ρ€ΠΊΠΈ полярных кондСнсаторов Π½ΡƒΠΆΠ½ΠΎ ΡΠΎΠ±Π»ΡŽΠ΄Π°Ρ‚ΡŒ ΠΏΠΎΠ»ΡΡ€Π½ΠΎΡΡ‚ΡŒ ΠΈ ΠΏΠΎΠ΄ΠΊΠ»ΡŽΡ‡Π°Ρ‚ΡŒ Ρ‰ΡƒΠΏΡ‹ ΠΊ Π½ΠΈΠΌ соотвСтствСнно: плюсовой ΠΊ Π½ΠΎΠΆΠΊΠ΅ Β«+Β», Π° минусовой ΠΊ Π½ΠΎΠΆΠΊΠ΅ Β«-Β».

ΠŸΡ€ΠΈ ΠΏΡ€ΠΎΠ²Π΅Ρ€ΠΊΠ΅ нСполярных Β«ΠΊΠΎΠ½Π΄Π΅Ρ€ΠΎΠ²Β» ΠΏΠΎΠ»ΡΡ€Π½ΠΎΡΡ‚ΡŒ Π² ΠΏΠΎΠ΄ΠΊΠ»ΡŽΡ‡Π΅Π½ΠΈΠΈ ΡΠΎΠ±Π»ΡŽΠ΄Π°Ρ‚ΡŒ Π½Π΅ Π½ΡƒΠΆΠ½ΠΎ, ΠΎΠ΄Π½Π°ΠΊΠΎ здСсь Π΅ΡΡ‚ΡŒ ΠΎΠ΄Π½Π° ΠΎΡΠΎΠ±Π΅Π½Π½ΠΎΡΡ‚ΡŒ Π½Π° ΠΊΠΎΡ‚ΠΎΡ€ΡƒΡŽ Π½ΡƒΠΆΠ½ΠΎ ΠΎΠ±Ρ€Π°Ρ‰Π°Ρ‚ΡŒ Π²Π½ΠΈΠΌΠ°Π½ΠΈΠ΅. Для ΠΏΡ€ΠΎΠ²Π΅Ρ€ΠΊΠΈ цСлостности ΠΊΠΎΠ½Π΄Π΅Ρ€Π° ΠΏΠ΅Ρ€Π΅ΠΊΠ»ΡŽΡ‡Π°Ρ‚Π΅Π»ΡŒ ΠΌΡƒΠ»ΡŒΡ‚ΠΈΠΌΠ΅Ρ‚Ρ€Π° Π½ΡƒΠΆΠ½ΠΎ Π²Ρ‹ΡΡ‚Π°Π²ΠΈΡ‚ΡŒ Π½Π° ΠΎΡ‚ΠΌΠ΅Ρ‚ΠΊΡƒ 2 МОм. Если Π±ΡƒΠ΄Π΅Ρ‚ мСньшС Ρ‚ΠΎ Π½Π° дисплСС Π±ΡƒΠ΄Π΅Ρ‚ ΠΎΡ‚ΠΎΠ±Ρ€Π°ΠΆΠ°Ρ‚ΡŒΡΡ — Β«1Β» (Π΅Π΄ΠΈΠ½ΠΈΡ†Π°), ΠΌΠΎΠΆΠ½ΠΎ Π»ΠΎΠΆΠ½ΠΎ ΠΏΠΎΠ΄ΡƒΠΌΠ°Ρ‚ΡŒ Ρ‡Ρ‚ΠΎ кондСнсатор нСисправСн.

ΠŸΡ€ΠΎΠ²Π΅Ρ€ΡΠ΅ΠΌ кондСнсатор ΠΌΡƒΠ»ΡŒΡ‚ΠΈΠΌΠ΅Ρ‚Ρ€ΠΎΠΌ Π² Ρ€Π΅ΠΆΠΈΠΌΠ΅ ΠΎΠΌΠΌΠ΅Ρ‚Ρ€Π°

Π’ нашСй сСгодняшнСй ΡΡ‚Π°Ρ‚ΡŒΠ΅ Π±ΡƒΠ΄Π΅ΠΌ ΠΏΡ€ΠΎΠ²Π΅Ρ€ΡΡ‚ΡŒ Ρ‡Π΅Ρ‚Ρ‹Ρ€Π΅ кондСнсатора: Π΄Π²Π° полярных (диэлСктричСских) ΠΈ Π΄Π²Π° нСполярных (кСрамичСских). ΠŸΠ΅Ρ€Π΅Π΄ Ρ‚Π΅ΠΌ ΠΊΠ°ΠΊ Π²Ρ‹ΠΏΠΎΠ»Π½ΡΡ‚ΡŒ ΠΏΡ€ΠΎΠ²Π΅Ρ€ΠΊΡƒ Π½Π΅ΠΎΠ±Ρ…ΠΎΠ΄ΠΈΠΌΠΎ Ρ€Π°Π·Ρ€ΡΠ΄ΠΈΡ‚ΡŒ кондСнсатор. Для этого Π½ΡƒΠΆΠ½ΠΎ Π·Π°ΠΌΠΊΠ½ΡƒΡ‚ΡŒ Π΅Π³ΠΎ Π²Ρ‹Π²ΠΎΠ΄Ρ‹ Π½Π° мСталличСский ΠΏΡ€Π΅Π΄ΠΌΠ΅Ρ‚.

ΠŸΠ΅Ρ€Π΅ΠΊΠ»ΡŽΡ‡Π°Ρ‚Π΅Π»ΡŒ ΠΌΡƒΠ»ΡŒΡ‚ΠΈΠΌΠ΅Ρ‚Ρ€Π° устанавливаСм Π² сСкторС измСрСния сопротивлСния (Ρ€Π΅ΠΆΠΈΠΌ ΠΎΠΌΠΌΠ΅Ρ‚Ρ€Π°). Π Π΅ΠΆΠΈΠΌ сопротивлСния даст Π½Π°ΠΌ ΠΏΠΎΠ½ΡΡ‚ΡŒ Π΅ΡΡ‚ΡŒ Π»ΠΈ Π²Π½ΡƒΡ‚Ρ€ΠΈ ΠΊΠΎΠ½Π΄Π΅Ρ€Π° ΠΎΠ±Ρ€Ρ‹Π² ΠΈΠ»ΠΈ ΠΊΠΎΡ€ΠΎΡ‚ΠΊΠΎΠ΅ Π·Π°ΠΌΡ‹ΠΊΠ°Π½ΠΈΠ΅.

ΠŸΡ€ΠΎΠ²Π΅Ρ€ΠΈΠΌ сначала полярныС ΠΊΠΎΠ½Π΄Π΅Ρ€Ρ‹ Π½ΠΎΠΌΠΈΠ½Π°Π»ΠΎΠΌ 5.6 ΠΌΠΊΠ€ ΠΈ 3.3 ΠΌΠΊΠ€ соотвСтствСнно (ΠΎΠ½ΠΈ ΠΌΠ½Π΅ Π΄ΠΎΡΡ‚Π°Π»ΠΈΡΡŒ ΠΎΡ‚ нСисправных ΡΠ½Π΅Ρ€Π³ΠΎΡΠ±Π΅Ρ€Π΅Π³Π°ΡŽΡ‰ΠΈΡ… Π»Π°ΠΌΠΏΠΎΡ‡Π΅ΠΊ).

Π”Ρ€ΡƒΠ·ΡŒΡ Π·Π°Π±Ρ‹Π» ΠΎΡ‚ΠΌΠ΅Ρ‚ΠΈΡ‚ΡŒ, ΠΏΠ΅Ρ€Π΅Π΄ Π²Ρ‹ΠΏΠΎΠ»Π½Π΅Π½ΠΈΠ΅ΠΌ ΠΏΡ€ΠΎΠ²Π΅Ρ€ΠΊΠΈ Π½Π΅ΠΎΠ±Ρ…ΠΎΠ΄ΠΈΠΌΠΎ Ρ€Π°Π·Ρ€ΡΠΆΠ°Ρ‚ΡŒ кондСнсатор. Для этого Π½Π΅ΠΎΠ±Ρ…ΠΎΠ΄ΠΈΠΌΠΎ Π·Π°ΠΊΠΎΡ€ΠΎΡ‚ΠΈΡ‚ΡŒ Π΅Π³ΠΎ Π²Ρ‹Π²ΠΎΠ΄Ρ‹ Π½Π° мСталличСский ΠΏΡ€Π΅Π΄ΠΌΠ΅Ρ‚ (ΠΎΡ‚Π²Π΅Ρ€Ρ‚ΠΊΡƒ, Ρ‰ΡƒΠΏ, ΠΏΡ€ΠΎΠ²ΠΎΠ΄ ΠΈ Ρ‚.ΠΏ.). Π’Π°ΠΊ показания Π±ΡƒΠ΄ΡƒΡ‚ Π±ΠΎΠ»Π΅Π΅ Ρ‚ΠΎΡ‡Π½Ρ‹ΠΌΠΈ.

Для этого выставляСм ΠΏΠ΅Ρ€Π΅ΠΊΠ»ΡŽΡ‡Π°Ρ‚Π΅Π»ΡŒ Π½Π° ΠΎΡ‚ΠΌΠ΅Ρ‚ΠΊΡƒ 2 МОм ΠΈ касаСмся Ρ‰ΡƒΠΏΠ°ΠΌΠΈ Π²Ρ‹Π²ΠΎΠ΄ΠΎΠ² кондСнсатора. Как Ρ‚ΠΎΠ»ΡŒΠΊΠΎ Ρ‰ΡƒΠΏΡ‹ Π±ΡƒΠ΄ΡƒΡ‚ ΠΏΠΎΠ΄ΠΊΠ»ΡŽΡ‡Π΅Π½Ρ‹, Π½Π° дисплСС ΠΌΠΎΠΆΠ½ΠΎ ΡƒΠ²ΠΈΠ΄Π΅Ρ‚ΡŒ ΡΡ‚Ρ€Π΅ΠΌΠΈΡ‚Π΅Π»ΡŒΠ½ΠΎ растущСС сопротивлСниС.

ΠŸΠΎΡ‡Π΅ΠΌΡƒ Ρ‚Π°ΠΊ происходит? ΠŸΠΎΡ‡Π΅ΠΌΡƒ Π½Π° дисплСС ΠΌΠΎΠΆΠ½ΠΎ Π½Π°Π±Π»ΡŽΠ΄Π°Ρ‚ΡŒ Β«ΠΏΠ»Π°Π²Π°ΡŽΡ‰ΠΈΠ΅ значСния сопротивлСния»? ВсС Π΄Π΅Π»ΠΎ Π² Ρ‚ΠΎΠΌ, Ρ‡Ρ‚ΠΎ ΠΏΡ€ΠΈ касании Ρ‰ΡƒΠΏΠ°ΠΌΠΈ Π²Ρ‹Π²ΠΎΠ΄ΠΎΠ² ΠΊ кондСнсатору прикладываСтся постоянноС напряТСниС (Π±Π°Ρ‚Π°Ρ€Π΅ΠΉΠΊΠ° ΠΏΡ€ΠΈΠ±ΠΎΡ€Π°) – ΠΎΠ½ Π½Π°Ρ‡ΠΈΠ½Π°Π΅Ρ‚ Π·Π°Ρ€ΡΠΆΠ°Ρ‚ΡŒΡΡ. Π§Π΅ΠΌ дольшС ΠΌΡ‹ Π΄Π΅Ρ€ΠΆΠΈΠΌ Ρ‰ΡƒΠΏΡ‹, Ρ‚Π΅ΠΌ большС кондСнсатор заряТаСтся, ΠΈ сопротивлСниС ΠΏΠ»Π°Π²Π½ΠΎ увСличиваСтся. Π‘ΠΊΠΎΡ€ΠΎΡΡ‚ΡŒ заряда Π½Π°ΠΏΡ€ΡΠΌΡƒΡŽ зависит ΠΎΡ‚ Смкости. Бпустя врСмя кондСнсатор зарядится ΠΈ Π΅Π³ΠΎ сопротивлСниС Π±ΡƒΠ΄Π΅Ρ‚ Ρ€Π°Π²Π½ΠΎ «бСсконСчности», Π° Π½Π° дисплСС ΠΌΡƒΠ»ΡŒΡ‚ΠΈΠΌΠ΅Ρ‚Ρ€Π° ΠΌΡ‹ ΡƒΠ²ΠΈΠ΄ΠΈΠΌ Β«1Β». Π­Ρ‚ΠΎ ΠΏΠΎΠΊΠ°Π·Π°Ρ‚Π΅Π»ΡŒ Ρ‚ΠΎΠ³ΠΎ Ρ‡Ρ‚ΠΎ кондСнсатор исправСн.

НС всС удаСтся ΠΏΠ΅Ρ€Π΅Π΄Π°Ρ‚ΡŒ фотографиями, Π½ΠΎ для экзСмпляра 5. 6 ΠΌΠΊΠ€ сопротивлСниС стартуСт с 200 кОм ΠΈ ΠΏΠ»Π°Π²Π½ΠΎ растСт, ΠΏΠΎΠΊΠ° Π½Π΅ ΠΏΠ΅Ρ€Π΅Π²Π°Π»ΠΈΡ‚ ΠΎΡ‚ΠΌΠ΅Ρ‚ΠΊΡƒ Π² 2 МОм. Длится вСсь процСсс, ΠΏΡ€ΠΈΠΌΠ΅Ρ€Π½ΠΎ 10 сСк.

Π‘ΠΎ Π²Ρ‚ΠΎΡ€Ρ‹ΠΌ кондСнсатором Π½ΠΎΠΌΠΈΠ½Π°Π»ΠΎΠΌ 3.3 ΠΌΠΊΠ€ происходит всС Π°Π½Π°Π»ΠΎΠ³ΠΈΡ‡Π½ΠΎ. НачинаСт Π·Π°Ρ€ΡΠΆΠ°Ρ‚ΡŒΡΡ, сопротивлСниС растСт, ΠΊΠ°ΠΊ Ρ‚ΠΎΠ»ΡŒΠΊΠΎ показания прСвысят ΠΎΡ‚ΠΌΠ΅Ρ‚ΠΊΡƒ 2 МОм Π½Π° дисплСС ΠΌΠΎΠΆΠ½ΠΎ ΡƒΠ²ΠΈΠ΄Π΅Ρ‚ΡŒ Β«1Β» Ρ‡Ρ‚ΠΎ соотвСтствуСт «бСсконСчности». По Π²Ρ€Π΅ΠΌΠ΅Π½ΠΈ процСсс длится мСньшС, ΠΏΡ€ΠΈΠΌΠ΅Ρ€Π½ΠΎ 5 сСк.

Π’ случаС со Π²Ρ‚ΠΎΡ€ΠΎΠΉ нСполярной ΠΏΠ°Ρ€ΠΎΠΉ кондСнсаторов Π΄Π΅Π»Π°Π΅ΠΌ всС Π°Π½Π°Π»ΠΎΠ³ΠΈΡ‡Π½ΠΎ. КасаСмся Ρ‰ΡƒΠΏΠ°ΠΌΠΈ Π²Ρ‹Π²ΠΎΠ΄ΠΎΠ² ΠΈ наблюдаСм Π·Π° ΠΈΠ·ΠΌΠ΅Π½Π΅Π½ΠΈΠ΅ΠΌ сопротивлСния Π½Π° ΠΏΡ€ΠΈΠ±ΠΎΡ€Π΅.

ΠŸΠ΅Ρ€Π²Ρ‹ΠΉ ΠΈΠ· Π½ΠΈΡ… ΠΊΠΎΠ½Π΄Π΅Ρ€ Β«104К» Π΅Π³ΠΎ сопротивлСниС сначала Π½Π΅ΠΌΠ½ΠΎΠ³ΠΎ сниТаСтся (Π΄ΠΎ 900 кОм) ΠΏΠΎΡ‚ΠΎΠΌ Π½Π°Ρ‡ΠΈΠ½Π°Π΅Ρ‚ ΠΏΠ»Π°Π²Π½ΠΎ расти, ΠΏΠΎΠΊΠ° Π½Π΅ ΠΏΠ΅Ρ€Π΅Π²Π°Π»ΠΈΡ‚ Π·Π° ΠΎΡ‚ΠΌΠ΅Ρ‚ΠΊΡƒ. ЗаряТаСтся дольшС, Ρ‡Π΅ΠΌ ΠΎΡΡ‚Π°Π»ΡŒΠ½Ρ‹Π΅ ΠΎΠΊΠΎΠ»ΠΎ 30 сСк.

Π’Ρ‚ΠΎΡ€ΠΎΠΉ ΠΏΡ€ΠΈΠΌΠ΅Ρ€ ΠΏΡ€ΠΎΠ²Π΅Ρ€ΠΊΠ° кондСнсатора ΠΌΡƒΠ»ΡŒΡ‚ΠΈΠΌΠ΅Ρ‚Ρ€ΠΎΠΌ Ρ‚ΠΈΠΏΠ° ΠœΠ‘Π“Πž Π΅ΠΌΠΊΠΎΡΡ‚ΡŒΡŽ 1 ΠΌΠΊΠ€. На Ρ„ΠΎΡ‚ΠΎ ΠΌΠΎΠΆΠ½ΠΎ Π²ΠΈΠ΄Π΅Ρ‚ΡŒ, ΠΊΠ°ΠΊ измСняСтся сопротивлСниС ΠΏΡ€ΠΈ ΠΏΡ€ΠΎΠ²Π΅Ρ€ΠΊΠ΅. Волько Π² этом случаС ΠΏΠ΅Ρ€Π΅ΠΊΠ»ΡŽΡ‡Π°Ρ‚Π΅Π»ΡŒ Π½ΡƒΠΆΠ½ΠΎ ΡƒΡΡ‚Π°Π½ΠΎΠ²ΠΈΡ‚ΡŒ Π½Π° ΠΎΡ‚ΠΌΠ΅Ρ‚ΠΊΡƒ 20 МОм (сопротивлСниС большоС, Π½Π° 2-ΠΊΠ΅ ΠΎΡ‡Π΅Π½ΡŒ быстро заряТаСтся).

Π‘ΠΏΠ΅Ρ€Π²Π° Π½ΡƒΠΆΠ½ΠΎ ΡΠ½ΡΡ‚ΡŒ заряд, для этого Π·Π°ΠΊΠΎΡ€Π°Ρ‡ΠΈΠ²Π°Π΅ΠΌ Π²Ρ‹Π²ΠΎΠ΄Ρ‹ ΠΎΡ‚Π²Π΅Ρ€Ρ‚ΠΊΠΎΠΉ:

На дисплСС ΠΏΡ€ΠΈΠ±ΠΎΡ€Π° наблюдаСм ΠΊΠ°ΠΊ Π½Π°Ρ‡ΠΈΠ½Π°Π΅Ρ‚ измСнятся сопротивлСниС:Β 

По Ρ€Π΅Π·ΡƒΠ»ΡŒΡ‚Π°Ρ‚Π°ΠΌ Π΄Π°Π½Π½ΠΎΠΉ ΠΏΡ€ΠΎΠ²Π΅Ρ€ΠΊΠΈ ΠΌΠΎΠΆΠ½ΠΎ ΡΠ΄Π΅Π»Π°Ρ‚ΡŒ Π²Ρ‹Π²ΠΎΠ΄, Ρ‡Ρ‚ΠΎ всС Π²Π°Ρ€ΠΈΠ°Π½Ρ‚Ρ‹ кондСнсаторов находятся Π² исправном состоянии.

Как ΠΏΡ€ΠΎΠ²Π΅Ρ€ΠΈΡ‚ΡŒ Π΅ΠΌΠΊΠΎΡΡ‚ΡŒ кондСнсатора ΠΌΡƒΠ»ΡŒΡ‚ΠΈΠΌΠ΅Ρ‚Ρ€ΠΎΠΌ

Одной ΠΈΠ· основных характСристик любого кондСнсатора являСтся Β«Π΅ΠΌΠΊΠΎΡΡ‚ΡŒΒ». Для Ρ‚ΠΎΠ³ΠΎ Ρ‡Ρ‚ΠΎΠ±Ρ‹ ΠΏΠΎΠ½ΡΡ‚ΡŒ Ρ€Π°Π±ΠΎΡ‡ΠΈΠΉ кондСнсатор ΠΈΠ»ΠΈ Π½Π΅Ρ‚ Π½Π΅ΠΎΠ±Ρ…ΠΎΠ΄ΠΈΠΌΠΎ ΠΈΠ·ΠΌΠ΅Ρ€ΠΈΡ‚ΡŒ Π΄Π°Π½Π½ΡƒΡŽ характСристику ΠΈ ΡΡ€Π°Π²Π½ΠΈΡ‚ΡŒ ΠΏΠΎΠΊΠ°Π·Π°Ρ‚Π΅Π»ΠΈ с Ρ‚Π΅ΠΌΠΈ ΠΊΠΎΡ‚ΠΎΡ€Ρ‹Π΅ ΡƒΠΊΠ°Π·Π°Π½Ρ‹ ΠΏΡ€ΠΎΠΈΠ·Π²ΠΎΠ΄ΠΈΡ‚Π΅Π»Π΅ΠΌ Π½Π° корпусС устройства. Если ΠΏΠΎΠ΄ Ρ€ΡƒΠΊΠΎΠΉ Π΅ΡΡ‚ΡŒ Ρ…ΠΎΡ€ΠΎΡˆΠΈΠΉ ΠΏΡ€ΠΈΠ±ΠΎΡ€, Ρ‚ΠΎ ΠΈΠ·ΠΌΠ΅Ρ€ΠΈΡ‚ΡŒ Π΅ΠΌΠΊΠΎΡΡ‚ΡŒ кондСнсатора ΠΌΡƒΠ»ΡŒΡ‚ΠΈΠΌΠ΅Ρ‚Ρ€ΠΎΠΌ Π½Π΅ составит Ρ‚Ρ€ΡƒΠ΄Π°. Но здСсь Π΅ΡΡ‚ΡŒ свои Π½ΡŽΠ°Π½ΡΡ‹.

Если ΠΏΡ‹Ρ‚Π°Ρ‚ΡŒΡΡ ΠΈΠ·ΠΌΠ΅Ρ€ΠΈΡ‚ΡŒ Π΅ΠΌΠΊΠΎΡΡ‚ΡŒ с ΠΏΠΎΠΌΠΎΡ‰ΡŒΡŽ Ρ‰ΡƒΠΏΠΎΠ² (ΠΊΠ°ΠΊ Π² ΠΌΠΎΠ΅ΠΌ случаС с ΠΌΡƒΠ»ΡŒΡ‚ΠΈΠΌΠ΅Ρ‚Ρ€ΠΎΠΌ DT9208A) Ρ‚ΠΎ Ρƒ Вас Π½ΠΈΡ‡Π΅Π³ΠΎ Π½Π΅ получится. Π”Π΅Π»ΠΎ Π² Ρ‚ΠΎΠΌ, Ρ‡Ρ‚ΠΎ Π΅ΠΌΠΊΠΎΡΡ‚ΡŒ нСльзя ΠΏΡ€ΠΎΠ²Π΅Ρ€ΠΈΡ‚ΡŒ, просто ΠΏΠΎΠ΄ΠΊΠ»ΡŽΡ‡ΠΈΠ² Ρ‰ΡƒΠΏΡ‹ ΠΊ кондСнсатору. Π’Π°ΠΊ ΠΊΠ°ΠΊ ΠΏΡ€ΠΎΠ²Π΅Ρ€ΠΈΡ‚ΡŒ Π΅ΠΌΠΊΠΎΡΡ‚ΡŒ кондСнсатора ΠΌΡƒΠ»ΡŒΡ‚ΠΈΠΌΠ΅Ρ‚Ρ€ΠΎΠΌ ΠΈ ΠΌΠΎΠΆΠ½ΠΎ Π»ΠΈ Π²ΠΎΠΎΠ±Ρ‰Π΅ это ΡΠ΄Π΅Π»Π°Ρ‚ΡŒ?

Для этой Ρ†Π΅Π»ΠΈ Π½Π° ΠΌΡƒΠ»ΡŒΡ‚ΠΈΠΌΠ΅Ρ‚Ρ€Π΅ Π΅ΡΡ‚ΡŒ ΡΠΏΠ΅Ρ†ΠΈΠ°Π»ΡŒΠ½Ρ‹Π΅ Ρ€Π°Π·ΡŠΠ΅ΠΌΡ‹ Β«Π³Π½Π΅Π·Π΄Π°Β» -CX+. Β«-Β» ΠΈ Β«+Β» ΠΎΠ·Π½Π°Ρ‡Π°ΡŽΡ‚ ΠΏΠΎΠ»ΡΡ€Π½ΠΎΡΡ‚ΡŒ ΠΏΠΎΠ΄ΠΊΠ»ΡŽΡ‡Π΅Π½ΠΈΡ.

Π”Π°Π²Π°ΠΉΡ‚Π΅ ΠΏΡ€ΠΎΠ²Π΅Ρ€ΠΈΠΌ Π΅ΠΌΠΊΠΎΡΡ‚ΡŒ кСрамичСского ΠΊΠΎΠ½Π΄Π΅Ρ€Π° Β«104К». Напомню, ΠΌΠ°Ρ€ΠΊΠΈΡ€ΠΎΠ²ΠΊΠ° 104 Ρ€Π°ΡΡˆΠΈΡ„Ρ€ΠΎΠ²Ρ‹Π²Π°Π΅Ρ‚ΡΡ: 10 – Π·Π½Π°Ρ‡Π΅Π½ΠΈΠ΅ Π² ΠΏΠ€, 4-количСство Π½ΡƒΠ»Π΅ΠΉ (100000 ΠΏΠ€ = 100 Π½Π€ = 0.1 ΠΌΠΊΠ€).

ВыставляСм ΠΏΠ΅Ρ€Π΅ΠΊΠ»ΡŽΡ‡Π°Ρ‚Π΅Π»ΡŒ ΠΌΡƒΠ»ΡŒΡ‚ΠΈΠΌΠ΅Ρ‚Ρ€Π° Π½Π° Π½Π΅ΠΎΠ±Ρ…ΠΎΠ΄ΠΈΠΌΡƒΡŽ ΠΎΡ‚ΠΌΠ΅Ρ‚ΠΊΡƒ — блиТайшСС большСС Π·Π½Π°Ρ‡Π΅Π½ΠΈΠ΅ (я установил Π½Π° ΠΎΡ‚ΠΌΠ΅Ρ‚ΠΊΠ΅ 200 Π½Π€). Π‘Π΅Ρ€Π΅ΠΌ кондСнсатор ΠΈ вставляСм Π½ΠΎΠΆΠΊΠΈ Π² Ρ€Π°Π·ΡŠΠ΅ΠΌΡ‹ ΠΌΡƒΠ»ΡŒΡ‚ΠΈΠΌΠ΅Ρ‚Ρ€Π° -CX+. Какой стороной Π²ΡΡ‚Π°Π²Π»ΡΡ‚ΡŒ Π½Π΅ Π²Π°ΠΆΠ½ΠΎ, Ρ‚Π°ΠΊ ΠΊΠ°ΠΊ Π΄Π°Π½Π½Ρ‹ΠΉ ΠΊΠΎΠ½Π΄Π΅Ρ€ — нСполярный. На дисплСС ΠΌΡ‹ Π²ΠΈΠ΄ΠΈΠΌ Π·Π½Π°Ρ‡Π΅Π½ΠΈΠ΅ Смкости – 102.6 Π½Π€. Π§Ρ‚ΠΎ соотвСтствуСт Π½ΠΎΠΌΠΈΠ½Π°Π»ΡŒΠ½Ρ‹ΠΌ характСристикам.

Π‘Π»Π΅Π΄ΡƒΡŽΡ‰ΠΈΠΉ экзСмпляр элСктролитичСский кондСнсатор с номинальной Π΅ΠΌΠΊΠΎΡΡ‚ΡŒΡŽ 3.3 ΠΌΠΊΠ€. ΠŸΠ΅Ρ€Π΅ΠΊΠ»ΡŽΡ‡Π°Ρ‚Π΅Π»ΡŒ выставляСм Π½Π° ΠΎΡ‚ΠΌΠ΅Ρ‚ΠΊΠ΅ 20 ΠΌΠΊΠ€. Π’Π΅ΠΏΠ΅Ρ€ΡŒ Π½ΡƒΠΆΠ½ΠΎ ΠΏΡ€Π°Π²ΠΈΠ»ΡŒΠ½ΠΎ Β«Π²ΠΎΡ‚ΠΊΠ½ΡƒΡ‚ΡŒΒ» ΠΊΠΎΠ½Π΄Π΅Ρ€ Π² Ρ€Π°Π·ΡŠΠ΅ΠΌΡ‹ с соблюдСниСм полярности. Для этого Π½ΡƒΠΆΠ½ΠΎ Π·Π½Π°Ρ‚ΡŒ какая Π½ΠΎΠΆΠΊΠ° «плюс», Π° какая «минус». Π£Π·Π½Π°Ρ‚ΡŒ это Π½Π΅ составит Ρ‚Ρ€ΡƒΠ΄Π°, Ρ‚Π°ΠΊ ΠΊΠ°ΠΊ ΠΏΡ€ΠΎΠΈΠ·Π²ΠΎΠ΄ΠΈΡ‚Π΅Π»ΡŒ ΡƒΠΆΠ΅ позаботился ΠΎΠ± этом. Если ΠΏΡ€ΠΈΡΠΌΠΎΡ‚Ρ€Π΅Ρ‚ΡŒΡΡ Π½Π° корпусС Π²ΠΈΠ΄Π½ΠΎ ΡΠΏΠ΅Ρ†ΠΈΠ°Π»ΡŒΠ½Π°Ρ ΠΎΡ‚ΠΌΠ΅Ρ‚ΠΊΠ° — чСрная полоса с ΠΎΠ±ΠΎΠ·Π½Π°Ρ‡Π΅Π½ΠΈΠ΅ΠΌ нуля. Π‘ΠΎ стороны этой Π½ΠΎΠΆΠΊΠΈ располагаСтся «минус», с ΠΏΡ€ΠΎΡ‚ΠΈΠ²ΠΎΠΏΠΎΠ»ΠΎΠΆΠ½ΠΎΠΉ «плюс».

ВставляСм наш кондСнсатор Π² посадочныС Π³Π½Π΅Π·Π΄Π° ΠΌΡƒΠ»ΡŒΡ‚ΠΈΠΌΠ΅Ρ‚Ρ€Π°. На Ρ„ΠΎΡ‚ΠΎ Π²ΠΈΠ΄Π½ΠΎ, Ρ‡Ρ‚ΠΎ Π΅ΠΌΠΊΠΎΡΡ‚ΡŒ Π΄Π°Π½Π½ΠΎΠ³ΠΎ экзСмпляра Ρ€Π°Π²Π½Π° 3.58 ΠΌΠΊΠ€, Ρ‡Ρ‚ΠΎ соотвСтствуСт Π½ΠΎΠΌΠΈΠ½Π°Π»ΡŒΠ½Ρ‹ΠΌ ΠΏΠ°Ρ€Π°ΠΌΠ΅Ρ‚Ρ€Π°ΠΌ. Π’Π°ΠΊΠΈΠΌ простым способом выполняСтся ΠΏΡ€ΠΎΠ²Π΅Ρ€ΠΊΠ° кондСнсатора ΠΌΡƒΠ»ΡŒΡ‚ΠΈΠΌΠ΅Ρ‚Ρ€ΠΎΠΌ.

Π”Ρ€ΡƒΠ³ΠΎΠΉ ΠΏΡ€ΠΈΠΌΠ΅Ρ€ ΠΊΠΎΠ½Π΄Π΅Ρ€ Π΅ΠΌΠΊΠΎΡΡ‚ΡŒΡŽ 5.6 ΠΌΠΊΠ€. ΠŸΡ€ΠΈ ΠΏΡ€ΠΎΠ²Π΅Ρ€ΠΊΠ΅ Π΄Π°Π½Π½Ρ‹ΠΉ экзСмпляр ΠΏΠΎΠΊΠ°Π·Π°Π» Π΅ΠΌΠΊΠΎΡΡ‚ΡŒ 5.9 ΠΌΠΊΠ€, Ρ‡Ρ‚ΠΎ Ρ‚ΠΎΠΆΠ΅ соотвСтствуСт Π½ΠΎΡ€ΠΌΠ΅.

ΠšΠΎΠ½Π΄Π΅Ρ€ ΠœΠ‘Π“Πž, Π΅ΠΌΠΊΠΎΡΡ‚ΡŒΡŽ 1 ΠΌΠΊΠ€ ΠΏΠΎΠΊΠ°Π·Π°Π» Ρ€Π΅Π·ΡƒΠ»ΡŒΡ‚Π°Ρ‚ 1.08, Ρ‡Ρ‚ΠΎ Ρ‚Π°ΠΊΠΆΠ΅ соотвСтствуСт Π½ΠΎΡ€ΠΌΠ΅.

Если ΠΏΡ€ΠΈ Π·Π°ΠΌΠ΅Ρ€Π°Ρ… окаТСтся Ρ‡Ρ‚ΠΎ Π΅ΠΌΠΊΠΎΡΡ‚ΡŒ сильно отличаСтся ΠΎΡ‚ Π½ΠΎΠΌΠΈΠ½Π°Π»ΡŒΠ½Ρ‹Ρ… Π·Π½Π°Ρ‡Π΅Π½ΠΈΠΉ (ΠΈΠ»ΠΈ вовсС Ρ€Π°Π²Π½Π° Π½ΡƒΠ»ΡŽ) это Π·Π½Π°Ρ‡ΠΈΡ‚, Ρ‡Ρ‚ΠΎ кондСнсатор нСисправСн ΠΈ Π΅Π³ΠΎ Π½ΡƒΠΆΠ½ΠΎ Π·Π°ΠΌΠ΅Π½ΠΈΡ‚ΡŒ.

Как ΠΏΡ€ΠΎΠ²Π΅Ρ€ΠΈΡ‚ΡŒ кондСнсатор тСстСром (стрСлочным ΠΏΡ€ΠΈΠ±ΠΎΡ€ΠΎΠΌ)

Π”Ρ€ΡƒΠ·ΡŒΡ завалялся Ρƒ мСня Π² Π³Π°Ρ€Π°ΠΆΠ΅ ΠΈΠ·ΠΌΠ΅Ρ€ΠΈΡ‚Π΅Π»ΡŒΠ½Ρ‹ΠΉ ΠΏΡ€ΠΈΠ±ΠΎΡ€ Π²Ρ€Π΅ΠΌΠ΅Π½ Π‘Π‘Π‘Π  — Π¦4313. Он Π²ΠΏΠΎΠ»Π½Π΅ Ρ€Π°Π±ΠΎΡ‡ΠΈΠΉ, поэтому я Ρ€Π΅ΡˆΠΈΠ» ΠΏΠΎΡΠΊΡΠΏΠ΅Ρ€ΠΈΠΌΠ΅Π½Ρ‚ΠΈΡ€ΠΎΠ²Π°Ρ‚ΡŒ ΠΈ Π²Ρ‹ΠΏΠΎΠ»Π½ΠΈΡ‚ΡŒ ΠΏΡ€ΠΎΠ²Π΅Ρ€ΠΊΡƒ ΠΈΠΌ.

ΠŸΠΎΡ‡Π΅ΠΌΡƒ я Ρ€Π΅ΡˆΠΈΠ» ΠΈΡΠΏΠΎΠ»ΡŒΠ·ΠΎΠ²Π°Ρ‚ΡŒ Π΅Π³ΠΎ? ΠœΠ΅Ρ‚ΠΎΠ΄ΠΈΠΊΠ° ΠΏΡ€ΠΎΠ²Π΅Ρ€ΠΊΠΈ Π½Π΅ измСняСтся Π½ΠΎ, Π°Π½Π°Π»ΠΎΠ³ΠΎΠ²Ρ‹ΠΌΠΈ ΠΏΡ€ΠΈΠ±ΠΎΡ€Π°ΠΌΠΈ (стрСлочными) Ρ€Π°Π±ΠΎΡ‚Ρƒ Π²Ρ‹ΠΏΠΎΠ»Π½ΡΡ‚ΡŒ наглядно ΠΏΡ€ΠΎΡ‰Π΅. ΠŸΡ€ΠΎΡ‰Π΅ Π² ΠΏΠ»Π°Π½Π΅ Π²ΠΈΠ·ΡƒΠ°Π»ΡŒΠ½ΠΎΠ³ΠΎ отслСТивания. Π—Π΄Π΅ΡΡŒ придСтся Π½Π°Π±Π»ΡŽΠ΄Π°Ρ‚ΡŒ Π½Π΅ Π·Π° ΠΈΠ·ΠΌΠ΅Π½Π΅Π½ΠΈΠ΅ΠΌ Ρ†ΠΈΡ„Ρ€ Π½Π° дисплСС, Π° Π·Π° ΠΎΡ‚ΠΊΠ»ΠΎΠ½Π΅Π½ΠΈΠ΅ΠΌ стрСлки ΠΏΡ€ΠΈΠ±ΠΎΡ€Π°. ΠŸΡ€ΠΈΡ‡Π΅ΠΌ стрСлка Π±ΡƒΠ΄Π΅Ρ‚ ΠΎΡ‚ΠΊΠ»ΠΎΠ½ΡΡ‚ΡŒΡΡ сначала Π² ΠΎΠ΄Π½Ρƒ сторону, Π·Π°Ρ‚Π΅ΠΌ Π² Π΄Ρ€ΡƒΠ³ΡƒΡŽ.

Π§Ρ‚ΠΎΠ±Ρ‹ Π½Π°ΡΡ‚Ρ€ΠΎΠΈΡ‚ΡŒ тСстСр Π¦4313 Π½Π° ΠΈΠ·ΠΌΠ΅Ρ€Π΅Π½ΠΈΠ΅ сопротивлСния Π½ΡƒΠΆΠ½ΠΎ Π½Π°ΠΆΠ°Ρ‚ΡŒ ΠΊΠ½ΠΎΠΏΠΊΡƒ Β«rxΒ». ВставляСм Ρ‰ΡƒΠΏΡ‹ ΠΏΡ€ΠΈΠ±ΠΎΡ€Π° Π² Ρ€Π°Π±ΠΎΡ‡ΠΈΠ΅ ΠΊΠΎΠ½Ρ‚Π°ΠΊΡ‚Ρ‹. Для Π½Π°Ρ‡Π°Π»Π° Π±Π΅Ρ€Π΅ΠΌ кондСнсатор ΠΈ разряТаСм Π΅Π³ΠΎ. Π—Π°Ρ‚Π΅ΠΌ касаСмся Ρ‰ΡƒΠΏΠ°ΠΌΠΈ ΠΊΠΎΠ½Ρ‚Π°ΠΊΡ‚ΠΎΠ² ΠΊΠΎΠ½Π΄Π΅Ρ€Π°. Если кондСнсатор исправный стрСлка сначала отклонится, Π° Π·Π°Ρ‚Π΅ΠΌ ΠΏΠΎ ΠΌΠ΅Ρ€Π΅ заряда ΠΏΠ»Π°Π²Π½ΠΎ возвратится Π² исходноС (Π½ΡƒΠ»Π΅Π²ΠΎΠ΅) ΠΏΠΎΠ»ΠΎΠΆΠ΅Π½ΠΈΠ΅. Π‘ΠΊΠΎΡ€ΠΎΡΡ‚ΡŒ пСрСмСщСния стрСлки зависит ΠΎΡ‚ Ρ‚ΠΎΠ³ΠΎ ΠΊΠ°ΠΊΠΎΠΉ Смкости испытуСмый кондСнсатор.

Если стрСлка ΠΏΡ€ΠΈΠ±ΠΎΡ€Π° Π½Π΅ отклоняСтся ΠΈΠ»ΠΈ ΠΎΡ‚ΠΊΠ»ΠΎΠ½ΠΈΠ»Π°ΡΡŒ ΠΈ зависла Π² ΠΎΠΏΡ€Π΅Π΄Π΅Π»Π΅Π½Π½ΠΎΠΌ ΠΏΠΎΠ»ΠΎΠΆΠ΅Π½ΠΈΠΈ, это Π³ΠΎΠ²ΠΎΡ€ΠΈΡ‚ ΠΎ Ρ‚ΠΎΠΌ, Ρ‡Ρ‚ΠΎ кондСнсатор нСисправный.

На этом всС Π΄ΠΎΡ€ΠΎΠ³ΠΈΠ΅ Π΄Ρ€ΡƒΠ·ΡŒΡ, надСюсь, данная ΡΡ‚Π°Ρ‚ΡŒΡ, ΠΊΠ°ΠΊ ΠΏΡ€ΠΎΠ²Π΅Ρ€ΠΈΡ‚ΡŒ кондСнсатор ΠΌΡƒΠ»ΡŒΡ‚ΠΈΠΌΠ΅Ρ‚Ρ€ΠΎΠΌ Ρ†ΠΈΡ„Ρ€ΠΎΠ²Ρ‹ΠΌ ΠΈ стрСлочным Π±Ρ‹Π»Π° для вас интСрСсной ΠΈ раскрыла всС вопросы. Если Ρ‡Ρ‚ΠΎ, Π½Π΅ ΡΡ‚Π΅ΡΠ½ΡΠΉΡ‚Π΅ΡΡŒ ΠΏΠΈΡΠ°Ρ‚ΡŒ ΠΊΠΎΠΌΠΌΠ΅Π½Ρ‚Π°Ρ€ΠΈΠΈ. Π’Π°ΠΊΠΆΠ΅ особая Π±Π»Π°Π³ΠΎΠ΄Π°Ρ€Π½ΠΎΡΡ‚ΡŒ Π·Π° Π Π•ΠŸΠžΠ‘Π’ Π² соц.сСтях.

ΠŸΠΎΡ…ΠΎΠΆΠΈΠ΅ ΠΌΠ°Ρ‚Π΅Ρ€ΠΈΠ°Π»Ρ‹ Π½Π° сайтС:

ΠŸΠΎΠ½Ρ€Π°Π²ΠΈΠ»Π°ΡΡŒ ΡΡ‚Π°Ρ‚ΡŒΡ — подСлись с Π΄Ρ€ΡƒΠ·ΡŒΡΠΌΠΈ!

Β 

Как ΠΏΡ€ΠΎΠ²Π΅Ρ€ΠΈΡ‚ΡŒ кондСнсатор самым простым, Π΄Π΅ΡˆΠ΅Π²Ρ‹ΠΌ ΠΌΡƒΠ»ΡŒΡ‚ΠΈΠΌΠ΅Ρ‚Ρ€ΠΎΠΌ

Как ΠΏΡ€ΠΎΠ²Π΅Ρ€ΠΈΡ‚ΡŒ ΠΎΠ±Ρ‹Ρ‡Π½Ρ‹ΠΌ ΠΌΡƒΠ»ΡŒΡ‚ΠΈΠΌΠ΅Ρ‚Ρ€ΠΎΠΌ ΠΈΡΠΏΡ€Π°Π²Π½ΠΎΡΡ‚ΡŒ кондСнсатора?

Π˜Ρ‚Π°ΠΊ, Ρƒ вас Π΅ΡΡ‚ΡŒ ΠΏΡ€ΠΎΠ±Π»Π΅ΠΌΠ° β€” Π½ΡƒΠΆΠ½ΠΎ ΠΏΡ€ΠΎΠ²Π΅Ρ€ΠΈΡ‚ΡŒ ΠΈΡΠΏΡ€Π°Π²Π½ΠΎΡΡ‚ΡŒ кондСнсатора, Π½ΠΎ подходящСго ΠΈΠ·ΠΌΠ΅Ρ€ΠΈΡ‚Π΅Π»ΡŒΠ½ΠΎΠ³ΠΎ ΠΏΡ€ΠΈΠ±ΠΎΡ€Π° с Ρ„ΡƒΠ½ΠΊΡ†ΠΈΠ΅ΠΉ измСрСния Смкости ΠΏΠΎΠ΄ Ρ€ΡƒΠΊΠΎΠΉ Π½Π΅Ρ‚. Π§Ρ‚ΠΎ ΠΆΠ΅ Π΄Π΅Π»Π°Ρ‚ΡŒ? Π‘Π΅ΠΆΠ°Ρ‚ΡŒ Π² ΠΌΠ°Π³Π°Π·ΠΈΠ½ ΠΈ ΠΊΡƒΠΏΠΈΡ‚ΡŒ Π½ΡƒΠΆΠ½Ρ‹ΠΉ ΠΌΡƒΠ»ΡŒΡ‚ΠΈΠΌΠ΅Ρ‚Ρ€? Если Π²Ρ‹ Π±ΡƒΠ΄Π΅Ρ‚Π΅ постоянно ΠΈΠΌΠ΅Ρ‚ΡŒ Π΄Π΅Π»ΠΎ с ΠΈΠ·ΠΌΠ΅Ρ€Π΅Π½ΠΈΠ΅ΠΌ Смкости ΠΈ ΠΏΡ€ΠΎΠ²Π΅Ρ€ΠΊΠΎΠΉ кондСнсаторов, Ρ‚Π°ΠΊΠΎΠΉ шаг Π±ΡƒΠ΄Π΅Ρ‚ Π±ΠΎΠ»Π΅Π΅ Ρ‡Π΅ΠΌ ΠΎΠΏΡ€Π°Π²Π΄Π°Π½, Π½ΠΎ для Ρ€Π°Π·ΠΎΠ²ΠΎΠΉ, простой ΠΏΡ€ΠΎΠ²Π΅Ρ€ΠΊΠΈ ΠΏΠΎΠ΄ΠΎΠΉΠ΄Π΅Ρ‚ ΠΈ ΠΎΠ±Ρ‹Ρ‡Π½Ρ‹ΠΉ, самый простой ΠΏΡ€ΠΈΠ±ΠΎΡ€.

Π’Π°ΠΊ Ρ‡Ρ‚ΠΎ Π΄Π°Π²Π°ΠΉΡ‚Π΅ ΡƒΠ·Π½Π°Π΅ΠΌ, ΠΊΠ°ΠΊ ΠΌΠΎΠΆΠ½ΠΎ ΠΏΡ€ΠΎΠ²Π΅Ρ€ΠΈΡ‚ΡŒ Ρ€Π°Π±ΠΎΡ‚ΠΎΡΠΏΠΎΡΠΎΠ±Π½ΠΎΡΡ‚ΡŒ кондСнсатора с ΠΏΠΎΠΌΠΎΡ‰ΡŒΡŽ Π΄Π°Π½Π½ΠΎΠ³ΠΎ ΠΈΠ·ΠΌΠ΅Ρ€ΠΈΡ‚Π΅Π»ΡŒΠ½ΠΎΠ³ΠΎ ΠΏΡ€ΠΈΠ±ΠΎΡ€Π°, ΠΊΠΎΡ‚ΠΎΡ€Ρ‹ΠΉ Π²ΠΎΠΎΠ±Ρ‰Π΅ Π½Π΅ ΠΈΠΌΠ΅Π΅Ρ‚ Ρ„ΡƒΠ½ΠΊΡ†ΠΈΠΈ измСрСния Смкости кондСнсаторов. ЕдинствСнный нСдостаток этого способа β€” ΠΈΠ·ΠΌΠ΅Ρ€Π΅Π½ΠΈΠ΅ Смкости кондСнсатора Ρ‚Π°ΠΊΠΈΠΌ способом просто Π½Π΅Π²ΠΎΠ·ΠΌΠΎΠΆΠ½ΠΎ.

Π’Π°ΠΊ Ρ‡Ρ‚ΠΎ ΠΆΠ΅ Π½ΡƒΠΆΠ½ΠΎ Π΄Π΅Π»Π°Ρ‚ΡŒ?

НачнСм ΠΏΡ€ΠΎΠ²Π΅Ρ€ΠΊΡƒ. ΠŸΡ€Π΅Π΄ΡΡ‚Π°Π²ΠΈΠΌ, Ρ‡Ρ‚ΠΎ Π²Ρ‹ ΡƒΠΆΠ΅ Ρ€Π°Π·ΠΎΠ±Ρ€Π°Π»ΠΈ ΠΏΡ€ΠΈΠ±ΠΎΡ€ ΠΈΠ»ΠΈ устройство Π½Π° ΠΊΠΎΡ‚ΠΎΡ€ΠΎΠΌ Π½ΡƒΠΆΠ½ΠΎ ΠΏΡ€ΠΎΠ²Π΅Ρ€ΠΈΡ‚ΡŒ кондСнсаторы, ΠΈΠ»ΠΈ ΠΆΠ΅ ΠΎΠ½ΠΈ ΠΈ вовсС отпаяны. Π‘ послСдними Ρ€Π°Π±ΠΎΡ‚Π°Ρ‚ΡŒ Π±ΡƒΠ΄Π΅Ρ‚ Π΄Π°ΠΆΠ΅ ΠΏΡ€ΠΎΡ‰Π΅. Но Ссли кондСнсаторы Π½ΡƒΠΆΠ½ΠΎ Ρ‚ΠΎΠ»ΡŒΠΊΠΎ ΠΏΡ€ΠΎΠ²Π΅Ρ€ΠΈΡ‚ΡŒ, Π»ΡƒΡ‡ΡˆΠ΅ Π½Π΅ Π²Ρ‹ΠΏΠ°ΠΈΠ²Π°Ρ‚ΡŒ ΠΈΡ… с устройства. ОсобСнно Ссли ΡΠΎΠΌΠ½Π΅Π²Π°Π΅Ρ‚Π΅ΡΡŒ, Ρ‡Ρ‚ΠΎ получится ΠΈΡ… Π²Ρ‹ΠΏΠ°ΡΡ‚ΡŒ ΠΈ ΠΏΡ€ΠΈΠΏΠ°ΡΡ‚ΡŒ Π½Π° мСсто.

  • Π˜Ρ‚Π°ΠΊ, Π²ΠΊΠ»ΡŽΡ‡Π°Π΅ΠΌ ΠΌΡƒΠ»ΡŒΡ‚ΠΈΠΌΠ΅Ρ‚Ρ€ Π² Ρ€Π΅ΠΆΠΈΠΌ измСрСния сопротивлСния. ΠŸΡ€ΠΈ этом выставляСм самый высокий ΠΏΡ€Π΅Π΄Π΅Π».

  • НСваТно, выпаян кондСнсатор ΠΈΠ»ΠΈ находится Π½Π° ΠΏΠ»Π°Ρ‚Π΅ β€” Π³Π»Π°Π²Π½ΠΎΠ΅ ΠΏΠΎΠ΄ΠΊΠ»ΡŽΡ‡ΠΈΡ‚ΡŒ Ρ‰ΡƒΠΏΡ‹ ΠΊ Π²Ρ‹Π²ΠΎΠ΄Π°ΠΌ кондСнсатора. Но Π½Π΅ΠΊΠΎΡ‚ΠΎΡ€Ρ‹Π΅ Ρ€Π°Π΄ΠΈΠΎΠ»ΡŽΠ±ΠΈΡ‚Π΅Π»ΠΈ ΡΠΎΠ²Π΅Ρ‚ΡƒΡŽΡ‚ ΠΎΡ‚ΠΏΠ°ΡΡ‚ΡŒ хотя Π±Ρ‹ ΠΎΠ΄Π½Ρƒ Π½ΠΎΠΆΠΊΡƒ кондСнсатора, Ρ‡Ρ‚ΠΎΠ±Ρ‹ ΡƒΡΡ‚Ρ€Π°Π½ΠΈΡ‚ΡŒ Β«ΠΏΠ°Ρ€Π°Π·ΠΈΡ‚Π½Ρ‹Π΅ ΠΏΠΎΠΌΠ΅Ρ…ΠΈΒ» ΠΏΡ€ΠΎΡ‡ΠΈΡ… ΠΊΠΎΠΌΠΏΠΎΠ½Π΅Π½Ρ‚ΠΎΠ² сСти.

  • Π’Π΅ΠΏΠ΅Ρ€ΡŒ наблюдаСм Π·Π° показаниями. На экранС устройства Π²Ρ‹ ΡƒΠ²ΠΈΠ΄ΠΈΡ‚Π΅, Ρ‡Ρ‚ΠΎ сопротивлСниС кондСнсатора постСпСнно возрастаСт. Если это Ρ‚Π°ΠΊ β€” кондСнсатор исправСн.
Как это Ρ€Π°Π±ΠΎΡ‚Π°Π΅Ρ‚?

Когда кондСнсатор Π½Π°Π±ΠΈΡ€Π°Π΅Ρ‚ заряд Π΅Π³ΠΎ сопротивлСниС, соотвСтствСнно, растСт. Если Π²Ρ‹ Π½Π°Π±Π»ΡŽΠ΄Π°Π΅Ρ‚Π΅ рост сопротивлСния, Π·Π½Π°Ρ‡ΠΈΡ‚, кондСнсатор заряТаСтся. ΠŸΡ€ΠΈ ΠΈΠ·ΠΌΠ΅Ρ€Π΅Π½ΠΈΠΈ сопротивлСния ΠΌΡƒΠ»ΡŒΡ‚ΠΈΠΌΠ΅Ρ‚Ρ€Ρ‹ ΠΏΠΎΠ΄Π°ΡŽΡ‚ Ρ‡Π΅Ρ€Π΅Π· Ρ‰ΡƒΠΏΡ‹ ΠΎΠΏΡ€Π΅Π΄Π΅Π»Π΅Π½Π½ΠΎΠ΅, фиксированноС напряТСниС. ИмСнно ΠΎΠ½ΠΎ ΠΈ заряТаСт кондСнсатор. Если сопротивлСниС остаСтся постоянным β€” кондСнсатор ΠΏΡ€ΠΎΠ±ΠΈΡ‚ ΠΈ Π½Π΅ Π½Π°Π±ΠΈΡ€Π°Π΅Ρ‚ заряд.

Для Ρ‚Π°ΠΊΠΎΠΉ Π²ΠΎΡ‚ ΠΏΡ€ΠΎΠ²Π΅Ρ€ΠΊΠΈ кондСнсатора Π³ΠΎΠ΄ΠΈΡ‚ΡŒΡΡ любая модСль, которая ΠΌΠΎΠΆΠ΅Ρ‚ ΠΈΠ·ΠΌΠ΅Ρ€ΡΡ‚ΡŒ сопротивлСниС. Π­Ρ‚ΠΎ ΠΌΠΎΠΆΠ΅Ρ‚ Π±Ρ‹Ρ‚ΡŒ ΠΊΠ°ΠΊ ΡƒΠ½ΠΈΠ²Π΅Ρ€ΡΠ°Π»ΡŒΠ½Ρ‹ΠΉ Ρ†ΠΈΡ„Ρ€ΠΎΠ²ΠΎΠΉ ΠΏΡ€ΠΈΠ±ΠΎΡ€, Ρ‚Π°ΠΊ ΠΈ простой, Π°Π½Π°Π»ΠΎΠ³ΠΎΠ²Ρ‹ΠΉ ΠΈΠ·ΠΌΠ΅Ρ€ΠΈΡ‚Π΅Π»ΡŒ. Но Π²ΠΎΡ‚ ΡΠ½ΠΈΠΌΠ°Ρ‚ΡŒ Π΄Π°Π½Π½Ρ‹Π΅ простым, Π°Π½Π°Π»ΠΎΠ³ΠΎΠ²Ρ‹ΠΌ инструмСнтом интСрСснСС.

  • Аналоговый ΠΌΡƒΠ»ΡŒΡ‚ΠΈΠΌΠ΅Ρ‚Ρ€ Π΄ΠΎΠ»ΠΆΠ΅Π½ Π±Ρ‹Ρ‚ΡŒ Π²ΠΊΠ»ΡŽΡ‡Π΅Π½ Π² Ρ€Π΅ΠΆΠΈΠΌ измСрСния сопротивлСния. МоТно Π²Ρ‹Π±Ρ€Π°Ρ‚ΡŒ срСдний Π΄ΠΈΠ°ΠΏΠ°Π·ΠΎΠ½.
  • Как ΠΈ Π² случаС с Ρ†ΠΈΡ„Ρ€ΠΎΠ²Ρ‹ΠΌ, Π΄ΠΎΡ‚Ρ€ΠΎΠ½ΡŒΡ‚Π΅ΡΡŒ Ρ‰ΡƒΠΏΠ°ΠΌΠΈ ΠΊ ΠΊΠΎΠ½Ρ‚Π°ΠΊΡ‚Π°ΠΌ кондСнсатора.
  • ΠΠ°Π±Π»ΡŽΠ΄Π°ΠΉΡ‚Π΅ Π·Π° стрСлкой. Она Π±ΡƒΠ΄Π΅Ρ‚ Π΄ΠΎ ΠΎΠΏΡ€Π΅Π΄Π΅Π»Π΅Π½Π½ΠΎΠ³ΠΎ ΠΌΠΎΠΌΠ΅Π½Ρ‚Π° ΠΏΠΎΠ»Π·Ρ‚ΠΈ Π²Π²Π΅Ρ€Ρ…, Π° ΠΏΠΎΡ‚ΠΎΠΌ ΠΏΠ°Π΄Π°Ρ‚ΡŒ Π½Π°Π·Π°Π΄. Если это происходит, Π·Π½Π°Ρ‡ΠΈΡ‚, кондСнсатор заряТаСтся ΠΈ разряТаСтся.
Как Π²ΠΈΠ΄ΠΈΡ‚Π΅, всС достаточно просто!

Π‘Ρ‚ΠΎΠΈΡ‚ Π·Π°ΠΌΠ΅Ρ‚ΠΈΡ‚ΡŒ, Ρ‡Ρ‚ΠΎ ΠΌΡƒΠ»ΡŒΡ‚ΠΈΠΌΠ΅Ρ‚Ρ€Ρ‹ Π½Π΅ смогут ΠΈΠ·ΠΌΠ΅Ρ€ΠΈΡ‚ΡŒ Π΅ΠΌΠΊΠΎΡΡ‚ΡŒ кондСнсатора. Π₯отя Π² Π±ΠΎΠ»ΡŒΡˆΠΈΠ½ΡΡ‚Π²Π΅ случаСв достаточно просто ΠΏΡ€ΠΎΠ²Π΅Ρ€ΠΈΡ‚ΡŒ Ρ€Π°Π±ΠΎΡ‚ΠΎΡΠΏΠΎΡΠΎΠ±Π½ΠΎΡΡ‚ΡŒ ΠΊΠΎΠΌΠΏΠΎΠ½Π΅Π½Ρ‚Π°.

ΠžΠΏΡƒΠ±Π»ΠΈΠΊΠΎΠ²Π°Π½ΠΎ: 2021-09-13 ОбновлСно: 2021-09-13

Автор: Магазин Electronoff

ΠŸΠΎΠ΄Π΅Π»ΠΈΡ‚ΡŒΡΡ Π² соцсСтях

Π’ΠžΠ§ΠΠžΠ• Π˜Π—ΠœΠ•Π Π•ΠΠ˜Π• Π•ΠœΠšΠžΠ‘Π’Π˜ ΠšΠžΠΠ”Π•ΠΠ‘ΠΠ’ΠžΠ Π

БСйчас практичСски ΠΊΠ°ΠΆΠ΄Ρ‹ΠΉ ΡƒΠ½ΠΈΠ²Π΅Ρ€ΡΠ°Π»ΡŒΠ½Ρ‹ΠΉ ΠΌΡƒΠ»ΡŒΡ‚ΠΈΠΌΠ΅Ρ‚Ρ€ ΠΈΠΌΠ΅Π΅Ρ‚ Π²ΠΎΠ·ΠΌΠΎΠΆΠ½ΠΎΡΡ‚ΡŒ измСрСния Смкости кондСнсаторов. Π­Ρ‚ΠΎ особСнно ΠΏΠΎΠ»Π΅Π·Π½ΠΎ, ΠΊΠΎΠ³Π΄Π° ΠΈΠΌΠ΅Π΅ΠΌ Π΄Π΅Π»ΠΎ с кондСнсаторами, ΠΌΠ°Ρ€ΠΊΠΈΡ€ΠΎΠ²ΠΊΠ° ΠΊΠΎΡ‚ΠΎΡ€Ρ‹Ρ… Π½Π΅Ρ‡ΠΈΡ‚Π°Π΅ΠΌΠ° ΠΈΠ»ΠΈ отсутствуСт. Π’ этом случаС достаточно измСрСния с Ρ‚ΠΎΡ‡Π½ΠΎΡΡ‚ΡŒΡŽ Π΄ΠΎ Π½Π΅ΡΠΊΠΎΠ»ΡŒΠΊΠΈΡ… ΠΏΡ€ΠΎΡ†Π΅Π½Ρ‚ΠΎΠ², ΠΏΠΎΡ‚ΠΎΠΌΡƒ Ρ‡Ρ‚ΠΎ Π²ΠΎ-ΠΏΠ΅Ρ€Π²Ρ‹Ρ…, сами кондСнсаторы Π½Π΅ Ρ‚Π°ΠΊ Ρ‚ΠΎΡ‡Π½Ρ‹, Π° Π²ΠΎ-Π²Ρ‚ΠΎΡ€Ρ‹Ρ…, для устройств этого Ρ…Π²Π°Ρ‚Π°Π΅Ρ‚. Но ΠΈΠ½ΠΎΠ³Π΄Π° Π½Π΅ΠΎΠ±Ρ…ΠΎΠ΄ΠΈΠΌΠΎ Π·Π½Π°Ρ‚ΡŒ Ρ‚ΠΎΡ‡Π½ΠΎΠ΅ Π·Π½Π°Ρ‡Π΅Π½ΠΈΠ΅ Смкости кондСнсатора. Π’Π΅Π΄ΡŒ ΠΏΡ€Π΅Ρ†ΠΈΠ·ΠΈΠΎΠ½Π½Ρ‹Π΅ кондСнсаторы труднодоступны ΠΈ довольно Π΄ΠΎΡ€ΠΎΠ³ΠΈ. ΠŸΠΎΡΡ‚ΠΎΠΌΡƒ просто Π±Π΅Ρ€Π΅ΠΌ ΡƒΠΏΠ°ΠΊΠΎΠ²ΠΊΡƒ ΠΎΠ΄ΠΈΠ½Π°ΠΊΠΎΠ²Ρ‹Ρ… ΠΈ ΠΏΠΎΠ΄Π±ΠΈΡ€Π°Π΅ΠΌ подходящий. Π’Π°ΠΊ ΠΊΠ°ΠΊ Ρ‚ΠΎΡ‡Π½ΠΎ измСряСтся Π΅ΠΌΠΊΠΎΡΡ‚ΡŒ кондСнсатора? Π•ΡΡ‚ΡŒ нСсколько способов ΡΠ΄Π΅Π»Π°Ρ‚ΡŒ это.

ΠœΠ΅Ρ‚ΠΎΠ΄ 1: мост Π’ΠΈΠ½Π°

Π­Ρ‚ΠΎ ΠΎΠ΄ΠΈΠ½ ΠΈΠ· ΠΏΠ΅Ρ€Π²Ρ‹Ρ… ΠΌΠ΅Ρ‚ΠΎΠ΄ΠΎΠ² Ρ‚ΠΎΡ‡Π½ΠΎΠ³ΠΎ измСрСния Смкости, ΠΈΠ·ΠΎΠ±Ρ€Π΅Ρ‚Π΅Π½Π½Ρ‹ΠΉ Максом Π’ΠΈΠ½ΠΎΠΌ Π² 1891 Π³ΠΎΠ΄Ρƒ. Π‘ ΠΏΠΎΠΌΠΎΡ‰ΡŒΡŽ моста Π’ΠΈΠ½Π° ΠΌΠΎΠΆΠ½ΠΎ Ρ‚ΠΎΡ‡Π½ΠΎ ΠΈΠ·ΠΌΠ΅Ρ€ΠΈΡ‚ΡŒ ΠΊΠ°ΠΊ Π΅ΠΌΠΊΠΎΡΡ‚ΡŒ, Ρ‚Π°ΠΊ ΠΈ сопротивлСниС. А послС прСобразования Π² мост МаксвСлла Π΅Ρ‰Π΅ ΠΈ ΠΈΠ½Π΄ΡƒΠΊΡ‚ΠΈΠ²Π½ΠΎΡΡ‚ΡŒ. ВсС Π°Π½Π°Π»ΠΎΠ³ΠΎΠ²Ρ‹Π΅ мосты RLC основаны Π½Π° ΠΏΡ€ΠΈΠ½Ρ†ΠΈΠΏΠ΅ этой схСмы.

Π’Ρ…ΠΎΠ΄ Uwe ΠΏΠΎΠ΄ΠΊΠ»ΡŽΡ‡Π΅Π½ ΠΊ Π³Π΅Π½Π΅Ρ€Π°Ρ‚ΠΎΡ€Ρƒ ΡΠΈΠ½ΡƒΡΠΎΠΈΠ΄Π°Π»ΡŒΠ½ΠΎΠΉ Π²ΠΎΠ»Π½Ρ‹ с фиксированной ΠΈΠ»ΠΈ Ρ€Π΅Π³ΡƒΠ»ΠΈΡ€ΡƒΠ΅ΠΌΠΎΠΉ частотой. К Uwy ΠΏΠΎΠ΄ΠΊΠ»ΡŽΡ‡Π΅Π½ Π²ΠΎΠ»ΡŒΡ‚ΠΌΠ΅Ρ‚Ρ€. Rx ΠΈ Cx — искомыС сопротивлСниС ΠΈ Π΅ΠΌΠΊΠΎΡΡ‚ΡŒ. R3 ΠΈ C2 извСстны ΠΈ постоянны. R2 ΠΈ R4 — ΠΏΠΎΡ‚Π΅Π½Ρ†ΠΈΠΎΠΌΠ΅Ρ‚Ρ€Ρ‹, снабТСнныС шкалами, с ΠΊΠΎΡ‚ΠΎΡ€Ρ‹Ρ… ΡΡ‡ΠΈΡ‚Ρ‹Π²Π°ΡŽΡ‚ΡΡ значСния Rx ΠΈ Cx. Π­Ρ‚ΠΈ ΠΏΠΎΡ‚Π΅Π½Ρ†ΠΈΠΎΠΌΠ΅Ρ‚Ρ€Ρ‹ Ρ€Π΅Π³ΡƒΠ»ΠΈΡ€ΡƒΡŽΡ‚ΡΡ Π΄ΠΎ Ρ‚Π΅Ρ… ΠΏΠΎΡ€, ΠΏΠΎΠΊΠ° мост Π½Π΅ Π±ΡƒΠ΄Π΅Ρ‚ сбалансирован ΠΈ Π²ΠΎΠ»ΡŒΡ‚ΠΌΠ΅Ρ‚Ρ€ Π½Π΅ ΠΏΠΎΠΊΠ°ΠΆΠ΅Ρ‚ ноль. Π’ΠΎΠ³Π΄Π° ΡƒΠ΄ΠΎΠ²Π»Π΅Ρ‚Π²ΠΎΡ€ΡΡŽΡ‚ΡΡ Π΄Π²Π΅ зависимости:

Β  Β  Β 

Π’ΠΎΡ‡Π½ΠΎΡΡ‚ΡŒ измСрСния зависит ΠΎΡ‚ ΡΡ‚Π°Π±ΠΈΠ»ΡŒΠ½ΠΎΡΡ‚ΠΈ Π³Π΅Π½Π΅Ρ€Π°Ρ‚ΠΎΡ€Π° ΠΏΠΈΡ‚Π°ΡŽΡ‰Π΅Π³ΠΎ мост, ΠΈ знания Π½ΠΎΠΌΠΈΠ½Π°Π»Π° рСзисторов ΠΈ Смкости C2. Π˜ΡΠΏΠΎΠ»ΡŒΠ·ΡƒΡ извСстныС значСния Rx ΠΈ Cx, Π΅Π³ΠΎ ΠΌΠΎΠΆΠ½ΠΎ ΠΎΡ‚ΠΊΠ°Π»ΠΈΠ±Ρ€ΠΎΠ²Π°Ρ‚ΡŒ.

ΠœΠ΅Ρ‚ΠΎΠ΄ 2: ΠΈΠ·ΠΌΠ΅Ρ€Π΅Π½ΠΈΠ΅ частоты LC-Π³Π΅Π½Π΅Ρ€Π°Ρ‚ΠΎΡ€Π°

Π’ схСмС использован простой LC-Π³Π΅Π½Π΅Ρ€Π°Ρ‚ΠΎΡ€ с ΠΊΠΎΠΌΠΏΠ°Ρ€Π°Ρ‚ΠΎΡ€ΠΎΠΌ. Π’ рСзонансном ΠΊΠΎΠ½Ρ‚ΡƒΡ€Π΅ Ρ€Π°Π±ΠΎΡ‚Π°ΡŽΡ‚ извСстная Π΅ΠΌΠΊΠΎΡΡ‚ΡŒ ΠΈ извСстная ΠΈΠ½Π΄ΡƒΠΊΡ‚ΠΈΠ²Π½ΠΎΡΡ‚ΡŒ. Π”ΠΎΠΏΠΎΠ»Π½ΠΈΡ‚Π΅Π»ΡŒΠ½Π°Ρ, ΠΏΠΎΠ΄ΠΊΠ»ΡŽΡ‡Π°Π΅ΠΌΠ°Ρ ΠΊ Ρ€Π΅Π»Π΅, позволяСт Ρ€Π°ΡΡΡ‡ΠΈΡ‚Π°Ρ‚ΡŒ Ρ‚ΠΎΡ‡Π½Ρ‹Π΅ значСния L ΠΈ C ΠΈΡΠΏΠΎΠ»ΡŒΠ·ΡƒΠ΅ΠΌΡ‹Ρ… ΠΊΠΎΠΌΠΏΠΎΠ½Π΅Π½Ρ‚ΠΎΠ². Π’ΠΎ врСмя измСрСния добавлСнная внСшняя Π΅ΠΌΠΊΠΎΡΡ‚ΡŒ ΠΈΠ»ΠΈ ΠΈΠ½Π΄ΡƒΠΊΡ‚ΠΈΠ²Π½ΠΎΡΡ‚ΡŒ измСняСт частоту ΠΊΠΎΠ»Π΅Π±Π°Π½ΠΈΠΉ Π³Π΅Π½Π΅Ρ€Π°Ρ‚ΠΎΡ€Π° ΠΈ это ΠΈΠ·ΠΌΠ΅Π½Π΅Π½ΠΈΠ΅ позволяСт Ρ€Π°ΡΡΡ‡ΠΈΡ‚Π°Ρ‚ΡŒ ΠΈΠ·ΠΌΠ΅Ρ€Π΅Π½Π½ΠΎΠ΅ Π·Π½Π°Ρ‡Π΅Π½ΠΈΠ΅.

Π­Ρ‚Π° схСма сущСствуСт Π² Π½Π΅ΡΠΊΠΎΠ»ΡŒΠΊΠΈΡ… Π²Π°Ρ€ΠΈΠ°Π½Ρ‚Π°Ρ…, часто с использованиСм встроСнных Π² ΠΌΠΈΠΊΡ€ΠΎΠΊΠΎΠ½Ρ‚Ρ€ΠΎΠ»Π»Π΅Ρ€ ΠΊΠΎΠΌΠΏΠ°Ρ€Π°Ρ‚ΠΎΡ€ΠΎΠ². Π’ΠΎΡ‡Π½ΠΎΡΡ‚ΡŒ расчСтов Π² исходной вСрсии — 0,1%. Π’ΠΎΡ‡Π½ΠΎΡΡ‚ΡŒ ΠΊΠ°Π»ΠΈΠ±Ρ€ΠΎΠ²ΠΊΠΈ зависит ΠΎΡ‚ точности ΠΊΠ°Π»ΠΈΠ±Ρ€ΠΎΠ²ΠΎΡ‡Π½ΠΎΠ³ΠΎ кондСнсатора.

ΠœΠ΅Ρ‚ΠΎΠ΄ 3: ΠΈΠ·ΠΌΠ΅Ρ€Π΅Π½ΠΈΠ΅ ёмкости с ΠΏΠΎΠΌΠΎΡ‰ΡŒΡŽ CTMU

CTMU ΠΈΠ»ΠΈ Π±Π»ΠΎΠΊ измСрСния Π²Ρ€Π΅ΠΌΠ΅Π½ΠΈ зарядки — это ΠΌΠΎΠ΄ΡƒΠ»ΡŒ ΠΈΠΌΠ΅ΡŽΡ‰ΠΈΠΉΡΡ Π²ΠΎ ΠΌΠ½ΠΎΠ³ΠΈΡ… ΠΌΠΈΠΊΡ€ΠΎΠΊΠΎΠ½Ρ‚Ρ€ΠΎΠ»Π»Π΅Ρ€Π°Ρ… PIC, ΠΏΡ€Π΅Π΄Π½Π°Π·Π½Π°Ρ‡Π΅Π½Π½Ρ‹ΠΉ Π² основном для управлСния ΠΊΠ»Π°Π²ΠΈΠ°Ρ‚ΡƒΡ€Π°ΠΌΠΈ ΠΈ сСнсорными интСрфСйсами. ΠœΠΎΠ΄ΡƒΠ»ΡŒ Ρ‚Π°ΠΊΠΆΠ΅ позволяСт Ρ‚ΠΎΡ‡Π½ΠΎ ΠΈΠ·ΠΌΠ΅Ρ€ΡΡ‚ΡŒ Π΅ΠΌΠΊΠΎΡΡ‚ΡŒ, измСряя напряТСниС Π½Π° тСстируСмом кондСнсаторС, ΠΏΠΈΡ‚Π°Π΅ΠΌΠΎΠΌ ΠΎΡ‚ источника Ρ‚ΠΎΠΊΠ° Π² Ρ‚Π΅Ρ‡Π΅Π½ΠΈΠ΅ ΠΎΠΏΡ€Π΅Π΄Π΅Π»Π΅Π½Π½ΠΎΠ³ΠΎ ΠΏΠ΅Ρ€ΠΈΠΎΠ΄Π° Π²Ρ€Π΅ΠΌΠ΅Π½ΠΈ. Π’ основС Ρ€Π°Π±ΠΎΡ‚Ρ‹ систСмы Π»Π΅ΠΆΠΈΡ‚ Ρ„ΠΎΡ€ΠΌΡƒΠ»Π° заряда:

ΠŸΠΎΡΠΊΠΎΠ»ΡŒΠΊΡƒ Π½Π°ΠΌ извСстны Ρ‚ΠΎΠΊ I ΠΈ врСмя t, ΠΈ ΠΌΠΎΠΆΠ΅ΠΌ ΠΈΠ·ΠΌΠ΅Ρ€ΠΈΡ‚ΡŒ напряТСниС V, Ρ‚ΠΎ Ρ‡Ρ‚ΠΎΠ± Π²Ρ‹Ρ‡ΠΈΡΠ»ΠΈΡ‚ΡŒ Π·Π½Π°Ρ‡Π΅Π½ΠΈΠ΅ C. ΠœΠ΅Ρ‚ΠΎΠ΄ Ρ€Π°Π±ΠΎΡ‚Ρ‹ ΠΏΠΎΠΊΠ°Π·Π°Π½ Π½Π° рисункС Π½ΠΈΠΆΠ΅ ΠΈΠ· Π΄ΠΎΠΊΡƒΠΌΠ΅Π½Ρ‚Π°Ρ†ΠΈΠΈ ΠΊ AN1375.

Как ΠΈΠ·ΠΌΠ΅Ρ€ΠΈΡ‚ΡŒ ΠΌΡƒΠ»ΡŒΡ‚ΠΈΠΌΠ΅Ρ‚Ρ€ΠΎΠΌ Ρ‘ΠΌΠΊΠΎΡΡ‚ΡŒ кондСнсатора?

ΠžΡ‚Π²Π΅Ρ‚ мастСра:

Π§Ρ‚ΠΎΠ±Ρ‹ ΠΈΠ·ΠΌΠ΅Ρ€ΠΈΡ‚ΡŒ Ρ‘ΠΌΠΊΠΎΡΡ‚ΡŒ кондСнсатора, ΠΌΠΎΠΆΠ½ΠΎ Π²ΠΎΡΠΏΠΎΠ»ΡŒΠ·ΠΎΠ²Π°Ρ‚ΡŒΡΡ Π»ΡŽΠ±Ρ‹ΠΌ Ρ†ΠΈΡ„Ρ€ΠΎΠ²Ρ‹ΠΌ ΠΌΡƒΠ»ΡŒΡ‚ΠΈΠΌΠ΅Ρ‚Ρ€ΠΎΠΌ. НСкоторыС ΠΈΡ… этих инструмСнтов ΠΌΠΎΠ³ΡƒΡ‚ ΠΈΠ·ΠΌΠ΅Ρ€ΠΈΡ‚ΡŒ Ρ‘ΠΌΠΊΠΎΡΡ‚ΡŒ нСпосрСдствСнно, Π° Π½Π΅ΠΊΠΎΡ‚ΠΎΡ€Ρ‹Π΅ ΠΏΠΎΠ·Π²ΠΎΠ»ΡΡŽΡ‚ это ΡΠ΄Π΅Π»Π°Ρ‚ΡŒ ΠΏΡ€ΠΈ использовании косвСнных ΠΌΠ΅Ρ‚ΠΎΠ΄ΠΎΠ² измСрСния.

УбСдившись, Ρ‡Ρ‚ΠΎ Π² вашСм ΠΌΡƒΠ»ΡŒΡ‚ΠΈΠΌΠ΅Ρ‚Ρ€Π΅ присутствуСт нСобходимая функция измСрСния ёмкости, Π΅Π³ΠΎ слСдуСт ΠΏΠΎΠ΄ΠΊΠ»ΡŽΡ‡ΠΈΡ‚ΡŒ ΠΊ кондСнсатору ΠΈ ΠΏΠ΅Ρ€Π΅ΠΊΠ»ΡŽΡ‡Π°Ρ‚Π΅Π»Π΅ΠΌ Π²Ρ‹Π±Ρ€Π°Ρ‚ΡŒ самый Ρ‚ΠΎΡ‡Π½Ρ‹ΠΉ ΠΏΡ€Π΅Π΄Π΅Π» измСрСния ёмкости. Если Π½Π° ΠΈΠ½Π΄ΠΈΠΊΠ°Ρ‚ΠΎΡ€Π΅ отобразится сообщСниС ΠΎ ΠΏΠ΅Ρ€Π΅Π³Ρ€ΡƒΠ·ΠΊΠ΅, Π½ΡƒΠΆΠ½ΠΎ ΠΏΠ΅Ρ€Π΅ΠΊΠ»ΡŽΡ‡ΠΈΡ‚ΡŒ инструмСнт Π½Π° ΠΌΠ΅Π½Π΅Π΅ Ρ‚ΠΎΡ‡Π½Ρ‹ΠΉ ΠΏΡ€Π΅Π΄Π΅Π». Π‘ΠΎΠ²Π΅Ρ€ΡˆΠ°ΠΉΡ‚Π΅ эти манипуляции Π΄ΠΎ Ρ‚ΠΎΠ³ΠΎ ΠΌΠΎΠΌΠ΅Π½Ρ‚Π°, ΠΏΠΎΠΊΠ° ΠΏΡ€ΠΈΠ±ΠΎΡ€ Π½Π΅ выдаст показания.

Π’ случаС, ΠΊΠΎΠ³Π΄Π° для измСрСния ёмкости ΠΈΡΠΏΠΎΠ»ΡŒΠ·ΡƒΠ΅Ρ‚ΡΡ мостовая приставка, слСдуСт Ρ€Π°Π±ΠΎΡ‚Π°Ρ‚ΡŒ с ΠΌΡƒΠ»ΡŒΡ‚ΠΈΠΌΠ΅Ρ‚Ρ€ΠΎΠΌ, ΠΊΠ°ΠΊ с устройством для опрСдСлСния баланса моста. ΠŸΠΎΠ΄ΠΊΠ»ΡŽΡ‡ΠΈΡ‚Π΅ Π΅Π³ΠΎ Ρ‡Π΅Ρ€Π΅Π· Π΄Π΅Ρ‚Π΅ΠΊΡ‚ΠΎΡ€ с Ρ„ΠΈΠ»ΡŒΡ‚Ρ€ΡƒΡŽΡ‰ΠΈΠΌ кондСнсатором ΠΊ Π²Ρ‹Π²ΠΎΠ΄Π°ΠΌ моста. УстановитС Π½Π° ΠΏΡ€ΠΈΠ±ΠΎΡ€Π΅ Ρ€Π΅ΠΆΠΈΠΌ ΠΌΠΈΠΊΡ€ΠΎΠ°ΠΌΠΏΠ΅Ρ€ΠΌΠ΅Ρ‚Ρ€Π° постоянного Ρ‚ΠΎΠΊΠ°. Π’Π΅ΠΏΠ΅Ρ€ΡŒ ΠΏΠΎΠ΄ΠΊΠ»ΡŽΡ‡ΠΈΡ‚Π΅ кондСнсатор ΠΊ мосту, сбалансируйтС послСдний Π΄ΠΎ ΠΌΠΈΠ½ΠΈΠΌΡƒΠΌΠ° ΠΏΠΎΠΊΠ°Π·Π°Π½ΠΈΠΉ. ΠŸΡ€ΠΎΡ‡Ρ‚ΠΈΡ‚Π΅ ΠΏΠΎΠ»ΡƒΡ‡Π΅Π½Π½Ρ‹Π΅ значСния ΠΏΠΎ шкалС моста.

Если Π² вашСм ΠΌΡƒΠ»ΡŒΡ‚ΠΈΠΌΠ΅Ρ‚Ρ€Π΅ Π½Π΅Ρ‚ возмоТности ΠΈΠ·ΠΌΠ΅Ρ€ΡΡ‚ΡŒ Ρ‘ΠΌΠΊΠΎΡΡ‚ΡŒ, ΠΈ Π½Π΅Ρ‚ мостовой приставки, Ρ‚ΠΎ слСдуСт ΠΈΡΠΏΠΎΠ»ΡŒΠ·ΠΎΠ²Π°Ρ‚ΡŒ ΡΠ»Π΅Π΄ΡƒΡŽΡ‰ΠΈΠΉ ΠΌΠ΅Ρ‚ΠΎΠ΄. Π’Π°ΠΌ понадобится Π³Π΅Π½Π΅Ρ€Π°Ρ‚ΠΎΡ€ стандартных сигналов, Π½Π° ΠΊΠΎΡ‚ΠΎΡ€ΠΎΠΌ Π½ΡƒΠΆΠ½ΠΎ ΡƒΡΡ‚Π°Π½ΠΎΠ²ΠΈΡ‚ΡŒ ΠΈΠ·Π²Π΅ΡΡ‚Π½ΡƒΡŽ Π°ΠΌΠΏΠ»ΠΈΡ‚ΡƒΠ΄Ρƒ сигнала, которая Ρ€Π°Π²Π½Π° нСскольким Π²ΠΎΠ»ΡŒΡ‚Π°ΠΌ. Π—Π°Ρ‚Π΅ΠΌ ΠΏΠΎΡΠ»Π΅Π΄ΠΎΠ²Π°Ρ‚Π΅Π»ΡŒΠ½ΠΎ Π²ΠΊΠ»ΡŽΡ‡Π°ΠΉΡ‚Π΅ ΠΌΡƒΠ»ΡŒΡ‚ΠΈΠΌΠ΅Ρ‚Ρ€ (ΠΊΠΎΡ‚ΠΎΡ€Ρ‹ΠΉ Π² зависимости ΠΎΡ‚ условий измСрСния Ρ€Π°Π±ΠΎΡ‚Π°Π΅Ρ‚ ΠΊΠ°ΠΊ ΠΌΠΈΠΊΡ€ΠΎΠ°ΠΌΠΏΠ΅Ρ€ΠΌΠ΅Ρ‚Ρ€ ΠΈΠ»ΠΈ ΠΌΠΈΠ»Π»ΠΈΠ°ΠΌΠΏΠ΅Ρ€ΠΌΠ΅Ρ‚Ρ€ ΠΏΠ΅Ρ€Π΅ΠΌΠ΅Π½Π½ΠΎΠ³ΠΎ Ρ‚ΠΎΠΊΠ°), Π³Π΅Π½Π΅Ρ€Π°Ρ‚ΠΎΡ€ ΠΈ кондСнсатор, ΠΎΠ±ΡŠΡ‘ΠΌ ΠΊΠΎΡ‚ΠΎΡ€ΠΎΠ³ΠΎ Π½Π΅ΠΎΠ±Ρ…ΠΎΠ΄ΠΈΠΌΠΎ ΠΈΠ·ΠΌΠ΅Ρ€ΠΈΡ‚ΡŒ.

УстановитС частоту, ΠΏΡ€ΠΈ ΠΊΠΎΡ‚ΠΎΡ€ΠΎΠΉ ΠΌΡƒΠ»ΡŒΡ‚ΠΈΠΌΠ΅Ρ‚Ρ€ ΠΏΠΎΠΊΠ°ΠΆΠ΅Ρ‚ Ρ‚ΠΎΠΊ, Π½Π΅ ΠΏΡ€Π΅Π²Ρ‹ΡˆΠ°ΡŽΡ‰ΠΈΠΉ Π² ΠΏΠ΅Ρ€Π²ΠΎΠΌ случаС 200 мкА, Π° Π²ΠΎ Π²Ρ‚ΠΎΡ€ΠΎΠΌ – 2 мА. ΠŸΡ€ΠΈ слишком ΠΌΠ°Π»ΠΎΠΉ частотС ΠΏΡ€ΠΈΠ±ΠΎΡ€ Π½ΠΈΡ‡Π΅Π³ΠΎ Π½Π΅ ΠΏΠΎΠΊΠ°ΠΆΠ΅Ρ‚. Π”Π°Π»Π΅Π΅ слСдуСт ΠΏΠΎΠ΄Π΅Π»ΠΈΡ‚ΡŒ Π°ΠΌΠΏΠ»ΠΈΡ‚ΡƒΠ΄Π½ΠΎΠ΅ Π·Π½Π°Ρ‡Π΅Π½ΠΈΠ΅ напряТСния, Π²Ρ‹Ρ€Π°ΠΆΠ΅Π½Π½ΠΎΠ³ΠΎ Π² Π²ΠΎΠ»ΡŒΡ‚Π°Ρ…, Π½Π° ΠΊΠ²Π°Π΄Ρ€Π°Ρ‚Π½Ρ‹ΠΉ ΠΊΠΎΡ€Π΅Π½ΡŒ ΠΈΠ· Π΄Π²ΡƒΡ…. Π’Π°ΠΊΠΈΠΌ ΠΎΠ±Ρ€Π°Π·ΠΎΠΌ ΠΏΠΎΠ»ΡƒΡ‡Π°Π΅ΠΌ Π΅Π³ΠΎ Π΄Π΅ΠΉΡΡ‚Π²ΡƒΡŽΡ‰Π΅Π΅ Π·Π½Π°Ρ‡Π΅Π½ΠΈΠ΅. ΠŸΠ΅Ρ€Π΅Π²Π΅Π΄ΠΈΡ‚Π΅ Ρ‚ΠΎΠΊ Π² Π°ΠΌΠΏΠ΅Ρ€Ρ‹, ΠΏΠΎΠ΄Π΅Π»ΠΈΡ‚Π΅ напряТСниС Π½Π° Ρ‚ΠΎΠΊ. ΠŸΠΎΠ»ΡƒΡ‡Π΅Π½Π½ΠΎΠ΅ Π·Π½Π°Ρ‡Π΅Π½ΠΈΠ΅ – ёмкостноС сопротивлСниС кондСнсатора Π² ΠΎΠΌΠ°Ρ…. Π˜ΡΠΏΠΎΠ»ΡŒΠ·ΡƒΠΉΡ‚Π΅ Π·Π½Π°Ρ‡Π΅Π½ΠΈΠ΅ частоты ΠΈ ёмкостного сопротивлСниС Π² Ρ„ΠΎΡ€ΠΌΡƒΠ»Π΅ для вычислСния ёмкости:

УстановитС Ρ‚Π°ΠΊΡƒΡŽ частоту, Ρ‡Ρ‚ΠΎΠ±Ρ‹ ΠΌΡƒΠ»ΡŒΡ‚ΠΈΠΌΠ΅Ρ‚Ρ€ ΠΏΠΎΠΊΠ°Π·Π°Π» Ρ‚ΠΎΠΊ, Π½Π΅ ΠΏΡ€Π΅Π²Ρ‹ΡˆΠ°ΡŽΡ‰ΠΈΠΉ Π² ΠΏΠ΅Ρ€Π²ΠΎΠΌ случаС 200 мкА, Π° Π²ΠΎ Π²Ρ‚ΠΎΡ€ΠΎΠΌ — 2 мА (Ссли частота слишком ΠΌΠ°Π»Π°, ΠΎΠ½ Π½Π΅ ΠΏΠΎΠΊΠ°ΠΆΠ΅Ρ‚ Π½ΠΈΡ‡Π΅Π³ΠΎ). Π—Π°Ρ‚Π΅ΠΌ ΠΏΠΎΠ΄Π΅Π»ΠΈΡ‚Π΅ Π°ΠΌΠΏΠ»ΠΈΡ‚ΡƒΠ΄Π½ΠΎΠ΅ Π·Π½Π°Ρ‡Π΅Π½ΠΈΠ΅ напряТСния, Π²Ρ‹Ρ€Π°ΠΆΠ΅Π½Π½ΠΎΠ³ΠΎ Π² Π²ΠΎΠ»ΡŒΡ‚Π°Ρ…, Π½Π° ΠΊΠ²Π°Π΄Ρ€Π°Ρ‚Π½Ρ‹ΠΉ ΠΊΠΎΡ€Π΅Π½ΡŒ ΠΈΠ· Π΄Π²ΡƒΡ…, Ρ‡Ρ‚ΠΎΠ±Ρ‹ ΠΏΠΎΠ»ΡƒΡ‡ΠΈΡ‚ΡŒ Π΄Π΅ΠΉΡΡ‚Π²ΡƒΡŽΡ‰Π΅Π΅ Π΅Π³ΠΎ Π·Π½Π°Ρ‡Π΅Π½ΠΈΠ΅. Π’ΠΎΠΊ ΠΏΠ΅Ρ€Π΅Π²Π΅Π΄ΠΈΡ‚Π΅ Π² Π°ΠΌΠΏΠ΅Ρ€Ρ‹, послС Ρ‡Π΅Π³ΠΎ ΠΏΠΎΠ΄Π΅Π»ΠΈΡ‚Π΅ напряТСниС Π½Π° Ρ‚ΠΎΠΊ, ΠΈ Π²Ρ‹ ΠΏΠΎΠ»ΡƒΡ‡ΠΈΡ‚Π΅ СмкостноС сопротивлСниС кондСнсатора, Π²Ρ‹Ρ€Π°ΠΆΠ΅Π½Π½ΠΎΠ΅ Π² ΠΎΠΌΠ°Ρ…. Π—Π°Ρ‚Π΅ΠΌ, зная частоту ΠΈ СмкостноС сопротивлСниС, вычислитС Π΅ΠΌΠΊΠΎΡΡ‚ΡŒ ΠΏΠΎ Ρ„ΠΎΡ€ΠΌΡƒΠ»Π΅: C=1/(2Ο€fR), Π³Π΄Π΅ C — Π΅ΠΌΠΊΠΎΡΡ‚ΡŒ Π² Ρ„Π°Ρ€Π°Π΄Π°Ρ…, Ο€ — матСматичСская константа Β«ΠΏΠΈΒ», f — частота Π² Π³Π΅Ρ€Ρ†Π°Ρ…, R — СмкостноС сопротивлСниС Π² ΠΎΠΌΠ°Ρ….

ВычислСнноС Π·Π½Π°Ρ‡Π΅Π½ΠΈΠ΅ ёмкости ΠΏΠ΅Ρ€Π΅Π²Π΅Π΄ΠΈΡ‚Π΅ Π² Π±ΠΎΠ»Π΅Π΅ ΡƒΠ΄ΠΎΠ±Π½Ρ‹Π΅ Π΅Π΄ΠΈΠ½ΠΈΡ†Ρ‹ измСрСния: ΠΏΠΈΠΊΠΎΡ„Π°Ρ€Π°Π΄Ρ‹, Π½Π°Π½ΠΎΡ„Π°Ρ€Π°Π΄Ρ‹ ΠΈΠ»ΠΈ ΠΌΠΈΠΊΡ€ΠΎΡ„Π°Ρ€Π°Π΄Ρ‹.

ΠŸΠΎΠΌΠ½ΠΈΡ‚Π΅, Ρ‡Ρ‚ΠΎ Ρ‚Π°ΠΊΠΎΠΉ ΠΌΠ΅Ρ‚ΠΎΠ΄ нСльзя ΠΏΡ€ΠΈΠΌΠ΅Π½ΡΡ‚ΡŒ для Π·Π°ΠΌΠ΅Ρ€Π° ёмкости оксидных кондСнсаторов.

ΠŸΠ΅Ρ€Π΅Π΄ Π΅Π³ΠΎ ΠΈΠ·ΠΌΠ΅Ρ€Π΅Π½ΠΈΠ΅ΠΌ кондСнсатор Π½ΡƒΠΆΠ½ΠΎ Ρ€Π°Π·Ρ€ΡΠ΄ΠΈΡ‚ΡŒ, ΠΈΡΠΏΠΎΠ»ΡŒΠ·ΡƒΡ бСзопасный способ.

Π’ Ρ‡Ρ‘ΠΌ измСряСтся Ρ‘ΠΌΠΊΠΎΡΡ‚ΡŒ кондСнсатора: ΠΊΠ°ΠΊ ΠΈΠ·ΠΌΠ΅Ρ€ΠΈΡ‚ΡŒ

ΠšΠΎΠ½Π΄Π΅Π½ΡΠ°Ρ‚ΠΎΡ€Ρ‹ ΡΠ²Π»ΡΡŽΡ‚ΡΡ ваТнСйшими пассивными ΠΊΠΎΠΌΠΏΠΎΠ½Π΅Π½Ρ‚Π°ΠΌΠΈ элСктричСских Ρ†Π΅ΠΏΠ΅ΠΉ. Π›ΡŽΠ±Π°Ρ элСктричСская схСма содСрТит в своСм составС Ρ‚Π°ΠΊΠΈΠ΅ элСмСнты Ρ€Π°Π·Π»ΠΈΡ‡Π½Ρ‹Ρ… Ρ‚ΠΈΠΏΠΎΠ² ΠΈΒ Π½ΠΎΠΌΠΈΠ½Π°Π»ΠΎΠ².

Π§Ρ‚ΠΎ это Ρ‚Π°ΠΊΠΎΠ΅

ΠšΠΎΠ½Π΄Π΅Π½ΡΠ°Ρ‚ΠΎΡ€Β β€” элСктричСский Π΄Π²ΡƒΡ…ΠΏΠΎΠ»ΡŽΡΠ½ΠΈΠΊ (элСмСнт с двумя Π²Ρ‹Π²ΠΎΠ΄Π°ΠΌΠΈ) с постоянным ΠΈΠ»ΠΈ измСняСмым Π·Π½Π°Ρ‡Π΅Π½ΠΈΠ΅ΠΌ Смкости. ΠžΠ±Π»Π°Π΄Π°Π΅Ρ‚ бСсконСчно большим сопротивлСниСм постоянному Ρ‚ΠΎΠΊΡƒ.

ΠŸΡ€ΠΎΡΡ‚Π΅ΠΉΡˆΠΈΠΉ кондСнсатор

Π’Π°ΠΆΠ½ΠΎ! БСсконСчно большим сопротивлСниСм ΠΎΠ±Π»Π°Π΄Π°Π΅Ρ‚ ΠΈΠ΄Π΅Π°Π»ΡŒΠ½Ρ‹ΠΉ кондСнсатор. Π Π΅Π°Π»ΡŒΠ½Ρ‹Π΅ устройства ΠΈΠΌΠ΅ΡŽΡ‚ Ρ‚ΠΎΠΊ ΡƒΡ‚Π΅Ρ‡ΠΊΠΈ, ΠΊΠΎΡ‚ΠΎΡ€Ρ‹ΠΉ Π½Π΅ΠΎΠ±Ρ…ΠΎΠ΄ΠΈΠΌΠΎ ΡƒΡ‡ΠΈΡ‚Ρ‹Π²Π°Ρ‚ΡŒ.

ОсновноС Π½Π°Π·Π½Π°Ρ‡Π΅Π½ΠΈΠ΅ устройства — Π½Π°ΠΊΠΎΠΏΠ»Π΅Π½ΠΈΠ΅ энСргии элСктричСского поля и заряда.

НСсмотря Π½Π°Β Ρ‚ΠΎ, Ρ‡Ρ‚ΠΎ кондСнсаторы ΡΠ²Π»ΡΡŽΡ‚ΡΡ ΡΠ°ΠΌΠΎΡΡ‚ΠΎΡΡ‚Π΅Π»ΡŒΠ½Ρ‹ΠΌΠΈ элСмСнтами, Π΅ΠΌΠΊΠΎΡΡ‚ΡŒΡŽ ΠΎΠ±Π»Π°Π΄Π°ΡŽΡ‚ Π»ΡŽΠ±Ρ‹Π΅ Π΄Ρ€ΡƒΠ³ΠΈΠ΅ устройства, Π΄Π°ΠΆΠ΅ Π΄ΠΈΠΎΠ΄ и транзистор.

Π₯арактСристики

Как элСмСнт элСктричСской Ρ†Π΅ΠΏΠΈ, кондСнсатор ΠΈΠΌΠ΅Π΅Ρ‚ Ρ‚Π°ΠΊΠΈΠ΅ ΠΏΠ°Ρ€Π°ΠΌΠ΅Ρ‚Ρ€Ρ‹:

  • ЭлСктричСская Π΅ΠΌΠΊΠΎΡΡ‚ΡŒ, которая характСризуСтся свойством накапливания элСктричСского заряда.
  • НоминальноС напряТСниС. Π—Π½Π°Ρ‡Π΅Π½ΠΈΠ΅ напряТСния Π½Π° ΠΎΠ±ΠΊΠ»Π°Π΄ΠΊΠ°Ρ…, ΠΏΡ€ΠΈ ΠΊΠΎΡ‚ΠΎΡ€ΠΎΠΌ элСмСнт Π² Ρ‚Π΅Ρ‡Π΅Π½ΠΈΠΈ срока слуТбы сохраняСт свои ΠΏΠ°Ρ€Π°ΠΌΠ΅Ρ‚Ρ€Ρ‹.

ΠŸΡ€ΠΈ Ρ€Π°Π±ΠΎΡ‚Π΅ с элСктричСскими цСпями Π½Π΅ΠΎΠ±Ρ…ΠΎΠ΄ΠΈΠΌΠΎ ΡƒΡ‡ΠΈΡ‚Ρ‹Π²Π°Ρ‚ΡŒ ΠΏΠ°Ρ€Π°Π·ΠΈΡ‚Π½Ρ‹Π΅ ΠΏΠ°Ρ€Π°ΠΌΠ΅Ρ‚Ρ€Ρ‹, ΠΊΠΎΡ‚ΠΎΡ€Ρ‹Π΅ ΡΠ²Π»ΡΡŽΡ‚ΡΡ Π½Π΅ΠΆΠ΅Π»Π°Ρ‚Π΅Π»ΡŒΠ½Ρ‹ΠΌΠΈ:

  • Π’ΠΎΠΊ ΡƒΡ‚Π΅Ρ‡ΠΊΠΈ, ΠΊΠΎΡ‚ΠΎΡ€Ρ‹ΠΉ появляСтся ΠΈΠ·-Π·Π° Π½Π΅ΡΠΎΠ²Π΅Ρ€ΡˆΠ΅Π½ΡΡ‚Π²Π° диэлСктрика, качСства изоляции ΠΎΠ±ΠΊΠ»Π°Π΄ΠΎΠΊ.
  • ΠŸΠΎΡΠ»Π΅Π΄ΠΎΠ²Π°Ρ‚Π΅Π»ΡŒΠ½ΠΎΠ΅ эквивалСнтноС сопротивлСниС, ΠΊΠΎΡ‚ΠΎΡ€ΠΎΠ΅ складываСтся ΠΈΠ· сопротивлСния Π²Ρ‹Π²ΠΎΠ΄ΠΎΠ², сопротивлСниС ΠΊΠΎΠ½Ρ‚Π°ΠΊΡ‚Π° Π²Ρ‹Π²ΠΎΠ΄-ΠΎΠ±ΠΊΠ»Π°Π΄ΠΊΠ°, Π²Π½ΡƒΡ‚Ρ€Π΅Π½Π½ΠΈΡ… свойств диэлСктрика.
  • ЭквивалСнтная ΠΈΠ½Π΄ΡƒΠΊΡ‚ΠΈΠ²Π½ΠΎΡΡ‚ΡŒ, Π² ΠΊΠΎΡ‚ΠΎΡ€ΡƒΡŽ входят ΠΈΠ½Π΄ΡƒΠΊΡ‚ΠΈΠ²Π½ΠΎΡΡ‚ΡŒ Π²Ρ‹Π²ΠΎΠ΄ΠΎΠ² ΠΈ ΠΎΠ±ΠΊΠ»Π°Π΄ΠΎΠΊ.
  • ВангСнс ΡƒΠ³Π»Π° диэлСктричСских ΠΏΠΎΡ‚Π΅Ρ€ΡŒ, Ρ…Π°Ρ€Π°ΠΊΡ‚Π΅Ρ€ΠΈΠ·ΡƒΡŽΡ‰ΠΈΠΉ элСктричСскиС ΠΏΠΎΡ‚Π΅Ρ€ΠΈ Π² кондСнсаторС Π½Π° высоких частотах.
  • Π’Π΅ΠΌΠΏΠ΅Ρ€Π°Ρ‚ΡƒΡ€Π½Ρ‹ΠΉ коэффициСнт Смкости, ΠΏΠΎΠΊΠ°Π·Ρ‹Π²Π°ΡŽΡ‰ΠΈΠΉ, ΠΊΠ°ΠΊ ΠΎΠ½Π° мСняСтся Π² зависимости ΠΎΡ‚ Ρ‚Π΅ΠΌΠΏΠ΅Ρ€Π°Ρ‚ΡƒΡ€Ρ‹.
  • ΠŸΠ°Ρ€Π°Π·ΠΈΡ‚Π½Ρ‹ΠΉ ΠΏΡŒΠ΅Π·ΠΎΡΡ„Ρ„Π΅ΠΊΡ‚, ΠΏΡ€ΠΎΡΠ²Π»ΡΡŽΡ‰ΠΈΠΉΡΡ ΠΊΠ°ΠΊ гСнСрация напряТСния ΠΏΡ€ΠΈ физичСском воздСйствии Π½Π° диэлСктрик (тряска, вибрация).
ЭквивалСнтная схСма

Устройство кондСнсатора

ΠŸΡ€ΠΎΡΡ‚Π΅ΠΉΡˆΠΈΠΉ кондСнсатор состоит ΠΈΠ·Β Π΄Π²ΡƒΡ… мСталличСских пластин (ΠΎΠ±ΠΊΠ»Π°Π΄ΠΎΠΊ), Ρ€Π°Π·Π΄Π΅Π»Π΅Π½Π½Ρ‹Ρ… слоСм диэлСктрика. Π•ΠΌΠΊΠΎΡΡ‚ΡŒ (ΡΠΏΠΎΡΠΎΠ±Π½ΠΎΡΡ‚ΡŒ Π½Π°ΠΊΠ°ΠΏΠ»ΠΈΠ²Π°Ρ‚ΡŒ элСктричСский заряд) увСличиваСтся с ростом ΠΏΠ»ΠΎΡ‰Π°Π΄ΠΈ пластин ΠΈΒ ΡΒ ΡƒΠΌΠ΅Π½ΡŒΡˆΠ΅Π½ΠΈΠ΅ΠΌ Ρ‚ΠΎΠ»Ρ‰ΠΈΠ½Ρ‹ ΠΈΠ·ΠΎΠ»ΠΈΡ€ΡƒΡŽΡ‰Π΅Π³ΠΎ слоя.

ΠŸΠ°Ρ€Π°ΠΌΠ΅Ρ‚Ρ€Ρ‹ ΠΏΡ€ΠΎΡΡ‚Π΅ΠΉΡˆΠ΅ΠΉ конструкции слишком ΠΌΠ°Π»Ρ‹. Для СС увСличСния Π΅ΡΡ‚ΡŒ Π΄Π²Π° ΠΏΡƒΡ‚ΠΈ:

  • Π£Π²Π΅Π»ΠΈΡ‡Π΅Π½ΠΈΠ΅ ΠΏΠ»ΠΎΡ‰Π°Π΄ΠΈ ΠΎΠ±ΠΊΠ»Π°Π΄ΠΎΠΊ, Ρ‡Ρ‚ΠΎ ΠΏΡ€ΠΈΠ²ΠΎΠ΄ΠΈΡ‚ ΠΊ ΡƒΠ²Π΅Π»ΠΈΡ‡Π΅Π½ΠΈΡŽ Π³Π°Π±Π°Ρ€ΠΈΡ‚ΠΎΠ².
  • УмСньшСниС Ρ‚ΠΎΠ»Ρ‰ΠΈΠ½Ρ‹ диэлСктрика, приводящСС ΠΊ сниТСнию номинального Ρ€Π°Π±ΠΎΡ‡Π΅Π³ΠΎ напряТСния ΠΈΠ·-Π·Π° элСктричСского пробоя.

Для Ρ‚ΠΎΠ³ΠΎ, Ρ‡Ρ‚ΠΎΠ±Ρ‹ ΠΈΠ·Π±Π΅ΠΆΠ°Ρ‚ΡŒ пСрСчислСнных ΠΏΡ€ΠΎΠ±Π»Π΅ΠΌ, Ρ€Π°Π·Ρ€Π°Π±ΠΎΡ‚Π°Π½Ρ‹ ΡΠΏΠ΅Ρ†ΠΈΠ°Π»ΡŒΠ½Ρ‹Π΅ конструкции. НапримСр, Ссли ΡΠ΄Π΅Π»Π°Ρ‚ΡŒ ΠΎΠ±ΠΊΠ»Π°Π΄ΠΊΠΈ нСбольшой ΡˆΠΈΡ€ΠΈΠ½Ρ‹ и большой Π΄Π»ΠΈΠ½Ρ‹, ΠΈΡ…Β ΠΌΠΎΠΆΠ½ΠΎ вмСстС с гибким диэлСктриком ΡΠ²Π΅Ρ€Π½ΡƒΡ‚ΡŒ Π²Β ΠΏΠ»ΠΎΡ‚Π½Ρ‹ΠΉ Ρ†ΠΈΠ»ΠΈΠ½Π΄Ρ€, получится цилиндричСский кондСнсатор. РазмСщая пластины с диэлСктриком ΠΏΠΎΠΏΠ΅Ρ€Π΅ΠΌΠ΅Π½Π½ΠΎ, Π²Β Π²ΠΈΠ΄Π΅ слоСного ΠΏΠΈΡ€ΠΎΠ³Π° и чСрСдуя ΠΏΠΎΠ΄ΠΊΠ»ΡŽΡ‡Π΅Π½ΠΈΠ΅ ΠΊΒ Π²Ρ‹Π²ΠΎΠ΄Π°ΠΌ, получаСтся ΠΏΡ€ΡΠΌΠΎΡƒΠ³ΠΎΠ»ΡŒΠ½Ρ‹ΠΉ ΠΊΠΎΠΌΠΏΠΎΠ½Π΅Π½Ρ‚ с большой эффСктивной ΠΏΠ»ΠΎΡ‰Π°Π΄ΡŒΡŽ ΠΎΠ±ΠΊΠ»Π°Π΄ΠΎΠΊ.

Π Π°Π·Π½Ρ‹Π΅ Ρ‚ΠΈΠΏΡ‹ конструкции

Π•Ρ‰Π΅ ΠΎΠ΄ΠΈΠ½ ΠΏΡƒΡ‚ΡŒΒ β€” использованиС в качСствС диэлСктрика Ρ‚ΠΎΠ½ΠΊΠΎΠ³ΠΎ оксидного слоя на повСрхности мСталличСской Ρ„ΠΎΠ»ΡŒΠ³ΠΈ и раствора проводящСго элСктролита в качСствС Π²Ρ‚ΠΎΡ€ΠΎΠΉ ΠΎΠ±ΠΊΠ»Π°Π΄ΠΊΠΈ. Π’Π°ΠΊΠΈΠΌ ΠΎΠ±Ρ€Π°Π·ΠΎΠΌ получаСтся элСктролитичСский кондСнсатор, конструкция ΠΊΠΎΡ‚ΠΎΡ€ΠΎΠ³ΠΎ ΠΎΠ±Π»Π°Π΄Π°Π΅Ρ‚ самой большой Π΅ΠΌΠΊΠΎΡΡ‚ΡŒΡŽ.

Π’Π°ΠΆΠ½ΠΎ! Π’Π°ΠΊΠΈΠ΅ устройства ΠΈΠΌΠ΅ΡŽΡ‚ нСдостаток — соблюдСниС полярности ΠΏΠΎΠ΄ΠΊΠ»ΡŽΡ‡Π΅Π½ΠΈΡ, Ρ‡Ρ‚ΠΎ ΠΎΠ³Ρ€Π°Π½ΠΈΡ‡ΠΈΠ²Π°Π΅Ρ‚ ΠΈΡ…Β ΠΏΡ€ΠΈΠΌΠ΅Π½Π΅Π½ΠΈΠ΅: ΠΎΠ½ΠΎ Π²ΠΎΠ·ΠΌΠΎΠΆΠ½ΠΎ Ρ‚ΠΎΠ»ΡŒΠΊΠΎ в цСпях постоянного Ρ‚ΠΎΠΊΠ° в качСствС ΡΠ³Π»Π°ΠΆΠΈΠ²Π°ΡŽΡ‰ΠΈΡ… Ρ„ΠΈΠ»ΡŒΡ‚Ρ€ΠΎΠ².

Π’Β Ρ‡Π΅ΠΌ измСряСтся

Π•Π΄ΠΈΠ½ΠΈΡ†Π΅ΠΉ Смкости слуТит Ρ„Π°Ρ€Π°Π΄Π°. Но это ΠΎΡ‡Π΅Π½ΡŒ большая Π²Π΅Π»ΠΈΡ‡ΠΈΠ½Π° и лишь Π½Π΅ΠΊΠΎΡ‚ΠΎΡ€Ρ‹Π΅ ΡΠΏΠ΅Ρ†ΠΈΠ°Π»ΡŒΠ½Ρ‹Π΅ Ρ‚ΠΈΠΏΡ‹ устройств ΠΈΠΌΠ΅ΡŽΡ‚ Π²Π΅Π»ΠΈΡ‡ΠΈΠ½Ρƒ нСсколько Ρ„Π°Ρ€Π°Π΄.

ΠžΠ±Ρ‹Ρ‡Π½ΠΎ ΠΈΡΠΏΠΎΠ»ΡŒΠ·ΡƒΡŽΡ‚ΡΡ ΠΊΡ€Π°Ρ‚Π½Ρ‹Π΅ Π²Π΅Π»ΠΈΡ‡ΠΈΠ½Ρ‹:

  • ΠœΠΈΠΊΡ€ΠΎΡ„Π°Ρ€Π°Π΄Π° β€” 10-6 Ρ„Π°Ρ€Π°Π΄Ρ‹β€” ΠΌΠΊΠ€, Β΅F.
  • Нанофарада β€” 10-9 Ρ„Π°Ρ€Π°Π΄Ρ‹β€” Π½Π€, nF.
  • ΠŸΠΈΠΊΠΎΡ„Π°Ρ€Π°Π΄Π° β€” 10-12 Ρ„Π°Ρ€Π°Π΄Ρ‹β€” ΠΏΠ€, pF.

Π”ΠΎΠ²ΠΎΠ»ΡŒΠ½ΠΎ часто в устройствах встрСчаСтся ΠΏΠΎΡΠ»Π΅Π΄ΠΎΠ²Π°Ρ‚Π΅Π»ΡŒΠ½ΠΎΠ΅ ΠΈΒ ΠΏΠ°Ρ€Π°Π»Π»Π΅Π»ΡŒΠ½ΠΎΠ΅ соСдинСниС. Как ΠΎΠΏΡ€Π΅Π΄Π΅Π»ΠΈΡ‚ΡŒ Π΅ΠΌΠΊΠΎΡΡ‚ΡŒ соСдинСнных кондСнсаторов? Π Π΅Π·ΡƒΠ»ΡŒΡ‚ΠΈΡ€ΡƒΡŽΡ‰Π΅Π΅ Π·Π½Π°Ρ‡Π΅Π½ΠΈΠ΅ для Ρ‚Π°ΠΊΠΈΡ… соСдинСний рассчитываСтся ΠΏΠΎ-Ρ€Π°Π·Π½ΠΎΠΌΡƒ.

ΠŸΠ°Ρ€Π°Π»Π»Π΅Π»ΡŒΠ½ΠΎΠ΅ ΠΈΒ ΠΏΠΎΡΠ»Π΅Π΄ΠΎΠ²Π°Ρ‚Π΅Π»ΡŒΠ½ΠΎΠ΅ соСдинСниС

ΠŸΠ°Ρ€Π°Π»Π»Π΅Π»ΡŒΠ½ΠΎΠ΅ соСдинСниС

ΠŸΡ€ΠΈ ΠΏΠ°Ρ€Π°Π»Π»Π΅Π»ΡŒΠ½ΠΎΠΌ соСдинСнии Смкости всСх элСмСнтов суммируСтся. НоминальноС Ρ€Π°Π±ΠΎΡ‡Π΅Π΅ напряТСниС равняСтся Π½Π°ΠΈΠΌΠ΅Π½ΡŒΡˆΠ΅ΠΌΡƒ из соСдинСнных элСмСнтов

ΠŸΠΎΡΠ»Π΅Π΄ΠΎΠ²Π°Ρ‚Π΅Π»ΡŒΠ½ΠΎΠ΅ соСдинСниС

Π’Β Π΄Π°Π½Π½ΠΎΠΌ случаС, Ρ‡Ρ‚ΠΎΠ±Ρ‹ ΡƒΠ·Π½Π°Ρ‚ΡŒ Ρ€Π΅Π·ΡƒΠ»ΡŒΡ‚ΠΈΡ€ΡƒΡŽΡ‰ΡƒΡŽ Π΅ΠΌΠΊΠΎΡΡ‚ΡŒ, придСтся ΠΏΡ€ΠΈΠ±Π΅Π³Π½ΡƒΡ‚ΡŒ к расчСтам.

Для Π΄Π²ΡƒΡ… элСмСнтов:

Π‘ = Π‘1Β·Π‘2/(Π‘1+Π‘2)

Для Ρ‚Ρ€Π΅Ρ… элСмСнтов:

Π‘=(Π‘1Β·Π‘2+Π‘1Β·Π‘3+Π‘2Β·Π‘3)/(Π‘1+Π‘2+Π‘3)

НапряТСниС равняСтся суммС напряТСний Π½Π°Β ΠΊΠ°ΠΆΠ΄ΠΎΠΌ элСмСнтС.

Π’Π°ΠΆΠ½ΠΎ! НапряТСниС Π½Π°Β ΠΎΡ‚Π΄Π΅Π»ΡŒΠ½Ρ‹Ρ… кондСнсаторах распрСдСляСтся Π½Π΅Ρ€Π°Π²Π½ΠΎΠΌΠ΅Ρ€Π½ΠΎ, Π°Β ΠΏΡ€ΠΎΠΏΠΎΡ€Ρ†ΠΈΠΎΠ½Π°Π»ΡŒΠ½ΠΎ Смкости.

ΠŸΡ€ΠΈΠ±ΠΎΡ€Ρ‹ для измСрСния Смкости

Π‘ΠΏΠ΅Ρ†ΠΈΠ°Π»ΡŒΠ½Ρ‹Π΅ ΠΏΡ€ΠΈΠ±ΠΎΡ€Ρ‹ для измСрСния Смкости ΠΈΡΠΏΠΎΠ»ΡŒΠ·ΡƒΡŽΡ‚ Ρ€Π°Π·Π»ΠΈΡ‡Π½Ρ‹Π΅ ΠΏΡ€ΠΈΠ½Ρ†ΠΈΠΏΡ‹. НаиболСС распространСны Ρ‚Π°ΠΊΠΈΠ΅:

  • Π˜Π·ΠΌΠ΅Ρ€Π΅Π½ΠΈΠ΅ Ρ€Π΅Π°ΠΊΡ‚ΠΈΠ²Π½ΠΎΠ³ΠΎ сопротивлСния;
  • Π˜Π·ΠΌΠ΅Ρ€Π΅Π½ΠΈΠ΅ частоты рСзонанса ΠΊΠΎΠ»Π΅Π±Π°Ρ‚Π΅Π»ΡŒΠ½ΠΎΠ³ΠΎ ΠΊΠΎΠ½Ρ‚ΡƒΡ€Π°.

ΠŸΠ΅Ρ€Π²Ρ‹ΠΉ Ρ‚ΠΈΠΏ ΠΏΡ€ΠΈΠ±ΠΎΡ€ΠΎΠ² Π½Π°ΠΈΠ±ΠΎΠ»Π΅Π΅ распространСн. ΠŸΡ€ΠΈΠ½Ρ†ΠΈΠΏ ΠΈΡ…Β Ρ€Π°Π±ΠΎΡ‚Ρ‹ основан Π½Π°Β Ρ‚ΠΎΠΌ, Ρ‡Ρ‚ΠΎ кондСнсатор ΠΎΠ±Π»Π°Π΄Π°Π΅Ρ‚ Ρ€Π΅Π°ΠΊΡ‚ΠΈΠ²Π½Ρ‹ΠΌ сопротивлСниСм, ΠΎΠ±Ρ€Π°Ρ‚Π½ΠΎ ΠΏΡ€ΠΎΠΏΠΎΡ€Ρ†ΠΈΠΎΠ½Π°Π»ΡŒΠ½Ρ‹ΠΌ частотС ΠΏΡ€ΠΈΠ»ΠΎΠΆΠ΅Π½Π½ΠΎΠ³ΠΎ напряТСния. Π’ΠΎΒ Π΅ΡΡ‚ΡŒ, Ρ‡Π΅ΠΌ Π²Ρ‹ΡˆΠ΅ частота сигнала, Ρ‚Π΅ΠΌ мСньшС сопротивлСниС. На клСммах ΠΏΡ€ΠΈΠ±ΠΎΡ€Π° присутствуСт напряТСниС Π·Π°Π΄Π°Π½Π½ΠΎΠΉ Π²Π΅Π»ΠΈΡ‡ΠΈΠ½Ρ‹ и частота, а шкала ΡƒΠΆΠ΅ ΠΎΡ‚ΠΊΠ°Π»ΠΈΠ±Ρ€ΠΎΠ²Π°Π½Π° Π²Β Π΅Π΄ΠΈΠ½ΠΈΡ†Π°Ρ… Смкости, поэтому Π½ΠΈΠΊΠ°ΠΊΠΈΡ… вычислСний ΠΏΡ€ΠΎΠΈΠ·Π²ΠΎΠ΄ΠΈΡ‚ΡŒ Π½Π΅Β Π½Π°Π΄ΠΎ, Π·Π°Β ΠΈΡΠΊΠ»ΡŽΡ‡Π΅Π½ΠΈΠ΅ΠΌ ΡƒΡ‡Π΅Ρ‚Π° полоТСния Π²Ρ…ΠΎΠ΄Π½Ρ‹Ρ… ΠΏΠ΅Ρ€Π΅ΠΊΠ»ΡŽΡ‡Π°Ρ‚Π΅Π»Π΅ΠΉ.

Π¦ΠΈΡ„Ρ€ΠΎΠ²Ρ‹Π΅ ΠΏΡ€ΠΈΠ±ΠΎΡ€Ρ‹ для измСрСния Смкости в эксплуатации Π΅Ρ‰Π΅ ΠΏΡ€ΠΎΡ‰Π΅. На цифровом ΠΈΠ½Π΄ΠΈΠΊΠ°Ρ‚ΠΎΡ€Π΅ сразу показываСтся Π·Π½Π°Ρ‡Π΅Π½ΠΈΠ΅ измСряСмого ΠΏΠ°Ρ€Π°ΠΌΠ΅Ρ‚Ρ€Π°.

Π¦ΠΈΡ„Ρ€ΠΎΠ²ΠΎΠΉ ΠΈΠ·ΠΌΠ΅Ρ€ΠΈΡ‚Π΅Π»ΡŒ

Для устройств Π²Ρ‚ΠΎΡ€ΠΎΠ³ΠΎ Ρ‚ΠΈΠΏΠ° ΠΈΡΠΏΠΎΠ»ΡŒΠ·ΡƒΠ΅Ρ‚ΡΡ явлСниС рСзонанса — скачкообразноС ΠΈΠ·ΠΌΠ΅Ρ€Π΅Π½ΠΈΠ΅ ΠΏΠ°Ρ€Π°ΠΌΠ΅Ρ‚Ρ€ΠΎΠ² ΠΊΠΎΠ»Π΅Π±Π°Ρ‚Π΅Π»ΡŒΠ½ΠΎΠ³ΠΎ ΠΊΠΎΠ½Ρ‚ΡƒΡ€Π° из соСдинСнных кондСнсатора ΠΈΒ ΠΊΠ°Ρ‚ΡƒΡˆΠΊΠΈ индуктивности.

Для опрСдСлСния Смкости измСряСмый элСмСнт ΠΏΠΎΠ΄ΠΊΠ»ΡŽΡ‡Π°Π΅Ρ‚ΡΡ ΠΊΒ ΠΊΠ°Ρ‚ΡƒΡˆΠΊΠ΅ индуктивности с точно ΠΎΠΏΡ€Π΅Π΄Π΅Π»Π΅Π½Π½Ρ‹ΠΌΠΈ ΠΏΠ°Ρ€Π°ΠΌΠ΅Ρ‚Ρ€Π°ΠΌΠΈ. ИзмСняя частоту сигнала, Π΄ΠΎΠ±ΠΈΠ²Π°ΡŽΡ‚ΡΡ рСзонанса ΠΈΒ ΠΎΡ‚ΡΡ‡ΠΈΡ‚Ρ‹Π²Π°ΡŽΡ‚ в этот ΠΌΠΎΠΌΠ΅Π½Ρ‚ Π΅ΠΌΠΊΠΎΡΡ‚ΡŒ кондСнсатора на шкалС ΠΏΡ€ΠΈΠ±ΠΎΡ€Π°.

Π’Π°ΠΊΠΆΠ΅ ΠΊΠ°ΠΊ ΠΈΒ ΠΏΠ΅Ρ€Π²Ρ‹Π΅, эти устройства ΠΌΠΎΠ³ΡƒΡ‚ Π±Ρ‹Ρ‚ΡŒ Π°Π½Π°Π»ΠΎΠ³ΠΎΠ²Ρ‹ΠΌΠΈ ΠΈΠ»ΠΈ Ρ†ΠΈΡ„Ρ€ΠΎΠ²Ρ‹ΠΌΠΈ.

НаиболСС часто ΠΈΡΠΏΠΎΠ»ΡŒΠ·ΡƒΡŽΡ‚ΡΡ ΠΊΠΎΠΌΠ±ΠΈΠ½ΠΈΡ€ΠΎΠ²Π°Π½Π½Ρ‹Π΅ ΠΈΠ·ΠΌΠ΅Ρ€ΠΈΡ‚Π΅Π»ΡŒΠ½Ρ‹Π΅ устройства, ΠΊΠΎΡ‚ΠΎΡ€Ρ‹ΠΌΠΈ ΠΌΠΎΠΆΠ½ΠΎ ΠΈΠ·ΠΌΠ΅Ρ€ΡΡ‚ΡŒ Π΄ΠΎΠΏΠΎΠ»Π½ΠΈΡ‚Π΅Π»ΡŒΠ½ΠΎ ΠΈΠ½Π΄ΡƒΠΊΡ‚ΠΈΠ²Π½ΠΎΡΡ‚ΡŒ и сопротивлСниС — RLC-ΠΌΠ΅Ρ‚Ρ€Ρ‹.

Π˜Π·ΠΌΠ΅Ρ€ΠΈΡ‚Π΅Π»ΡŒ RLC

Π‘ΠΏΠ΅Ρ†ΠΈΠ°Π»ΡŒΠ½Ρ‹ΠΉ ΠΈΠ·ΠΌΠ΅Ρ€ΠΈΡ‚Π΅Π»ΡŒ ΠΌΠΎΠΆΠ΅Ρ‚ ΠΎΠΏΡ€Π΅Π΄Π΅Π»ΡΡ‚ΡŒ эквивалСнтноС ΠΏΠΎΡΠ»Π΅Π΄ΠΎΠ²Π°Ρ‚Π΅Π»ΡŒΠ½ΠΎΠ΅ сопротивлСниС (ЭПБ, ESR) и тангСнс ΡƒΠ³Π»Π° ΠΏΠΎΡ‚Π΅Ρ€ΡŒ.

ΠžΡ†Π΅Π½ΠΈΡ‚ΡŒ Π΅ΠΌΠΊΠΎΡΡ‚ΡŒ элСктролитичСского кондСнсатора ΠΌΠΎΠΆΠ½ΠΎ, ΠΈΡΠΏΠΎΠ»ΡŒΠ·ΡƒΡ ΠΎΠ±Ρ‹Ρ‡Π½Ρ‹ΠΉ ΠΌΡƒΠ»ΡŒΡ‚ΠΈΠΌΠ΅Ρ‚Ρ€ Π²Β Ρ€Π΅ΠΆΠΈΠΌΠ΅ измСрСния сопротивлСния. ВрСмя заряда косвСнно Π±ΡƒΠ΄Π΅Ρ‚ ΡΠ²ΠΈΠ΄Π΅Ρ‚Π΅Π»ΡŒΡΡ‚Π²ΠΎΠ²Π°Ρ‚ΡŒ ΠΎΒ Π²Π΅Π»ΠΈΡ‡ΠΈΠ½Π΅ Смкости (Π§Π΅ΠΌ большС Π²Π΅Π»ΠΈΡ‡ΠΈΠ½Π°, Ρ‚Π΅ΠΌ ΠΌΠ΅Π΄Π»Π΅Π½Π½Π΅Π΅ Π±ΡƒΠ΄ΡƒΡ‚ измСнСния ΠΏΠΎΠΊΠ°Π·Π°Π½ΠΈΠΉ).

Как ΠΏΡ€Π°Π²ΠΈΠ»ΡŒΠ½ΠΎ ΠΈΠ·ΠΌΠ΅Ρ€ΡΡ‚ΡŒ Π΅ΠΌΠΊΠΎΡΡ‚ΡŒ

Как ΠΈΠ·ΠΌΠ΅Ρ€ΠΈΡ‚ΡŒ Ρ‘ΠΌΠΊΠΎΡΡ‚ΡŒ кондСнсатора, нС имСя ΡΠΏΠ΅Ρ†ΠΈΠ°Π»ΡŒΠ½ΠΎΠ³ΠΎ оборудования? НуТно ΠΎΠΏΡ€Π΅Π΄Π΅Π»ΠΈΡ‚ΡŒ Π²Π΅Π»ΠΈΡ‡ΠΈΠ½Ρƒ Ρ‚ΠΎΠΊΠ°, ΠΏΡ€ΠΎΡ‚Π΅ΠΊΠ°ΡŽΡ‰ΡƒΡŽ Ρ‡Π΅Ρ€Π΅Π· Ρ†Π΅ΠΏΡŒ с кондСнсатором ΠΈΒ ΠΏΠ°Π΄Π΅Π½ΠΈΠ΅ напряТСния Π½Π°Β Π½Π΅ΠΌ. Π—Π½Π°Ρ‡Π΅Π½ΠΈΠ΅ измСряСмого ΠΏΠ°Ρ€Π°ΠΌΠ΅Ρ‚Ρ€Π° Π²Ρ‹Ρ‡ΠΈΡΠ»ΡΡŽΡ‚ на основании Ρ„ΠΎΡ€ΠΌΡƒΠ»Ρ‹:

Xc = 1/2·π·fΒ·C,

Π“Π΄Π΅ Π₯с — Ρ€Π΅Π°ΠΊΡ‚ΠΈΠ²Π½ΠΎΠ΅ сопротивлСниС кондСнсатора,

π — число пи, Ρ€Π°Π²Π½ΠΎΠ΅ 3. 14,

fΒ β€” частота Ρ‚ΠΎΠΊΠ°.

Π˜Π·Β ΠΏΡ€ΠΈΠ²Π΅Π΄Π΅Π½Π½ΠΎΠΉ Ρ„ΠΎΡ€ΠΌΡƒΠ»Ρ‹ ΠΌΠΎΠΆΠ½ΠΎ Π½Π°ΠΉΡ‚ΠΈ Π·Π½Π°Ρ‡Π΅Π½ΠΈΠ΅ Смкости:

Π‘ = 1/2·π·fΒ·Π₯с

Π Π΅Π°ΠΊΡ‚ΠΈΠ²Π½ΠΎΠ΅ сопротивлСниС Π₯с находят ΠΈΠ·Β ΠΏΠΎΠΊΠ°Π·Π°Π½ΠΈΠΉ ΠΈΠ·ΠΌΠ΅Ρ€ΠΈΡ‚Π΅Π»ΡŒΠ½Ρ‹Ρ… ΠΏΡ€ΠΈΠ±ΠΎΡ€ΠΎΠ²:

Π₯с = U/I.

Π‘Π°ΠΌΠΎΡΡ‚ΠΎΡΡ‚Π΅Π»ΡŒΠ½ΠΎΠ΅ ΠΈΠ·ΠΌΠ΅Ρ€Π΅Π½ΠΈΠ΅ Смкости кондСнсаторов ΠΏΡ€ΠΈ ΠΏΠΎΠΌΠΎΡ‰ΠΈ ΠΏΡ€ΠΎΡΡ‚Π΅ΠΉΡˆΠΈΡ… ΠΏΡ€ΠΈΠ±ΠΎΡ€ΠΎΠ² достаточно Ρ‚Ρ€ΡƒΠ΄ΠΎΠ΅ΠΌΠΊΠΎΠ΅ ΠΈΒ Π½Π΅Β Π΄Π°Π΅Ρ‚ Π½Π΅ΠΎΠ±Ρ…ΠΎΠ΄ΠΈΠΌΠΎΠΉ точности. Π›ΡƒΡ‡ΡˆΠΈΠ΅ Ρ€Π΅Π·ΡƒΠ»ΡŒΡ‚Π°Ρ‚Ρ‹ ΠΌΠΎΠΆΠ½ΠΎ ΠΏΠΎΠ»ΡƒΡ‡ΠΈΡ‚ΡŒ, ΠΈΡΠΏΠΎΠ»ΡŒΠ·ΡƒΡ спСциализированныС ΠΈΠ·ΠΌΠ΅Ρ€ΠΈΡ‚Π΅Π»ΡŒΠ½Ρ‹Π΅ устройства.

5 способов с ΠΌΡƒΠ»ΡŒΡ‚ΠΈΠΌΠ΅Ρ‚Ρ€ΠΎΠΌ ΠΈ Π±Π΅Π· Π½Π΅Π³ΠΎ

ΠžΠΏΠΈΡΡ‹Π²Π°Π΅Ρ‚, ΠΊΠ°ΠΊ ΠΏΡ€ΠΎΠ²Π΅Ρ€ΠΈΡ‚ΡŒ кондСнсатор с Ρ„ΡƒΠ½ΠΊΡ†ΠΈΠ΅ΠΉ измСрСния Смкости ΠΈ Π±Π΅Π· Π½Π΅Π΅ Π½Π° ΠΌΡƒΠ»ΡŒΡ‚ΠΈΠΌΠ΅Ρ‚Ρ€Π΅, ΠΊΠ°ΠΊ ΠΏΡ€ΠΎΠ²Π΅Ρ€ΠΈΡ‚ΡŒ кондСнсатор с ΠΏΠΎΠΌΠΎΡ‰ΡŒΡŽ тСстСра нСпрСрывности Ρ†Π΅ΠΏΠΈ ΠΈΠ»ΠΈ с ΠΏΠΎΠΌΠΎΡ‰ΡŒΡŽ ΠΎΠΌΠΌΠ΅Ρ‚Ρ€Π°, Π° Ρ‚Π°ΠΊΠΆΠ΅ провСсти Β«Π³Ρ€ΡƒΠ±ΡƒΡŽ ΠΏΡ€ΠΎΠ²Π΅Ρ€ΠΊΡƒΒ» ΠΏΡƒΡ‚Π΅ΠΌ Π΅Π³ΠΎ ΠΊΠΎΡ€ΠΎΡ‚ΠΊΠΎΠ³ΠΎ замыкания.

Найти большС руководств, совСтов ΠΈ совСтов ΠΏΠΎ автомобилям ΠΈ ΠΌΠΎΡ‚ΠΎΡ†ΠΈΠΊΠ»Π°ΠΌ

Π‘ΠžΠ”Π•Π Π–ΠΠΠ˜Π•
Π§Ρ‚ΠΎ Ρ‚Π°ΠΊΠΎΠ΅ кондСнсатор
Π’ΠΈΠ·ΡƒΠ°Π»ΡŒΠ½Ρ‹ΠΉ осмотр
ΠŸΡ€ΠΎΠ²Π΅Ρ€ΠΊΠ° работоспособности
1. Как ΠΏΡ€ΠΎΠ²Π΅Ρ€ΠΈΡ‚ΡŒ кондСнсатор Π±Π΅Π· измСрСния Смкости
2.Как ΠΏΡ€ΠΎΠ²Π΅Ρ€ΠΈΡ‚ΡŒ кондСнсатор с ΠΏΠΎΠΌΠΎΡ‰ΡŒΡŽ ΠΌΡƒΠ»ΡŒΡ‚ΠΈΠΌΠ΅Ρ‚Ρ€Π° для ΠΏΡ€ΠΎΠ²Π΅Ρ€ΠΊΠΈ цСлостности Ρ†Π΅ΠΏΠΈ
3. ИспользованиС ΠΌΡƒΠ»ΡŒΡ‚ΠΈΠΌΠ΅Ρ‚Ρ€Π° с ΠΈΠ·ΠΌΠ΅Ρ€Π΅Π½ΠΈΠ΅ΠΌ Смкости
4. Как ΠΏΡ€ΠΎΠ²Π΅Ρ€ΠΈΡ‚ΡŒ кондСнсатор с ΠΏΠΎΠΌΠΎΡ‰ΡŒΡŽ ΠΎΠΌΠΌΠ΅Ρ‚Ρ€Π°
5. Как ΠΏΡ€ΠΎΠ²Π΅Ρ€ΠΈΡ‚ΡŒ кондСнсатор ΠΏΡƒΡ‚Π΅ΠΌ Π΅Π³ΠΎ ΠΊΠΎΡ€ΠΎΡ‚ΠΊΠΎΠ³ΠΎ замыкания

ΠœΡƒΠ»ΡŒΡ‚ΠΈΠΌΠ΅Ρ‚Ρ€ являСтся ΠΏΡ€Π΅Π΄ΠΏΠΎΡ‡Ρ‚ΠΈΡ‚Π΅Π»ΡŒΠ½Ρ‹ΠΌ ΠΈΠ·ΠΌΠ΅Ρ€ΠΈΡ‚Π΅Π»ΡŒΠ½Ρ‹ΠΌ ΠΏΡ€ΠΈΠ±ΠΎΡ€ΠΎΠΌ, ΠΊΠΎΠ³Π΄Π° Ρ€Π΅Ρ‡ΡŒ ΠΈΠ΄Π΅Ρ‚ ΠΎ ΠΏΡ€ΠΎΠ²Π΅Ρ€ΠΊΠ΅ Π²ΠΎΠ·ΠΌΠΎΠΆΠ½ΠΎ нСисправного кондСнсатора. Π•ΡΡ‚ΡŒ нСсколько способов ΠΏΡ€ΠΎΠ²Π΅Ρ€ΠΈΡ‚ΡŒ кондСнсатор с ΠΏΠΎΠΌΠΎΡ‰ΡŒΡŽ ΠΌΡƒΠ»ΡŒΡ‚ΠΈΠΌΠ΅Ρ‚Ρ€Π°.

Π’ основном, ΠΎΠ΄Π½Π°ΠΊΠΎ:

ΠœΡƒΠ»ΡŒΡ‚ΠΈΠΌΠ΅Ρ‚Ρ€Ρƒ трСбуСтся ΡΠΏΠ΅Ρ†ΠΈΠ°Π»ΡŒΠ½ΠΎΠ΅ ΠΈΠ·ΠΌΠ΅Ρ€ΠΈΡ‚Π΅Π»ΡŒΠ½ΠΎΠ΅ устройство, Ρ‡Ρ‚ΠΎΠ±Ρ‹ ΠΈΠΌΠ΅Ρ‚ΡŒ Π²ΠΎΠ·ΠΌΠΎΠΆΠ½ΠΎΡΡ‚ΡŒ ΠΏΡ€ΠΎΠ²Π΅Ρ€ΡΡ‚ΡŒ кондСнсаторы ΠΈ, Ρ‚Π°ΠΊΠΈΠΌ ΠΎΠ±Ρ€Π°Π·ΠΎΠΌ, ΠΎΠΏΡ€Π΅Π΄Π΅Π»ΡΡ‚ΡŒ Ρ‚ΠΎΡ‡Π½Ρ‹Π΅ значСния Смкости кондСнсатора.Если Π½Π΅Ρ‚ Ρ„ΡƒΠ½ΠΊΡ†ΠΈΠΈ измСрСния Смкости, ΠΌΠΎΠΆΠ½ΠΎ Ρ‚ΠΎΠ»ΡŒΠΊΠΎ ΠΎΠΏΡ€Π΅Π΄Π΅Π»ΠΈΡ‚ΡŒ, Π΅ΡΡ‚ΡŒ Π»ΠΈ Ρƒ кондСнсатора ΠΊΠΎΡ€ΠΎΡ‚ΠΊΠΎΠ΅ Π·Π°ΠΌΡ‹ΠΊΠ°Π½ΠΈΠ΅ ΠΈΠ»ΠΈ ΠΎΠ½ заряТаСтся. Для этого ΠΌΠΎΠΆΠ½ΠΎ Π²Ρ‹ΠΏΠΎΠ»Π½ΠΈΡ‚ΡŒ ΠΏΡ€ΠΎΠ²Π΅Ρ€ΠΊΡƒ нСпрСрывности ΠΈΠ»ΠΈ ΠΈΠ·ΠΌΠ΅Ρ€Π΅Π½ΠΈΠ΅ сопротивлСния Π² омичСском Π΄ΠΈΠ°ΠΏΠ°Π·ΠΎΠ½Π΅.

Π§Ρ‚ΠΎ Ρ‚Π°ΠΊΠΎΠ΅ кондСнсатор?

ΠšΠΎΠ½Π΄Π΅Π½ΡΠ°Ρ‚ΠΎΡ€Ρ‹ β€” это пассивный элСктронный ΠΊΠΎΠΌΠΏΠΎΠ½Π΅Π½Ρ‚, ΠΊΠΎΡ‚ΠΎΡ€Ρ‹ΠΉ ΠΈΡΠΏΠΎΠ»ΡŒΠ·ΡƒΠ΅Ρ‚ΡΡ ΠΏΠΎΡ‡Ρ‚ΠΈ Π²ΠΎ всСх элСктричСских устройствах. Π’Ρ‹ ΠΌΠΎΠΆΠ΅Ρ‚Π΅ Π½Π°ΠΉΡ‚ΠΈ ΠΈΡ… Π² ΠΊΠΎΠΌΠΏΡŒΡŽΡ‚Π΅Ρ€Π°Ρ…, Ρ‚Π΅Π»Π΅Π²ΠΈΠ·ΠΎΡ€Π°Ρ…, ΠΊΡƒΡ…ΠΎΠ½Π½ΠΎΠΉ Ρ‚Π΅Ρ…Π½ΠΈΠΊΠ΅, рСмСслСнных станках, транспортных срСдствах ΠΈ ΠΌΠ½ΠΎΠ³ΠΈΡ… Π΄Ρ€ΡƒΠ³ΠΈΡ… устройствах.

Π’ основном кондСнсаторы состоят ΠΈΠ· Π΄Π²ΡƒΡ… элСктропроводящих повСрхностСй, ΠΊΠΎΡ‚ΠΎΡ€Ρ‹Π΅ ΠΎΡ‚Π΄Π΅Π»Π΅Π½Ρ‹ Π΄Ρ€ΡƒΠ³ ΠΎΡ‚ Π΄Ρ€ΡƒΠ³Π° изоляционным ΠΌΠ°Ρ‚Π΅Ρ€ΠΈΠ°Π»ΠΎΠΌ. Однако ΡΡƒΡ‰Π΅ΡΡ‚Π²ΡƒΡŽΡ‚ Ρ€Π°Π·Π½Ρ‹Π΅ Ρ‚ΠΈΠΏΡ‹ ΠΈ Ρ„ΠΎΡ€ΠΌΡ‹ кондСнсаторов. Одним ΠΈΠ· самых извСстных являСтся элСктролитичСский кондСнсатор. Π­Ρ‚ΠΎ поляризованный кондСнсатор. Напротив, кСрамичСскиС кондСнсаторы, Π½Π°ΠΏΡ€ΠΈΠΌΠ΅Ρ€, ΠΈΡΠΏΠΎΠ»ΡŒΠ·ΡƒΡŽΡ‚ΡΡ Π² качСствС нСполяризованных кондСнсаторов. Π’ области кондСнсаторов Π΄Π²ΠΈΠ³Π°Ρ‚Π΅Π»Π΅ΠΉ Ρ‚Π°ΠΊΠΆΠ΅ ΠΈΡΠΏΠΎΠ»ΡŒΠ·ΡƒΡŽΡ‚ΡΡ пусковыС кондСнсаторы.

ΠŸΠΎΡΠΊΠΎΠ»ΡŒΠΊΡƒ кондСнсаторы Π±Π»ΠΎΠΊΠΈΡ€ΡƒΡŽΡ‚ постоянный Ρ‚ΠΎΠΊ ΠΈ ΠΏΡ€ΠΎΠΏΡƒΡΠΊΠ°ΡŽΡ‚ ΠΏΠ΅Ρ€Π΅ΠΌΠ΅Π½Π½Ρ‹ΠΉ Ρ‚ΠΎΠΊ, ΠΎΠ½ΠΈ Π²Ρ‹ΠΏΠΎΠ»Π½ΡΡŽΡ‚ Ρ€Π°Π·Π½Ρ‹Π΅ Ρ„ΡƒΠ½ΠΊΡ†ΠΈΠΈ.Π’ Ρ†Π΅ΠΏΠΈ ΠΏΠ΅Ρ€Π΅ΠΌΠ΅Π½Π½ΠΎΠ³ΠΎ Ρ‚ΠΎΠΊΠ° кондСнсатор ΠΈΡΠΏΠΎΠ»ΡŒΠ·ΡƒΠ΅Ρ‚ΡΡ ΠΊΠ°ΠΊ рСзистор ΠΏΠ΅Ρ€Π΅ΠΌΠ΅Π½Π½ΠΎΠ³ΠΎ Ρ‚ΠΎΠΊΠ°, Π² Ρ†Π΅ΠΏΠΈ постоянного Ρ‚ΠΎΠΊΠ° ΠΎΠ½ ΠΌΠΎΠΆΠ΅Ρ‚ Π½Π°ΠΊΠ°ΠΏΠ»ΠΈΠ²Π°Ρ‚ΡŒ элСктричСский заряд. Π­Ρ‚ΠΎ сохранСнноС напряТСниС называСтся элСктричСской Π΅ΠΌΠΊΠΎΡΡ‚ΡŒΡŽ (C) ΠΈ измСряСтся Π² Ρ„Π°Ρ€Π°Π΄Π°Ρ… (F).

ΠŸΠΎΡΠΊΠΎΠ»ΡŒΠΊΡƒ элСктролитичСскиС кондСнсаторы со Π²Ρ€Π΅ΠΌΠ΅Π½Π΅ΠΌ ΠΈΠ·Π½Π°ΡˆΠΈΠ²Π°ΡŽΡ‚ΡΡ, ΠΌΠΎΠΆΠ΅Ρ‚ ΠΏΠΎΡ‚Ρ€Π΅Π±ΠΎΠ²Π°Ρ‚ΡŒΡΡ ΠΏΡ€ΠΎΠ²Π΅Ρ€ΠΊΠ° ΠΈΡ… работоспособности. ΠšΠΎΠ½Π΄Π΅Π½ΡΠ°Ρ‚ΠΎΡ€ ΠΌΠΎΠΆΠ½ΠΎ ΠΈΠ·ΠΌΠ΅Ρ€ΠΈΡ‚ΡŒ ΠΌΡƒΠ»ΡŒΡ‚ΠΈΠΌΠ΅Ρ‚Ρ€ΠΎΠΌ. Π•ΡΡ‚ΡŒ Π΄Π²Π° ΠΏΠΎΠ΄Ρ…ΠΎΠ΄Π°: Π²Ρ‹ просто Ρ…ΠΎΡ‚ΠΈΡ‚Π΅ ΠΏΡ€ΠΎΠ²Π΅Ρ€ΠΈΡ‚ΡŒ состояниС кондСнсатора с ΠΏΠΎΠΌΠΎΡ‰ΡŒΡŽ ΠΌΡƒΠ»ΡŒΡ‚ΠΈΠΌΠ΅Ρ‚Ρ€Π° ΠΈΠ»ΠΈ Ρ…ΠΎΡ‚ΠΈΡ‚Π΅ ΠΈΠ·ΠΌΠ΅Ρ€ΠΈΡ‚ΡŒ Ρ‚ΠΎΡ‡Π½ΡƒΡŽ Π΅ΠΌΠΊΠΎΡΡ‚ΡŒ кондСнсатора?

Π’ΠΈΠ·ΡƒΠ°Π»ΡŒΠ½Ρ‹ΠΉ осмотр

  • ΠŸΠ»Π°ΡΡ‚ΠΈΠΊΠΎΠ²Ρ‹ΠΉ корпус: Π“Π΄Π΅-Ρ‚ΠΎ Π½Π° корпусС появляСтся нСопрСдСлимая масса? Π•ΡΡ‚ΡŒ Π»ΠΈ Π½Π° корпусС Ρ‚Ρ€Π΅Ρ‰ΠΈΠ½Π° ΠΈΠ»ΠΈ Π΄Π°ΠΆΠ΅ Π΄Ρ‹Ρ€Π°?
  • ΠΠ»ΡŽΠΌΠΈΠ½ΠΈΠ΅Π²Ρ‹ΠΉ корпус: ΠΏΡ€ΠΎΡ‚Π΅ΠΊΠ°Π΅Ρ‚ Π»ΠΈ ΠΆΠΈΠ΄ΠΊΠΎΡΡ‚ΡŒ? Π‘Ρ€Π°Π±ΠΎΡ‚Π°Π»Π° Π·Π°Ρ‰ΠΈΡ‚Π° ΠΎΡ‚ ΠΈΠ·Π±Ρ‹Ρ‚ΠΎΡ‡Π½ΠΎΠ³ΠΎ давлСния?

Если Π²Ρ‹ ΠΌΠΎΠΆΠ΅Ρ‚Π΅ ΠΎΡ‚Π²Π΅Ρ‚ΠΈΡ‚ΡŒ Β«Π”Π°Β» Π½Π° ΠΎΠ΄ΠΈΠ½ ΠΈΠ· этих вопросов, скорСС всСго, кондСнсатор нСисправСн.

Π’ ΡΠ»Π΅Π΄ΡƒΡŽΡ‰Π΅ΠΌ Ρ€Π°Π·Π΄Π΅Π»Π΅ прСдставлСны Ρ€Π°Π·Π»ΠΈΡ‡Π½Ρ‹Π΅ ΠΌΠ΅Ρ‚ΠΎΠ΄Ρ‹ ΠΏΡ€ΠΎΠ²Π΅Ρ€ΠΊΠΈ кондСнсатора с ΠΏΠΎΠΌΠΎΡ‰ΡŒΡŽ ΠΌΡƒΠ»ΡŒΡ‚ΠΈΠΌΠ΅Ρ‚Ρ€Π°.

Π€ΡƒΠ½ΠΊΡ†ΠΈΠΎΠ½Π°Π»ΡŒΠ½Ρ‹ΠΉ тСст

Π”Π²ΠΈΠ³Π°Ρ‚Π΅Π»ΡŒ с нСисправным кондСнсатором Π»ΠΈΠ±ΠΎ Π³ΡƒΠ΄ΠΈΡ‚ ΠΏΠ΅Ρ€Π΅Π΄ пуском, Π»ΠΈΠ±ΠΎ запускаСтся с ΠΎΡ‚Ρ‡Π΅Ρ‚Π»ΠΈΠ²ΠΎ ΡΠ»Ρ‹ΡˆΠΈΠΌΡ‹ΠΌ Π³ΡƒΠ»ΠΎΠΌ. Π­Ρ‚ΠΎ явныС ΠΏΡ€ΠΈΠ·Π½Π°ΠΊΠΈ ΠΏΠΎΡ‚Π΅Ρ€ΠΈ Смкости ΠΈ, ΡΠ»Π΅Π΄ΠΎΠ²Π°Ρ‚Π΅Π»ΡŒΠ½ΠΎ, нСисправности кондСнсатора.

Π’Ρ‹ Π΄ΠΎΠ»ΠΆΠ½Ρ‹ Π±Ρ‹Ρ‚ΡŒ ΠΎΡ‡Π΅Π½ΡŒ остороТны с этим Ρ‚ΠΈΠΏΠΎΠΌ тСста, Ρ‚Π°ΠΊ ΠΊΠ°ΠΊ сущСствуСт большой риск Ρ‚Ρ€Π°Π²ΠΌΡ‹. ΠŸΡ€Π΅ΠΆΠ΄Π΅ всСго, Π½ΠΈΠΊΠΎΠ³Π΄Π° Π½Π΅ тСстируйтС ΠΏΠΈΠ»Ρ‹ ΠΈΠ»ΠΈ газонокосилки Ρ‚Π°ΠΊΠΈΠΌ ΠΎΠ±Ρ€Π°Π·ΠΎΠΌ.МногиС люди ΠΏΠ΅Ρ€Π΅ΠΎΡ†Π΅Π½ΠΈΠ²Π°ΡŽΡ‚ свои рСфлСксы ΠΈ Π½Π΅ ΠΌΠΎΠ³ΡƒΡ‚ достаточно быстро ΡƒΠ±Ρ€Π°Ρ‚ΡŒ ΠΏΠ°Π»ΡŒΡ†Ρ‹ ΠΈΠ· опасной Π·ΠΎΠ½Ρ‹ ΠΏΡ€ΠΈ Π²Π½Π΅Π·Π°ΠΏΠ½ΠΎΠΌ запускС двигатСля. К соТалСнию, ΠΌΠ½ΠΎΠ³ΠΈΠ΅ нСсчастныС случаи с ΠΎΡ‚Ρ€ΡƒΠ±Π»Π΅Π½Π½Ρ‹ΠΌΠΈ ΠΏΠ°Π»ΡŒΡ†Π°ΠΌΠΈ говорят сами Π·Π° сСбя.

Если Π΄Π²ΠΈΠ³Π°Ρ‚Π΅Π»ΡŒ вращаСтся Π² Π½Π΅ΠΏΡ€Π°Π²ΠΈΠ»ΡŒΠ½ΠΎΠΌ Π½Π°ΠΏΡ€Π°Π²Π»Π΅Π½ΠΈΠΈ, это Ρ‚Π°ΠΊΠΆΠ΅ ΠΌΠΎΠΆΠ΅Ρ‚ ΡƒΠΊΠ°Π·Ρ‹Π²Π°Ρ‚ΡŒ Π½Π° Π½Π΅ΠΈΡΠΏΡ€Π°Π²Π½ΠΎΡΡ‚ΡŒ кондСнсатора. Π’ΠΎ ΠΆΠ΅ самоС относится ΠΊ ΠΎΡ‡Π΅Π½ΡŒ ΠΌΠ΅Π΄Π»Π΅Π½Π½ΠΎΠΌΡƒ ΠΈΠ»ΠΈ Π±Π΅ΡΡΠΈΠ»ΡŒΠ½ΠΎΠΌΡƒ запуску ΠΌΠ°ΡˆΠΈΠ½Ρ‹. Если машина Π·Π°Π³Ρ€ΡƒΠΆΠ΅Π½Π°, ΡΠΊΠΎΡ€ΠΎΡΡ‚ΡŒ Π² Ρ‚Π°ΠΊΠΎΠΌ случаС ΠΏΠ°Π΄Π°Π΅Ρ‚ ΠΎΡ‡Π΅Π½ΡŒ быстро. Если ваш ΡΠ»Π΅ΠΊΡ‚Ρ€ΠΎΠ΄Π²ΠΈΠ³Π°Ρ‚Π΅Π»ΡŒ Ρ€Π°Π±ΠΎΡ‚Π°Π΅Ρ‚ Π½Π΅ Π² Ρ‚Ρƒ сторону ΠΈΠ»ΠΈ Ρƒ Π½Π΅Π³ΠΎ заканчиваСтся ΠΌΠΎΡ‰Π½ΠΎΡΡ‚ΡŒ, ΠΏΠΎΠΌΠΈΠΌΠΎ Π΄Π΅Ρ„Π΅ΠΊΡ‚Π° кондСнсатора ΠΌΠΎΠΆΠ΅Ρ‚ Π±Ρ‹Ρ‚ΡŒ Π²ΠΈΠ½ΠΎΠ²Π°Ρ‚Π° нСисправная ΠΎΠ±ΠΌΠΎΡ‚ΠΊΠ° двигатСля.

1. Как ΠΏΡ€ΠΎΠ²Π΅Ρ€ΠΈΡ‚ΡŒ кондСнсатор Π±Π΅Π· измСрСния Смкости

Если имССтся Ρ‚ΠΎΠ»ΡŒΠΊΠΎ простой ΠΌΡƒΠ»ΡŒΡ‚ΠΈΠΌΠ΅Ρ‚Ρ€ Π±Π΅Π· Ρ„ΡƒΠ½ΠΊΡ†ΠΈΠΈ измСрСния Смкости, Ρ‚ΠΎ ΠΌΠΎΠΆΠ½ΠΎ ΠΏΡ€ΠΎΠ²Π΅Ρ€ΠΈΡ‚ΡŒ Ρ‚ΠΎΠ»ΡŒΠΊΠΎ ΠΏΡ€ΠΈΠ±Π»ΠΈΠ·ΠΈΡ‚Π΅Π»ΡŒΠ½ΡƒΡŽ Ρ€Π°Π±ΠΎΡ‚ΠΎΡΠΏΠΎΡΠΎΠ±Π½ΠΎΡΡ‚ΡŒ кондСнсатора ΠΈΠ»ΠΈ элСктролитичСского кондСнсатора (элСктролитичСского кондСнсатора). Π’Ρ‹ΠΏΠΎΠ»Π½ΠΈΡ‚Π΅ ΡΠ»Π΅Π΄ΡƒΡŽΡ‰ΠΈΠ΅ дСйствия:

1. ΠžΡ‚ΠΊΡ€Ρ‹Ρ‚ΡŒ кондСнсатор

Π’ ΠΏΠ΅Ρ€Π²ΡƒΡŽ ΠΎΡ‡Π΅Ρ€Π΅Π΄ΡŒ слСдуСт ΠΏΠΎΠ»Π½ΠΎΡΡ‚ΡŒΡŽ Π²Ρ‹Π½ΡƒΡ‚ΡŒ ΠΈΠ· Ρ†Π΅ΠΏΠΈ провСряСмый кондСнсатор. ВсС ΠΊΠΎΠ½Ρ‚Π°ΠΊΡ‚Ρ‹ Ρ†Π΅ΠΏΠΈ Π΄ΠΎΠ»ΠΆΠ½Ρ‹ Π±Ρ‹Ρ‚ΡŒ ΡƒΠ΄Π°Π»Π΅Π½Ρ‹, Π° полюса кондСнсатора Π΄ΠΎΠ»ΠΆΠ½Ρ‹ Π±Ρ‹Ρ‚ΡŒ доступны.

2. Π’ΠΈΠ·ΡƒΠ°Π»ΡŒΠ½ΠΎ ΠΏΡ€ΠΎΠ²Π΅Ρ€ΡŒΡ‚Π΅ кондСнсатор

ΠŸΠ΅Ρ€Π΅Π΄ ΠΈΠ·ΠΌΠ΅Ρ€Π΅Π½ΠΈΠ΅ΠΌ кондСнсатора ΠΌΡƒΠ»ΡŒΡ‚ΠΈΠΌΠ΅Ρ‚Ρ€ΠΎΠΌ Π΅Π³ΠΎ слСдуСт Π²ΠΈΠ·ΡƒΠ°Π»ΡŒΠ½ΠΎ ΠΏΡ€ΠΎΠ²Π΅Ρ€ΠΈΡ‚ΡŒ Π½Π° Π½Π°Π»ΠΈΡ‡ΠΈΠ΅ явных ΠΏΠΎΠ²Ρ€Π΅ΠΆΠ΄Π΅Π½ΠΈΠΉ. ΠžΠ±Ρ€Π°Ρ‚ΠΈΡ‚Π΅ Π²Π½ΠΈΠΌΠ°Π½ΠΈΠ΅ Π½Π° нСбольшиС нСровности ΠΈΠ»ΠΈ ΠΌΠ΅Π»ΠΊΠΈΠ΅ Ρ‚Ρ€Π΅Ρ‰ΠΈΠ½Ρ‹ Π½Π° повСрхности. Π£Ρ‚Π΅Ρ‡ΠΊΠ° Тидкости Ρ‚Π°ΠΊΠΆΠ΅ ΡƒΠΊΠ°Π·Ρ‹Π²Π°Π΅Ρ‚ Π½Π° Π½Π΅ΠΈΡΠΏΡ€Π°Π²Π½ΠΎΡΡ‚ΡŒ кондСнсатора, ΠΊΠΎΡ‚ΠΎΡ€Ρ‹ΠΉ слСдуСт Π·Π°ΠΌΠ΅Π½ΠΈΡ‚ΡŒ.

3. Разрядный кондСнсатор

Π‘Π»Π΅Π΄ΡƒΡŽΡ‰ΠΈΠΌ шагом являСтся полная разрядка кондСнсатора. Π§Ρ‚ΠΎΠ±Ρ‹ Π² кондСнсаторС Π½Π΅ Π±Ρ‹Π»ΠΎ остаточного Ρ‚ΠΎΠΊΠ°, Π΅Π³ΠΎ ΠΌΠΎΠΆΠ½ΠΎ ΠΏΠΎΠ΄ΠΊΠ»ΡŽΡ‡ΠΈΡ‚ΡŒ ΠΊ ΠΏΠΎΡ‚Ρ€Π΅Π±ΠΈΡ‚Π΅Π»ΡŽ, Π½Π°ΠΏΡ€ΠΈΠΌΠ΅Ρ€ ΠΊ простой Π»Π°ΠΌΠΏΠΎΡ‡ΠΊΠ΅.Π’Π°ΠΊΠΈΠΌ ΠΎΠ±Ρ€Π°Π·ΠΎΠΌ, вся накоплСнная энСргия ΠΌΠΎΠΆΠ΅Ρ‚ Π±Ρ‹Ρ‚ΡŒ ΠΏΠΎΠ»Π½ΠΎΡΡ‚ΡŒΡŽ разряТСна.

4. Набор ΠΌΡƒΠ»ΡŒΡ‚ΠΈΠΌΠ΅Ρ‚Ρ€Π°

Π’Π΅ΠΏΠ΅Ρ€ΡŒ ΠΌΡƒΠ»ΡŒΡ‚ΠΈΠΌΠ΅Ρ‚Ρ€ Π΄ΠΎΠ»ΠΆΠ΅Π½ Π±Ρ‹Ρ‚ΡŒ настроСн Π½Π° Ρ„ΡƒΠ½ΠΊΡ†ΠΈΡŽ измСрСния сопротивлСния (ΠΈΠ·ΠΌΠ΅Ρ€Π΅Π½Π½Ρ‹Π΅ значСния Π² ΠΎΠΌΠ°Ρ…). Π”ΠΈΠ°ΠΏΠ°Π·ΠΎΠ½ измСрСния 1000 Ом, Ρ‚. Π΅. 1 кОм, Π΄ΠΎΠ»ΠΆΠ΅Π½ Π±Ρ‹Ρ‚ΡŒ Π²Ρ‹Π±Ρ€Π°Π½ Ρ‚Π°ΠΊ, Ρ‡Ρ‚ΠΎΠ±Ρ‹ ΠΌΠΎΠΆΠ½ΠΎ Π±Ρ‹Π»ΠΎ ΠΎΠΏΡ€Π΅Π΄Π΅Π»ΠΈΡ‚ΡŒ ΠΏΡ€ΠΈΠ³ΠΎΠ΄Π½Ρ‹Π΅ для использования Ρ€Π΅Π·ΡƒΠ»ΡŒΡ‚Π°Ρ‚Ρ‹.

5. Π˜Π·ΠΌΠ΅Ρ€ΡŒΡ‚Π΅ кондСнсатор ΠΌΡƒΠ»ΡŒΡ‚ΠΈΠΌΠ΅Ρ‚Ρ€ΠΎΠΌ

Π’Π΅ΠΏΠ΅Ρ€ΡŒ Π΄Π²Π΅ ΠΈΠ·ΠΌΠ΅Ρ€ΠΈΡ‚Π΅Π»ΡŒΠ½Ρ‹Π΅ Π»ΠΈΠ½ΠΈΠΈ ΠΌΡƒΠ»ΡŒΡ‚ΠΈΠΌΠ΅Ρ‚Ρ€Π° ΠΌΠΎΠΆΠ½ΠΎ ΠΏΠΎΠ΄ΠΊΠ»ΡŽΡ‡ΠΈΡ‚ΡŒ ΠΊ полюсам кондСнсатора.Для ΠΏΠΎΠ»Π½ΠΎΠΉ ΠΏΡ€ΠΎΠ²Π΅Ρ€ΠΊΠΈ кондСнсатора ΠΈΠ·ΠΌΠ΅Ρ€ΠΈΡ‚Π΅Π»ΡŒΠ½Ρ‹Π΅ Π»ΠΈΠ½ΠΈΠΈ Π½Π΅ΠΎΠ±Ρ…ΠΎΠ΄ΠΈΠΌΠΎ ΠΏΡ€ΠΈΠΌΠ΅Π½ΠΈΡ‚ΡŒ Π΄Π²Π°ΠΆΠ΄Ρ‹ ΠΈ ΡΡ€Π°Π²Π½ΠΈΡ‚ΡŒ Ρ€Π΅Π°ΠΊΡ†ΠΈΡŽ ΠΎΠ±ΠΎΠΈΡ… процСссов:

На дисплСС Ρ†ΠΈΡ„Ρ€ΠΎΠ²ΠΎΠ³ΠΎ ΠΌΡƒΠ»ΡŒΡ‚ΠΈΠΌΠ΅Ρ‚Ρ€Π° Ρ‚Π΅ΠΏΠ΅Ρ€ΡŒ Π΄ΠΎΠ»ΠΆΠ½ΠΎ ΠΎΡ‚ΠΎΠ±Ρ€Π°ΠΆΠ°Ρ‚ΡŒΡΡ ΠΈΠ·ΠΌΠ΅Ρ€Π΅Π½Π½ΠΎΠ΅ Π·Π½Π°Ρ‡Π΅Π½ΠΈΠ΅ Π² Ρ‚Π΅Ρ‡Π΅Π½ΠΈΠ΅ Π΄ΠΎΠ»ΠΈ сСкунды, ΠΊΠΎΡ‚ΠΎΡ€ΡƒΡŽ Π²Ρ‹ Π΄ΠΎΠ»ΠΆΠ½Ρ‹ Π·Π°ΠΏΠΎΠΌΠ½ΠΈΡ‚ΡŒ. ПослС этого дисплСй ΠΈΠ·ΠΌΠ΅Ρ€Π΅Π½ΠΈΠΉ Π½Π΅ΠΌΠ΅Π΄Π»Π΅Π½Π½ΠΎ ΠΏΠ΅Ρ€Π΅ΠΉΠ΄Π΅Ρ‚ ΠΊ OL (ΠžΡ‚ΠΊΡ€Ρ‹Ρ‚Π°Ρ линия). Если ΠΈΠ·ΠΌΠ΅Ρ€ΠΈΡ‚Π΅Π»ΡŒΠ½Ρ‹Π΅ Π»ΠΈΠ½ΠΈΠΈ ΡƒΠ΄Π°Π»ΡΡŽΡ‚ΡΡ ΠΈ снова ΠΏΠΎΠ΄ΠΊΠ»ΡŽΡ‡Π°ΡŽΡ‚ΡΡ, Ρ‚ΠΎ ΠΆΠ΅ самоС ΠΈΠ·ΠΌΠ΅Ρ€Π΅Π½Π½ΠΎΠ΅ Π·Π½Π°Ρ‡Π΅Π½ΠΈΠ΅, Π° Π·Π°Ρ‚Π΅ΠΌ OL Π΄ΠΎΠ»ΠΆΠ½ΠΎ снова ΠΏΠΎΡΠ²ΠΈΡ‚ΡŒΡΡ Π½Π° дисплСС. Если это Ρ‚Π°ΠΊ, Ρ‚ΠΎ кондСнсатор Π² порядкС.

2. Как ΠΏΡ€ΠΎΠ²Π΅Ρ€ΠΈΡ‚ΡŒ кондСнсатор с ΠΏΠΎΠΌΠΎΡ‰ΡŒΡŽ ΠΌΡƒΠ»ΡŒΡ‚ΠΈΠΌΠ΅Ρ‚Ρ€Π° для ΠΏΡ€ΠΎΠ²Π΅Ρ€ΠΊΠΈ цСлостности Ρ†Π΅ΠΏΠΈ

ВСстСр цСлостности Ρ†Π΅ΠΏΠΈ с ΠΏΡ€ΠΎΠ²Π΅Ρ€ΠΊΠΎΠΉ Π΄ΠΈΠΎΠ΄ΠΎΠ² встроСн Π²ΠΎ ΠΌΠ½ΠΎΠ³ΠΈΠ΅ ΠΌΠΎΠ΄Π΅Π»ΠΈ ΠΌΡƒΠ»ΡŒΡ‚ΠΈΠΌΠ΅Ρ‚Ρ€ΠΎΠ². Π­Ρ‚ΠΎ Ρ‚Π°ΠΊΠΆΠ΅ ΠΌΠΎΠΆΠ΅Ρ‚ Π±Ρ‹Ρ‚ΡŒ использовано для ΠΏΡ€ΠΎΠ²Π΅Ρ€ΠΊΠΈ кондСнсатора. Однако Ρ‚Π°ΠΊΠΈΠΌ ΠΎΠ±Ρ€Π°Π·ΠΎΠΌ ΠΌΠΎΠΆΠ½ΠΎ Ρ‚ΠΎΠ»ΡŒΠΊΠΎ ΠΎΠΏΡ€Π΅Π΄Π΅Π»ΠΈΡ‚ΡŒ, заряТаСтся Π»ΠΈ кондСнсатор.

Π’ΠΎΠΊ ΠΎΡ‚ ΠΈΠ·ΠΌΠ΅Ρ€ΠΈΡ‚Π΅Π»ΡŒΠ½ΠΎΠ³ΠΎ ΠΏΡ€ΠΈΠ±ΠΎΡ€Π° сначала Ρ‚Π΅Ρ‡Π΅Ρ‚ Π² кондСнсатор, ΠΏΠΎΠΊΠ° ΠΎΠ½ Π½Π΅ зарядится ΠΏΠΎΠ»Π½ΠΎΡΡ‚ΡŒΡŽ. Π—Π°Ρ‚Π΅ΠΌ ΠΌΠΎΠΆΠ½ΠΎ провСсти ΠΈΠ·ΠΌΠ΅Ρ€Π΅Π½ΠΈΠ΅ сопротивлСния. Π—Π°Ρ‚Π΅ΠΌ показания Π½Π° дисплСС Π±ΡƒΠ΄ΡƒΡ‚ постоянно ΡƒΠ²Π΅Π»ΠΈΡ‡ΠΈΠ²Π°Ρ‚ΡŒΡΡ, ΠΏΠΎΠΊΠ° Π½Π΅ Π±ΡƒΠ΄Π΅Ρ‚ достигнут ΠΏΡ€Π΅Π΄Π΅Π» Π΄ΠΈΠ°ΠΏΠ°Π·ΠΎΠ½Π° измСрСния ΠΈ Π½Π΅ Π±ΡƒΠ΄Π΅Ρ‚ ΠΎΡ‚ΠΎΠ±Ρ€Π°ΠΆΠ°Ρ‚ΡŒΡΡ Ρ‚ΠΎΠ»ΡŒΠΊΠΎ 1.

ΠŸΡ€ΠΎΠ²Π΅Ρ€ΠΊΠ° нСпрСрывности с ΠΏΠΎΠΌΠΎΡ‰ΡŒΡŽ Π·Π²ΡƒΠΊΠΎΠ²ΠΎΠ³ΠΎ сигнала

ΠœΡƒΠ»ΡŒΡ‚ΠΈΠΌΠ΅Ρ‚Ρ€Ρ‹, оснащСнныС тСстСром нСпрСрывности с акустичСским сигналом, ΠΎΠ±Π΅ΡΠΏΠ΅Ρ‡ΠΈΠ²Π°ΡŽΡ‚ ΡΠ»Π΅Π΄ΡƒΡŽΡ‰ΡƒΡŽ ΠΎΠ±Ρ€Π°Ρ‚Π½ΡƒΡŽ связь:

  • НСпрСрывный Π·Π²ΡƒΠΊΠΎΠ²ΠΎΠΉ сигнал ΠΈΠ»ΠΈ Π΅Π³ΠΎ отсутствиС ΠΎΠ·Π½Π°Ρ‡Π°Π΅Ρ‚, Ρ‡Ρ‚ΠΎ кондСнсатор нСисправСн.
  • ИзмСнСниС громкости ΠΈΠ»ΠΈ высоты Π·Π²ΡƒΠΊΠ° акустичСского сигнала ΠΎΠ·Π½Π°Ρ‡Π°Π΅Ρ‚, Ρ‡Ρ‚ΠΎ кондСнсатор Π² порядкС.

Π’ ΠΎΠ±ΠΎΠΈΡ… Π²Π°Ρ€ΠΈΠ°Π½Ρ‚Π°Ρ… кондСнсатор ΠΌΠΎΠΆΠ½ΠΎ ΠΏΡ€ΠΎΠ²Π΅Ρ€ΠΈΡ‚ΡŒ Ρ‚ΠΎΠ»ΡŒΠΊΠΎ Π½Π° ΠΊΠΎΡ€ΠΎΡ‚ΠΊΠΎΠ΅ Π·Π°ΠΌΡ‹ΠΊΠ°Π½ΠΈΠ΅ ΠΈΠ»ΠΈ ΠΌΠΎΠΆΠ½ΠΎ ΠΏΡ€ΠΎΠ²Π΅Ρ€ΠΈΡ‚ΡŒ процСсс зарядки. Π’ΠΎΡ‡Π½ΡƒΡŽ Π΅ΠΌΠΊΠΎΡΡ‚ΡŒ кондСнсатора Ρ‚Π°ΠΊΠΈΠΌ способом ΠΈΠ·ΠΌΠ΅Ρ€ΠΈΡ‚ΡŒ нСльзя.

Π’Ρ‹ Ρ‚Π°ΠΊΠΆΠ΅ Π΄ΠΎΠ»ΠΆΠ½Ρ‹ ΡƒΡ‡ΠΈΡ‚Ρ‹Π²Π°Ρ‚ΡŒ, Ρ‡Ρ‚ΠΎ кондСнсаторы ΠΌΠΎΠ³ΡƒΡ‚ Ρ€Π΅Π°Π³ΠΈΡ€ΠΎΠ²Π°Ρ‚ΡŒ ΠΈΠ½Π°Ρ‡Π΅, ΠΊΠΎΠ³Π΄Π° ΠΎΠ½ΠΈ ΡƒΠ΄Π°Π»Π΅Π½Ρ‹, Ρ‡Π΅ΠΌ ΠΊΠΎΠ³Π΄Π° ΠΎΠ½ΠΈ встроСны Π² Ρ†Π΅ΠΏΡŒ. Π‘ малСнькими кондСнсаторами Π² Π΄ΠΈΠ°ΠΏΠ°Π·ΠΎΠ½Π΅ ΠΏΠ€ ΠΈΠ»ΠΈ Π½Π€ измСрСния, бСзусловно, ΠΈΠΌΠ΅ΡŽΡ‚ смысл, Π½ΠΎ с большими кондСнсаторами ΠΎΡ‚ 10 ΠΌΠΊΠ€ становятся Π½Π΅Ρ‚ΠΎΡ‡Π½Ρ‹ΠΌΠΈ, Ρ‚Π°ΠΊ ΠΊΠ°ΠΊ ΠΎΠ½ΠΈ Π²Π΅Π΄ΡƒΡ‚ сСбя ΠΏΡ€ΠΈ ΠΈΠ·ΠΌΠ΅Ρ€Π΅Π½ΠΈΠΈ ΠΈΠ½Π°Ρ‡Π΅, Ρ‡Π΅ΠΌ ΠΏΡ€ΠΈ Π½ΠΎΡ€ΠΌΠ°Π»ΡŒΠ½ΠΎΠΉ Ρ€Π°Π±ΠΎΡ‚Π΅ Π² Ρ€Π΅Π°Π»ΡŒΠ½Ρ‹Ρ… условиях. Π˜Π·ΠΌΠ΅Ρ€Π΅Π½ΠΈΠ΅ кондСнсаторов Π² Ρ†Π΅ΠΏΠΈ, Π½ΠΎ это большС для профСссионалов, Ρ‡Π΅ΠΌ для элСктриков-Π»ΡŽΠ±ΠΈΡ‚Π΅Π»Π΅ΠΉ.

Π£Π·Π½Π°ΠΉΡ‚Π΅ большС ΠΎ Ρ‚ΠΎΡ‡Π½ΠΎΠΉ ΠΏΡ€ΠΎΡ†Π΅Π΄ΡƒΡ€Π΅ провСдСния ΠΏΡ€ΠΎΠ²Π΅Ρ€ΠΊΠΈ нСпрСрывности с ΠΏΠΎΠΌΠΎΡ‰ΡŒΡŽ ΠΌΡƒΠ»ΡŒΡ‚ΠΈΠΌΠ΅Ρ‚Ρ€Π° Π² руководствС ΠΏΠΎ эксплуатации ΠΌΡƒΠ»ΡŒΡ‚ΠΈΠΌΠ΅Ρ‚Ρ€Π° ΠΈ ΡƒΠ·Π½Π°ΠΉΡ‚Π΅ всС, Ρ‡Ρ‚ΠΎ Π²Π°ΠΌ Π½ΡƒΠΆΠ½ΠΎ ΡƒΡ‡ΠΈΡ‚Ρ‹Π²Π°Ρ‚ΡŒ.

3. ИспользованиС ΠΌΡƒΠ»ΡŒΡ‚ΠΈΠΌΠ΅Ρ‚Ρ€Π° с ΠΈΠ·ΠΌΠ΅Ρ€Π΅Π½ΠΈΠ΅ΠΌ Смкости

ΠŸΡ€ΠΈ Π½Π°Π»ΠΈΡ‡ΠΈΠΈ ΠΌΡƒΠ»ΡŒΡ‚ΠΈΠΌΠ΅Ρ‚Ρ€Π°, способного ΠΈΠ·ΠΌΠ΅Ρ€ΡΡ‚ΡŒ Смкости, ΠΌΠΎΠΆΠ½ΠΎ провСсти прямоС ΠΈΠ·ΠΌΠ΅Ρ€Π΅Π½ΠΈΠ΅ Смкости кондСнсатора ΠΈΠ»ΠΈ элСктролитичСского кондСнсатора (элСктролитичСского кондСнсатора). Π’Ρ‹ΠΏΠΎΠ»Π½ΠΈΡ‚Π΅ ΡΠ»Π΅Π΄ΡƒΡŽΡ‰ΠΈΠ΅ дСйствия:

1. Π’Ρ‹ΡΡ‚Π°Π²ΠΈΡ‚ΡŒ кондСнсатор

Π—Π΄Π΅ΡΡŒ Ρ‚ΠΎΠΆΠ΅ ΠΏΠ΅Ρ€Π²Ρ‹ΠΌ Π΄Π΅Π»ΠΎΠΌ Π½ΡƒΠΆΠ½ΠΎ ΠΏΠΎΠ»Π½ΠΎΡΡ‚ΡŒΡŽ Π²Ρ‹Π½ΡƒΡ‚ΡŒ ΠΈΠ· схСмы провСряСмый кондСнсатор. ВсС ΠΊΠΎΠ½Ρ‚Π°ΠΊΡ‚Ρ‹ Ρ†Π΅ΠΏΠΈ Π΄ΠΎΠ»ΠΆΠ½Ρ‹ Π±Ρ‹Ρ‚ΡŒ ΡƒΠ΄Π°Π»Π΅Π½Ρ‹, Π° Π΄Π²Π° полюса кондСнсатора Π΄ΠΎΠ»ΠΆΠ½Ρ‹ Π±Ρ‹Ρ‚ΡŒ доступны.

2. Π’ΠΈΠ·ΡƒΠ°Π»ΡŒΠ½ΠΎ ΠΏΡ€ΠΎΠ²Π΅Ρ€ΡŒΡ‚Π΅ кондСнсатор

ΠŸΠ΅Ρ€Π΅Π΄ ΠΈΠ·ΠΌΠ΅Ρ€Π΅Π½ΠΈΠ΅ΠΌ Смкости ΠΌΡƒΠ»ΡŒΡ‚ΠΈΠΌΠ΅Ρ‚Ρ€ΠΎΠΌ слСдуСт ΠΏΡ€ΠΎΠ²Π΅Ρ€ΠΈΡ‚ΡŒ кондСнсатор Π½Π° Π½Π°Π»ΠΈΡ‡ΠΈΠ΅ ΠΏΠΎΠ²Ρ€Π΅ΠΆΠ΄Π΅Π½ΠΈΠΉ. Если Π½Π° повСрхности Π²ΠΈΠ΄Π½Ρ‹ нСбольшиС выпуклости, ΠΌΠ΅Π»ΠΊΠΈΠ΅ Ρ‚Ρ€Π΅Ρ‰ΠΈΠ½Ρ‹ ΠΈΠ»ΠΈ Π΄Π°ΠΆΠ΅ Π²Ρ‹Ρ‚Π΅ΠΊΠ°ΡŽΡ‰ΠΈΠ΅ Тидкости, это ΠΌΠΎΠΆΠ΅Ρ‚ ΡƒΠΊΠ°Π·Ρ‹Π²Π°Ρ‚ΡŒ Π½Π° Π½Π΅ΠΈΡΠΏΡ€Π°Π²Π½ΠΎΡΡ‚ΡŒ кондСнсатора.

3. Разрядный кондСнсатор

Π‘Π»Π΅Π΄ΡƒΡŽΡ‰ΠΈΠΌ шагом являСтся полная разрядка кондСнсатора. Π§Ρ‚ΠΎΠ±Ρ‹ ΡΠ½ΡΡ‚ΡŒ вСсь остаточный Ρ‚ΠΎΠΊ с кондСнсатора, Π΅Π³ΠΎ ΠΌΠΎΠΆΠ½ΠΎ ΠΏΠΎΠ΄ΠΊΠ»ΡŽΡ‡ΠΈΡ‚ΡŒ ΠΊ ΠΏΠΎΡ‚Ρ€Π΅Π±ΠΈΡ‚Π΅Π»ΡŽ. Π—Π΄Π΅ΡΡŒ Ρ‚ΠΎΠΆΠ΅ ΠΏΠΎΠ΄ΠΎΠΉΠ΄Π΅Ρ‚ Π»Π°ΠΌΠΏΠΎΡ‡ΠΊΠ°, Π½Π°ΠΏΡ€ΠΈΠΌΠ΅Ρ€, Ρ‡Ρ‚ΠΎΠ±Ρ‹ ΠΏΠΎΠ»Π½ΠΎΡΡ‚ΡŒΡŽ Ρ€Π°Π·Ρ€ΡΠ΄ΠΈΡ‚ΡŒ ΡΠ½Π΅Ρ€Π³ΠΈΡŽ кондСнсатора.

4. Набор ΠΌΡƒΠ»ΡŒΡ‚ΠΈΠΌΠ΅Ρ‚Ρ€Π°

Π’Π΅ΠΏΠ΅Ρ€ΡŒ ΠΌΡƒΠ»ΡŒΡ‚ΠΈΠΌΠ΅Ρ‚Ρ€ Π΄ΠΎΠ»ΠΆΠ΅Π½ Π±Ρ‹Ρ‚ΡŒ настроСн Π½Π° Ρ„ΡƒΠ½ΠΊΡ†ΠΈΡŽ измСрСния Смкости (ΠΈΠ·ΠΌΠ΅Ρ€Π΅Π½Π½Ρ‹Π΅ значСния Π² Ρ„Π°Ρ€Π°Π΄Π°Ρ…).Π”ΠΈΠ°ΠΏΠ°Π·ΠΎΠ½ измСрСния здСсь ΠΎΠ±Ρ‹Ρ‡Π½ΠΎ автоматичСски настраиваСтся устройством.

5. Π˜Π·ΠΌΠ΅Ρ€ΠΈΡ‚ΡŒ Π΅ΠΌΠΊΠΎΡΡ‚ΡŒ кондСнсатора ΠΌΡƒΠ»ΡŒΡ‚ΠΈΠΌΠ΅Ρ‚Ρ€ΠΎΠΌ

Π’Π΅ΠΏΠ΅Ρ€ΡŒ ΠΎΠ±Π΅ ΠΈΠ·ΠΌΠ΅Ρ€ΠΈΡ‚Π΅Π»ΡŒΠ½Ρ‹Π΅ Π»ΠΈΠ½ΠΈΠΈ ΠΌΠΎΠΆΠ½ΠΎ ΠΏΠΎΠ΄ΠΊΠ»ΡŽΡ‡ΠΈΡ‚ΡŒ ΠΊ полюсам кондСнсатора. Π’Π΅ΠΏΠ΅Ρ€ΡŒ Π½Π° дисплСС ΠΌΡƒΠ»ΡŒΡ‚ΠΈΠΌΠ΅Ρ‚Ρ€Π° Π΄ΠΎΠ»ΠΆΠ½ΠΎ ΠΎΡ‚ΠΎΠ±Ρ€Π°ΠΆΠ°Ρ‚ΡŒΡΡ ΠΏΠΎΠΊΠ°Π·Π°Π½ΠΈΠ΅, ΠΏΡ€ΠΈΠΌΠ΅Ρ€Π½ΠΎ ΡΠΎΠΎΡ‚Π²Π΅Ρ‚ΡΡ‚Π²ΡƒΡŽΡ‰Π΅Π΅ Π·Π½Π°Ρ‡Π΅Π½ΠΈΡŽ, ΡƒΠΊΠ°Π·Π°Π½Π½ΠΎΠΌΡƒ Π½Π° кондСнсаторС. Если Π΄Π²Π° значСния ΠΎΡ‡Π΅Π½ΡŒ ΠΏΠΎΡ…ΠΎΠΆΠΈ, кондСнсатор исправСн. Если ΠΎΠΏΡ€Π΅Π΄Π΅Π»Π΅Π½Π½ΠΎΠ΅ ΠΈΠ·ΠΌΠ΅Ρ€Π΅Π½Π½ΠΎΠ΅ Π·Π½Π°Ρ‡Π΅Π½ΠΈΠ΅ Π·Π½Π°Ρ‡ΠΈΡ‚Π΅Π»ΡŒΠ½ΠΎ Π½ΠΈΠΆΠ΅ значСния, ΡƒΠΊΠ°Π·Π°Π½Π½ΠΎΠ³ΠΎ Π½Π° кондСнсаторС, ΠΈΠ»ΠΈ Ссли ΠΈΠ·ΠΌΠ΅Ρ€Π΅Π½Π½ΠΎΠ΅ Π·Π½Π°Ρ‡Π΅Π½ΠΈΠ΅ Π²ΠΎΠΎΠ±Ρ‰Π΅ Π½Π΅ отобраТаСтся, Ρ‚ΠΎ кондСнсатор нСисправСн ΠΈ ΠΏΠΎΠ΄Π»Π΅ΠΆΠΈΡ‚ Π·Π°ΠΌΠ΅Π½Π΅.

ΠžΠ±Ρ‰Π΅Π΅ ΠΏΡ€ΠΈΠΌΠ΅Ρ‡Π°Π½ΠΈΠ΅:

Π’Π°ΠΊ ΠΊΠ°ΠΊ кондСнсаторы ΠΈΠ»ΠΈ элСктролитичСскиС кондСнсаторы хранят элСктричСский Ρ‚ΠΎΠΊ, ΠΎΠ½ΠΈ Π΄ΠΎΠ»ΠΆΠ½Ρ‹ Π±Ρ‹Ρ‚ΡŒ ΠΏΠΎΠ»Π½ΠΎΡΡ‚ΡŒΡŽ разряТСны, ΠΏΡ€Π΅ΠΆΠ΄Π΅ Ρ‡Π΅ΠΌ Π²Ρ‹ смоТСтС ΠΏΡ€ΠΎΠ²Π΅Ρ€ΠΈΡ‚ΡŒ кондСнсатор с ΠΏΠΎΠΌΠΎΡ‰ΡŒΡŽ ΠΌΡƒΠ»ΡŒΡ‚ΠΈΠΌΠ΅Ρ‚Ρ€Π°.

Π‘ ΠΏΠΎΠΌΠΎΡ‰ΡŒΡŽ простых ΠΌΡƒΠ»ΡŒΡ‚ΠΈΠΌΠ΅Ρ‚Ρ€ΠΎΠ² ΠΌΠΎΠΆΠ½ΠΎ Ρ‚ΠΎΠ»ΡŒΠΊΠΎ ΠΎΠΏΡ€Π΅Π΄Π΅Π»ΠΈΡ‚ΡŒ, ΠΈΠΌΠ΅Π΅Ρ‚ Π»ΠΈ кондСнсатор ΠΊΠΎΡ€ΠΎΡ‚ΠΊΠΎΠ΅ Π·Π°ΠΌΡ‹ΠΊΠ°Π½ΠΈΠ΅ ΠΈΠ»ΠΈ ΠΎΠ½ заряТаСтся. Π’ΠΎΡ‡Π½Ρ‹Π΅ ΠΈΠ·ΠΌΠ΅Ρ€Π΅Π½Π½Ρ‹Π΅ значСния Смкости кондСнсатора ΠΌΠΎΠΆΠ½ΠΎ ΠΎΠΏΡ€Π΅Π΄Π΅Π»ΠΈΡ‚ΡŒ Ρ‚ΠΎΠ»ΡŒΠΊΠΎ с ΠΏΠΎΠΌΠΎΡ‰ΡŒΡŽ ΡΠΎΠΎΡ‚Π²Π΅Ρ‚ΡΡ‚Π²ΡƒΡŽΡ‰ΠΈΠΌ ΠΎΠ±Ρ€Π°Π·ΠΎΠΌ ΠΎΠ±ΠΎΡ€ΡƒΠ΄ΠΎΠ²Π°Π½Π½Ρ‹Ρ… ΠΈΠ·ΠΌΠ΅Ρ€ΠΈΡ‚Π΅Π»ΡŒΠ½Ρ‹Ρ… ΠΏΡ€ΠΈΠ±ΠΎΡ€ΠΎΠ².

4.Как ΠΏΡ€ΠΎΠ²Π΅Ρ€ΠΈΡ‚ΡŒ кондСнсатор с ΠΏΠΎΠΌΠΎΡ‰ΡŒΡŽ ΠΎΠΌΠΌΠ΅Ρ‚Ρ€Π°

ΠŸΡ€ΠΎΠ²Π΅Ρ€ΠΈΡ‚ΡŒ кондСнсатор Π² элСктродвигатСлС ΠΌΠΎΠΆΠ½ΠΎ Ρ‚Π°ΠΊΠΆΠ΅ ΠΏΡƒΡ‚Π΅ΠΌ измСрСния сопротивлСния ΠΎΠΌΠΌΠ΅Ρ‚Ρ€ΠΎΠΌ. Π’ этом ΠΈΠ·ΠΌΠ΅Ρ€Π΅Π½ΠΈΠΈ сопротивлСниС Π΄ΠΎΠ»ΠΆΠ½ΠΎ Π½Π°Ρ‡ΠΈΠ½Π°Ρ‚ΡŒΡΡ с Π½ΠΈΠ·ΠΊΠΎΠ³ΠΎ уровня ΠΈ постСпСнно ΡƒΠ²Π΅Π»ΠΈΡ‡ΠΈΠ²Π°Ρ‚ΡŒΡΡ ΠΏΠΎ ΠΌΠ΅Ρ€Π΅ зарядки кондСнсатора. НаиболСС ΠΈΠ½Ρ„ΠΎΡ€ΠΌΠ°Ρ‚ΠΈΠ²Π½Ρ‹ΠΌ ΠΈΠ· ΠΎΠ±ΠΎΠΈΡ… ΠΌΠ΅Ρ‚ΠΎΠ΄ΠΎΠ² измСрСния являСтся сравнСниС с ΠΎΠΏΡ€Π΅Π΄Π΅Π»Π΅Π½Π½ΠΎ Ρ€Π°Π±ΠΎΡ‚Π°ΡŽΡ‰ΠΈΠΌ кондСнсатором двигатСля с Ρ‚Π΅ΠΌΠΈ ΠΆΠ΅ тСхничСскими характСристиками. Если отклонСния указатСля Π²Π΅Π΄ΡƒΡ‚ сСбя ΠΎΠ΄ΠΈΠ½Π°ΠΊΠΎΠ²ΠΎ с Ρ‚ΠΎΡ‡ΠΊΠΈ зрСния интСнсивности ΠΈ Π²Ρ€Π΅ΠΌΠ΅Π½Π½ΠΎΠΉ прогрСссии, кондСнсатор, вСроятно, Π² порядкС.

5. Как ΠΏΡ€ΠΎΠ²Π΅Ρ€ΠΈΡ‚ΡŒ кондСнсатор ΠΏΡƒΡ‚Π΅ΠΌ Π΅Π³ΠΎ ΠΊΠΎΡ€ΠΎΡ‚ΠΊΠΎΠ³ΠΎ замыкания

Π’ Π½Π΅ΠΊΠΎΡ‚ΠΎΡ€Ρ‹Ρ… ситуациях состояниС элСктролитичСского кондСнсатора ΠΌΠΎΠΆΠ½ΠΎ ΠΏΡ€ΠΎΠ²Π΅Ρ€ΠΈΡ‚ΡŒ Ρ‚ΠΎΠ»ΡŒΠΊΠΎ Π±Π΅Π· ΠΎΠΌΠΌΠ΅Ρ‚Ρ€Π° ΠΈΠ»ΠΈ Π²ΠΎΠ»ΡŒΡ‚ΠΌΠ΅Ρ‚Ρ€Π° ΠΏΡ€ΠΈ Π½Π°Π»ΠΈΡ‡ΠΈΠΈ подходящСго источника напряТСния. ΠšΠΎΠ½Π΄Π΅Π½ΡΠ°Ρ‚ΠΎΡ€ заряТаСтся Π·Π° 1-2 сСкунды. Π—Π°Ρ‚Π΅ΠΌ Π½ΡƒΠΆΠ½ΠΎ Π·Π°ΠΌΠΊΠ½ΡƒΡ‚ΡŒ ΠΊΠΎΠ½Ρ‚Π°ΠΊΡ‚Ρ‹ мСталличСской ΠΎΡ‚Π²Π΅Ρ€Ρ‚ΠΊΠΎΠΉ.

Π˜ΡΠΏΡ€Π°Π²Π½Ρ‹ΠΉ кондСнсатор Π΄ΠΎΠ»ΠΆΠ΅Π½ Π΄Π°Π²Π°Ρ‚ΡŒ ΡΡ€ΠΊΡƒΡŽ искру. Если ΠΎΠ½ тусклый ΠΈΠ»ΠΈ Π΅Π΄Π²Π° Π·Π°ΠΌΠ΅Ρ‚Π΅Π½, это Π³ΠΎΠ²ΠΎΡ€ΠΈΡ‚ ΠΎ Ρ‚ΠΎΠΌ, Ρ‡Ρ‚ΠΎ кондСнсатор нСисправСн ΠΈ ΠΏΠ»ΠΎΡ…ΠΎ Π΄Π΅Ρ€ΠΆΠΈΡ‚ заряд.

US Tech Online -> Π’ΠΎΡ‡Π½ΠΎΠ΅ ΠΈΠ·ΠΌΠ΅Ρ€Π΅Π½ΠΈΠ΅ Ρ‡Ρ€Π΅Π·Π²Ρ‹Ρ‡Π°ΠΉΠ½ΠΎ ΠΌΠ°Π»Ρ‹Ρ… Π·Π½Π°Ρ‡Π΅Π½ΠΈΠΉ Смкости

ΠŸΠΎΠ»ΡŒΠ·ΠΎΠ²Π°Ρ‚Π΅Π»ΠΈ Ρ‚Π°ΠΊΠΆΠ΅ ΠΌΠΎΠ³ΡƒΡ‚ Π²Ρ€ΡƒΡ‡Π½ΡƒΡŽ Π²Ρ‹Π±Ρ€Π°Ρ‚ΡŒ Ρ€Π΅ΠΆΠΈΠΌ измСрСния, Π° частоту тСстового сигнала ΠΌΠΎΠΆΠ½ΠΎ Π²Ρ‹Π±Ρ€Π°Ρ‚ΡŒ Π² Π΄ΠΈΠ°ΠΏΠ°Π·ΠΎΠ½Π΅ фиксированных Π·Π½Π°Ρ‡Π΅Π½ΠΈΠΉ ΠΎΡ‚ 100 Π“Ρ† Π΄ΠΎ 100 ΠΊΠ“Ρ†. Π˜ΡΠΏΡ‹Ρ‚Π°Ρ‚Π΅Π»ΡŒΠ½ΠΎΠ΅ напряТСниС ΠΌΠΎΠΆΠ΅Ρ‚ Π±Ρ‹Ρ‚ΡŒ установлСно Π½Π° 1,0, 0,5 ΠΈ 0,1 Π’ (срСднСквадратичноС Π·Π½Π°Ρ‡Π΅Π½ΠΈΠ΅).

ΠŸΡƒΡ‚Π΅ΠΌ пропускания постоянного Ρ‚ΠΎΠΊΠ° Ρ‡Π΅Ρ€Π΅Π· измСряСмый ΠΊΠΎΠΌΠΏΠΎΠ½Π΅Π½Ρ‚ ΠΌΠΎΠΆΠ½ΠΎ ΠΈΠ·ΠΌΠ΅Ρ€ΠΈΡ‚ΡŒ напряТСниС ΠΈ Ρ‚ΠΎΠΊ. По Π·Π°ΠΊΠΎΠ½Ρƒ Ома рассчитываСтся сопротивлСниС постоянному Ρ‚ΠΎΠΊΡƒ (RDC). ΠŸΠΎΠ΄Π°Ρ‡Π΅ΠΉ постоянного напряТСния Π² прямом ΠΈ ΠΎΠ±Ρ€Π°Ρ‚Π½ΠΎΠΌ Π½Π°ΠΏΡ€Π°Π²Π»Π΅Π½ΠΈΠΈ ΠΎΠ±Π½Π°Ρ€ΡƒΠΆΠΈΠ²Π°ΡŽΡ‚ΡΡ Π΄ΠΈΠΎΠ΄Ρ‹ ΠΈ опрСдСляСтся ΠΏΠΎΠ»ΡΡ€Π½ΠΎΡΡ‚ΡŒ p-n ΠΏΠ΅Ρ€Π΅Ρ…ΠΎΠ΄Π°.

Для кондСнсаторов Π΅ΠΌΠΊΠΎΡΡ‚ΡŒΡŽ Π±ΠΎΠ»Π΅Π΅ 40 ΠΌΠ€ Π΅ΠΌΠΊΠΎΡΡ‚ΡŒ рассчитываСтся с использованиСм измСнСния напряТСния Π½Π° ΠΈΠ·ΠΌΠ΅Ρ€Π΅Π½Π½ΠΎΠΌ кондСнсаторС ΠΏΡ€ΠΈ Π΅Π³ΠΎ зарядкС Π² Ρ‚Π΅Ρ‡Π΅Π½ΠΈΠ΅ ΠΎΠΏΡ€Π΅Π΄Π΅Π»Π΅Π½Π½ΠΎΠ³ΠΎ ΠΈΠ½Ρ‚Π΅Ρ€Π²Π°Π»Π° Π²Ρ€Π΅ΠΌΠ΅Π½ΠΈ ΠΈ ΠΏΡ€ΠΈΠ»ΠΎΠΆΠ΅Π½Π½ΠΎΠ³ΠΎ Ρ‚ΠΎΠΊΠ°. ΠŸΡ€ΠΈΠ½Ρ†ΠΈΠΏ Ρ€Π°Π±ΠΎΡ‚Ρ‹ частотомСра основан Π½Π° подсчСтС ΠΈΠΌΠΏΡƒΠ»ΡŒΡΠΎΠ² ΠΎΠΏΠΎΡ€Π½ΠΎΠ³ΠΎ Π³Π΅Π½Π΅Ρ€Π°Ρ‚ΠΎΡ€Π° ΠΌΠ΅ΠΆΠ΄Ρƒ двумя Ρ€Π°ΠΌΠΏΠ°ΠΌΠΈ Π²Ρ…ΠΎΠ΄Π½ΠΎΠ³ΠΎ сигнала Π·Π° ΠΎΠΏΡ€Π΅Π΄Π΅Π»Π΅Π½Π½Ρ‹ΠΉ ΠΏΠ΅Ρ€ΠΈΠΎΠ΄. ΠŸΡ€ΠΈ этом Ρ‚Π°ΠΊΠΆΠ΅ подсчитываСтся количСство ΠΏΠ΅Ρ€ΠΈΠΎΠ΄ΠΎΠ² Π²Ρ…ΠΎΠ΄Π½ΠΎΠ³ΠΎ сигнала.

Π—Π°Ρ‚Π΅ΠΌ вычисляСтся частота ΠΏΡƒΡ‚Π΅ΠΌ дСлСния количСства ΠΏΠ΅Ρ€ΠΈΠΎΠ΄ΠΎΠ² Π²Ρ…ΠΎΠ΄Π½ΠΎΠ³ΠΎ сигнала Π½Π° количСство ΠΈΠΌΠΏΡƒΠ»ΡŒΡΠΎΠ² ΠΎΡ‚ ΠΎΠΏΠΎΡ€Π½ΠΎΠ³ΠΎ Π³Π΅Π½Π΅Ρ€Π°Ρ‚ΠΎΡ€Π° ΠΈ умноТСния Π½Π° частоту ΠΎΠΏΠΎΡ€Π½ΠΎΠ³ΠΎ Π³Π΅Π½Π΅Ρ€Π°Ρ‚ΠΎΡ€Π°.По сути, ΠΏΡ€ΠΈΠ½Ρ†ΠΈΠΏ измСрСния напряТСния основан Π½Π° сравнСнии Π²Ρ…ΠΎΠ΄Π½ΠΎΠ³ΠΎ сигнала с ΠΎΠΏΠΎΡ€Π½Ρ‹ΠΌ напряТСниСм.

ΠšΠ°Π»ΠΈΠ±Ρ€ΠΎΠ²ΠΊΠ° смСщСния Смкости
Siborg Ρ‚Π°ΠΊΠΆΠ΅ ΠΏΡ€Π΅Π΄Π»Π°Π³Π°Π΅Ρ‚ ΠΊΠ°Π»ΠΈΠ±Ρ€ΠΎΠ²ΠΎΡ‡Π½ΡƒΡŽ ΠΏΠ»Π°Ρ‚Ρƒ смСщСния Смкости, которая обСспСчиваСт Π½Π°Π΄Π΅ΠΆΠ½Ρ‹ΠΉ ΠΌΠ΅Ρ‚ΠΎΠ΄ опрСдСлСния ΠΏΠ°Ρ€Π°Π·ΠΈΡ‚Π½ΠΎΠ³ΠΎ смСщСния ΠΌΠ΅ΠΆΠ΄Ρƒ ΠΈΠ·ΠΌΠ΅Ρ€ΠΈΡ‚Π΅Π»ΡŒΠ½Ρ‹ΠΌΠΈ Π²Ρ‹Π²ΠΎΠ΄Π°ΠΌΠΈ. ΠœΠ°ΠΊΠ΅Ρ‚ ΠΏΠ΅Ρ‡Π°Ρ‚Π½ΠΎΠΉ ΠΏΠ»Π°Ρ‚Ρ‹ ΠΈΡΠΏΠΎΠ»ΡŒΠ·ΡƒΠ΅Ρ‚ отвСрстия для прСдставлСния ΠΊΠΎΠΌΠΏΠΎΠ½Π΅Π½Ρ‚ΠΎΠ² Ρ€Π°Π·Π»ΠΈΡ‡Π½Ρ‹Ρ… Ρ€Π°Π·ΠΌΠ΅Ρ€ΠΎΠ².

ПослС Π½Π°Π΄Π»Π΅ΠΆΠ°Ρ‰Π΅Π³ΠΎ провСдСния ΠΎΡ‚ΠΊΡ€Ρ‹Ρ‚ΠΎΠΉ ΠΊΠ°Π»ΠΈΠ±Ρ€ΠΎΠ²ΠΊΠΈ ΠΏΡ€ΠΈΠ±ΠΎΡ€Π° для ΠΊΠΎΠ½ΠΊΡ€Π΅Ρ‚Π½ΠΎΠ³ΠΎ Ρ€Π°Π·ΠΌΠ΅Ρ€Π° ΠΊΠΎΠΌΠΏΠΎΠ½Π΅Π½Ρ‚Π° ΠΌΠΎΠΆΠ½ΠΎ ΠΏΡ€ΠΎΠΈΠ·Π²ΠΎΠ΄ΠΈΡ‚ΡŒ Π°Π±ΡΠΎΠ»ΡŽΡ‚Π½Ρ‹Π΅ измСрСния Π·Π½Π°Ρ‡Π΅Π½ΠΈΠΉ ΠΊΠΎΠΌΠΏΠΎΠ½Π΅Π½Ρ‚ΠΎΠ² с Ρ‚ΠΎΡ‡Π½ΠΎΡΡ‚ΡŒΡŽ Π΄ΠΎ Ρ‚Ρ€Π΅Ρ… Ρ„Π΅ΠΌΡ‚ΠΎΡ„Π°Ρ€Π°Π΄.

НапримСр, открытая ΠΊΠ°Π»ΠΈΠ±Ρ€ΠΎΠ²ΠΊΠ° Π±Ρ‹Π»Π° Π²Ρ‹ΠΏΠΎΠ»Π½Π΅Π½Π° с Ρ€Π°Π·ΠΌΠ΅Ρ€ΠΎΠΌ ΠΊΠΎΠΌΠΏΠΎΠ½Π΅Π½Ρ‚Π° 2920 (7,4 ΠΌΠΌ ΠΌΠ΅ΠΆΠ΄Ρƒ ΠΊΠΎΠ½Ρ‡ΠΈΠΊΠ°ΠΌΠΈ ΠΏΠΈΠ½Ρ†Π΅Ρ‚Π°). Π Π΅Π·ΡƒΠ»ΡŒΡ‚Π°Ρ‚Ρ‹ Ρ€Π°Π·Π»ΠΈΡ‡Π°ΡŽΡ‚ΡΡ Π½Π΅Π·Π½Π°Ρ‡ΠΈΡ‚Π΅Π»ΡŒΠ½ΠΎ, Π² зависимости ΠΎΡ‚ расстояния ΠΌΠ΅ΠΆΠ΄Ρƒ ΠΊΠΎΠ½Ρ‡ΠΈΠΊΠ°ΠΌΠΈ ΠΏΠΈΠ½Ρ†Π΅Ρ‚Π° ΠΈ ΠΈΡ… ΠΎΠΊΡ€ΡƒΠΆΠ΅Π½ΠΈΠ΅ΠΌ. ПоднСсСниС Ρ€ΡƒΠΊΠΈ ΠΊ ΠΏΠΈΠ½Ρ†Π΅Ρ‚Ρƒ ΠΌΠΎΠΆΠ΅Ρ‚ привСсти ΠΊ смСнС Π½Π΅ΡΠΊΠΎΠ»ΡŒΠΊΠΈΡ… Ρ„Π΅ΠΌΡ‚ΠΎΡ„Π°Ρ€Π°Π΄.

На ΠΏΡ€Π°ΠΊΡ‚ΠΈΠΊΠ΅ ΠΏΡ€ΠΈ использовании ΠΊΠΎΠΌΠΏΠΎΠ½Π΅Π½Ρ‚Π° 01005 ΠΈ расстояния ΠΌΠ΅ΠΆΠ΄Ρƒ ΠΊΠΎΠ½Ρ†Π°ΠΌΠΈ ΠΏΠΈΠ½Ρ†Π΅Ρ‚Π° 0,4 ΠΌΠΌ измСрСнная Π΅ΠΌΠΊΠΎΡΡ‚ΡŒ составляСт 0,249 ΠΏΠΈΠΊΠΎΡ„Π°Ρ€Π°Π΄. 0201 с расстояниСм ΠΌΠ΅ΠΆΠ΄Ρƒ ΠΊΠΎΠ½Ρ‡ΠΈΠΊΠ°ΠΌΠΈ ΠΏΠΈΠ½Ρ†Π΅Ρ‚Π° 0,6 ΠΌΠΌ ΠΏΠΎΠΊΠ°Π·Ρ‹Π²Π°Π΅Ρ‚ Π·Π½Π°Ρ‡Π΅Π½ΠΈΠ΅ 0.225 ΠΏΠΈΠΊΠΎΡ„Π°Ρ€Π°Π΄. Π”Ρ€ΡƒΠ³ΠΈΠΌ ΠΏΡ€ΠΈΠΌΠ΅Ρ€ΠΎΠΌ являСтся Π·Π½Π°Ρ‡Π΅Π½ΠΈΠ΅ измСрСния 0,177 ΠΏΠΈΠΊΠΎΡ„Π°Ρ€Π°Π΄ для ΠΊΠΎΠΌΠΏΠΎΠ½Π΅Π½Ρ‚Π° 0402 с 1 ΠΌΠΌ ΠΌΠ΅ΠΆΠ΄Ρƒ ΠΊΠΎΠ½Ρ‡ΠΈΠΊΠ°ΠΌΠΈ ΠΏΠΈΠ½Ρ†Π΅Ρ‚Π°.

Siborg Systems Ρ€Π°Π·Ρ€Π°Π±Π°Ρ‚Ρ‹Π²Π°Π΅Ρ‚ ΠΈ ΠΏΡ€ΠΎΠΈΠ·Π²ΠΎΠ΄ΠΈΡ‚ Ρ†ΠΈΡ„Ρ€ΠΎΠ²Ρ‹Π΅ ΠΌΡƒΠ»ΡŒΡ‚ΠΈΠΌΠ΅Ρ‚Ρ€Ρ‹ с 2004 Π³ΠΎΠ΄Π°. Основная Π»ΠΈΠ½Π΅ΠΉΠΊΠ° ΠΏΡ€ΠΎΠ΄ΡƒΠΊΡ‚ΠΎΠ² ΠΊΠΎΠΌΠΏΠ°Π½ΠΈΠΈ β€” это ΠΌΡƒΠ»ΡŒΡ‚ΠΈΠΌΠ΅Ρ‚Ρ€Ρ‹ LCR-Reader, ΠΊΠΎΡ‚ΠΎΡ€Ρ‹Π΅ ΡΠ²Π»ΡΡŽΡ‚ΡΡ ΡƒΠ½ΠΈΠ²Π΅Ρ€ΡΠ°Π»ΡŒΠ½Ρ‹ΠΌΠΈ ΠΈ Ρ‚ΠΎΡ‡Π½Ρ‹ΠΌΠΈ ΠΏΠΈΠ½Ρ†Π΅Ρ‚Π°ΠΌΠΈ ΠΈ ΠΎΠ±Π΅ΡΠΏΠ΅Ρ‡ΠΈΠ²Π°ΡŽΡ‚ Π±Π°Π·ΠΎΠ²ΡƒΡŽ Ρ‚ΠΎΡ‡Π½ΠΎΡΡ‚ΡŒ 0,1 ΠΏΡ€ΠΎΡ†Π΅Π½Ρ‚Π° для ΠΈΠ·ΠΌΠ΅Ρ€Π΅Π½ΠΈΠΉ L-C-R.

Как ΠΈΠ·ΠΌΠ΅Ρ€ΠΈΡ‚ΡŒ Π΅ΠΌΠΊΠΎΡΡ‚ΡŒ — Как

Какой самый Π½Π΅Π΄ΠΎΡ€ΠΎΠ³ΠΎΠΉ способ ΠΈΠ·ΠΌΠ΅Ρ€ΠΈΡ‚ΡŒ Π΅ΠΌΠΊΠΎΡΡ‚ΡŒ? Π£ мСня Π΅ΡΡ‚ΡŒ нСсколько кондСнсаторов для Ρ„ΠΎΡ‚ΠΎΠ²ΡΠΏΡ‹ΡˆΠ΅ΠΊ, Π½Π° ΠΊΠΎΡ‚ΠΎΡ€Ρ‹Ρ… Π½Π΅ ΡƒΠΊΠ°Π·Π°Π½Π° ΠΈΡ… Π΅ΠΌΠΊΠΎΡΡ‚ΡŒ.Π― пошСл Π² Ρ€Π°Π΄ΠΈΠΎΠ»Π°Π²ΠΊΡƒ, ΠΈ самый Π΄Π΅ΡˆΠ΅Π²Ρ‹ΠΉ ΠΌΡƒΠ»ΡŒΡ‚ΠΈΠΌΠ΅Ρ‚Ρ€, ΠΊΠΎΡ‚ΠΎΡ€Ρ‹ΠΉ ΠΈΠ·ΠΌΠ΅Ρ€ΠΈΠ» это, стоил ΠΎΠΊΠΎΠ»ΠΎ 60 Π΄ΠΎΠ»Π»Π°Ρ€ΠΎΠ². Π•ΡΡ‚ΡŒ Π»ΠΈ Π΄Π΅ΡˆΠ΅Π²Ρ‹ΠΉ способ ΡƒΠ·Π½Π°Ρ‚ΡŒ Π½ΠΎΠΌΠΈΠ½Π°Π» ΠΌΠΎΠΈΡ… кондСнсаторов?

ΠžΠ±ΡΡƒΠΆΠ΄Π΅Π½ΠΈΡ

Π’Ρ‹, ΠΎΡ‡Π΅Π²ΠΈΠ΄Π½ΠΎ, довольно ΡƒΠΌΠ½Ρ‹ Π² этой области, поэтому я задаю Π²Π°ΠΌ вопрос.

Как ΡƒΠ·Π½Π°Ρ‚ΡŒ, какая Π΅ΠΌΠΊΠΎΡΡ‚ΡŒ ΠΌΠ½Π΅ Π½ΡƒΠΆΠ½Π°? Π•ΡΡ‚ΡŒ ΡƒΡ€Π°Π²Π½Π΅Π½ΠΈΠ΅? МнС Π½ΡƒΠΆΠ½ΠΎ 5 Π’, разряТСнноС Π² Ρ‚Π΅Ρ‡Π΅Π½ΠΈΠ΅ 2 сСкунд ΠΎΡ‚ кондСнсатора для ΠΌΠΎΠ΅Π³ΠΎ ΠΏΡ€ΠΎΠ΅ΠΊΡ‚Π° «сдСлай сам», Π½ΠΎ Π½ΠΈΠΊΡ‚ΠΎ Π½Π΅ ΠΌΠΎΠΆΠ΅Ρ‚ Π΄Π°Ρ‚ΡŒ ΠΌΠ½Π΅ прямой ΠΎΡ‚Π²Π΅Ρ‚ Π½Π° Π·Π½Π°Ρ‡Π΅Π½ΠΈΠ΅ Смкости, ΠΊΠΎΡ‚ΠΎΡ€ΠΎΠ΅ ΠΌΠ½Π΅ Π½ΡƒΠΆΠ½ΠΎ.КакиС-Π»ΠΈΠ±ΠΎ прСдлоТСния?

ΠžΡ‚Π²Π΅Ρ‚ 8 Π»Π΅Ρ‚ Π½Π°Π·Π°Π΄

Π―Ρ†Π΅ΠΊ, заглянитС Π½Π° эту страницу:

ΠΈ ΠΈΡΠΏΠΎΠ»ΡŒΠ·ΡƒΠΉΡ‚Π΅ ΠΏΡ€Π΅Π΄ΠΎΡΡ‚Π°Π²Π»Π΅Π½Π½ΡƒΡŽ Ρ„ΠΎΡ€ΠΌΡƒΠ»Ρƒ. Π’Π°ΠΌ Π²Ρ‹ Π²Π²ΠΎΠ΄ΠΈΡ‚Π΅ 5 Π²ΠΎΠ»ΡŒΡ‚ Π² качСствС Π½Π°Ρ‡Π°Π»ΡŒΠ½ΠΎΠ³ΠΎ напряТСния, Π° Π·Π°Ρ‚Π΅ΠΌ устанавливаСтС врСмя Ρ€Π°Π²Π½Ρ‹ΠΌ 2 сСкундам. Π—Π°Ρ‚Π΅ΠΌ Π²Ρ‹ ΠΌΠΎΠΆΠ΅Ρ‚Π΅ Π½Π°ΠΉΡ‚ΠΈ Π·Π½Π°Ρ‡Π΅Π½ΠΈΠ΅ RC.

Π― Π½Π΅ знаю, ΠΊΠ°ΠΊ Ρ‚ΠΎΡ‡Π½ΠΎ ΠΈΠ·ΠΌΠ΅Ρ€ΠΈΡ‚ΡŒ Π΅ΠΌΠΊΠΎΡΡ‚ΡŒ кондСнсатора, Π½ΠΎ я ΠΌΠΎΠ³Ρƒ ΠΎΡ†Π΅Π½ΠΈΡ‚ΡŒ Π΅ΠΌΠΊΠΎΡΡ‚ΡŒ кондСнсатора Π±Π»ΠΈΠ·ΠΊΠΎ ΠΊ фактичСскому Π·Π½Π°Ρ‡Π΅Π½ΠΈΡŽ с ΠΏΠΎΠΌΠΎΡ‰ΡŒΡŽ Π΄Π΅ΡˆΠ΅Π²Ρ‹Ρ… Π΄Π΅Ρ‚Π°Π»Π΅ΠΉ.

ВсС, Ρ‡Ρ‚ΠΎ Π²Π°ΠΌ Π½ΡƒΠΆΠ½ΠΎ ΡΠ΄Π΅Π»Π°Ρ‚ΡŒ, это ΠΏΠΎΠ΄ΠΊΠ»ΡŽΡ‡ΠΈΡ‚ΡŒ рСзистор с извСстным Π·Π½Π°Ρ‡Π΅Π½ΠΈΠ΅ΠΌ (Π² ΠΌΠ΅Π³Π°ΠΎΠΌΠ°Ρ…), Ρ†ΠΈΡ„Ρ€ΠΎΠ²ΠΎΠΉ ΠΌΡƒΠ»ΡŒΡ‚ΠΈΠΌΠ΅Ρ‚Ρ€ ΠΈ кондСнсатор, ΠΊΠΎΡ‚ΠΎΡ€Ρ‹ΠΉ Π±ΡƒΠ΄Π΅Ρ‚ ΠΈΠ·ΠΌΠ΅Ρ€ΡΡ‚ΡŒΡΡ ΠΏΠ°Ρ€Π°Π»Π»Π΅Π»ΡŒΠ½ΠΎ.

ΠŸΡ€Π΅ΠΆΠ΄Π΅ Ρ‡Π΅ΠΌ ΠΏΠΎΠ΄ΠΊΠ»ΡŽΡ‡ΠΈΡ‚ΡŒ кондСнсатор с рСзистором ΠΈ ΠΌΡƒΠ»ΡŒΡ‚ΠΈΠΌΠ΅Ρ‚Ρ€ΠΎΠΌ ΠΏΠ°Ρ€Π°Π»Π»Π΅Π»ΡŒΠ½ΠΎ,
зарядитС кондСнсатор извСстным источником постоянного напряТСния (Π»ΡƒΡ‡ΡˆΠ΅ всСго Ρ€Π°Π±ΠΎΡ‚Π°Π΅Ρ‚ батарСя). ПослС Ρ‚ΠΎΠ³ΠΎ, ΠΊΠ°ΠΊ кондСнсатор ΠΏΠΎΠ»Π½ΠΎΡΡ‚ΡŒΡŽ заряТСн, ΠΏΠΎΠ΄Π³ΠΎΡ‚ΠΎΠ²ΡŒΡ‚Π΅ сСкундомСр ΠΈ Π΄Π°ΠΉΡ‚Π΅ сСкундомСру Π½Π°Ρ‡Π°Ρ‚ΡŒ отсчСт, ΠΊΠ°ΠΊ Ρ‚ΠΎΠ»ΡŒΠΊΠΎ Π²Ρ‹ ΠΏΠΎΠ΄ΠΊΠ»ΡŽΡ‡ΠΈΡ‚Π΅ кондСнсатор ΠΏΠ°Ρ€Π°Π»Π»Π΅Π»ΡŒΠ½ΠΎ.

УстановитС ΠΎΠΏΠΎΡ€Π½ΠΎΠ΅ напряТСниС Ρ‚Π°ΠΊΠΈΠΌ ΠΎΠ±Ρ€Π°Π·ΠΎΠΌ, Ρ‡Ρ‚ΠΎΠ±Ρ‹ ΠΎΡΡ‚Π°Π½ΠΎΠ²ΠΈΡ‚ΡŒ отсчСт сСкундомСра послС Ρ‚ΠΎΠ³ΠΎ, ΠΊΠ°ΠΊ Π²Ρ‹ ΡƒΠ²ΠΈΠ΄ΠΈΡ‚Π΅ Π·Π½Π°Ρ‡Π΅Π½ΠΈΠ΅ ΠΎΠΏΠΎΡ€Π½ΠΎΠ³ΠΎ напряТСния, ΠΎΡ‚ΠΎΠ±Ρ€Π°ΠΆΠ°Π΅ΠΌΠΎΠ΅ ΠΌΡƒΠ»ΡŒΡ‚ΠΈΠΌΠ΅Ρ‚Ρ€ΠΎΠΌ, Ρ‚. Π΅. 100 ΠΌΠ’.(-t/CR)
ln(V) = ln(Vo)-t/CR
ln(V/Vo) = -t/CR
ln(Vo/V) = t/CR

НаконСц,
C = t/[ln(Vo/V)R], Π² Ρ„Π°Ρ€Π°Π΄Π°Ρ…

ΠŸΠΎΡΠΊΠΎΠ»ΡŒΠΊΡƒ кондСнсатор ΠΈ ΠΌΡƒΠ»ΡŒΡ‚ΠΈΠΌΠ΅Ρ‚Ρ€ ΠΈΠΌΠ΅ΡŽΡ‚ Π²Π½ΡƒΡ‚Ρ€Π΅Π½Π½Π΅Π΅ сопротивлСниС, ΠΈΠ·ΠΌΠ΅Ρ€Π΅Π½Π½ΠΎΠ΅ Π·Π½Π°Ρ‡Π΅Π½ΠΈΠ΅ Π±ΡƒΠ΄Π΅Ρ‚ Π½Π΅ΠΌΠ½ΠΎΠ³ΠΎ ΠΎΡ‚Π»ΠΈΡ‡Π°Ρ‚ΡŒΡΡ ΠΎΡ‚ фактичСского значСния. Π― ΠΏΡ€ΠΎΠ±ΠΎΠ²Π°Π» это Ρ€Π°Π½ΡŒΡˆΠ΅, ΠΈ ΠΎΠ½ Π΄Π΅ΠΉΡΡ‚Π²ΠΈΡ‚Π΅Π»ΡŒΠ½ΠΎ ΠΎΡ†Π΅Π½ΠΈΠ²Π°Π΅Ρ‚ Π΅ΠΌΠΊΠΎΡΡ‚ΡŒ нСизвСстного кондСнсатора.

НадСюсь, это ΠΏΠΎΠΌΠΎΠΆΠ΅Ρ‚.

ΠžΡ‚Π²Π΅Ρ‚ 9 Π»Π΅Ρ‚ Π½Π°Π·Π°Π΄

Π§Ρ‚ΠΎ Π΄Π΅Π»Π°Ρ‚ΡŒ, Ссли Π²ΠΎ врСмя экзамСна я Π½Π΅ ΠΌΠΎΠ³Ρƒ Π²ΡΠΏΠΎΠΌΠ½ΠΈΡ‚ΡŒ Ρ„ΠΎΡ€ΠΌΡƒΠ»Ρƒ.

ΠŸΠ΅Ρ€Π²Ρ‹ΠΉ ΠΌΠ΅Ρ‚ΠΎΠ΄

Π½Π° этой страницС β€” с RC-Ρ†Π΅ΠΏΡŒΡŽ Π²Ρ€Π΅ΠΌΠ΅Π½ΠΈ. ΠΎΠ½ ΡƒΠΊΠ°Π·Ρ‹Π²Π°Π΅Ρ‚, Ρ‡Ρ‚ΠΎ Π²Ρ‹ Π½ΠΈΠΊΠΎΠ³Π΄Π° Π½Π΅ смоТСтС ΠΈΠ·ΠΌΠ΅Ρ€ΠΈΡ‚ΡŒ это врСмя, ΠΏΠΎΡ‚ΠΎΠΌΡƒ Ρ‡Ρ‚ΠΎ ΠΎΠ½ΠΎ Ρ‚Π°ΠΊΠΎΠ΅ малСнькоС, Π½ΠΎ это Π½Π΅ Ρ‚Π°ΠΊ, Ссли Ρƒ вас Π΅ΡΡ‚ΡŒ ΠΌΠΈΠΊΡ€ΠΎΠΊΠΎΠ½Ρ‚Ρ€ΠΎΠ»Π»Π΅Ρ€ Ρ‚ΠΈΠΏΠ° Π±Π°Π·ΠΎΠ²ΠΎΠ³ΠΎ ΡˆΡ‚Π°ΠΌΠΏΠ° 2.
Π² Ρ†Π΅ΠΏΠΈ Π²Ρ€Π΅ΠΌΠ΅Π½ΠΈ RC, сопротивлСниС Π² ΠΎΠΌΠ°Ρ…, ΡƒΠΌΠ½ΠΎΠΆΠ΅Π½Π½ΠΎΠ΅ Π½Π° Π΅ΠΌΠΊΠΎΡΡ‚ΡŒ Π² Π€Π°Ρ€Π°Π΄Ρ‹ Ρ€Π°Π²Π½Ρ‹ Π²Ρ€Π΅ΠΌΠ΅Π½ΠΈ Π² сСкундах. ΡΠ»Π΅Π΄ΠΎΠ²Π°Ρ‚Π΅Π»ΡŒΠ½ΠΎ, Π΅ΠΌΠΊΠΎΡΡ‚ΡŒ Ρ€Π°Π²Π½Π° сСкундам, Π΄Π΅Π»Π΅Π½Π½Ρ‹ΠΌ Π½Π° сопротивлСниС. Π‘ = Π’/Π 

Π²ΠΎΡ‚ ΠΊΠ°ΠΊ Π±ΡƒΠ΄Π΅Ρ‚ Π²Ρ‹Π³Π»ΡΠ΄Π΅Ρ‚ΡŒ схСма:

β€”β€”β€”β€”β€”β€”β€”β€”β€”> Π½Π° Π²Ρ…ΠΎΠ΄Π½ΠΎΠΉ ΠΏΠΈΠ½ основного ΡˆΡ‚Π°ΠΌΠΏΠ°
| |
нСизвСстно C извСстно R
| |
β€”β€”β€”β€”β€”β€”β€”β€”β€”> Π½Π° массу

, Ρ‚Π°ΠΊ Ρ‡Ρ‚ΠΎ Π²Ρ‹ Π΄Π΅Π»Π°Π΅Ρ‚Π΅, Π²Ρ‹ устанавливаСтС ΠΊΠΎΠ½Ρ‚Π°ΠΊΡ‚ Π½Π° нСсколько миллисСкунд, Ρ‡Ρ‚ΠΎΠ±Ρ‹ Π·Π°Ρ€ΡΠ΄ΠΈΡ‚ΡŒ ΠΊΠΎΠ»ΠΏΠ°Ρ‡ΠΎΠΊ.Π·Π°Ρ‚Π΅ΠΌ Π²Ρ‹ мСняСтС ΠΊΠΎΠ½Ρ‚Π°ΠΊΡ‚ Π½Π° Π²Ρ…ΠΎΠ΄, запускаСтС счСтчик ΠΈ ΡƒΠΊΠ°Π·Ρ‹Π²Π°Π΅Ρ‚Π΅ Π±Π°Π·ΠΎΠ²ΠΎΠΉ ΠΌΠ°Ρ€ΠΊΠ΅ ΡΠ»Π΅Π΄ΠΈΡ‚ΡŒ Π·Π° Π½ΠΈΠ·ΠΊΠΈΠΌ логичСским ΡƒΡ€ΠΎΠ²Π½Π΅ΠΌ.

, ΠΊΠΎΠ½Π΅Ρ‡Π½ΠΎ, Π²Π°ΠΌ Π½ΡƒΠΆΠ½ΠΎ сначала Β«ΠΎΡ‚ΠΊΠ°Π»ΠΈΠ±Ρ€ΠΎΠ²Π°Ρ‚ΡŒΒ» ваш Π±Π°Π·ΠΎΠ²Ρ‹ΠΉ ΡˆΡ‚Π°ΠΌΠΏ, ΠΈΠ·ΠΌΠ΅Ρ€ΠΈΠ², сколько Π²Ρ€Π΅ΠΌΠ΅Π½ΠΈ трСбуСтся, Ρ‡Ρ‚ΠΎΠ±Ρ‹ ΠΏΡ€ΠΎΠΉΡ‚ΠΈ ΡΡ‡Π΅Ρ‚Π½ΡƒΡŽ ΠΎΠ±Π»Π°ΡΡ‚ΡŒ ΠΊΠΎΠ΄Π°. Π½Π°ΠΏΡ€ΠΈΠΌΠ΅Ρ€, скаТитС Π΅ΠΌΡƒ ΠΏΡ€ΠΎΠΉΡ‚ΠΈ Ρ†ΠΈΠΊΠ» счСта, Π½Π°ΠΏΡ€ΠΈΠΌΠ΅Ρ€, Π΄Π΅ΡΡΡ‚ΡŒ тысяч Ρ€Π°Π·, Π° Π·Π°Ρ‚Π΅ΠΌ Π·Π°ΠΌΠ΅Ρ€ΡŒΡ‚Π΅, сколько Π²Ρ€Π΅ΠΌΠ΅Π½ΠΈ это Π·Π°ΠΉΠΌΠ΅Ρ‚, с ΠΏΠΎΠΌΠΎΡ‰ΡŒΡŽ сСкундомСра. Π±ΠΎΠ»Π΅Π΅ высокоС число даст Π²Π°ΠΌ Ρ…ΠΎΡ€ΠΎΡˆΠ΅Π΅ срСднСС послС выполнСния ΠΊΠ°Π»ΠΈΠ±Ρ€ΠΎΠ²ΠΊΠΈ нСсколько Ρ€Π°Π·. Π±Π°Π·ΠΎΠ²Ρ‹ΠΉ ΡˆΡ‚Π°ΠΌΠΏ Ρ€Π°Π±ΠΎΡ‚Π°Π΅Ρ‚ Π² миллисСкундах, поэтому Π΄Π΅ΡΡΡ‚ΡŒ тысяч Ρ†ΠΈΠΊΠ»ΠΎΠ² ΠΌΠΎΠ³ΡƒΡ‚ Π±Ρ‹Ρ‚ΡŒ Π±Π»ΠΈΠ·ΠΊΠΈ ΠΊ дСсяти сСкундам, Π½ΠΎ это зависит ΠΎΡ‚ слоТности Ρ†ΠΈΠΊΠ»Π° ΠΈ Π½Π°Π±ΠΎΡ€Π° ΠΏΠ΅Ρ€Π΅ΠΌΠ΅Π½Π½Ρ‹Ρ… Ρ‚Π΅ΠΎΡ€ΠΈΠΈ хаоса.

Ρ‚Π°ΠΊ ΠΈΠ»ΠΈ ΠΈΠ½Π°Ρ‡Π΅, Ρ‚Π΅ΠΏΠ΅Ρ€ΡŒ, ΠΊΠΎΠ³Π΄Π° Π²Ρ‹ Π·Π½Π°Π΅Ρ‚Π΅, сколько Π²Ρ€Π΅ΠΌΠ΅Π½ΠΈ Π·Π°Π½ΠΈΠΌΠ°Π΅Ρ‚ ΠΊΠ°ΠΆΠ΄Ρ‹ΠΉ счСт Π² вашСм счСтчикС, Π²Ρ‹ ΠΌΠΎΠΆΠ΅Ρ‚Π΅ ввСсти фактичСскоС врСмя Π² сСкундах Π² ΡƒΡ€Π°Π²Π½Π΅Π½ΠΈΠ΅, Ρ‡Ρ‚ΠΎΠ±Ρ‹ Π½Π°ΠΉΡ‚ΠΈ ΠΎΡ‡Π΅Π½ΡŒ Ρ…ΠΎΡ€ΠΎΡˆΠ΅Π΅ ΠΏΡ€ΠΈΠ±Π»ΠΈΠΆΠ΅Π½ΠΈΠ΅ Смкости.

Π― Π±Ρ‹ пошСл Π½Π° www.dealextreme.com ΠΈ Π·Π°ΠΊΠ°Π·Π°Π» Ρ‚Π°ΠΌ Π½Π΅Π΄ΠΎΡ€ΠΎΠ³ΠΎΠΉ ΠΌΡƒΠ»ΡŒΡ‚ΠΈΠΌΠ΅Ρ‚Ρ€. Π’Ρ‹ ΠΌΠΎΠΆΠ΅Ρ‚Π΅ ΠΏΠΎΠ»ΡƒΡ‡ΠΈΡ‚ΡŒ ΠΎΠ΄ΠΈΠ½ с Ρ„ΡƒΠ½ΠΊΡ†ΠΈΠ΅ΠΉ Смкости Π·Π° 12 $ ΠΈΠ»ΠΈ мСньшС с бСсплатной доставкой.

Π‘Ρ…Π΅ΠΌΡƒ, ΠΎΠΏΠΈΡΠ°Π½Π½ΡƒΡŽ orksecurity, ΠΎΡ‡Π΅Π½ΡŒ вСсСло Π΄Π΅Π»Π°Ρ‚ΡŒ, ΠΈ ΠΎΠ½Π° стоит мСньшС Π΄ΠΎΠ»Π»Π°Ρ€Π° (ΠΏΡ€Π΅ΠΎΠ±Ρ€Π°Π·ΠΎΠ²Π°Ρ‚Π΅Π»ΠΈ Ρ‚Ρ€ΠΈΠ³Π³Π΅Ρ€Π° Π¨ΠΌΠΈΡ‚Ρ‚Π° ΠΎΡ‚Π»ΠΈΡ‡Π½ΠΎ подходят, Ρ‚Π°ΠΉΠΌΠ΅Ρ€Ρ‹ 555 Ρ‚ΠΎΠΆΠ΅ ΠΏΠΎΠ΄ΠΎΠΉΠ΄ΡƒΡ‚), хотя, ΠΏΠΎ ΠΌΠΎΠ΅ΠΌΡƒ ΠΎΠΏΡ‹Ρ‚Ρƒ, Π²Π°ΠΌ понадобится осциллограф (Π±ΠΎΠ»Π΅Π΅ Π΄ΠΎΡ€ΠΎΠ³ΠΎΠΉ, Ρ‡Π΅ΠΌ ΠΌΡƒΠ»ΡŒΡ‚ΠΈΠΌΠ΅Ρ‚Ρ€), ΠΈΠ»ΠΈ ΠΌΡƒΠ»ΡŒΡ‚ΠΈΠΌΠ΅Ρ‚Ρ€ с Ρ„ΡƒΠ½ΠΊΡ†ΠΈΠ΅ΠΉ подсчСта частоты, Ρ‡Ρ‚ΠΎΠ±Ρ‹ ΠΏΠΎΠ»ΡŒΠ·ΠΎΠ²Π°Ρ‚ΡŒΡΡ ΠΈΠΌ с ΠΏΡ€ΠΈΠ΅ΠΌΠ»Π΅ΠΌΡ‹ΠΌ удобством ΠΈ Ρ‚ΠΎΡ‡Π½ΠΎΡΡ‚ΡŒΡŽ.Π’Ρ‹ ΠΌΠΎΠΆΠ΅Ρ‚Π΅ ΠΏΡ€ΠΎΠ΄ΠΎΠ»ΠΆΠ°Ρ‚ΡŒ Π²ΠΊΠ»ΡŽΡ‡Π°Ρ‚ΡŒ ΠΈ Π²Ρ‹ΠΊΠ»ΡŽΡ‡Π°Ρ‚ΡŒ ΠΊΡƒΡ‡Ρƒ извСстных СмкостСй, ΠΏΠΎΠΊΠ° Π½Π΅ пСрСстанСтС ΡΠ»Ρ‹ΡˆΠ°Ρ‚ΡŒ сигнал. Π½ΠΎ этот ΠΌΠ΅Ρ‚ΠΎΠ΄ мСня Π½Π΅ ΠΎΡ‡Π΅Π½ΡŒ ΠΏΡ€ΠΈΠ²Π»Π΅ΠΊΠ°Π΅Ρ‚!

Π‘ΠΎΠ»Π΅Π΅ Π΄Π΅ΡˆΠ΅Π²Ρ‹ΠΉ ΠΈ ΡƒΠ΄ΠΎΠ±Π½Ρ‹ΠΉ ΠΌΠ΅Ρ‚ΠΎΠ΄ ΠΌΠΎΠΆΠ΅Ρ‚ ΡΠΎΡΡ‚ΠΎΡΡ‚ΡŒ Π² Ρ‚ΠΎΠΌ, Ρ‡Ρ‚ΠΎΠ±Ρ‹ Π£Π‘Π•Π”Π˜Π’Π¬Π‘Π―, Ρ‡Ρ‚ΠΎ ΠΎΠ½ΠΈ сначала разряТСны, Π·Π°Ρ‚Π΅ΠΌ Π·Π°Ρ€ΡΠ΄ΠΈΡ‚ΡŒ ΠΈΡ… Π±Π°Ρ‚Π°Ρ€Π΅Π΅ΠΉ 9 Π’ ΠΈΠ»ΠΈ 12 Π’ ΠΈ ΡΠΎΠ΅Π΄ΠΈΠ½ΠΈΡ‚ΡŒ ΠΈΡ… ΠΏΠΎΡΠ»Π΅Π΄ΠΎΠ²Π°Ρ‚Π΅Π»ΡŒΠ½ΠΎ с рСзистором ΠΈ свСтодиодом. Π’Π°ΠΌ Π½ΡƒΠΆΠ½ΠΎ Π±ΡƒΠ΄Π΅Ρ‚ ΡƒΠΌΠ΅Ρ‚ΡŒ ΡΡ‡ΠΈΡ‚Π°Ρ‚ΡŒ ΠΈ ΠΏΠΎΠ»ΡŒΠ·ΠΎΠ²Π°Ρ‚ΡŒΡΡ сСкундомСром.

Из Ρ‚Π°Π±Π»ΠΈΡ†Ρ‹ Π΄Π°Π½Π½Ρ‹Ρ… свСтодиода Π²Ρ‹ ΠΌΠΎΠΆΠ΅Ρ‚Π΅ ΡƒΠ²ΠΈΠ΄Π΅Ρ‚ΡŒ ΠΏΠ°Π΄Π΅Π½ΠΈΠ΅ напряТСния Π½Π° свСтодиодС (ΠΈΠ»ΠΈ ΠΈΡΠΏΠΎΠ»ΡŒΠ·ΠΎΠ²Π°Ρ‚ΡŒ ΠΌΡƒΠ»ΡŒΡ‚ΠΈΠΌΠ΅Ρ‚Ρ€ с Ρ„ΡƒΠ½ΠΊΡ†ΠΈΠ΅ΠΉ ΠΏΡ€ΠΎΠ²Π΅Ρ€ΠΊΠΈ Π΄ΠΈΠΎΠ΄ΠΎΠ²).Когда ΠΏΡ€ΠΈΠ»ΠΎΠΆΠ΅Π½Π½ΠΎΠ΅ напряТСниС ΡƒΠΏΠ°Π΄Π΅Ρ‚ Π½ΠΈΠΆΠ΅ этого ΠΏΠΎΡ€ΠΎΠ³Π°, ΠΎΠ½ Π²Ρ‹ΠΊΠ»ΡŽΡ‡ΠΈΡ‚ΡΡ. Π’Ρ‹Ρ…ΠΎΠ΄Π½ΠΎΠ΅ напряТСниС кондСнсатора Π½Π° рСзисторС прСдставляСт собой Ρ„ΡƒΠ½ΠΊΡ†ΠΈΡŽ затухания, ΠΊΠ°ΠΊ описано здСсь: http://en.wikipedia.org/wiki/Capacitor#DC_circuits

Π’Ρ‹ Π² основном ΠΈΡΠΏΠΎΠ»ΡŒΠ·ΡƒΠ΅Ρ‚Π΅ свСтодиод Π² качСствС ΠΏΡ€ΠΈΠΌΠΈΡ‚ΠΈΠ²Π½ΠΎΠ³ΠΎ ΠΈΠ½Π΄ΠΈΠΊΠ°Ρ‚ΠΎΡ€Π° напряТСния. Π― ΠΏΡ€Π΅Π΄Π»Π°Π³Π°ΡŽ красный ΠΈΠ·-Π·Π° Π½ΠΈΠ·ΠΊΠΎΠ³ΠΎ ΠΏΠΎΡ€ΠΎΠ³ΠΎΠ²ΠΎΠ³ΠΎ напряТСния. ВрСмя, Π² Ρ‚Π΅Ρ‡Π΅Π½ΠΈΠ΅ ΠΊΠΎΡ‚ΠΎΡ€ΠΎΠ³ΠΎ свСтодиод остаСтся Π²ΠΊΠ»ΡŽΡ‡Π΅Π½Π½Ρ‹ΠΌ, ΠΏΠΎΠ·Π²ΠΎΠ»ΠΈΡ‚ Π²Π°ΠΌ Ρ€Π°ΡΡΡ‡ΠΈΡ‚Π°Ρ‚ΡŒ Π΅ΠΌΠΊΠΎΡΡ‚ΡŒ, Ρ‚Π°ΠΊ ΠΊΠ°ΠΊ Ρ‚Π΅ΠΏΠ΅Ρ€ΡŒ Π²Ρ‹ Π·Π½Π°Π΅Ρ‚Π΅ врСмя, ΠΊΠΎΡ‚ΠΎΡ€ΠΎΠ΅ ΠΏΠΎΡ‚Ρ€Π΅Π±ΠΎΠ²Π°Π»ΠΎΡΡŒ для ΠΏΠ΅Ρ€Π΅Ρ…ΠΎΠ΄Π° ΠΎΡ‚ извСстного Π½Π°Ρ‡Π°Π»ΡŒΠ½ΠΎΠ³ΠΎ напряТСния ΠΊ извСстному ΠΊΠΎΠ½Π΅Ρ‡Π½ΠΎΠΌΡƒ Π½Π°ΠΏΡ€ΡΠΆΠ΅Π½ΠΈΡŽ ΠΏΡ€ΠΈ извСстном сопротивлСнии.РСзистор большСго Ρ€Π°Π·ΠΌΠ΅Ρ€Π° даст Π±ΠΎΠ»Π΅Π΅ Ρ‚ΠΎΡ‡Π½Ρ‹Π΅ Ρ€Π΅Π·ΡƒΠ»ΡŒΡ‚Π°Ρ‚Ρ‹ (ΠΈΠ·-Π·Π° Π²Π½ΡƒΡ‚Ρ€Π΅Π½Π½Π΅Π³ΠΎ сопротивлСния кондСнсатора ΠΈ Π±ΠΎΠ»Π΅Π΅ Π΄Π»ΠΈΡ‚Π΅Π»ΡŒΠ½ΠΎΠ³ΠΎ Π²Ρ€Π΅ΠΌΠ΅Π½ΠΈ свСчСния свСтодиода), Π½ΠΎ свСтодиод Π±ΡƒΠ΄Π΅Ρ‚ тусклСС; Π½Π°ΠΉΡ‚ΠΈ Ρ…ΠΎΡ€ΠΎΡˆΠΈΠΉ баланс. Π‘Π½Π°Ρ‡Π°Π»Π° ΠΏΠΎΠΏΡ€ΠΎΠ±ΡƒΠΉΡ‚Π΅ нСсколько сотСн Ом.

ΠšΠΎΠ½Π΅Ρ‡Π½ΠΎ, Ссли Ρƒ вас Π΅ΡΡ‚ΡŒ ΠΌΡƒΠ»ΡŒΡ‚ΠΈΠΌΠ΅Ρ‚Ρ€ с Ρ„ΡƒΠ½ΠΊΡ†ΠΈΠ΅ΠΉ измСрСния напряТСния, просто ΠΏΠΎΠ΄ΠΊΠ»ΡŽΡ‡ΠΈΡ‚Π΅ Π΅Π³ΠΎ ΠΏΠ°Ρ€Π°Π»Π»Π΅Π»ΡŒΠ½ΠΎ кондСнсатору (ΠΏΡ€ΠΈ зарядкС 9Π’ ΠΈΠ»ΠΈ 12Π’ это Π½Π΅ ΠΏΡ€ΠΎΠ±Π»Π΅ΠΌΠ°), ΠΎΠ½ Π±ΡƒΠ΄Π΅Ρ‚ Π½Π°ΠΌΠ½ΠΎΠ³ΠΎ Ρ‡ΡƒΠ²ΡΡ‚Π²ΠΈΡ‚Π΅Π»ΡŒΠ½Π΅Π΅ свСтодиода. Π’Ρ‹ ΡƒΠ²ΠΈΠ΄ΠΈΡ‚Π΅ ΠΏΠ°Π΄Π΅Π½ΠΈΠ΅ напряТСния ΠΏΠΎ ΠΌΠ΅Ρ€Π΅ Ρ‚ΠΎΠ³ΠΎ, ΠΊΠ°ΠΊ ΠΎΠ½ΠΎ спадаСт Π½Π° рСзисторС, ΠΈ Π²Ρ‹ ΠΌΠΎΠΆΠ΅Ρ‚Π΅ ΡΠΈΠ½Ρ…Ρ€ΠΎΠ½ΠΈΠ·ΠΈΡ€ΠΎΠ²Π°Ρ‚ΡŒ Π΅Π³ΠΎ с ΠΊΠ°ΠΊΠΎΠΉ-Ρ‚ΠΎ ΠΏΡ€ΠΎΠΈΠ·Π²ΠΎΠ»ΡŒΠ½ΠΎΠΉ Ρ‚ΠΎΡ‡ΠΊΠΎΠΉ.

По ΠΌΠΎΠ΅ΠΌΡƒ (ΠΎΠ³Ρ€Π°Π½ΠΈΡ‡Π΅Π½Π½ΠΎΠΌΡƒ) ΠΎΠΏΡ‹Ρ‚Ρƒ ΠΈ (ΠΎΡ‡Π΅Π½ΡŒ ΠΎΠ³Ρ€Π°Π½ΠΈΡ‡Π΅Π½Π½ΠΎΠΉ) памяти кондСнсаторы Ρ„ΠΎΡ‚ΠΎΠ²ΡΠΏΡ‹ΡˆΠ΅ΠΊ рассчитаны ΠΏΡ€ΠΈΠΌΠ΅Ρ€Π½ΠΎ Π½Π° 330 Π’ ΠΈ ΠΈΠΌΠ΅ΡŽΡ‚ Π΅ΠΌΠΊΠΎΡΡ‚ΡŒ ΠΏΡ€ΠΈΠΌΠ΅Ρ€Π½ΠΎ ΠΎΡ‚ 100 Π΄ΠΎ 300 ΠΌΠΈΠΊΡ€ΠΎΡ„Π°Ρ€Π°Π΄. Π—Π½Π°Π½ΠΈΠ΅ Смкости Π½Π΅ Π³ΠΎΠ²ΠΎΡ€ΠΈΡ‚ Π²Π°ΠΌ ΠΎ бСзопасном номинальном напряТСнии, ΠžΠ§Π•ΠΠ¬ ΠžΠ‘ΠžΠ—ΠΠΠ™Π’Π• это. Π‘ΠΎΠ΅Π΄ΠΈΠ½Π΅Π½ΠΈΠ΅ ΠΈΡ… ΠΏΠΎΡΠ»Π΅Π΄ΠΎΠ²Π°Ρ‚Π΅Π»ΡŒΠ½ΠΎ ΠΈ Π½Π°ΠΈΠ²Π½ΠΎΠ΅ ΠΏΡ€Π΅Π΄ΠΏΠΎΠ»ΠΎΠΆΠ΅Π½ΠΈΠ΅, Ρ‡Ρ‚ΠΎ это ΡƒΠ²Π΅Π»ΠΈΡ‡ΠΈΠ²Π°Π΅Ρ‚ допуск ΠΏΠΎ Π½Π°ΠΏΡ€ΡΠΆΠ΅Π½ΠΈΡŽ, Ρ‚Π°ΠΊΠΆΠ΅ ΠΌΠΎΠΆΠ΅Ρ‚ привСсти ΠΊ катастрофичСским ΠΎΡ‚ΠΊΠ°Π·Π°ΠΌ.

ΠœΡƒΠ»ΡŒΡ‚ΠΈΠΌΠ΅Ρ‚Ρ€ опрСдСляСт Π΅ΠΌΠΊΠΎΡΡ‚ΡŒ, заряТая кондСнсатор извСстным Ρ‚ΠΎΠΊΠΎΠΌ, измСряя ΠΏΠΎΠ»ΡƒΡ‡Π΅Π½Π½ΠΎΠ΅ напряТСниС ΠΈ Π·Π°Ρ‚Π΅ΠΌ вычисляя Π΅ΠΌΠΊΠΎΡΡ‚ΡŒ.

ΠŸΡ€Π΅Π΄ΡƒΠΏΡ€Π΅ΠΆΠ΄Π΅Π½ΠΈΠ΅: Π₯ΠΎΡ€ΠΎΡˆΠΈΠΉ кондСнсатор Π½Π°ΠΊΠ°ΠΏΠ»ΠΈΠ²Π°Π΅Ρ‚ элСктричСский заряд ΠΈ ΠΌΠΎΠΆΠ΅Ρ‚ ΠΎΡΡ‚Π°Π²Π°Ρ‚ΡŒΡΡ ΠΏΠΎΠ΄ напряТСниСм послС ΠΎΡ‚ΠΊΠ»ΡŽΡ‡Π΅Π½ΠΈΡ питания. ΠŸΡ€Π΅ΠΆΠ΄Π΅ Ρ‡Π΅ΠΌ ΠΏΡ€ΠΈΠΊΠ°ΡΠ°Ρ‚ΡŒΡΡ ΠΊ Π½Π΅ΠΌΡƒ ΠΈΠ»ΠΈ ΠΏΡ€ΠΎΠ²ΠΎΠ΄ΠΈΡ‚ΡŒ измСрСния, Π°) Π²Ρ‹ΠΊΠ»ΡŽΡ‡ΠΈΡ‚Π΅ всС ΠΏΠΈΡ‚Π°Π½ΠΈΠ΅, Π±) ΠΈΡΠΏΠΎΠ»ΡŒΠ·ΡƒΠΉΡ‚Π΅ ΠΌΡƒΠ»ΡŒΡ‚ΠΈΠΌΠ΅Ρ‚Ρ€, Ρ‡Ρ‚ΠΎΠ±Ρ‹ ΡƒΠ±Π΅Π΄ΠΈΡ‚ΡŒΡΡ, Ρ‡Ρ‚ΠΎ ΠΏΠΈΡ‚Π°Π½ΠΈΠ΅ ΠΎΡ‚ΠΊΠ»ΡŽΡ‡Π΅Π½ΠΎ, ΠΈ Π²) остороТно разрядитС кондСнсатор, ΠΏΠΎΠ΄ΠΊΠ»ΡŽΡ‡ΠΈΠ² рСзистор ΠΊ ΠΏΡ€ΠΎΠ²ΠΎΠ΄Π°ΠΌ (ΠΊΠ°ΠΊ ΡƒΠΊΠ°Π·Π°Π½ΠΎ Π² ΡΠ»Π΅Π΄ΡƒΡŽΡ‰Π΅ΠΌ Π°Π±Π·Π°Ρ†Π΅). ΠžΠ±ΡΠ·Π°Ρ‚Π΅Π»ΡŒΠ½ΠΎ ΠΈΡΠΏΠΎΠ»ΡŒΠ·ΡƒΠΉΡ‚Π΅ ΡΠΎΠΎΡ‚Π²Π΅Ρ‚ΡΡ‚Π²ΡƒΡŽΡ‰ΠΈΠ΅ срСдства ΠΈΠ½Π΄ΠΈΠ²ΠΈΠ΄ΡƒΠ°Π»ΡŒΠ½ΠΎΠΉ Π·Π°Ρ‰ΠΈΡ‚Ρ‹.

Для бСзопасного разряда кондСнсатора: ПослС ΠΎΡ‚ΠΊΠ»ΡŽΡ‡Π΅Π½ΠΈΡ питания ΠΏΠΎΠ΄ΠΊΠ»ΡŽΡ‡ΠΈΡ‚Π΅ рСзистор 20 000 Ом, 5 Π’Ρ‚ ΠΊ ΠΊΠ»Π΅ΠΌΠΌΠ°ΠΌ кондСнсатора Π½Π° ΠΏΡΡ‚ΡŒ сСкунд.Π‘ ΠΏΠΎΠΌΠΎΡ‰ΡŒΡŽ ΠΌΡƒΠ»ΡŒΡ‚ΠΈΠΌΠ΅Ρ‚Ρ€Π° ΡƒΠ±Π΅Π΄ΠΈΡ‚Π΅ΡΡŒ, Ρ‡Ρ‚ΠΎ кондСнсатор ΠΏΠΎΠ»Π½ΠΎΡΡ‚ΡŒΡŽ разряТСн.

  1. Π‘ ΠΏΠΎΠΌΠΎΡ‰ΡŒΡŽ Ρ†ΠΈΡ„Ρ€ΠΎΠ²ΠΎΠ³ΠΎ ΠΌΡƒΠ»ΡŒΡ‚ΠΈΠΌΠ΅Ρ‚Ρ€Π° (DMM) ΡƒΠ±Π΅Π΄ΠΈΡ‚Π΅ΡΡŒ, Ρ‡Ρ‚ΠΎ ΠΏΠΈΡ‚Π°Π½ΠΈΠ΅ Ρ†Π΅ΠΏΠΈ ΠΎΡ‚ΠΊΠ»ΡŽΡ‡Π΅Π½ΠΎ. Если кондСнсатор ΠΈΡΠΏΠΎΠ»ΡŒΠ·ΡƒΠ΅Ρ‚ΡΡ Π² Ρ†Π΅ΠΏΠΈ ΠΏΠ΅Ρ€Π΅ΠΌΠ΅Π½Π½ΠΎΠ³ΠΎ Ρ‚ΠΎΠΊΠ°, установитС ΠΌΡƒΠ»ΡŒΡ‚ΠΈΠΌΠ΅Ρ‚Ρ€ Π½Π° ΠΈΠ·ΠΌΠ΅Ρ€Π΅Π½ΠΈΠ΅ напряТСния ΠΏΠ΅Ρ€Π΅ΠΌΠ΅Π½Π½ΠΎΠ³ΠΎ Ρ‚ΠΎΠΊΠ°. Если ΠΎΠ½ ΠΈΡΠΏΠΎΠ»ΡŒΠ·ΡƒΠ΅Ρ‚ΡΡ Π² Ρ†Π΅ΠΏΠΈ постоянного Ρ‚ΠΎΠΊΠ°, настройтС Ρ†ΠΈΡ„Ρ€ΠΎΠ²ΠΎΠΉ ΠΌΡƒΠ»ΡŒΡ‚ΠΈΠΌΠ΅Ρ‚Ρ€ Π½Π° ΠΈΠ·ΠΌΠ΅Ρ€Π΅Π½ΠΈΠ΅ напряТСния постоянного Ρ‚ΠΎΠΊΠ°.
  2. ΠžΡΠΌΠΎΡ‚Ρ€ΠΈΡ‚Π΅ кондСнсатор. Если ΡƒΡ‚Π΅Ρ‡ΠΊΠΈ, Ρ‚Ρ€Π΅Ρ‰ΠΈΠ½Ρ‹, выпуклости ΠΈΠ»ΠΈ Π΄Ρ€ΡƒΠ³ΠΈΠ΅ ΠΏΡ€ΠΈΠ·Π½Π°ΠΊΠΈ износа ΠΎΡ‡Π΅Π²ΠΈΠ΄Π½Ρ‹, Π·Π°ΠΌΠ΅Π½ΠΈΡ‚Π΅ кондСнсатор.
  3. ΠŸΠΎΠ²Π΅Ρ€Π½ΠΈΡ‚Π΅ Ρ†ΠΈΡ„Π΅Ρ€Π±Π»Π°Ρ‚ Π² Ρ€Π΅ΠΆΠΈΠΌ измСрСния Смкости.Π‘ΠΈΠΌΠ²ΠΎΠ» часто Π΄Π΅Π»ΠΈΡ‚ мСсто Π½Π° Ρ†ΠΈΡ„Π΅Ρ€Π±Π»Π°Ρ‚Π΅ с Π΄Ρ€ΡƒΠ³ΠΎΠΉ Ρ„ΡƒΠ½ΠΊΡ†ΠΈΠ΅ΠΉ. Π’ Π΄ΠΎΠΏΠΎΠ»Π½Π΅Π½ΠΈΠ΅ ΠΊ Ρ€Π΅Π³ΡƒΠ»ΠΈΡ€ΠΎΠ²ΠΊΠ΅ диска ΠΎΠ±Ρ‹Ρ‡Π½ΠΎ Π½Π΅ΠΎΠ±Ρ…ΠΎΠ΄ΠΈΠΌΠΎ Π½Π°ΠΆΠ°Ρ‚ΡŒ Ρ„ΡƒΠ½ΠΊΡ†ΠΈΠΎΠ½Π°Π»ΡŒΠ½ΡƒΡŽ ΠΊΠ½ΠΎΠΏΠΊΡƒ, Ρ‡Ρ‚ΠΎΠ±Ρ‹ Π°ΠΊΡ‚ΠΈΠ²ΠΈΡ€ΠΎΠ²Π°Ρ‚ΡŒ ΠΈΠ·ΠΌΠ΅Ρ€Π΅Π½ΠΈΠ΅. Π˜Π½ΡΡ‚Ρ€ΡƒΠΊΡ†ΠΈΠΈ см. Π² руководствС ΠΏΠΎΠ»ΡŒΠ·ΠΎΠ²Π°Ρ‚Π΅Π»Ρ ΠΌΡƒΠ»ΡŒΡ‚ΠΈΠΌΠ΅Ρ‚Ρ€Π°.

4. Для ΠΏΡ€Π°Π²ΠΈΠ»ΡŒΠ½ΠΎΠ³ΠΎ измСрСния кондСнсатор Π½Π΅ΠΎΠ±Ρ…ΠΎΠ΄ΠΈΠΌΠΎ ΡƒΠ΄Π°Π»ΠΈΡ‚ΡŒ ΠΈΠ· Ρ†Π΅ΠΏΠΈ. РазрядитС кондСнсатор, ΠΊΠ°ΠΊ описано Π² ΠΏΡ€Π΅Π΄ΡƒΠΏΡ€Π΅ΠΆΠ΄Π΅Π½ΠΈΠΈ Π²Ρ‹ΡˆΠ΅.

ΠŸΡ€ΠΈΠΌΠ΅Ρ‡Π°Π½ΠΈΠ΅: НСкоторыС ΠΌΡƒΠ»ΡŒΡ‚ΠΈΠΌΠ΅Ρ‚Ρ€Ρ‹ ΠΏΡ€Π΅Π΄Π»Π°Π³Π°ΡŽΡ‚ ΠΎΡ‚Π½ΠΎΡΠΈΡ‚Π΅Π»ΡŒΠ½Ρ‹ΠΉ (REL) Ρ€Π΅ΠΆΠΈΠΌ. ΠŸΡ€ΠΈ ΠΈΠ·ΠΌΠ΅Ρ€Π΅Π½ΠΈΠΈ ΠΌΠ°Π»Ρ‹Ρ… Π·Π½Π°Ρ‡Π΅Π½ΠΈΠΉ Смкости ΠΌΠΎΠΆΠ½ΠΎ ΠΈΡΠΏΠΎΠ»ΡŒΠ·ΠΎΠ²Π°Ρ‚ΡŒ ΠΎΡ‚Π½ΠΎΡΠΈΡ‚Π΅Π»ΡŒΠ½Ρ‹ΠΉ Ρ€Π΅ΠΆΠΈΠΌ для удалСния Смкости ΠΈΠ·ΠΌΠ΅Ρ€ΠΈΡ‚Π΅Π»ΡŒΠ½Ρ‹Ρ… ΠΏΡ€ΠΎΠ²ΠΎΠ΄ΠΎΠ².Π§Ρ‚ΠΎΠ±Ρ‹ пСрСвСсти ΠΌΡƒΠ»ΡŒΡ‚ΠΈΠΌΠ΅Ρ‚Ρ€ Π² ΠΎΡ‚Π½ΠΎΡΠΈΡ‚Π΅Π»ΡŒΠ½Ρ‹ΠΉ Ρ€Π΅ΠΆΠΈΠΌ измСрСния Смкости, ΠΎΡΡ‚Π°Π²ΡŒΡ‚Π΅ ΠΈΠ·ΠΌΠ΅Ρ€ΠΈΡ‚Π΅Π»ΡŒΠ½Ρ‹Π΅ ΠΏΡ€ΠΎΠ²ΠΎΠ΄Π° ΠΎΡ‚ΠΊΡ€Ρ‹Ρ‚Ρ‹ΠΌΠΈ ΠΈ Π½Π°ΠΆΠΌΠΈΡ‚Π΅ ΠΊΠ½ΠΎΠΏΠΊΡƒ REL. Π­Ρ‚ΠΎ удаляСт Π·Π½Π°Ρ‡Π΅Π½ΠΈΠ΅ остаточной Смкости ΠΈΠ·ΠΌΠ΅Ρ€ΠΈΡ‚Π΅Π»ΡŒΠ½Ρ‹Ρ… ΠΏΡ€ΠΎΠ²ΠΎΠ΄ΠΎΠ².

  • ΠŸΠΎΠ΄ΡΠΎΠ΅Π΄ΠΈΠ½ΠΈΡ‚Π΅ ΠΈΠ·ΠΌΠ΅Ρ€ΠΈΡ‚Π΅Π»ΡŒΠ½Ρ‹Π΅ ΠΏΡ€ΠΎΠ²ΠΎΠ΄Π° ΠΊ ΠΊΠ»Π΅ΠΌΠΌΠ°ΠΌ кондСнсатора. Π”Π΅Ρ€ΠΆΠΈΡ‚Π΅ ΠΈΠ·ΠΌΠ΅Ρ€ΠΈΡ‚Π΅Π»ΡŒΠ½Ρ‹Π΅ ΠΏΡ€ΠΎΠ²ΠΎΠ΄Π° ΠΏΠΎΠ΄ΠΊΠ»ΡŽΡ‡Π΅Π½Π½Ρ‹ΠΌΠΈ Π² Ρ‚Π΅Ρ‡Π΅Π½ΠΈΠ΅ Π½Π΅ΡΠΊΠΎΠ»ΡŒΠΊΠΈΡ… сСкунд, Ρ‡Ρ‚ΠΎΠ±Ρ‹ ΠΌΡƒΠ»ΡŒΡ‚ΠΈΠΌΠ΅Ρ‚Ρ€ автоматичСски Π²Ρ‹Π±Ρ€Π°Π» Π½ΡƒΠΆΠ½Ρ‹ΠΉ Π΄ΠΈΠ°ΠΏΠ°Π·ΠΎΠ½.
  • ΠŸΡ€ΠΎΡ‡ΠΈΡ‚Π°ΠΉΡ‚Π΅ ΠΎΡ‚ΠΎΠ±Ρ€Π°ΠΆΠ°Π΅ΠΌΠΎΠ΅ ΠΈΠ·ΠΌΠ΅Ρ€Π΅Π½ΠΈΠ΅. Если Π·Π½Π°Ρ‡Π΅Π½ΠΈΠ΅ Смкости находится Π² ΠΏΡ€Π΅Π΄Π΅Π»Π°Ρ… Π΄ΠΈΠ°ΠΏΠ°Π·ΠΎΠ½Π° измСрСния, ΠΌΡƒΠ»ΡŒΡ‚ΠΈΠΌΠ΅Ρ‚Ρ€ ΠΎΡ‚ΠΎΠ±Ρ€Π°Π·ΠΈΡ‚ Π·Π½Π°Ρ‡Π΅Π½ΠΈΠ΅ Смкости.Он Π±ΡƒΠ΄Π΅Ρ‚ ΠΎΡ‚ΠΎΠ±Ρ€Π°ΠΆΠ°Ρ‚ΡŒ OL, Ссли Π°) Π·Π½Π°Ρ‡Π΅Π½ΠΈΠ΅ Смкости Π²Ρ‹ΡˆΠ΅ Π΄ΠΈΠ°ΠΏΠ°Π·ΠΎΠ½Π° измСрСния ΠΈΠ»ΠΈ Π±) кондСнсатор нСисправСн.
  • ΠžΠ±Π·ΠΎΡ€ измСрСния Смкости

    Поиск ΠΈ устранСниС нСисправностСй ΠΎΠ΄Π½ΠΎΡ„Π°Π·Π½Ρ‹Ρ… Π΄Π²ΠΈΠ³Π°Ρ‚Π΅Π»Π΅ΠΉ β€” ΠΎΠ΄Π½ΠΎ ΠΈΠ· Π½Π°ΠΈΠ±ΠΎΠ»Π΅Π΅ ΠΏΡ€Π°ΠΊΡ‚ΠΈΡ‡Π½Ρ‹Ρ… ΠΏΡ€ΠΈΠΌΠ΅Π½Π΅Π½ΠΈΠΉ Ρ„ΡƒΠ½ΠΊΡ†ΠΈΠΈ измСрСния Смкости Ρ†ΠΈΡ„Ρ€ΠΎΠ²ΠΎΠ³ΠΎ ΠΌΡƒΠ»ΡŒΡ‚ΠΈΠΌΠ΅Ρ‚Ρ€Π°.

    ΠžΠ΄Π½ΠΎΡ„Π°Π·Π½Ρ‹ΠΉ Π΄Π²ΠΈΠ³Π°Ρ‚Π΅Π»ΡŒ с кондСнсаторным пуском, ΠΊΠΎΡ‚ΠΎΡ€Ρ‹ΠΉ Π½Π΅ запускаСтся, являСтся ΠΏΡ€ΠΈΠ·Π½Π°ΠΊΠΎΠΌ нСисправного кондСнсатора. Π’Π°ΠΊΠΈΠ΅ Π΄Π²ΠΈΠ³Π°Ρ‚Π΅Π»ΠΈ Π±ΡƒΠ΄ΡƒΡ‚ ΠΏΡ€ΠΎΠ΄ΠΎΠ»ΠΆΠ°Ρ‚ΡŒ Ρ€Π°Π±ΠΎΡ‚Π°Ρ‚ΡŒ послС запуска, Ρ‡Ρ‚ΠΎ услоТняСт поиск ΠΈ устранСниС нСисправностСй. Π’Ρ‹Ρ…ΠΎΠ΄ ΠΈΠ· строя кондСнсатора ТСсткого пуска компрСссоров HVAC являСтся Ρ…ΠΎΡ€ΠΎΡˆΠΈΠΌ ΠΏΡ€ΠΈΠΌΠ΅Ρ€ΠΎΠΌ этой ΠΏΡ€ΠΎΠ±Π»Π΅ΠΌΡ‹.Π”Π²ΠΈΠ³Π°Ρ‚Π΅Π»ΡŒ компрСссора ΠΌΠΎΠΆΠ΅Ρ‚ Π·Π°ΠΏΡƒΡΡ‚ΠΈΡ‚ΡŒΡΡ, Π½ΠΎ вскорС пСрСгрССтся, Ρ‡Ρ‚ΠΎ ΠΏΡ€ΠΈΠ²Π΅Π΄Π΅Ρ‚ ΠΊ ΠΎΡ‚ΠΊΠ»ΡŽΡ‡Π΅Π½ΠΈΡŽ Π²Ρ‹ΠΊΠ»ΡŽΡ‡Π°Ρ‚Π΅Π»Ρ.

    ΠžΠ΄Π½ΠΎΡ„Π°Π·Π½Ρ‹Π΅ Π΄Π²ΠΈΠ³Π°Ρ‚Π΅Π»ΠΈ с Ρ‚Π°ΠΊΠΈΠΌΠΈ ΠΏΡ€ΠΎΠ±Π»Π΅ΠΌΠ°ΠΌΠΈ ΠΈ ΡˆΡƒΠΌΠ½Ρ‹Π΅ ΠΎΠ΄Π½ΠΎΡ„Π°Π·Π½Ρ‹Π΅ Π΄Π²ΠΈΠ³Π°Ρ‚Π΅Π»ΠΈ с кондСнсаторами Ρ‚Ρ€Π΅Π±ΡƒΡŽΡ‚ ΠΌΡƒΠ»ΡŒΡ‚ΠΈΠΌΠ΅Ρ‚Ρ€Π° для ΠΏΡ€ΠΎΠ²Π΅Ρ€ΠΊΠΈ исправности кондСнсаторов. ΠŸΠΎΡ‡Ρ‚ΠΈ всС кондСнсаторы Π΄Π²ΠΈΠ³Π°Ρ‚Π΅Π»Π΅ΠΉ ΠΈΠΌΠ΅ΡŽΡ‚ Π½ΠΎΠΌΠΈΠ½Π°Π» Π² ΠΌΠΈΠΊΡ€ΠΎΡ„Π°Ρ€Π°Π΄Π°Ρ…, ΡƒΠΊΠ°Π·Π°Π½Π½Ρ‹ΠΉ Π½Π° кондСнсаторС.

    Π’Ρ€Π΅Ρ…Ρ„Π°Π·Π½Ρ‹Π΅ кондСнсаторы для ΠΊΠΎΡ€Ρ€Π΅ΠΊΡ†ΠΈΠΈ коэффициСнта мощности ΠΎΠ±Ρ‹Ρ‡Π½ΠΎ Π·Π°Ρ‰ΠΈΡ‰Π΅Π½Ρ‹ прСдохранитСлями. Если ΠΎΠ΄ΠΈΠ½ ΠΈΠ»ΠΈ нСсколько ΠΈΠ· этих кондСнсаторов Π²Ρ‹ΠΉΠ΄ΡƒΡ‚ ΠΈΠ· строя, это ΠΏΡ€ΠΈΠ²Π΅Π΄Π΅Ρ‚ ΠΊ нСэффСктивности систСмы, скорСС всСго, увСличатся счСта Π·Π° ΠΊΠΎΠΌΠΌΡƒΠ½Π°Π»ΡŒΠ½Ρ‹Π΅ услуги ΠΈ ΠΌΠΎΠ³ΡƒΡ‚ ΠΏΡ€ΠΎΠΈΠ·ΠΎΠΉΡ‚ΠΈ Π½Π΅ΠΏΡ€Π΅Π΄Π½Π°ΠΌΠ΅Ρ€Π΅Π½Π½Ρ‹Π΅ ΠΎΡ‚ΠΊΠ»ΡŽΡ‡Π΅Π½ΠΈΡ оборудования.Π’ случаС пСрСгорания прСдохранитСля кондСнсатора Π½Π΅ΠΎΠ±Ρ…ΠΎΠ΄ΠΈΠΌΠΎ ΠΈΠ·ΠΌΠ΅Ρ€ΠΈΡ‚ΡŒ ΠΏΡ€Π΅Π΄ΠΏΠΎΠ»Π°Π³Π°Π΅ΠΌΠΎΠ΅ Π·Π½Π°Ρ‡Π΅Π½ΠΈΠ΅ нСисправного кондСнсатора Π² ΠΌΠΈΠΊΡ€ΠΎΡ„Π°Ρ€Π°Π΄Π°Ρ… ΠΈ ΡƒΠ±Π΅Π΄ΠΈΡ‚ΡŒΡΡ, Ρ‡Ρ‚ΠΎ ΠΎΠ½ΠΎ находится Π² ΠΏΡ€Π΅Π΄Π΅Π»Π°Ρ… Π΄ΠΈΠ°ΠΏΠ°Π·ΠΎΠ½Π°, ΡƒΠΊΠ°Π·Π°Π½Π½ΠΎΠ³ΠΎ Π½Π° кондСнсаторС.

    Π‘Ρ‚ΠΎΠΈΡ‚ Π·Π½Π°Ρ‚ΡŒ Π½Π΅ΠΊΠΎΡ‚ΠΎΡ€Ρ‹Π΅ Π΄ΠΎΠΏΠΎΠ»Π½ΠΈΡ‚Π΅Π»ΡŒΠ½Ρ‹Π΅ Ρ„Π°ΠΊΡ‚ΠΎΡ€Ρ‹, связанныС с Π΅ΠΌΠΊΠΎΡΡ‚ΡŒΡŽ:

    • ΠšΠΎΠ½Π΄Π΅Π½ΡΠ°Ρ‚ΠΎΡ€Ρ‹ ΠΈΠΌΠ΅ΡŽΡ‚ ΠΎΠ³Ρ€Π°Π½ΠΈΡ‡Π΅Π½Π½Ρ‹ΠΉ срок слуТбы ΠΈ часто ΡΠ²Π»ΡΡŽΡ‚ΡΡ ΠΏΡ€ΠΈΡ‡ΠΈΠ½ΠΎΠΉ нСисправности.
    • НСисправныС кондСнсаторы ΠΌΠΎΠ³ΡƒΡ‚ ΠΈΠΌΠ΅Ρ‚ΡŒ ΠΊΠΎΡ€ΠΎΡ‚ΠΊΠΎΠ΅ Π·Π°ΠΌΡ‹ΠΊΠ°Π½ΠΈΠ΅, ΠΎΠ±Ρ€Ρ‹Π² Ρ†Π΅ΠΏΠΈ ΠΈΠ»ΠΈ ΠΌΠΎΠ³ΡƒΡ‚ физичСски ΠΈΡΠΏΠΎΡ€Ρ‚ΠΈΡ‚ΡŒΡΡ Π΄ΠΎ Ρ‚ΠΎΡ‡ΠΊΠΈ ΠΎΡ‚ΠΊΠ°Π·Π°.
    • ΠŸΡ€ΠΈ ΠΊΠΎΡ€ΠΎΡ‚ΠΊΠΎΠΌ Π·Π°ΠΌΡ‹ΠΊΠ°Π½ΠΈΠΈ кондСнсатора ΠΌΠΎΠΆΠ΅Ρ‚ ΠΏΠ΅Ρ€Π΅Π³ΠΎΡ€Π΅Ρ‚ΡŒ ΠΏΡ€Π΅Π΄ΠΎΡ…Ρ€Π°Π½ΠΈΡ‚Π΅Π»ΡŒ ΠΈΠ»ΠΈ ΠΏΠΎΠ²Ρ€Π΅Π΄ΠΈΡ‚ΡŒΡΡ Π΄Ρ€ΡƒΠ³ΠΈΠ΅ ΠΊΠΎΠΌΠΏΠΎΠ½Π΅Π½Ρ‚Ρ‹.
    • Когда кондСнсатор открываСтся ΠΈΠ»ΠΈ ΠΈΠ·Π½Π°ΡˆΠΈΠ²Π°Π΅Ρ‚ΡΡ, Ρ†Π΅ΠΏΡŒ ΠΈΠ»ΠΈ Π΅Π΅ ΠΊΠΎΠΌΠΏΠΎΠ½Π΅Π½Ρ‚Ρ‹ ΠΌΠΎΠ³ΡƒΡ‚ Π½Π΅ Ρ€Π°Π±ΠΎΡ‚Π°Ρ‚ΡŒ.
    • Износ Ρ‚Π°ΠΊΠΆΠ΅ ΠΌΠΎΠΆΠ΅Ρ‚ ΠΈΠ·ΠΌΠ΅Π½ΠΈΡ‚ΡŒ Π·Π½Π°Ρ‡Π΅Π½ΠΈΠ΅ Смкости кондСнсатора, Ρ‡Ρ‚ΠΎ ΠΌΠΎΠΆΠ΅Ρ‚ Π²Ρ‹Π·Π²Π°Ρ‚ΡŒ ΠΏΡ€ΠΎΠ±Π»Π΅ΠΌΡ‹.

    ΠšΠΎΠ½Π΄Π΅Π½ΡΠ°Ρ‚ΠΎΡ€Ρ‹ ΡΠ²Π»ΡΡŽΡ‚ΡΡ ΠΎΠ΄Π½ΠΈΠΌΠΈ ΠΈΠ· самых ΠΏΠΎΠ»Π΅Π·Π½Ρ‹Ρ… элСктронных ΠΊΠΎΠΌΠΏΠΎΠ½Π΅Π½Ρ‚ΠΎΠ². А Π΅ΠΌΠΊΠΎΡΡ‚ΡŒ β€” это Ρ‚Π΅Ρ€ΠΌΠΈΠ½, ΠΎΠ±ΠΎΠ·Π½Π°Ρ‡Π°ΡŽΡ‰ΠΈΠΉ ΡΠΏΠΎΡΠΎΠ±Π½ΠΎΡΡ‚ΡŒ кондСнсатора Π½Π°ΠΊΠ°ΠΏΠ»ΠΈΠ²Π°Ρ‚ΡŒ заряд. Π­Ρ‚ΠΎ Ρ‚Π°ΠΊΠΆΠ΅ ΠΈΠ·ΠΌΠ΅Ρ€Π΅Π½ΠΈΠ΅, ΠΈΡΠΏΠΎΠ»ΡŒΠ·ΡƒΠ΅ΠΌΠΎΠ΅ для указания Ρ‚ΠΎΠ³ΠΎ, сколько энСргии ΠΌΠΎΠΆΠ΅Ρ‚ Ρ…Ρ€Π°Π½ΠΈΡ‚ΡŒ ΠΊΠΎΠ½ΠΊΡ€Π΅Ρ‚Π½Ρ‹ΠΉ кондСнсатор. Π§Π΅ΠΌ большС Π΅ΠΌΠΊΠΎΡΡ‚ΡŒ кондСнсатора, Ρ‚Π΅ΠΌ большС заряда ΠΎΠ½ ΠΌΠΎΠΆΠ΅Ρ‚ Ρ…Ρ€Π°Π½ΠΈΡ‚ΡŒ.

    Π•ΠΌΠΊΠΎΡΡ‚ΡŒ измСряСтся Π² Π΅Π΄ΠΈΠ½ΠΈΡ†Π°Ρ…, Π½Π°Π·Ρ‹Π²Π°Π΅ΠΌΡ‹Ρ… Ρ„Π°Ρ€Π°Π΄ (сокращСнно F). ΠžΠΏΡ€Π΅Π΄Π΅Π»Π΅Π½ΠΈΠ΅ ΠΎΠ΄Π½ΠΎΠ³ΠΎ Ρ„Π°Ρ€Π°Π΄Π° ΠΎΠ±ΠΌΠ°Π½Ρ‡ΠΈΠ²ΠΎ простоС. ΠšΠΎΠ½Π΄Π΅Π½ΡΠ°Ρ‚ΠΎΡ€ Π΅ΠΌΠΊΠΎΡΡ‚ΡŒΡŽ ΠΎΠ΄ΠΈΠ½ Ρ„Π°Ρ€Π°Π΄ ΡƒΠ΄Π΅Ρ€ΠΆΠΈΠ²Π°Π΅Ρ‚ напряТСниС Π½Π° пластинах Ρ€ΠΎΠ²Π½ΠΎ ΠΎΠ΄ΠΈΠ½ Π²ΠΎΠ»ΡŒΡ‚, ΠΊΠΎΠ³Π΄Π° ΠΎΠ½ заряТаСтся Ρ‚ΠΎΠΊΠΎΠΌ Ρ€ΠΎΠ²Π½ΠΎ ΠΎΠ΄ΠΈΠ½ Π°ΠΌΠΏΠ΅Ρ€ Π² сСкунду.

    ΠžΠ±Ρ€Π°Ρ‚ΠΈΡ‚Π΅ Π²Π½ΠΈΠΌΠ°Π½ΠΈΠ΅, Ρ‡Ρ‚ΠΎ Π² этом ΠΎΠΏΡ€Π΅Π΄Π΅Π»Π΅Π½ΠΈΠΈ Ρ‡Π°ΡΡ‚ΡŒ Β«ΠΎΠ΄ΠΈΠ½ Π°ΠΌΠΏΠ΅Ρ€ Ρ‚ΠΎΠΊΠ° Π² сСкунду» Π½Π° самом Π΄Π΅Π»Π΅ относится ΠΊ количСству заряда, ΠΏΡ€ΠΈΡΡƒΡ‚ΡΡ‚Π²ΡƒΡŽΡ‰Π΅ΠΌΡƒ Π² кондСнсаторС. НСт ΠΏΡ€Π°Π²ΠΈΠ»Π°, согласно ΠΊΠΎΡ‚ΠΎΡ€ΠΎΠΌΡƒ Ρ‚ΠΎΠΊ Π΄ΠΎΠ»ΠΆΠ΅Π½ Ρ‚Π΅Ρ‡ΡŒ Ρ†Π΅Π»ΡƒΡŽ сСкунду. Π­Ρ‚ΠΎ ΠΌΠΎΠΆΠ΅Ρ‚ Π±Ρ‹Ρ‚ΡŒ ΠΎΠ΄ΠΈΠ½ Π°ΠΌΠΏΠ΅Ρ€ Π·Π° ΠΎΠ΄Π½Ρƒ сСкунду, ΠΈΠ»ΠΈ Π΄Π²Π° Π°ΠΌΠΏΠ΅Ρ€Π° Π·Π° полсСкунды, ΠΈΠ»ΠΈ ΠΏΠΎΠ»Π°ΠΌΠΏΠ΅Ρ€Π° Π·Π° Π΄Π²Π΅ сСкунды. Или это ΠΌΠΎΠΆΠ΅Ρ‚ Π±Ρ‹Ρ‚ΡŒ 100 мА Π² Ρ‚Π΅Ρ‡Π΅Π½ΠΈΠ΅ 10 сСкунд ΠΈΠ»ΠΈ 10 мА Π² Ρ‚Π΅Ρ‡Π΅Π½ΠΈΠ΅ 100 сСкунд.

    Один Π°ΠΌΠΏΠ΅Ρ€ Π² сСкунду соотвСтствуСт стандартной Π΅Π΄ΠΈΠ½ΠΈΡ†Π΅ измСрСния элСктричСского заряда, Π½Π°Π·Ρ‹Π²Π°Π΅ΠΌΠΎΠΉ ΠΊΡƒΠ»ΠΎΠ½ . Π’Π°ΠΊΠΈΠΌ ΠΎΠ±Ρ€Π°Π·ΠΎΠΌ, Π΄Ρ€ΡƒΠ³ΠΎΠΉ способ ΡƒΠΊΠ°Π·Π°Ρ‚ΡŒ Π·Π½Π°Ρ‡Π΅Π½ΠΈΠ΅ ΠΎΠ΄Π½ΠΎΠ³ΠΎ Ρ„Π°Ρ€Π°Π΄Π° β€” это ΡΠΊΠ°Π·Π°Ρ‚ΡŒ, Ρ‡Ρ‚ΠΎ это количСство Смкости, ΠΊΠΎΡ‚ΠΎΡ€ΠΎΠ΅ ΠΌΠΎΠΆΠ΅Ρ‚ Ρ…Ρ€Π°Π½ΠΈΡ‚ΡŒ ΠΎΠ΄ΠΈΠ½ ΠΊΡƒΠ»ΠΎΠ½ ΠΏΡ€ΠΈ напряТСнии Π² ΠΎΠ΄ΠΈΠ½ Π²ΠΎΠ»ΡŒΡ‚ Π½Π° пластинах.

    ΠŸΠΎΠ»ΡƒΡ‡Π°Π΅Ρ‚ΡΡ, Ρ‡Ρ‚ΠΎ ΠΎΠ΄ΠΈΠ½ Ρ„Π°Ρ€Π°Π΄ β€” это ΠΎΠ³Ρ€ΠΎΠΌΠ½ΠΎΠ΅ количСство Смкости, просто ΠΏΠΎΡ‚ΠΎΠΌΡƒ, Ρ‡Ρ‚ΠΎ ΠΎΠ΄ΠΈΠ½ ΠΊΡƒΠ»ΠΎΠ½ β€” это ΠΎΡ‡Π΅Π½ΡŒ большоС количСство заряда.Π§Ρ‚ΠΎΠ±Ρ‹ ΠΏΡ€Π΅Π΄ΡΡ‚Π°Π²ΠΈΡ‚ΡŒ это Π² пСрспСктивС, ΠΎΠ±Ρ‰ΠΈΠΉ заряд, содСрТащийся Π² срСднСм разрядС ΠΌΠΎΠ»Π½ΠΈΠΈ, составляСт ΠΎΠΊΠΎΠ»ΠΎ пяти ΠΊΡƒΠ»ΠΎΠ½ΠΎΠ², ΠΈ Π²Π°ΠΌ Π½ΡƒΠΆΠ½ΠΎ всСго ΠΏΡΡ‚ΡŒ кондСнсаторов Π΅ΠΌΠΊΠΎΡΡ‚ΡŒΡŽ ΠΎΠ΄ΠΈΠ½ Ρ„Π°Ρ€Π°Π΄, Ρ‡Ρ‚ΠΎΠ±Ρ‹ ΡΠΎΡ…Ρ€Π°Π½ΠΈΡ‚ΡŒ заряд, содСрТащийся Π² ΡƒΠ΄Π°Ρ€Π΅ ΠΌΠΎΠ»Π½ΠΈΠΈ. (НСкоторыС ΡƒΠ΄Π°Ρ€Ρ‹ ΠΌΠΎΠ»Π½ΠΈΠΈ Π³ΠΎΡ€Π°Π·Π΄ΠΎ ΠΌΠΎΡ‰Π½Π΅Π΅, Π΄ΠΎ 350 ΠΊΡƒΠ»ΠΎΠ½ΠΎΠ².)

    ΠŸΡ€Π΅Π΄ΠΏΠΎΠ»Π°Π³Π°Π΅Ρ‚ΡΡ, Ρ‡Ρ‚ΠΎ кондСнсатор ΠΏΠΎΡ‚ΠΎΠΊΠ° Π”ΠΎΠΊΠ° Π‘Ρ€Π°ΡƒΠ½Π° Π±Ρ‹Π» Π² Ρ„Π°Ρ€Π°Π΄Π½ΠΎΠΌ Π΄ΠΈΠ°ΠΏΠ°Π·ΠΎΠ½Π΅, ΠΏΠΎΡ‚ΠΎΠΌΡƒ Ρ‡Ρ‚ΠΎ Π”ΠΎΠΊ зарядил Π΅Π³ΠΎ ΡƒΠ΄Π°Ρ€ΠΎΠΌ ΠΌΠΎΠ»Π½ΠΈΠΈ. А Π²ΠΎΡ‚ кондСнсаторы, примСняСмыС Π² элСктроникС, Π·Π°Ρ€ΡΠΆΠ°ΡŽΡ‚ΡΡ ΠΎΡ‚ Π³ΠΎΡ€Π°Π·Π΄ΠΎ Π±ΠΎΠ»Π΅Π΅ скромных источников. Π“ΠΎΡ€Π°Π·Π΄ΠΎ скромнСС.

    На самом Π΄Π΅Π»Π΅ самыС большиС кондСнсаторы, ΠΊΠΎΡ‚ΠΎΡ€Ρ‹Π΅ Π²Ρ‹, вСроятно, Π±ΡƒΠ΄Π΅Ρ‚Π΅ ΠΈΡΠΏΠΎΠ»ΡŒΠ·ΠΎΠ²Π°Ρ‚ΡŒ, ΠΈΠΌΠ΅ΡŽΡ‚ Π΅ΠΌΠΊΠΎΡΡ‚ΡŒ, ΠΈΠ·ΠΌΠ΅Ρ€ΡΠ΅ΠΌΡƒΡŽ Π² ΠΌΠΈΠ»Π»ΠΈΠΎΠ½Π½Ρ‹Ρ… долях Ρ„Π°Ρ€Π°Π΄, Π½Π°Π·Ρ‹Π²Π°Π΅ΠΌΡƒΡŽ ΠΌΠΈΠΊΡ€ΠΎΡ„Π°Ρ€Π°Π΄ ΠΈ сокращСнно ΠΌΠΊ Π€. . называСтся ΠΏΠΈΠΊΠΎΡ„Π°Ρ€Π°Π΄ ΠΈ сокращСнно ΠΏΠ€.

    Π’ΠΎΡ‚ Π΅Ρ‰Π΅ нСсколько Π²Π΅Ρ‰Π΅ΠΉ, ΠΊΠΎΡ‚ΠΎΡ€Ρ‹Π΅ Π²Ρ‹ Π΄ΠΎΠ»ΠΆΠ½Ρ‹ Π·Π½Π°Ρ‚ΡŒ ΠΎΠ± измСрСниях кондСнсаторов:

    Как ΠΈ рСзисторы, кондСнсаторы Π½Π΅ ΠΈΠ·Π³ΠΎΡ‚Π°Π²Π»ΠΈΠ²Π°ΡŽΡ‚ΡΡ идСально.ВмСсто этого Π±ΠΎΠ»ΡŒΡˆΠΈΠ½ΡΡ‚Π²ΠΎ кондСнсаторов ΠΈΠΌΠ΅ΡŽΡ‚ ΠΏΡ€Π΅Π΄Π΅Π» ΠΏΠΎΠ³Ρ€Π΅ΡˆΠ½ΠΎΡΡ‚ΠΈ, Ρ‚Π°ΠΊΠΆΠ΅ Π½Π°Π·Ρ‹Π²Π°Π΅ΠΌΡ‹ΠΉ допуском . Π’ Π½Π΅ΠΊΠΎΡ‚ΠΎΡ€Ρ‹Ρ… случаях ΠΏΠΎΠ³Ρ€Π΅ΡˆΠ½ΠΎΡΡ‚ΡŒ ΠΌΠΎΠΆΠ΅Ρ‚ Π΄ΠΎΡΡ‚ΠΈΠ³Π°Ρ‚ΡŒ 80 %. К ΡΡ‡Π°ΡΡ‚ΡŒΡŽ, такая ΡΡ‚Π΅ΠΏΠ΅Π½ΡŒ впСчатлСния Ρ€Π΅Π΄ΠΊΠΎ ΠΎΠΊΠ°Π·Ρ‹Π²Π°Π΅Ρ‚ Π·Π°ΠΌΠ΅Ρ‚Π½ΠΎΠ΅ влияниС Π½Π° Π±ΠΎΠ»ΡŒΡˆΠΈΠ½ΡΡ‚Π²ΠΎ схСм.

    ΞΌ Π² ΞΌ F Π½Π΅ являСтся курсивом u ; это грСчСская Π±ΡƒΠΊΠ²Π° mu , которая являСтся ΠΎΠ±Ρ‰Π΅ΠΉ Π°Π±Π±Ρ€Π΅Π²ΠΈΠ°Ρ‚ΡƒΡ€ΠΎΠΉ для ΠΌΠΈΠΊΡ€ΠΎ .

    ΠžΠ±Ρ‹Ρ‡Π½ΠΎ значСния 1000 ΠΏΠ€ ΠΈ Π±ΠΎΠ»Π΅Π΅ ΠΏΡ€Π΅Π΄ΡΡ‚Π°Π²Π»ΡΡŽΡ‚ΡΡ Π² ΠΌΠΊΠ€, Π° Π½Π΅ Π² ΠΏΠ€. НапримСр, 1000 ΠΏΠ€ записываСтся ΠΊΠ°ΠΊ 0,001 ΠΌΠΊΠ€, Π° 22 000 ΠΏΠ€ записываСтся ΠΊΠ°ΠΊ 0,022 ΠΌΠΊΠ€.

    Как Ρ†ΠΈΡ„Ρ€ΠΎΠ²Ρ‹Π΅ ΠΌΡƒΠ»ΡŒΡ‚ΠΈΠΌΠ΅Ρ‚Ρ€Ρ‹ (DMM) ΠΈΠ·ΠΌΠ΅Ρ€ΡΡŽΡ‚ Π΅ΠΌΠΊΠΎΡΡ‚ΡŒ Ρ‡Π΅Ρ€Π΅Π· ΠΈΡ… Ρ‚ΠΈΠΏΠΈΡ‡Π½ΠΎΠ΅ Π²Ρ…ΠΎΠ΄Π½ΠΎΠ΅/Π²Ρ‹Ρ…ΠΎΠ΄Π½ΠΎΠ΅ сопротивлСниС 10 МОм?

    ΠŸΡ€ΠΈ обСспСчСнии логичСского уровня 3,3 Π’ ΠΏΠΎΠΏΡ‹Ρ‚ΠΊΠ° измСрСния 1F Π±ΡƒΠ΄Π΅Ρ‚ ΠΎΠ·Π½Π°Ρ‡Π°Ρ‚ΡŒ ΠΏΠΎΡΡ‚ΠΎΡΠ½Π½ΡƒΡŽ Π²Ρ€Π΅ΠΌΠ΅Π½ΠΈ 10 ΠΌΠΈΠ»Π»ΠΈΠΎΠ½ΠΎΠ² сСкунд (R x C), поэтому ΠΏΠΎΠ²Ρ‹ΡˆΠ΅Π½ΠΈΠ΅ напряТСния Π½Π° кондСнсаторС Π±ΡƒΠ΄Π΅Ρ‚ Π½Π΅ΠΈΠ·ΠΌΠ΅Ρ€ΠΈΠΌΡ‹ΠΌ (ΠΏΠΎ ΡƒΡ€ΠΎΠ²Π½ΡŽ ΡˆΡƒΠΌΠ°). Они Ρ‚Π°ΠΊΠΆΠ΅ Π΄Π΅Π»Π°ΡŽΡ‚ это Π² Ρ‚Π΅Ρ‡Π΅Π½ΠΈΠ΅ сСкунды. ΠΈΠ»ΠΈ ΠΎΠΊΠΎΠ»ΠΎ Ρ‚ΠΎΠ³ΠΎ с Ρ‚ΠΎΡ‡Π½ΠΎΡΡ‚ΡŒΡŽ 3%. Как это достигаСтся?

    3 ΠΎΡ‚Π²Π΅Ρ‚Π° 3

    БущСствуСт мноТСство способов измСрСния Смкости. Если Ρƒ вас Π΅ΡΡ‚ΡŒ Π³Π΅Π½Π΅Ρ€Π°Ρ‚ΠΎΡ€ сигналов, Π²Ρ‹ ΠΌΠΎΠΆΠ΅Ρ‚Π΅ ΠΈΡΠΏΠΎΠ»ΡŒΠ·ΠΎΠ²Π°Ρ‚ΡŒ ΠΏΡ€ΡΠΌΠΎΡƒΠ³ΠΎΠ»ΡŒΠ½Ρ‹ΠΉ сигнал ΠΈ ΠΈΠ·ΠΌΠ΅Ρ€ΠΈΡ‚ΡŒ врСмя нарастания.Или синусоида ΠΈ ΠΈΠ·ΠΌΠ΅Ρ€ΠΈΡ‚ΡŒ Ρ‚ΠΎΠΊ ΠΈ напряТСниС. Если Π²Ρ‹ Π·Π½Π°Π΅Ρ‚Π΅ Ρ‚ΠΎΠΊ ΠΈ напряТСниС, Π²Ρ‹ Π·Π½Π°Π΅Ρ‚Π΅, какая Ρƒ вас Π½Π°Π³Ρ€ΡƒΠ·ΠΊΠ°. Если Π½Π°Π³Ρ€ΡƒΠ·ΠΊΠΎΠΉ являСтся кондСнсатор, Π²Π°ΠΌ Ρ‚Π°ΠΊΠΆΠ΅ потрСбуСтся информация ΠΎ Ρ„Π°Π·Π΅. Бсылки Π½ΠΈΠΆΠ΅ Π±ΠΎΠ»Π΅Π΅ ΠΏΠΎΠ΄Ρ€ΠΎΠ±Π½ΠΎ Ρ€Π°ΡΡΠΊΠ°Π·Ρ‹Π²Π°ΡŽΡ‚ ΠΎ Ρ‚ΠΎΠΌ, ΠΊΠ°ΠΊ это дСлаСтся. ВмСсто Π³Π΅Π½Π΅Ρ€Π°Ρ‚ΠΎΡ€Π° сигналов Ρ†ΠΈΡ„Ρ€ΠΎΠ²Ρ‹Π΅ ΠΌΡƒΠ»ΡŒΡ‚ΠΈΠΌΠ΅Ρ‚Ρ€Ρ‹ ΠΎΠ±Ρ‹Ρ‡Π½ΠΎ ΠΈΠΌΠ΅ΡŽΡ‚ Π±ΠΎΠ»Π΅Π΅ ΠΏΡ€ΠΎΡΡ‚ΡƒΡŽ схСму (ΠΎΠ±Ρ‹Ρ‡Π½ΠΎ Π³Π΅Π½Π΅Ρ€ΠΈΡ€ΡƒΡŽΡ‰ΡƒΡŽ Ρ‚ΠΎΠ»ΡŒΠΊΠΎ ΠΎΠ΄Π½Ρƒ ΠΈΠ»ΠΈ нСсколько частот). ВмСсто схСмы осциллографа, ΠΈΠ·ΠΌΠ΅Ρ€ΡΡŽΡ‰Π΅ΠΉ Ρ„Π°Π·Ρƒ ΠΈ Π°ΠΌΠΏΠ»ΠΈΡ‚ΡƒΠ΄Ρƒ, Π΄Π΅Π»Π°Ρ‚ΡŒ расчСты.

    Π‘Π°ΠΌΠΎΠ΅ интСрСсноС, Ρ‡Ρ‚ΠΎ Ссли Ρƒ вас Π΅ΡΡ‚ΡŒ осциллограф ΠΈ Π³Π΅Π½Π΅Ρ€Π°Ρ‚ΠΎΡ€ сигналов, Π²Ρ‹ Ρ‚Π°ΠΊΠΆΠ΅ ΠΌΠΎΠΆΠ΅Ρ‚Π΅ ΠΈΠ·ΠΌΠ΅Ρ€ΡΡ‚ΡŒ Π΅ΠΌΠΊΠΎΡΡ‚ΡŒ, ΠΈΠ½ΠΎΠ³Π΄Π° Π»ΡƒΡ‡ΡˆΠ΅, Ρ‡Π΅ΠΌ Ρ†ΠΈΡ„Ρ€ΠΎΠ²ΠΎΠΉ ΠΌΡƒΠ»ΡŒΡ‚ΠΈΠΌΠ΅Ρ‚Ρ€.Π­Ρ‚ΠΎ Ρ‚Π°ΠΊΠΆΠ΅ Ρ€Π°Π±ΠΎΡ‚Π°Π΅Ρ‚ для индуктивности.


    Π˜ΡΡ‚ΠΎΡ‡Π½ΠΈΠΊ: https://meettechniek.info/passive/capacitance.html


    Π˜ΡΡ‚ΠΎΡ‡Π½ΠΈΠΊ: https://meettechniek.info/passive/capacitance.html

    Π•ΠΌΠΊΠΎΡΡ‚ΡŒ кондСнсатора β€” это ΡΠΏΠΎΡΠΎΠ±Π½ΠΎΡΡ‚ΡŒ кондСнсатора Π½Π°ΠΊΠ°ΠΏΠ»ΠΈΠ²Π°Ρ‚ΡŒ элСктричСский заряд Π½Π° Π΅Π΄ΠΈΠ½ΠΈΡ†Ρƒ напряТСния Π½Π° своих ΠΎΠ±ΠΊΠ»Π°Π΄ΠΊΠ°Ρ… кондСнсатора. Π•ΠΌΠΊΠΎΡΡ‚ΡŒ находится ΠΏΡƒΡ‚Π΅ΠΌ дСлСния элСктричСского заряда Π½Π° напряТСниС ΠΏΠΎ Ρ„ΠΎΡ€ΠΌΡƒΠ»Π΅ C=Q/V. Π•Π³ΠΎ Π΅Π΄ΠΈΠ½ΠΈΡ†Π΅ΠΉ являСтся Π€Π°Ρ€Π°Π΄Π°.

    Π€ΠΎΡ€ΠΌΡƒΠ»Π°

    Π•Π³ΠΎ Ρ„ΠΎΡ€ΠΌΡƒΠ»Π°:

    Π“Π΄Π΅ C β€” Π΅ΠΌΠΊΠΎΡΡ‚ΡŒ, Q β€” напряТСниС, Π° V β€” напряТСниС.ΠœΡ‹ Ρ‚Π°ΠΊΠΆΠ΅ ΠΌΠΎΠΆΠ΅ΠΌ Π½Π°ΠΉΡ‚ΠΈ заряд Q ΠΈ напряТСниС V, ΠΏΠ΅Ρ€Π΅Ρ„ΠΎΡ€ΠΌΡƒΠ»ΠΈΡ€ΠΎΠ²Π°Π² ΠΏΡ€ΠΈΠ²Π΅Π΄Π΅Π½Π½ΡƒΡŽ Π²Ρ‹ΡˆΠ΅ Ρ„ΠΎΡ€ΠΌΡƒΠ»Ρƒ ΡΠ»Π΅Π΄ΡƒΡŽΡ‰ΠΈΠΌ ΠΎΠ±Ρ€Π°Π·ΠΎΠΌ:

    .

    Π€Π°Ρ€Π°Π΄ β€” Π΅Π΄ΠΈΠ½ΠΈΡ†Π° измСрСния Смкости. Один Ρ„Π°Ρ€Π°Π΄ — это Π²Π΅Π»ΠΈΡ‡ΠΈΠ½Π° Смкости, ΠΊΠΎΠ³Π΄Π° ΠΎΠ΄ΠΈΠ½ ΠΊΡƒΠ»ΠΎΠ½ заряда хранится с ΠΎΠ΄Π½ΠΈΠΌ Π²ΠΎΠ»ΡŒΡ‚ΠΎΠΌ Π½Π° Π΅Π³ΠΎ пластинах.

    Π‘ΠΎΠ»ΡŒΡˆΠΈΠ½ΡΡ‚Π²ΠΎ кондСнсаторов, ΠΈΡΠΏΠΎΠ»ΡŒΠ·ΡƒΠ΅ΠΌΡ‹Ρ… Π² элСктроникС, ΠΈΠΌΠ΅ΡŽΡ‚ Π΅ΠΌΠΊΠΎΡΡ‚ΡŒ, ΡƒΠΊΠ°Π·Π°Π½Π½ΡƒΡŽ Π² ΠΌΠΈΠΊΡ€ΠΎΡ„Π°Ρ€Π°Π΄Π°Ρ… (ΠΌΠΊΠ€) ΠΈ ΠΏΠΈΠΊΠΎΡ„Π°Ρ€Π°Π΄Π°Ρ… (ΠΏΠ€). ΠœΠΈΠΊΡ€ΠΎΡ„Π°Ρ€Π°Π΄ β€” это ΠΎΠ΄Π½Π° миллионная Ρ‡Π°ΡΡ‚ΡŒ Ρ„Π°Ρ€Π°Π΄Π°, Π° ΠΏΠΈΠΊΠΎΡ„Π°Ρ€Π°Π΄ β€” ΠΎΠ΄Π½Π° триллионная Ρ‡Π°ΡΡ‚ΡŒ Ρ„Π°Ρ€Π°Π΄Π°.

    КакиС Ρ„Π°ΠΊΡ‚ΠΎΡ€Ρ‹ Π²Π»ΠΈΡΡŽΡ‚ Π½Π° Π΅ΠΌΠΊΠΎΡΡ‚ΡŒ кондСнсатора?

    Зависит ΠΎΡ‚ ΡΠ»Π΅Π΄ΡƒΡŽΡ‰ΠΈΡ… Ρ„Π°ΠΊΡ‚ΠΎΡ€ΠΎΠ²:

    ΠŸΠ»ΠΎΡ‰Π°Π΄ΡŒ пластин

    Π•ΠΌΠΊΠΎΡΡ‚ΡŒ прямо ΠΏΡ€ΠΎΠΏΠΎΡ€Ρ†ΠΈΠΎΠ½Π°Π»ΡŒΠ½Π° физичСскому Ρ€Π°Π·ΠΌΠ΅Ρ€Ρƒ пластин, опрСдСляСмому ΠΏΠ»ΠΎΡ‰Π°Π΄ΡŒΡŽ пластины, A.Π‘ΠΎΠ»ΡŒΡˆΠ°Ρ ΠΏΠ»ΠΎΡ‰Π°Π΄ΡŒ пластины Π΄Π°Π΅Ρ‚ Π±ΠΎΠ»ΡŒΡˆΡƒΡŽ Π΅ΠΌΠΊΠΎΡΡ‚ΡŒ ΠΈ ΠΌΠ΅Π½ΡŒΡˆΡƒΡŽ Π΅ΠΌΠΊΠΎΡΡ‚ΡŒ. На рис. (Π°) ΠΏΠΎΠΊΠ°Π·Π°Π½ΠΎ, Ρ‡Ρ‚ΠΎ ΠΏΠ»ΠΎΡ‰Π°Π΄ΡŒ пластины кондСнсатора с ΠΏΠ°Ρ€Π°Π»Π»Π΅Π»ΡŒΠ½Ρ‹ΠΌΠΈ пластинами Ρ€Π°Π²Π½Π° ΠΏΠ»ΠΎΡ‰Π°Π΄ΠΈ ΠΎΠ΄Π½ΠΎΠΉ ΠΈΠ· пластин. Если пластины ΠΏΠ΅Ρ€Π΅ΠΌΠ΅Ρ‰Π°ΡŽΡ‚ΡΡ ΠΎΡ‚Π½ΠΎΡΠΈΡ‚Π΅Π»ΡŒΠ½ΠΎ Π΄Ρ€ΡƒΠ³ Π΄Ρ€ΡƒΠ³Π°, ΠΊΠ°ΠΊ ΠΏΠΎΠΊΠ°Π·Π°Π½ΠΎ Π½Π° рис. (b), ΠΏΠ»ΠΎΡ‰Π°Π΄ΡŒ пСрСкрытия опрСдСляСт ΡΡ„Ρ„Π΅ΠΊΡ‚ΠΈΠ²Π½ΡƒΡŽ ΠΏΠ»ΠΎΡ‰Π°Π΄ΡŒ пластины. Π­Ρ‚ΠΎ ΠΈΠ·ΠΌΠ΅Π½Π΅Π½ΠΈΠ΅ эффСктивной ΠΏΠ»ΠΎΡ‰Π°Π΄ΠΈ пластины являСтся основным для ΠΎΠΏΡ€Π΅Π΄Π΅Π»Π΅Π½Π½ΠΎΠ³ΠΎ Ρ‚ΠΈΠΏΠ° ΠΏΠ΅Ρ€Π΅ΠΌΠ΅Π½Π½ΠΎΠ³ΠΎ кондСнсатора.

    Π Π°Π·Π΄Π΅Π»ΠΈΡ‚Π΅Π»ΡŒΠ½Ρ‹Π΅ пластины

    `Π•ΠΌΠΊΠΎΡΡ‚ΡŒ ΠΎΠ±Ρ€Π°Ρ‚Π½ΠΎ ΠΏΡ€ΠΎΠΏΠΎΡ€Ρ†ΠΈΠΎΠ½Π°Π»ΡŒΠ½Π° Ρ€Π°ΡΡΡ‚ΠΎΡΠ½ΠΈΡŽ ΠΌΠ΅ΠΆΠ΄Ρƒ пластинами. Π Π°Π·Π΄Π΅Π»Π΅Π½ΠΈΠ΅ пластин ΠΎΠ±ΠΎΠ·Π½Π°Ρ‡Π΅Π½ΠΎ Π±ΡƒΠΊΠ²ΠΎΠΉ d, ΠΊΠ°ΠΊ ΠΏΠΎΠΊΠ°Π·Π°Π½ΠΎ Π½Π° рис. (Π°). Π§Π΅ΠΌ большС расстояниС ΠΌΠ΅ΠΆΠ΄Ρƒ пластинами, Ρ‚Π΅ΠΌ мСньшС Π΅ΠΌΠΊΠΎΡΡ‚ΡŒ, ΠΊΠ°ΠΊ ΠΏΠΎΠΊΠ°Π·Π°Π½ΠΎ Π½Π° рис. (b). Как ΠΎΠ±ΡΡƒΠΆΠ΄Π°Π»ΠΎΡΡŒ Ρ€Π°Π½Π΅Π΅, напряТСниС пробоя прямо ΠΏΡ€ΠΎΠΏΠΎΡ€Ρ†ΠΈΠΎΠ½Π°Π»ΡŒΠ½ΠΎ Ρ€Π°ΡΡΡ‚ΠΎΡΠ½ΠΈΡŽ ΠΌΠ΅ΠΆΠ΄Ρƒ пластинами. Π§Π΅ΠΌ дальшС разнСсСны пластины, Ρ‚Π΅ΠΌ большС напряТСниС пробоя .

    ДиэлСктричСская ΠΏΡ€ΠΎΠ½ΠΈΡ†Π°Π΅ΠΌΠΎΡΡ‚ΡŒ ΠΌΠ°Ρ‚Π΅Ρ€ΠΈΠ°Π»Π°

    Как извСстно, изоляционный ΠΌΠ°Ρ‚Π΅Ρ€ΠΈΠ°Π» ΠΌΠ΅ΠΆΠ΄Ρƒ ΠΎΠ±ΠΊΠ»Π°Π΄ΠΊΠ°ΠΌΠΈ кондСнсатора называСтся диэлСктриком. ДиэлСктричСскиС ΠΌΠ°Ρ‚Π΅Ρ€ΠΈΠ°Π»Ρ‹ ΠΈΠΌΠ΅ΡŽΡ‚ Ρ‚Π΅Π½Π΄Π΅Π½Ρ†ΠΈΡŽ ΡƒΠΌΠ΅Π½ΡŒΡˆΠ°Ρ‚ΡŒ напряТСниС ΠΌΠ΅ΠΆΠ΄Ρƒ пластинами для Π΄Π°Π½Π½ΠΎΠ³ΠΎ заряда ΠΈ, Ρ‚Π°ΠΊΠΈΠΌ ΠΎΠ±Ρ€Π°Π·ΠΎΠΌ, ΡƒΠ²Π΅Π»ΠΈΡ‡ΠΈΠ²Π°Ρ‚ΡŒ Π΅ΠΌΠΊΠΎΡΡ‚ΡŒ.Если напряТСниС фиксировано, ΠΈΠ·-Π·Π° присутствия диэлСктрика ΠΌΠΎΠΆΠ΅Ρ‚ Π±Ρ‹Ρ‚ΡŒ сохранСно большС заряда, Ρ‡Π΅ΠΌ Π±Π΅Π· диэлСктрика. ΠœΠ΅Ρ€Π° способности ΠΌΠ°Ρ‚Π΅Ρ€ΠΈΠ°Π»Π° ΡΠΎΠ·Π΄Π°Π²Π°Ρ‚ΡŒ элСктричСскоС ΠΏΠΎΠ»Π΅ называСтся диэлСктричСской ΠΏΡ€ΠΎΠ½ΠΈΡ†Π°Π΅ΠΌΠΎΡΡ‚ΡŒΡŽ ΠΈΠ»ΠΈ ΠΎΡ‚Π½ΠΎΡΠΈΡ‚Π΅Π»ΡŒΠ½ΠΎΠΉ диэлСктричСской ΠΏΡ€ΠΎΠ½ΠΈΡ†Π°Π΅ΠΌΠΎΡΡ‚ΡŒΡŽ, ΠΎΠ±ΠΎΠ·Π½Π°Ρ‡Π°Π΅ΠΌΠΎΠΉ ΠΊΠ°ΠΊ ∈ r .

    Π•ΠΌΠΊΠΎΡΡ‚ΡŒ прямо ΠΏΡ€ΠΎΠΏΠΎΡ€Ρ†ΠΈΠΎΠ½Π°Π»ΡŒΠ½Π° диэлСктричСской проницаСмости. ДиэлСктричСская ΠΏΡ€ΠΎΠ½ΠΈΡ†Π°Π΅ΠΌΠΎΡΡ‚ΡŒ Π²Π°ΠΊΡƒΡƒΠΌΠ° опрСдСляСтся ΠΊΠ°ΠΊ 1, Π° диэлСктричСская ΠΏΡ€ΠΎΠ½ΠΈΡ†Π°Π΅ΠΌΠΎΡΡ‚ΡŒ Π²ΠΎΠ·Π΄ΡƒΡ…Π° ΠΎΡ‡Π΅Π½ΡŒ Π±Π»ΠΈΠ·ΠΊΠ° ΠΊ 1. Π­Ρ‚ΠΈ значСния ΠΈΡΠΏΠΎΠ»ΡŒΠ·ΡƒΡŽΡ‚ΡΡ Π² качСствС справочных, ΠΈ всС Π΄Ρ€ΡƒΠ³ΠΈΠ΅ ΠΌΠ°Ρ‚Π΅Ρ€ΠΈΠ°Π»Ρ‹ ΠΈΠΌΠ΅ΡŽΡ‚ значСния Ξ΅r, ΡƒΠΊΠ°Π·Π°Π½Π½Ρ‹Π΅ ΠΏΠΎ ΠΎΡ‚Π½ΠΎΡˆΠ΅Π½ΠΈΡŽ ΠΊ Π²Π°ΠΊΡƒΡƒΠΌΡƒ ΠΈΠ»ΠΈ Π²ΠΎΠ·Π΄ΡƒΡ…Ρƒ.НапримСр, ΠΌΠ°Ρ‚Π΅Ρ€ΠΈΠ°Π» с Ξ΅r=8 ΠΌΠΎΠΆΠ΅Ρ‚ ΠΈΠΌΠ΅Ρ‚ΡŒ Π΅ΠΌΠΊΠΎΡΡ‚ΡŒ, Π² восСмь Ρ€Π°Π· ΠΏΡ€Π΅Π²Ρ‹ΡˆΠ°ΡŽΡ‰ΡƒΡŽ Π΅ΠΌΠΊΠΎΡΡ‚ΡŒ Π²ΠΎΠ·Π΄ΡƒΡ…Π°, ΠΏΡ€ΠΈ ΠΏΡ€ΠΎΡ‡ΠΈΡ… Ρ€Π°Π²Π½Ρ‹Ρ… условиях.

    ДиэлСктричСская ΠΏΡ€ΠΎΠ½ΠΈΡ†Π°Π΅ΠΌΠΎΡΡ‚ΡŒ ∈r Π±Π΅Π·Ρ€Π°Π·ΠΌΠ΅Ρ€Π½Π°, ΠΏΠΎΡΠΊΠΎΠ»ΡŒΠΊΡƒ являСтся ΠΎΡ‚Π½ΠΎΡΠΈΡ‚Π΅Π»ΡŒΠ½ΠΎΠΉ ΠΌΠ΅Ρ€ΠΎΠΉ. Π­Ρ‚ΠΎ ΠΎΡ‚Π½ΠΎΡˆΠ΅Π½ΠΈΠ΅ Π°Π±ΡΠΎΠ»ΡŽΡ‚Π½ΠΎΠΉ диэлСктричСской проницаСмости ΠΌΠ°Ρ‚Π΅Ρ€ΠΈΠ°Π»Π°,∈r, ΠΊ Π°Π±ΡΠΎΠ»ΡŽΡ‚Π½ΠΎΠΉ диэлСктричСской проницаСмости Π²Π°ΠΊΡƒΡƒΠΌΠ°,∈ 0 , Π²Ρ‹Ρ€Π°ΠΆΠ°Π΅ΠΌΠΎΠ΅ ΡΠ»Π΅Π΄ΡƒΡŽΡ‰Π΅ΠΉ Ρ„ΠΎΡ€ΠΌΡƒΠ»ΠΎΠΉ:

    НиТС ΠΏΡ€ΠΈΠ²Π΅Π΄Π΅Π½Ρ‹ Π½Π΅ΠΊΠΎΡ‚ΠΎΡ€Ρ‹Π΅ распространСнныС диэлСктричСскиС ΠΌΠ°Ρ‚Π΅Ρ€ΠΈΠ°Π»Ρ‹ ΠΈ Ρ‚ΠΈΠΏΠΈΡ‡Π½Ρ‹Π΅ диэлСктричСскиС постоянныС для ΠΊΠ°ΠΆΠ΄ΠΎΠ³ΠΎ ΠΈΠ· Π½ΠΈΡ…. ЗначСния ΠΌΠΎΠ³ΡƒΡ‚ Π²Π°Ρ€ΡŒΠΈΡ€ΠΎΠ²Π°Ρ‚ΡŒΡΡ, ΠΏΠΎΡΠΊΠΎΠ»ΡŒΠΊΡƒ зависят ΠΎΡ‚ ΠΊΠΎΠ½ΠΊΡ€Π΅Ρ‚Π½ΠΎΠ³ΠΎ состава ΠΌΠ°Ρ‚Π΅Ρ€ΠΈΠ°Π»Π°.

    ΠœΠ°Ρ‚Π΅Ρ€ΠΈΠ°Π» Π’ΠΈΠΏΠΈΡ‡Π½Ρ‹Π΅ значСния ∈r

    • Π’ΠΎΠ·Π΄ΡƒΡ… 1.0
    • Π’Π΅Ρ„Π»ΠΎΠ½ 2,0
    • Π‘ΡƒΠΌΠ°Π³Π° 2,5
    • Масло 4.0
    • Блюда 5. 0
    • Π‘Ρ‚Π΅ΠΊΠ»ΠΎ 7,5
    • ΠšΠ΅Ρ€Π°ΠΌΠΈΠΊΠ° 1200

    ДиэлСктричСская ΠΏΡ€ΠΎΠ½ΠΈΡ†Π°Π΅ΠΌΠΎΡΡ‚ΡŒ ∈r Π±Π΅Π·Ρ€Π°Π·ΠΌΠ΅Ρ€Π½Π°, ΠΏΠΎΡΠΊΠΎΠ»ΡŒΠΊΡƒ являСтся ΠΎΡ‚Π½ΠΎΡΠΈΡ‚Π΅Π»ΡŒΠ½ΠΎΠΉ ΠΌΠ΅Ρ€ΠΎΠΉ. Π­Ρ‚ΠΎ ΠΎΡ‚Π½ΠΎΡˆΠ΅Π½ΠΈΠ΅ Π°Π±ΡΠΎΠ»ΡŽΡ‚Π½ΠΎΠΉ диэлСктричСской проницаСмости ΠΌΠ°Ρ‚Π΅Ρ€ΠΈΠ°Π»Π°,∈r, ΠΊ Π°Π±ΡΠΎΠ»ΡŽΡ‚Π½ΠΎΠΉ диэлСктричСской проницаСмости Π²Π°ΠΊΡƒΡƒΠΌΠ°,∈0, Π²Ρ‹Ρ€Π°ΠΆΠ°Π΅ΠΌΠΎΠ΅ ΡΠ»Π΅Π΄ΡƒΡŽΡ‰Π΅ΠΉ Ρ„ΠΎΡ€ΠΌΡƒΠ»ΠΎΠΉ:

    Π—Π½Π°Ρ‡Π΅Π½ΠΈΠ΅ ∈0 Ρ€Π°Π²Π½ΠΎ 8,85Γ—10-12 Π€/ΠΌ.

    Π€ΠΎΡ€ΠΌΡƒΠ»Π° Смкости Ρ‡Π΅Ρ€Π΅Π· физичСскиС ΠΏΠ°Ρ€Π°ΠΌΠ΅Ρ‚Ρ€Ρ‹

    Π’Ρ‹ Π²ΠΈΠ΄Π΅Π»ΠΈ, Ρ‡Ρ‚ΠΎ Π΅ΠΌΠΊΠΎΡΡ‚ΡŒ Π½Π°ΠΏΡ€ΡΠΌΡƒΡŽ связана с ΠΏΠ»ΠΎΡ‰Π°Π΄ΡŒΡŽ пластины, A, ΠΈ диэлСктричСской ΠΏΡ€ΠΎΠ½ΠΈΡ†Π°Π΅ΠΌΠΎΡΡ‚ΡŒΡŽ,Ξ΅r, ΠΈ ΠΎΠ±Ρ€Π°Ρ‚Π½ΠΎ ΠΏΡ€ΠΎΠΏΠΎΡ€Ρ†ΠΈΠΎΠ½Π°Π»ΡŒΠ½Π° Ρ€Π°ΡΡΡ‚ΠΎΡΠ½ΠΈΡŽ ΠΌΠ΅ΠΆΠ΄Ρƒ пластинами, d. Вочная Ρ„ΠΎΡ€ΠΌΡƒΠ»Π° для расчСта Смкости Ρ‡Π΅Ρ€Π΅Π· эти Ρ‚Ρ€ΠΈ Π²Π΅Π»ΠΈΡ‡ΠΈΠ½Ρ‹:

    ΠŸΡ€ΠΎΠΈΠ·Π²ΠΎΠ΄Π½Π°Ρ Π΅ΠΌΠΊΠΎΡΡ‚ΡŒ ΠΏΠ»ΠΎΡΠΊΠΎΠΏΠ°Ρ€Π°Π»Π»Π΅Π»ΡŒΠ½ΠΎΠ³ΠΎ кондСнсатора

    Рассмотрим кондСнсатор с плоскими пластинами. Π Π°Π·ΠΌΠ΅Ρ€ пластины большой, Π° расстояниС ΠΌΠ΅ΠΆΠ΄Ρƒ пластинами ΠΎΡ‡Π΅Π½ΡŒ малСнькоС, поэтому элСктричСскоС ΠΏΠΎΠ»Π΅ ΠΌΠ΅ΠΆΠ΄Ρƒ пластинами ΠΎΠ΄Π½ΠΎΡ€ΠΎΠ΄Π½ΠΎ.

    ЭлСктричСскоС ΠΏΠΎΠ»Π΅ Β«EΒ» ΠΌΠ΅ΠΆΠ΄Ρƒ плоским кондСнсатором составляСт:

    Π•ΠΌΠΊΠΎΡΡ‚ΡŒ цилиндричСских кондСнсаторов Ρ„ΠΈΠ·ΠΈΠΊΠ°

    Рассмотрим цилиндричСский кондСнсатор Π΄Π»ΠΈΠ½ΠΎΠΉ L, ΠΎΠ±Ρ€Π°Π·ΠΎΠ²Π°Π½Π½Ρ‹ΠΉ двумя ΠΊΠΎΠ°ΠΊΡΠΈΠ°Π»ΡŒΠ½Ρ‹ΠΌΠΈ Ρ†ΠΈΠ»ΠΈΠ½Π΄Ρ€Π°ΠΌΠΈ радиусов Β«aΒ» ΠΈ Β«bΒ». ΠŸΡ€Π΅Π΄ΠΏΠΎΠ»ΠΎΠΆΠΈΠΌ, Ρ‡Ρ‚ΠΎ L >> b, Ρ‚Π°ΠΊ Ρ‡Ρ‚ΠΎ Π½Π° ΠΊΠΎΠ½Ρ†Π°Ρ… Ρ†ΠΈΠ»ΠΈΠ½Π΄Ρ€ΠΎΠ² Π½Π΅Ρ‚ ΠΊΡ€Π°Π΅Π²ΠΎΠ³ΠΎ поля.

    ΠŸΡƒΡΡ‚ΡŒ Β«qΒ» β€” заряд кондСнсатора, Π° Β«VΒ» β€” Ρ€Π°Π·Π½ΠΎΡΡ‚ΡŒ ΠΏΠΎΡ‚Π΅Π½Ρ†ΠΈΠ°Π»ΠΎΠ² ΠΌΠ΅ΠΆΠ΄Ρƒ пластинами. Π’Π½ΡƒΡ‚Ρ€Π΅Π½Π½ΠΈΠΉ Ρ†ΠΈΠ»ΠΈΠ½Π΄Ρ€ заряТСн ΠΏΠΎΠ»ΠΎΠΆΠΈΡ‚Π΅Π»ΡŒΠ½ΠΎ, Π° внСшний Ρ†ΠΈΠ»ΠΈΠ½Π΄Ρ€ заряТСн ΠΎΡ‚Ρ€ΠΈΡ†Π°Ρ‚Π΅Π»ΡŒΠ½ΠΎ.ΠœΡ‹ Ρ…ΠΎΡ‚ΠΈΠΌ Π½Π°ΠΉΡ‚ΠΈ Π²Ρ‹Ρ€Π°ΠΆΠ΅Π½ΠΈΠ΅ для Смкости цилиндричСского кондСнсатора. Для этого рассмотрим Ρ†ΠΈΠ»ΠΈΠ½Π΄Ρ€ΠΈΡ‡Π΅ΡΠΊΡƒΡŽ гауссову ΠΏΠΎΠ²Π΅Ρ€Ρ…Π½ΠΎΡΡ‚ΡŒ радиуса Β«rΒ», Ρ‚Π°ΠΊΡƒΡŽ, Ρ‡Ρ‚ΠΎ Π’Π΅Π³ΠΈ

    Π’ этой ΡΡ‚Π°Ρ‚ΡŒΠ΅ ΠΌΡ‹ рассмотрим Ρ€Π°Π·Π»ΠΈΡ‡Π½Ρ‹Π΅ тСсты, ΠΊΠΎΡ‚ΠΎΡ€Ρ‹Π΅ ΠΌΡ‹ ΠΌΠΎΠΆΠ΅ΠΌ ΠΈΡΠΏΠΎΠ»ΡŒΠ·ΠΎΠ²Π°Ρ‚ΡŒ, Ρ‡Ρ‚ΠΎΠ±Ρ‹ ΠΎΠΏΡ€Π΅Π΄Π΅Π»ΠΈΡ‚ΡŒ, Ρ…ΠΎΡ€ΠΎΡˆ кондСнсатор ΠΈΠ»ΠΈ Π½Π΅Ρ‚, ΠΈΡΠΏΠΎΠ»ΡŒΠ·ΡƒΡ Ρ„ΡƒΠ½ΠΊΡ†ΠΈΠΈ Ρ†ΠΈΡ„Ρ€ΠΎΠ²ΠΎΠ³ΠΎ ΠΌΡƒΠ»ΡŒΡ‚ΠΈΠΌΠ΅Ρ‚Ρ€Π°.

    ΠœΡ‹ ΠΌΠΎΠΆΠ΅ΠΌ Π²Ρ‹ΠΏΠΎΠ»Π½ΠΈΡ‚ΡŒ мноТСство ΠΏΡ€ΠΎΠ²Π΅Ρ€ΠΎΠΊ, Ρ‡Ρ‚ΠΎΠ±Ρ‹ ΡƒΠ±Π΅Π΄ΠΈΡ‚ΡŒΡΡ, Ρ‡Ρ‚ΠΎ кондСнсатор Ρ€Π°Π±ΠΎΡ‚Π°Π΅Ρ‚ Π΄ΠΎΠ»ΠΆΠ½Ρ‹ΠΌ ΠΎΠ±Ρ€Π°Π·ΠΎΠΌ. ΠœΡ‹ Π±ΡƒΠ΄Π΅ΠΌ ΠΈΡΠΏΠΎΠ»ΡŒΠ·ΠΎΠ²Π°Ρ‚ΡŒ ΠΈ ΠΈΡΠΏΠΎΠ»ΡŒΠ·ΠΎΠ²Π°Ρ‚ΡŒ характСристики ΠΈ ΠΏΠΎΠ²Π΅Π΄Π΅Π½ΠΈΠ΅, ΠΊΠΎΡ‚ΠΎΡ€Ρ‹Π΅ Π΄ΠΎΠ»ΠΆΠ΅Π½ ΠΏΠΎΠΊΠ°Π·Ρ‹Π²Π°Ρ‚ΡŒ кондСнсатор, Ссли ΠΎΠ½ исправСн, ΠΈ, Ρ‚Π°ΠΊΠΈΠΌ ΠΎΠ±Ρ€Π°Π·ΠΎΠΌ, ΠΎΠΏΡ€Π΅Π΄Π΅Π»ΡΡ‚ΡŒ, исправСн ΠΎΠ½ ΠΈΠ»ΠΈ нСисправСн.

    ΠŸΡ€ΠΎΠ²Π΅Ρ€ΠΊΠ° кондСнсатора ΠΎΠΌΠΌΠ΅Ρ‚Ρ€ΠΎΠΌ ΠΌΡƒΠ»ΡŒΡ‚ΠΈΠΌΠ΅Ρ‚Ρ€Π°

    ΠžΡ‡Π΅Π½ΡŒ Ρ…ΠΎΡ€ΠΎΡˆΠΈΠΉ тСст, ΠΊΠΎΡ‚ΠΎΡ€Ρ‹ΠΉ Π²Ρ‹ ΠΌΠΎΠΆΠ΅Ρ‚Π΅ ΡΠ΄Π΅Π»Π°Ρ‚ΡŒ, это ΠΏΡ€ΠΎΠ²Π΅Ρ€ΠΈΡ‚ΡŒ кондСнсатор с ΠΏΠΎΠΌΠΎΡ‰ΡŒΡŽ ΠΌΡƒΠ»ΡŒΡ‚ΠΈΠΌΠ΅Ρ‚Ρ€Π°, настроСнного Π½Π° настройку ΠΎΠΌΠΌΠ΅Ρ‚Ρ€Π°.

    Π˜Π·ΠΌΠ΅Ρ€ΠΈΠ² сопротивлСниС кондСнсатора, ΠΌΡ‹ ΠΌΠΎΠΆΠ΅ΠΌ ΠΎΠΏΡ€Π΅Π΄Π΅Π»ΠΈΡ‚ΡŒ, Ρ…ΠΎΡ€ΠΎΡˆΠΈΠΉ ΠΎΠ½ ΠΈΠ»ΠΈ ΠΏΠ»ΠΎΡ…ΠΎΠΉ.

    Π§Ρ‚ΠΎΠ±Ρ‹ провСсти этот тСст, ΠΌΡ‹ Π±Π΅Ρ€Π΅ΠΌ ΠΎΠΌΠΌΠ΅Ρ‚Ρ€ ΠΈ ΠΏΠΎΠΌΠ΅Ρ‰Π°Π΅ΠΌ Ρ‰ΡƒΠΏΡ‹ Π½Π° Π²Ρ‹Π²ΠΎΠ΄Ρ‹ кондСнсатора. ΠžΡ€ΠΈΠ΅Π½Ρ‚Π°Ρ†ΠΈΡ Π½Π΅ ΠΈΠΌΠ΅Π΅Ρ‚ значСния, ΠΏΠΎΡ‚ΠΎΠΌΡƒ Ρ‡Ρ‚ΠΎ сопротивлСниС Π½Π΅ поляризовано.

    Если ΠΌΡ‹ считываСм ΠΎΡ‡Π΅Π½ΡŒ Π½ΠΈΠ·ΠΊΠΎΠ΅ сопротивлСниС (ΠΎΠΊΠΎΠ»ΠΎ 0 Ом) Π½Π° кондСнсаторС, ΠΌΡ‹ Π·Π½Π°Π΅ΠΌ, Ρ‡Ρ‚ΠΎ кондСнсатор нСисправСн.Он читаСтся Ρ‚Π°ΠΊ, ΠΊΠ°ΠΊ Π±ΡƒΠ΄Ρ‚ΠΎ Π½Π° Π½Π΅ΠΌ ΠΊΠΎΡ€ΠΎΡ‚ΠΊΠΎΠ΅ Π·Π°ΠΌΡ‹ΠΊΠ°Π½ΠΈΠ΅.

    Если ΠΌΡ‹ Ρ‡ΠΈΡ‚Π°Π΅ΠΌ ΠΎΡ‡Π΅Π½ΡŒ высокоС сопротивлСниС Π½Π° кондСнсаторС (нСсколько МОм), это ΠΏΡ€ΠΈΠ·Π½Π°ΠΊ Ρ‚ΠΎΠ³ΠΎ, Ρ‡Ρ‚ΠΎ кондСнсатор, вСроятно, Ρ‚ΠΎΠΆΠ΅ нСисправСн. Π­Ρ‚ΠΎ читаСтся, ΠΊΠ°ΠΊ Π±ΡƒΠ΄Ρ‚ΠΎ Π΅ΡΡ‚ΡŒ ΠΎΠ±Ρ€Ρ‹Π² Ρ†Π΅ΠΏΠΈ Π½Π° кондСнсаторС.

    ΠžΠ±Ρ‹Ρ‡Π½Ρ‹ΠΉ кондСнсатор Π±ΡƒΠ΄Π΅Ρ‚ ΠΈΠΌΠ΅Ρ‚ΡŒ сопротивлСниС Π³Π΄Π΅-Ρ‚ΠΎ ΠΌΠ΅ΠΆΠ΄Ρƒ этими двумя ΠΊΡ€Π°ΠΉΠ½ΠΈΠΌΠΈ значСниями, скаТСм, Π³Π΄Π΅-Ρ‚ΠΎ Π² дСсятках тысяч ΠΈΠ»ΠΈ сотнях тысяч Ом. Но Π½Π΅ 0 Ом ΠΈ Π½Π΅ нСсколько МОм.

    Π­Ρ‚ΠΎ простой, Π½ΠΎ эффСктивный ΠΌΠ΅Ρ‚ΠΎΠ΄ опрСдСлСния нСисправности кондСнсатора.

    ΠŸΡ€ΠΎΠ²Π΅Ρ€ΠΊΠ° кондСнсатора с ΠΏΠΎΠΌΠΎΡ‰ΡŒΡŽ ΠΌΡƒΠ»ΡŒΡ‚ΠΈΠΌΠ΅Ρ‚Ρ€Π° Π² настройкС Смкости

    Π•Ρ‰Π΅ ΠΎΠ΄Π½Π° ΠΏΡ€ΠΎΠ²Π΅Ρ€ΠΊΠ°, ΠΊΠΎΡ‚ΠΎΡ€ΡƒΡŽ Π²Ρ‹ ΠΌΠΎΠΆΠ΅Ρ‚Π΅ ΡΠ΄Π΅Π»Π°Ρ‚ΡŒ, это ΠΏΡ€ΠΎΠ²Π΅Ρ€ΠΈΡ‚ΡŒ Π΅ΠΌΠΊΠΎΡΡ‚ΡŒ кондСнсатора с ΠΏΠΎΠΌΠΎΡ‰ΡŒΡŽ ΠΌΡƒΠ»ΡŒΡ‚ΠΈΠΌΠ΅Ρ‚Ρ€Π°, Ссли Ρƒ вас Π΅ΡΡ‚ΡŒ ΠΈΠ·ΠΌΠ΅Ρ€ΠΈΡ‚Π΅Π»ΡŒ Смкости Π½Π° вашСм ΠΌΡƒΠ»ΡŒΡ‚ΠΈΠΌΠ΅Ρ‚Ρ€Π΅. ВсС, Ρ‡Ρ‚ΠΎ Π²Π°ΠΌ Π½ΡƒΠΆΠ½ΠΎ ΡΠ΄Π΅Π»Π°Ρ‚ΡŒ, это ΠΏΡ€ΠΎΡ‡ΠΈΡ‚Π°Ρ‚ΡŒ Π΅ΠΌΠΊΠΎΡΡ‚ΡŒ, которая находится Π½Π° внСшнСй сторонС кондСнсатора, Π²Π·ΡΡ‚ΡŒ Ρ‰ΡƒΠΏΡ‹ ΠΌΡƒΠ»ΡŒΡ‚ΠΈΠΌΠ΅Ρ‚Ρ€Π° ΠΈ ΠΏΠΎΠΌΠ΅ΡΡ‚ΠΈΡ‚ΡŒ ΠΈΡ… Π½Π° Π²Ρ‹Π²ΠΎΠ΄Ρ‹ кондСнсатора. ΠŸΠΎΠ»ΡΡ€Π½ΠΎΡΡ‚ΡŒ Π½Π΅ ΠΈΠΌΠ΅Π΅Ρ‚ значСния.

    Π­Ρ‚ΠΎ Ρ‚ΠΎ ΠΆΠ΅ самоС, Ρ‡Ρ‚ΠΎ ΠΈ пСрвая ΠΈΠ»Π»ΡŽΡΡ‚Ρ€Π°Ρ†ΠΈΡ, Ρ‚ΠΎΠ»ΡŒΠΊΠΎ Ρ‚Π΅ΠΏΠ΅Ρ€ΡŒ ΠΌΡƒΠ»ΡŒΡ‚ΠΈΠΌΠ΅Ρ‚Ρ€ настроСн Π½Π° настройку Смкости.

    Π’Ρ‹ Π΄ΠΎΠ»ΠΆΠ½Ρ‹ ΠΏΡ€ΠΎΡ‡ΠΈΡ‚Π°Ρ‚ΡŒ Π·Π½Π°Ρ‡Π΅Π½ΠΈΠ΅ рядом с номинальной Π΅ΠΌΠΊΠΎΡΡ‚ΡŒΡŽ кондСнсатора. Из-Π·Π° допуска ΠΈ Ρ‚ΠΎΠ³ΠΎ, Ρ‡Ρ‚ΠΎ (Π² частности, элСктролитичСскиС кондСнсаторы) ΠΌΠΎΠ³ΡƒΡ‚ Π²Ρ‹ΡΠΎΡ…Π½ΡƒΡ‚ΡŒ, Π²Ρ‹ ΠΌΠΎΠΆΠ΅Ρ‚Π΅ ΠΏΡ€ΠΎΡ‡ΠΈΡ‚Π°Ρ‚ΡŒ Π½Π΅ΠΌΠ½ΠΎΠ³ΠΎ мСньшС Π½ΠΎΠΌΠΈΠ½Π°Π»Π°, Ρ‡Π΅ΠΌ Π΅ΠΌΠΊΠΎΡΡ‚ΡŒ Π½ΠΎΠΌΠΈΠ½Π°Π»Π°. Π­Ρ‚ΠΎ Ρ…ΠΎΡ€ΠΎΡˆΠΎ. Если ΠΎΠ½ Π½Π΅ΠΌΠ½ΠΎΠ³ΠΎ Π½ΠΈΠΆΠ΅, это всС Π΅Ρ‰Π΅ Ρ…ΠΎΡ€ΠΎΡˆΠΈΠΉ кондСнсатор. Однако, Ссли Π²Ρ‹ Ρ‡ΠΈΡ‚Π°Π΅Ρ‚Π΅ Π·Π½Π°Ρ‡ΠΈΡ‚Π΅Π»ΡŒΠ½ΠΎ Π±ΠΎΠ»Π΅Π΅ Π½ΠΈΠ·ΠΊΡƒΡŽ Π΅ΠΌΠΊΠΎΡΡ‚ΡŒ ΠΈΠ»ΠΈ Π²ΠΎΠΎΠ±Ρ‰Π΅ Π΅Π΅ Π½Π΅Ρ‚, это Π²Π΅Ρ€Π½Ρ‹ΠΉ ΠΏΡ€ΠΈΠ·Π½Π°ΠΊ Ρ‚ΠΎΠ³ΠΎ, Ρ‡Ρ‚ΠΎ кондСнсатор нСисправСн ΠΈ нуТдаСтся Π² Π·Π°ΠΌΠ΅Π½Π΅.

    ΠŸΡ€ΠΎΠ²Π΅Ρ€ΠΊΠ° Смкости кондСнсатора β€” ΠΎΡ‚Π»ΠΈΡ‡Π½Ρ‹ΠΉ способ ΠΎΠΏΡ€Π΅Π΄Π΅Π»ΠΈΡ‚ΡŒ, исправСн Π»ΠΈ кондСнсатор.

    ΠŸΡ€ΠΎΠ²Π΅Ρ€ΠΊΠ° кондСнсатора с ΠΏΠΎΠΌΠΎΡ‰ΡŒΡŽ Π²ΠΎΠ»ΡŒΡ‚ΠΌΠ΅Ρ‚Ρ€Π°

    Π•Ρ‰Π΅ ΠΎΠ΄ΠΈΠ½ тСст, ΠΊΠΎΡ‚ΠΎΡ€Ρ‹ΠΉ Π²Ρ‹ ΠΌΠΎΠΆΠ΅Ρ‚Π΅ ΡΠ΄Π΅Π»Π°Ρ‚ΡŒ, Ρ‡Ρ‚ΠΎΠ±Ρ‹ ΠΏΡ€ΠΎΠ²Π΅Ρ€ΠΈΡ‚ΡŒ, исправСн Π»ΠΈ кондСнсатор, β€” это тСст напряТСния.

    Π’ ΠΊΠΎΠ½Ρ†Π΅ ΠΊΠΎΠ½Ρ†ΠΎΠ², кондСнсаторы β€” это Π½Π°ΠΊΠΎΠΏΠΈΡ‚Π΅Π»ΠΈ. Они хранят Ρ€Π°Π·Π½ΠΎΡΡ‚ΡŒ ΠΏΠΎΡ‚Π΅Π½Ρ†ΠΈΠ°Π»ΠΎΠ² зарядов Π½Π° своСй пластинС, которая являСтся напряТСниСм. Анод ΠΈΠΌΠ΅Π΅Ρ‚ ΠΏΠΎΠ»ΠΎΠΆΠΈΡ‚Π΅Π»ΡŒΠ½ΠΎΠ΅ напряТСниС, Π° ΠΊΠ°Ρ‚ΠΎΠ΄ ΠΈΠΌΠ΅Π΅Ρ‚ ΠΎΡ‚Ρ€ΠΈΡ†Π°Ρ‚Π΅Π»ΡŒΠ½ΠΎΠ΅ напряТСниС.

    Π’Ρ‹ ΠΌΠΎΠΆΠ΅Ρ‚Π΅ ΠΏΡ€ΠΎΠ²Π΅Ρ€ΠΈΡ‚ΡŒ, Ρ€Π°Π±ΠΎΡ‚Π°Π΅Ρ‚ Π»ΠΈ кондСнсатор Π½ΠΎΡ€ΠΌΠ°Π»ΡŒΠ½ΠΎ, зарядив Π΅Π³ΠΎ напряТСниСм, Π° Π·Π°Ρ‚Π΅ΠΌ ΡΡ‡ΠΈΡ‚Π°Ρ‚ΡŒ напряТСниС Π½Π° ΠΊΠ»Π΅ΠΌΠΌΠ°Ρ….Если ΠΎΠ½ ΠΏΠΎΠΊΠ°Π·Ρ‹Π²Π°Π΅Ρ‚ напряТСниС, Π΄ΠΎ ΠΊΠΎΡ‚ΠΎΡ€ΠΎΠ³ΠΎ Π²Ρ‹ Π΅Π³ΠΎ зарядили, Ρ‚ΠΎ кондСнсатор выполняСт свою Ρ€Π°Π±ΠΎΡ‚Ρƒ ΠΈ ΠΌΠΎΠΆΠ΅Ρ‚ ΡƒΠ΄Π΅Ρ€ΠΆΠΈΠ²Π°Ρ‚ΡŒ напряТСниС Π½Π° своих ΠΊΠ»Π΅ΠΌΠΌΠ°Ρ…. Если ΠΎΠ½ Π½Π΅ заряТаСтся ΠΈ Π½Π΅ считываСт напряТСниС, это ΠΏΡ€ΠΈΠ·Π½Π°ΠΊ Ρ‚ΠΎΠ³ΠΎ, Ρ‡Ρ‚ΠΎ кондСнсатор нСисправСн.

    Π§Ρ‚ΠΎΠ±Ρ‹ Π·Π°Ρ€ΡΠ΄ΠΈΡ‚ΡŒ кондСнсатор напряТСниСм, ΠΏΠΎΠ΄Π°ΠΉΡ‚Π΅ напряТСниС постоянного Ρ‚ΠΎΠΊΠ° Π½Π° Π²Ρ‹Π²ΠΎΠ΄Ρ‹ кондСнсатора. Π’Π΅ΠΏΠ΅Ρ€ΡŒ ΠΏΠΎΠ»ΡΡ€Π½ΠΎΡΡ‚ΡŒ ΠΎΡ‡Π΅Π½ΡŒ Π²Π°ΠΆΠ½Π° для поляризованных кондСнсаторов (элСктролитичСских кондСнсаторов). Если Π²Ρ‹ ΠΈΠΌΠ΅Π΅Ρ‚Π΅ Π΄Π΅Π»ΠΎ с поляризованным кондСнсатором, Ρ‚ΠΎ Π½Π΅ΠΎΠ±Ρ…ΠΎΠ΄ΠΈΠΌΠΎ ΡΠΎΠ±Π»ΡŽΠ΄Π°Ρ‚ΡŒ ΠΏΠΎΠ»ΡΡ€Π½ΠΎΡΡ‚ΡŒ ΠΈ ΠΏΡ€Π°Π²ΠΈΠ»ΡŒΠ½ΠΎΠ΅ Π½Π°Π·Π½Π°Ρ‡Π΅Π½ΠΈΠ΅ Π²Ρ‹Π²ΠΎΠ΄ΠΎΠ².ΠŸΠΎΠ»ΠΎΠΆΠΈΡ‚Π΅Π»ΡŒΠ½ΠΎΠ΅ напряТСниС поступаСт Π½Π° Π°Π½ΠΎΠ΄ (Π±ΠΎΠ»Π΅Π΅ Π΄Π»ΠΈΠ½Π½Ρ‹ΠΉ Π²Ρ‹Π²ΠΎΠ΄) кондСнсатора, Π° ΠΎΡ‚Ρ€ΠΈΡ†Π°Ρ‚Π΅Π»ΡŒΠ½ΠΎΠ΅ ΠΈΠ»ΠΈ Π·Π°Π·Π΅ΠΌΠ»Π΅Π½ΠΈΠ΅ поступаСт Π½Π° ΠΊΠ°Ρ‚ΠΎΠ΄ (Π±ΠΎΠ»Π΅Π΅ ΠΊΠΎΡ€ΠΎΡ‚ΠΊΠΈΠΉ Π²Ρ‹Π²ΠΎΠ΄) кондСнсатора. ΠŸΠΎΠ΄Π°ΠΉΡ‚Π΅ Π½Π° нСсколько сСкунд напряТСниС мСньшС номинального напряТСния кондСнсатора. НапримСр, ΠΏΠΎΠ΄Π°ΠΉΡ‚Π΅ Π½Π° кондСнсатор 25 Π’ 9 Π²ΠΎΠ»ΡŒΡ‚ ΠΈ Π΄Π°ΠΉΡ‚Π΅ 9 Π²ΠΎΠ»ΡŒΡ‚Π°ΠΌ Π·Π°Ρ€ΡΠ΄ΠΈΡ‚ΡŒ Π΅Π³ΠΎ Π² Ρ‚Π΅Ρ‡Π΅Π½ΠΈΠ΅ Π½Π΅ΡΠΊΠΎΠ»ΡŒΠΊΠΈΡ… сСкунд. Пока Π²Ρ‹ Π½Π΅ ΠΈΡΠΏΠΎΠ»ΡŒΠ·ΡƒΠ΅Ρ‚Π΅ ΠΎΠ³Ρ€ΠΎΠΌΠ½Ρ‹ΠΉ-ΠΎΠ³Ρ€ΠΎΠΌΠ½Ρ‹ΠΉ кондСнсатор, ΠΎΠ½ Π±ΡƒΠ΄Π΅Ρ‚ Π·Π°Ρ€ΡΠΆΠ°Ρ‚ΡŒΡΡ Π·Π° ΠΎΡ‡Π΅Π½ΡŒ ΠΊΠΎΡ€ΠΎΡ‚ΠΊΠΈΠΉ ΠΏΡ€ΠΎΠΌΠ΅ΠΆΡƒΡ‚ΠΎΠΊ Π²Ρ€Π΅ΠΌΠ΅Π½ΠΈ, всСго Π·Π° нСсколько сСкунд. ПослС Π·Π°Π²Π΅Ρ€ΡˆΠ΅Π½ΠΈΡ зарядки отсоСдинитС кондСнсатор ΠΎΡ‚ источника напряТСния ΠΈ ΠΈΠ·ΠΌΠ΅Ρ€ΡŒΡ‚Π΅ Π΅Π³ΠΎ напряТСниС с ΠΏΠΎΠΌΠΎΡ‰ΡŒΡŽ ΠΌΡƒΠ»ΡŒΡ‚ΠΈΠΌΠ΅Ρ‚Ρ€Π°.Π‘Π½Π°Ρ‡Π°Π»Π° напряТСниС Π΄ΠΎΠ»ΠΆΠ½ΠΎ Π±Ρ‹Ρ‚ΡŒ ΠΎΠΊΠΎΠ»ΠΎ 9 Π²ΠΎΠ»ΡŒΡ‚ (ΠΈΠ»ΠΈ любого Π΄Ρ€ΡƒΠ³ΠΎΠ³ΠΎ напряТСния), ΠΊΠΎΡ‚ΠΎΡ€ΠΎΠ΅ Π²Ρ‹ Π½Π° Π½Π΅Π³ΠΎ ΠΏΠΎΠ΄Π°Π»ΠΈ. ΠžΠ±Ρ€Π°Ρ‚ΠΈΡ‚Π΅ Π²Π½ΠΈΠΌΠ°Π½ΠΈΠ΅, Ρ‡Ρ‚ΠΎ напряТСниС Π±ΡƒΠ΄Π΅Ρ‚ быстро Ρ€Π°Π·Ρ€ΡΠΆΠ°Ρ‚ΡŒΡΡ ΠΈ ΡƒΠΏΠ°Π΄Π΅Ρ‚ Π΄ΠΎ 0 Π’, ΠΏΠΎΡ‚ΠΎΠΌΡƒ Ρ‡Ρ‚ΠΎ кондСнсатор разряТаСт своС напряТСниС Ρ‡Π΅Ρ€Π΅Π· ΠΌΡƒΠ»ΡŒΡ‚ΠΈΠΌΠ΅Ρ‚Ρ€. Π’Π΅ΠΌ Π½Π΅ ΠΌΠ΅Π½Π΅Π΅, Π²Ρ‹ Π΄ΠΎΠ»ΠΆΠ½Ρ‹ сначала ΡΡ‡ΠΈΡ‚Π°Ρ‚ΡŒ Π·Π½Π°Ρ‡Π΅Π½ΠΈΠ΅ зарядного напряТСния, ΠΏΡ€Π΅ΠΆΠ΄Π΅ Ρ‡Π΅ΠΌ ΠΎΠ½ΠΎ быстро ΡƒΠΏΠ°Π΄Π΅Ρ‚. Π­Ρ‚ΠΎ ΠΏΠΎΠ²Π΅Π΄Π΅Π½ΠΈΠ΅ Π·Π΄ΠΎΡ€ΠΎΠ²ΠΎΠ³ΠΎ ΠΈ Ρ…ΠΎΡ€ΠΎΡˆΠ΅Π³ΠΎ кондСнсатора. Если ΠΎΠ½ Π½Π΅ сохраняСт напряТСниС, ΠΎΠ½ нСисправСн ΠΈ ΠΏΠΎΠ΄Π»Π΅ΠΆΠΈΡ‚ Π·Π°ΠΌΠ΅Π½Π΅.

    Π˜Ρ‚Π°ΠΊ, Ρƒ вас Π΅ΡΡ‚ΡŒ 3 ΡΠ΅Ρ€ΡŒΠ΅Π·Π½Ρ‹Ρ… тСста, ΠΊΠΎΡ‚ΠΎΡ€Ρ‹Π΅ Π²Ρ‹ ΠΌΠΎΠΆΠ΅Ρ‚Π΅ Π²Ρ‹ΠΏΠΎΠ»Π½ΠΈΡ‚ΡŒ (всС ΠΈΠ»ΠΈ Π»ΠΈΠ±ΠΎ/ΠΈΠ»ΠΈ), Ρ‡Ρ‚ΠΎΠ±Ρ‹ ΠΏΡ€ΠΎΠ²Π΅Ρ€ΠΈΡ‚ΡŒ, исправСн кондСнсатор ΠΈΠ»ΠΈ Π½Π΅Ρ‚.

    ΠœΡƒΠ»ΡŒΡ‚ΠΈΠΌΠ΅Ρ‚Ρ€ опрСдСляСт Π΅ΠΌΠΊΠΎΡΡ‚ΡŒ, заряТая кондСнсатор извСстным Ρ‚ΠΎΠΊΠΎΠΌ, измСряя ΠΏΠΎΠ»ΡƒΡ‡Π΅Π½Π½ΠΎΠ΅ напряТСниС ΠΈ Π·Π°Ρ‚Π΅ΠΌ вычисляя Π΅ΠΌΠΊΠΎΡΡ‚ΡŒ.

    ΠŸΡ€Π΅Π΄ΡƒΠΏΡ€Π΅ΠΆΠ΄Π΅Π½ΠΈΠ΅: Π₯ΠΎΡ€ΠΎΡˆΠΈΠΉ кондСнсатор Π½Π°ΠΊΠ°ΠΏΠ»ΠΈΠ²Π°Π΅Ρ‚ элСктричСский заряд ΠΈ ΠΌΠΎΠΆΠ΅Ρ‚ ΠΎΡΡ‚Π°Π²Π°Ρ‚ΡŒΡΡ ΠΏΠΎΠ΄ напряТСниСм послС ΠΎΡ‚ΠΊΠ»ΡŽΡ‡Π΅Π½ΠΈΡ питания. ΠŸΡ€Π΅ΠΆΠ΄Π΅ Ρ‡Π΅ΠΌ ΠΏΡ€ΠΈΠΊΠ°ΡΠ°Ρ‚ΡŒΡΡ ΠΊ Π½Π΅ΠΌΡƒ ΠΈΠ»ΠΈ ΠΏΡ€ΠΎΠ²ΠΎΠ΄ΠΈΡ‚ΡŒ измСрСния, Π°) Π²Ρ‹ΠΊΠ»ΡŽΡ‡ΠΈΡ‚Π΅ всС ΠΏΠΈΡ‚Π°Π½ΠΈΠ΅, Π±) ΠΈΡΠΏΠΎΠ»ΡŒΠ·ΡƒΠΉΡ‚Π΅ ΠΌΡƒΠ»ΡŒΡ‚ΠΈΠΌΠ΅Ρ‚Ρ€, Ρ‡Ρ‚ΠΎΠ±Ρ‹ ΡƒΠ±Π΅Π΄ΠΈΡ‚ΡŒΡΡ, Ρ‡Ρ‚ΠΎ ΠΏΠΈΡ‚Π°Π½ΠΈΠ΅ ΠΎΡ‚ΠΊΠ»ΡŽΡ‡Π΅Π½ΠΎ, ΠΈ Π²) остороТно разрядитС кондСнсатор, ΠΏΠΎΠ΄ΠΊΠ»ΡŽΡ‡ΠΈΠ² рСзистор ΠΊ ΠΏΡ€ΠΎΠ²ΠΎΠ΄Π°ΠΌ (ΠΊΠ°ΠΊ ΡƒΠΊΠ°Π·Π°Π½ΠΎ Π² ΡΠ»Π΅Π΄ΡƒΡŽΡ‰Π΅ΠΌ Π°Π±Π·Π°Ρ†Π΅).ΠžΠ±ΡΠ·Π°Ρ‚Π΅Π»ΡŒΠ½ΠΎ ΠΈΡΠΏΠΎΠ»ΡŒΠ·ΡƒΠΉΡ‚Π΅ ΡΠΎΠΎΡ‚Π²Π΅Ρ‚ΡΡ‚Π²ΡƒΡŽΡ‰ΠΈΠ΅ срСдства ΠΈΠ½Π΄ΠΈΠ²ΠΈΠ΄ΡƒΠ°Π»ΡŒΠ½ΠΎΠΉ Π·Π°Ρ‰ΠΈΡ‚Ρ‹.

    Для бСзопасного разряда кондСнсатора: ПослС ΠΎΡ‚ΠΊΠ»ΡŽΡ‡Π΅Π½ΠΈΡ питания ΠΏΠΎΠ΄ΠΊΠ»ΡŽΡ‡ΠΈΡ‚Π΅ рСзистор 20 000 Ом, 5 Π’Ρ‚ ΠΊ ΠΊΠ»Π΅ΠΌΠΌΠ°ΠΌ кондСнсатора Π½Π° ΠΏΡΡ‚ΡŒ сСкунд. Π‘ ΠΏΠΎΠΌΠΎΡ‰ΡŒΡŽ ΠΌΡƒΠ»ΡŒΡ‚ΠΈΠΌΠ΅Ρ‚Ρ€Π° ΡƒΠ±Π΅Π΄ΠΈΡ‚Π΅ΡΡŒ, Ρ‡Ρ‚ΠΎ кондСнсатор ΠΏΠΎΠ»Π½ΠΎΡΡ‚ΡŒΡŽ разряТСн.

    1. Π‘ ΠΏΠΎΠΌΠΎΡ‰ΡŒΡŽ Ρ†ΠΈΡ„Ρ€ΠΎΠ²ΠΎΠ³ΠΎ ΠΌΡƒΠ»ΡŒΡ‚ΠΈΠΌΠ΅Ρ‚Ρ€Π° (DMM) ΡƒΠ±Π΅Π΄ΠΈΡ‚Π΅ΡΡŒ, Ρ‡Ρ‚ΠΎ ΠΏΠΈΡ‚Π°Π½ΠΈΠ΅ Ρ†Π΅ΠΏΠΈ ΠΎΡ‚ΠΊΠ»ΡŽΡ‡Π΅Π½ΠΎ. Если кондСнсатор ΠΈΡΠΏΠΎΠ»ΡŒΠ·ΡƒΠ΅Ρ‚ΡΡ Π² Ρ†Π΅ΠΏΠΈ ΠΏΠ΅Ρ€Π΅ΠΌΠ΅Π½Π½ΠΎΠ³ΠΎ Ρ‚ΠΎΠΊΠ°, установитС ΠΌΡƒΠ»ΡŒΡ‚ΠΈΠΌΠ΅Ρ‚Ρ€ Π½Π° ΠΈΠ·ΠΌΠ΅Ρ€Π΅Π½ΠΈΠ΅ напряТСния ΠΏΠ΅Ρ€Π΅ΠΌΠ΅Π½Π½ΠΎΠ³ΠΎ Ρ‚ΠΎΠΊΠ°. Если ΠΎΠ½ ΠΈΡΠΏΠΎΠ»ΡŒΠ·ΡƒΠ΅Ρ‚ΡΡ Π² Ρ†Π΅ΠΏΠΈ постоянного Ρ‚ΠΎΠΊΠ°, настройтС Ρ†ΠΈΡ„Ρ€ΠΎΠ²ΠΎΠΉ ΠΌΡƒΠ»ΡŒΡ‚ΠΈΠΌΠ΅Ρ‚Ρ€ Π½Π° ΠΈΠ·ΠΌΠ΅Ρ€Π΅Π½ΠΈΠ΅ напряТСния постоянного Ρ‚ΠΎΠΊΠ°.
    2. ΠžΡΠΌΠΎΡ‚Ρ€ΠΈΡ‚Π΅ кондСнсатор. Если ΡƒΡ‚Π΅Ρ‡ΠΊΠΈ, Ρ‚Ρ€Π΅Ρ‰ΠΈΠ½Ρ‹, выпуклости ΠΈΠ»ΠΈ Π΄Ρ€ΡƒΠ³ΠΈΠ΅ ΠΏΡ€ΠΈΠ·Π½Π°ΠΊΠΈ износа ΠΎΡ‡Π΅Π²ΠΈΠ΄Π½Ρ‹, Π·Π°ΠΌΠ΅Π½ΠΈΡ‚Π΅ кондСнсатор.
    3. ΠŸΠΎΠ²Π΅Ρ€Π½ΠΈΡ‚Π΅ Ρ†ΠΈΡ„Π΅Ρ€Π±Π»Π°Ρ‚ Π² Ρ€Π΅ΠΆΠΈΠΌ измСрСния Смкости. Π‘ΠΈΠΌΠ²ΠΎΠ» часто Π΄Π΅Π»ΠΈΡ‚ мСсто Π½Π° Ρ†ΠΈΡ„Π΅Ρ€Π±Π»Π°Ρ‚Π΅ с Π΄Ρ€ΡƒΠ³ΠΎΠΉ Ρ„ΡƒΠ½ΠΊΡ†ΠΈΠ΅ΠΉ. Π’ Π΄ΠΎΠΏΠΎΠ»Π½Π΅Π½ΠΈΠ΅ ΠΊ Ρ€Π΅Π³ΡƒΠ»ΠΈΡ€ΠΎΠ²ΠΊΠ΅ диска ΠΎΠ±Ρ‹Ρ‡Π½ΠΎ Π½Π΅ΠΎΠ±Ρ…ΠΎΠ΄ΠΈΠΌΠΎ Π½Π°ΠΆΠ°Ρ‚ΡŒ Ρ„ΡƒΠ½ΠΊΡ†ΠΈΠΎΠ½Π°Π»ΡŒΠ½ΡƒΡŽ ΠΊΠ½ΠΎΠΏΠΊΡƒ, Ρ‡Ρ‚ΠΎΠ±Ρ‹ Π°ΠΊΡ‚ΠΈΠ²ΠΈΡ€ΠΎΠ²Π°Ρ‚ΡŒ ΠΈΠ·ΠΌΠ΅Ρ€Π΅Π½ΠΈΠ΅. Π˜Π½ΡΡ‚Ρ€ΡƒΠΊΡ†ΠΈΠΈ см. Π² руководствС ΠΏΠΎΠ»ΡŒΠ·ΠΎΠ²Π°Ρ‚Π΅Π»Ρ ΠΌΡƒΠ»ΡŒΡ‚ΠΈΠΌΠ΅Ρ‚Ρ€Π°.

    4. Для ΠΏΡ€Π°Π²ΠΈΠ»ΡŒΠ½ΠΎΠ³ΠΎ измСрСния кондСнсатор Π½Π΅ΠΎΠ±Ρ…ΠΎΠ΄ΠΈΠΌΠΎ ΡƒΠ΄Π°Π»ΠΈΡ‚ΡŒ ΠΈΠ· Ρ†Π΅ΠΏΠΈ.РазрядитС кондСнсатор, ΠΊΠ°ΠΊ описано Π² ΠΏΡ€Π΅Π΄ΡƒΠΏΡ€Π΅ΠΆΠ΄Π΅Π½ΠΈΠΈ Π²Ρ‹ΡˆΠ΅.

    ΠŸΡ€ΠΈΠΌΠ΅Ρ‡Π°Π½ΠΈΠ΅: НСкоторыС ΠΌΡƒΠ»ΡŒΡ‚ΠΈΠΌΠ΅Ρ‚Ρ€Ρ‹ ΠΏΡ€Π΅Π΄Π»Π°Π³Π°ΡŽΡ‚ ΠΎΡ‚Π½ΠΎΡΠΈΡ‚Π΅Π»ΡŒΠ½Ρ‹ΠΉ (REL) Ρ€Π΅ΠΆΠΈΠΌ. ΠŸΡ€ΠΈ ΠΈΠ·ΠΌΠ΅Ρ€Π΅Π½ΠΈΠΈ ΠΌΠ°Π»Ρ‹Ρ… Π·Π½Π°Ρ‡Π΅Π½ΠΈΠΉ Смкости ΠΌΠΎΠΆΠ½ΠΎ ΠΈΡΠΏΠΎΠ»ΡŒΠ·ΠΎΠ²Π°Ρ‚ΡŒ ΠΎΡ‚Π½ΠΎΡΠΈΡ‚Π΅Π»ΡŒΠ½Ρ‹ΠΉ Ρ€Π΅ΠΆΠΈΠΌ для удалСния Смкости ΠΈΠ·ΠΌΠ΅Ρ€ΠΈΡ‚Π΅Π»ΡŒΠ½Ρ‹Ρ… ΠΏΡ€ΠΎΠ²ΠΎΠ΄ΠΎΠ². Π§Ρ‚ΠΎΠ±Ρ‹ пСрСвСсти ΠΌΡƒΠ»ΡŒΡ‚ΠΈΠΌΠ΅Ρ‚Ρ€ Π² ΠΎΡ‚Π½ΠΎΡΠΈΡ‚Π΅Π»ΡŒΠ½Ρ‹ΠΉ Ρ€Π΅ΠΆΠΈΠΌ измСрСния Смкости, ΠΎΡΡ‚Π°Π²ΡŒΡ‚Π΅ ΠΈΠ·ΠΌΠ΅Ρ€ΠΈΡ‚Π΅Π»ΡŒΠ½Ρ‹Π΅ ΠΏΡ€ΠΎΠ²ΠΎΠ΄Π° ΠΎΡ‚ΠΊΡ€Ρ‹Ρ‚Ρ‹ΠΌΠΈ ΠΈ Π½Π°ΠΆΠΌΠΈΡ‚Π΅ ΠΊΠ½ΠΎΠΏΠΊΡƒ REL. Π­Ρ‚ΠΎ удаляСт Π·Π½Π°Ρ‡Π΅Π½ΠΈΠ΅ остаточной Смкости ΠΈΠ·ΠΌΠ΅Ρ€ΠΈΡ‚Π΅Π»ΡŒΠ½Ρ‹Ρ… ΠΏΡ€ΠΎΠ²ΠΎΠ΄ΠΎΠ².

  • ΠŸΠΎΠ΄ΡΠΎΠ΅Π΄ΠΈΠ½ΠΈΡ‚Π΅ ΠΈΠ·ΠΌΠ΅Ρ€ΠΈΡ‚Π΅Π»ΡŒΠ½Ρ‹Π΅ ΠΏΡ€ΠΎΠ²ΠΎΠ΄Π° ΠΊ ΠΊΠ»Π΅ΠΌΠΌΠ°ΠΌ кондСнсатора. Π”Π΅Ρ€ΠΆΠΈΡ‚Π΅ ΠΈΠ·ΠΌΠ΅Ρ€ΠΈΡ‚Π΅Π»ΡŒΠ½Ρ‹Π΅ ΠΏΡ€ΠΎΠ²ΠΎΠ΄Π° ΠΏΠΎΠ΄ΠΊΠ»ΡŽΡ‡Π΅Π½Π½Ρ‹ΠΌΠΈ Π² Ρ‚Π΅Ρ‡Π΅Π½ΠΈΠ΅ Π½Π΅ΡΠΊΠΎΠ»ΡŒΠΊΠΈΡ… сСкунд, Ρ‡Ρ‚ΠΎΠ±Ρ‹ ΠΌΡƒΠ»ΡŒΡ‚ΠΈΠΌΠ΅Ρ‚Ρ€ автоматичСски Π²Ρ‹Π±Ρ€Π°Π» Π½ΡƒΠΆΠ½Ρ‹ΠΉ Π΄ΠΈΠ°ΠΏΠ°Π·ΠΎΠ½.
  • ΠŸΡ€ΠΎΡ‡ΠΈΡ‚Π°ΠΉΡ‚Π΅ ΠΎΡ‚ΠΎΠ±Ρ€Π°ΠΆΠ°Π΅ΠΌΠΎΠ΅ ΠΈΠ·ΠΌΠ΅Ρ€Π΅Π½ΠΈΠ΅. Если Π·Π½Π°Ρ‡Π΅Π½ΠΈΠ΅ Смкости находится Π² ΠΏΡ€Π΅Π΄Π΅Π»Π°Ρ… Π΄ΠΈΠ°ΠΏΠ°Π·ΠΎΠ½Π° измСрСния, ΠΌΡƒΠ»ΡŒΡ‚ΠΈΠΌΠ΅Ρ‚Ρ€ ΠΎΡ‚ΠΎΠ±Ρ€Π°Π·ΠΈΡ‚ Π·Π½Π°Ρ‡Π΅Π½ΠΈΠ΅ Смкости. Он Π±ΡƒΠ΄Π΅Ρ‚ ΠΎΡ‚ΠΎΠ±Ρ€Π°ΠΆΠ°Ρ‚ΡŒ OL, Ссли Π°) Π·Π½Π°Ρ‡Π΅Π½ΠΈΠ΅ Смкости Π²Ρ‹ΡˆΠ΅ Π΄ΠΈΠ°ΠΏΠ°Π·ΠΎΠ½Π° измСрСния ΠΈΠ»ΠΈ Π±) кондСнсатор нСисправСн.
  • ΠžΠ±Π·ΠΎΡ€ измСрСния Смкости

    Поиск ΠΈ устранСниС нСисправностСй ΠΎΠ΄Π½ΠΎΡ„Π°Π·Π½Ρ‹Ρ… Π΄Π²ΠΈΠ³Π°Ρ‚Π΅Π»Π΅ΠΉ β€” ΠΎΠ΄Π½ΠΎ ΠΈΠ· Π½Π°ΠΈΠ±ΠΎΠ»Π΅Π΅ ΠΏΡ€Π°ΠΊΡ‚ΠΈΡ‡Π½Ρ‹Ρ… ΠΏΡ€ΠΈΠΌΠ΅Π½Π΅Π½ΠΈΠΉ Ρ„ΡƒΠ½ΠΊΡ†ΠΈΠΈ измСрСния Смкости Ρ†ΠΈΡ„Ρ€ΠΎΠ²ΠΎΠ³ΠΎ ΠΌΡƒΠ»ΡŒΡ‚ΠΈΠΌΠ΅Ρ‚Ρ€Π°.

    ΠžΠ΄Π½ΠΎΡ„Π°Π·Π½Ρ‹ΠΉ Π΄Π²ΠΈΠ³Π°Ρ‚Π΅Π»ΡŒ с кондСнсаторным пуском, ΠΊΠΎΡ‚ΠΎΡ€Ρ‹ΠΉ Π½Π΅ запускаСтся, являСтся ΠΏΡ€ΠΈΠ·Π½Π°ΠΊΠΎΠΌ нСисправного кондСнсатора.Π’Π°ΠΊΠΈΠ΅ Π΄Π²ΠΈΠ³Π°Ρ‚Π΅Π»ΠΈ Π±ΡƒΠ΄ΡƒΡ‚ ΠΏΡ€ΠΎΠ΄ΠΎΠ»ΠΆΠ°Ρ‚ΡŒ Ρ€Π°Π±ΠΎΡ‚Π°Ρ‚ΡŒ послС запуска, Ρ‡Ρ‚ΠΎ услоТняСт поиск ΠΈ устранСниС нСисправностСй. Π’Ρ‹Ρ…ΠΎΠ΄ ΠΈΠ· строя кондСнсатора ТСсткого пуска компрСссоров HVAC являСтся Ρ…ΠΎΡ€ΠΎΡˆΠΈΠΌ ΠΏΡ€ΠΈΠΌΠ΅Ρ€ΠΎΠΌ этой ΠΏΡ€ΠΎΠ±Π»Π΅ΠΌΡ‹. Π”Π²ΠΈΠ³Π°Ρ‚Π΅Π»ΡŒ компрСссора ΠΌΠΎΠΆΠ΅Ρ‚ Π·Π°ΠΏΡƒΡΡ‚ΠΈΡ‚ΡŒΡΡ, Π½ΠΎ вскорС пСрСгрССтся, Ρ‡Ρ‚ΠΎ ΠΏΡ€ΠΈΠ²Π΅Π΄Π΅Ρ‚ ΠΊ ΠΎΡ‚ΠΊΠ»ΡŽΡ‡Π΅Π½ΠΈΡŽ Π²Ρ‹ΠΊΠ»ΡŽΡ‡Π°Ρ‚Π΅Π»Ρ.

    ΠžΠ΄Π½ΠΎΡ„Π°Π·Π½Ρ‹Π΅ Π΄Π²ΠΈΠ³Π°Ρ‚Π΅Π»ΠΈ с Ρ‚Π°ΠΊΠΈΠΌΠΈ ΠΏΡ€ΠΎΠ±Π»Π΅ΠΌΠ°ΠΌΠΈ ΠΈ ΡˆΡƒΠΌΠ½Ρ‹Π΅ ΠΎΠ΄Π½ΠΎΡ„Π°Π·Π½Ρ‹Π΅ Π΄Π²ΠΈΠ³Π°Ρ‚Π΅Π»ΠΈ с кондСнсаторами Ρ‚Ρ€Π΅Π±ΡƒΡŽΡ‚ ΠΌΡƒΠ»ΡŒΡ‚ΠΈΠΌΠ΅Ρ‚Ρ€Π° для ΠΏΡ€ΠΎΠ²Π΅Ρ€ΠΊΠΈ исправности кондСнсаторов. ΠŸΠΎΡ‡Ρ‚ΠΈ всС кондСнсаторы Π΄Π²ΠΈΠ³Π°Ρ‚Π΅Π»Π΅ΠΉ ΠΈΠΌΠ΅ΡŽΡ‚ Π½ΠΎΠΌΠΈΠ½Π°Π» Π² ΠΌΠΈΠΊΡ€ΠΎΡ„Π°Ρ€Π°Π΄Π°Ρ…, ΡƒΠΊΠ°Π·Π°Π½Π½Ρ‹ΠΉ Π½Π° кондСнсаторС.

    Π’Ρ€Π΅Ρ…Ρ„Π°Π·Π½Ρ‹Π΅ кондСнсаторы для ΠΊΠΎΡ€Ρ€Π΅ΠΊΡ†ΠΈΠΈ коэффициСнта мощности ΠΎΠ±Ρ‹Ρ‡Π½ΠΎ Π·Π°Ρ‰ΠΈΡ‰Π΅Π½Ρ‹ прСдохранитСлями.Если ΠΎΠ΄ΠΈΠ½ ΠΈΠ»ΠΈ нСсколько ΠΈΠ· этих кондСнсаторов Π²Ρ‹ΠΉΠ΄ΡƒΡ‚ ΠΈΠ· строя, это ΠΏΡ€ΠΈΠ²Π΅Π΄Π΅Ρ‚ ΠΊ нСэффСктивности систСмы, скорСС всСго, увСличатся счСта Π·Π° ΠΊΠΎΠΌΠΌΡƒΠ½Π°Π»ΡŒΠ½Ρ‹Π΅ услуги ΠΈ ΠΌΠΎΠ³ΡƒΡ‚ ΠΏΡ€ΠΎΠΈΠ·ΠΎΠΉΡ‚ΠΈ Π½Π΅ΠΏΡ€Π΅Π΄Π½Π°ΠΌΠ΅Ρ€Π΅Π½Π½Ρ‹Π΅ ΠΎΡ‚ΠΊΠ»ΡŽΡ‡Π΅Π½ΠΈΡ оборудования. Π’ случаС пСрСгорания прСдохранитСля кондСнсатора Π½Π΅ΠΎΠ±Ρ…ΠΎΠ΄ΠΈΠΌΠΎ ΠΈΠ·ΠΌΠ΅Ρ€ΠΈΡ‚ΡŒ ΠΏΡ€Π΅Π΄ΠΏΠΎΠ»Π°Π³Π°Π΅ΠΌΠΎΠ΅ Π·Π½Π°Ρ‡Π΅Π½ΠΈΠ΅ нСисправного кондСнсатора Π² ΠΌΠΈΠΊΡ€ΠΎΡ„Π°Ρ€Π°Π΄Π°Ρ… ΠΈ ΡƒΠ±Π΅Π΄ΠΈΡ‚ΡŒΡΡ, Ρ‡Ρ‚ΠΎ ΠΎΠ½ΠΎ находится Π² ΠΏΡ€Π΅Π΄Π΅Π»Π°Ρ… Π΄ΠΈΠ°ΠΏΠ°Π·ΠΎΠ½Π°, ΡƒΠΊΠ°Π·Π°Π½Π½ΠΎΠ³ΠΎ Π½Π° кондСнсаторС.

    Π‘Ρ‚ΠΎΠΈΡ‚ Π·Π½Π°Ρ‚ΡŒ Π½Π΅ΠΊΠΎΡ‚ΠΎΡ€Ρ‹Π΅ Π΄ΠΎΠΏΠΎΠ»Π½ΠΈΡ‚Π΅Π»ΡŒΠ½Ρ‹Π΅ Ρ„Π°ΠΊΡ‚ΠΎΡ€Ρ‹, связанныС с Π΅ΠΌΠΊΠΎΡΡ‚ΡŒΡŽ:

    • ΠšΠΎΠ½Π΄Π΅Π½ΡΠ°Ρ‚ΠΎΡ€Ρ‹ ΠΈΠΌΠ΅ΡŽΡ‚ ΠΎΠ³Ρ€Π°Π½ΠΈΡ‡Π΅Π½Π½Ρ‹ΠΉ срок слуТбы ΠΈ часто ΡΠ²Π»ΡΡŽΡ‚ΡΡ ΠΏΡ€ΠΈΡ‡ΠΈΠ½ΠΎΠΉ нСисправности.
    • НСисправныС кондСнсаторы ΠΌΠΎΠ³ΡƒΡ‚ ΠΈΠΌΠ΅Ρ‚ΡŒ ΠΊΠΎΡ€ΠΎΡ‚ΠΊΠΎΠ΅ Π·Π°ΠΌΡ‹ΠΊΠ°Π½ΠΈΠ΅, ΠΎΠ±Ρ€Ρ‹Π² Ρ†Π΅ΠΏΠΈ ΠΈΠ»ΠΈ ΠΌΠΎΠ³ΡƒΡ‚ физичСски ΠΈΡΠΏΠΎΡ€Ρ‚ΠΈΡ‚ΡŒΡΡ Π΄ΠΎ Ρ‚ΠΎΡ‡ΠΊΠΈ ΠΎΡ‚ΠΊΠ°Π·Π°.
    • ΠŸΡ€ΠΈ ΠΊΠΎΡ€ΠΎΡ‚ΠΊΠΎΠΌ Π·Π°ΠΌΡ‹ΠΊΠ°Π½ΠΈΠΈ кондСнсатора ΠΌΠΎΠΆΠ΅Ρ‚ ΠΏΠ΅Ρ€Π΅Π³ΠΎΡ€Π΅Ρ‚ΡŒ ΠΏΡ€Π΅Π΄ΠΎΡ…Ρ€Π°Π½ΠΈΡ‚Π΅Π»ΡŒ ΠΈΠ»ΠΈ ΠΏΠΎΠ²Ρ€Π΅Π΄ΠΈΡ‚ΡŒΡΡ Π΄Ρ€ΡƒΠ³ΠΈΠ΅ ΠΊΠΎΠΌΠΏΠΎΠ½Π΅Π½Ρ‚Ρ‹.
    • Когда кондСнсатор открываСтся ΠΈΠ»ΠΈ ΠΈΠ·Π½Π°ΡˆΠΈΠ²Π°Π΅Ρ‚ΡΡ, Ρ†Π΅ΠΏΡŒ ΠΈΠ»ΠΈ Π΅Π΅ ΠΊΠΎΠΌΠΏΠΎΠ½Π΅Π½Ρ‚Ρ‹ ΠΌΠΎΠ³ΡƒΡ‚ Π½Π΅ Ρ€Π°Π±ΠΎΡ‚Π°Ρ‚ΡŒ.
    • Износ Ρ‚Π°ΠΊΠΆΠ΅ ΠΌΠΎΠΆΠ΅Ρ‚ ΠΈΠ·ΠΌΠ΅Π½ΠΈΡ‚ΡŒ Π·Π½Π°Ρ‡Π΅Π½ΠΈΠ΅ Смкости кондСнсатора, Ρ‡Ρ‚ΠΎ ΠΌΠΎΠΆΠ΅Ρ‚ Π²Ρ‹Π·Π²Π°Ρ‚ΡŒ ΠΏΡ€ΠΎΠ±Π»Π΅ΠΌΡ‹.

    МнС Π½ΡƒΠΆΠ½ΠΎ ΠΈΠ·ΠΌΠ΅Ρ€ΠΈΡ‚ΡŒ ΠΈΠ»ΠΈ ΠΈΠ·ΠΌΠ΅Ρ€ΠΈΡ‚ΡŒ Π΅ΠΌΠΊΠΎΡΡ‚ΡŒ Π² Π΄ΠΈΠ°ΠΏΠ°Π·ΠΎΠ½Π΅ ΠΎΡ‚ 0 ΠΏΠ€ Π΄ΠΎ 5 ΠΏΠ€ с Ρ‚ΠΎΡ‡Π½ΠΎΡΡ‚ΡŒΡŽ 0,1 ΠΏΠ€ ΠΈΠ»ΠΈ Π²Ρ‹ΡˆΠ΅. Π― знаю, Ρ‡Ρ‚ΠΎ Ρ‡ΠΈΠΏΡ‹ прСобразования Смкости Π² Ρ†ΠΈΡ„Ρ€ΠΎΠ²ΠΎΠ΅ ΠΏΡ€Π΅ΠΎΠ±Ρ€Π°Π·ΠΎΠ²Π°Π½ΠΈΠ΅ ΡƒΡ‚Π²Π΅Ρ€ΠΆΠ΄Π°ΡŽΡ‚, Ρ‡Ρ‚ΠΎ Π΄Π΅Π»Π°ΡŽΡ‚ это, Π½ΠΎ Π΅ΡΡ‚ΡŒ Π»ΠΈ Π±ΠΎΠ»Π΅Π΅ простой / Π»Π΅Π³ΠΊΠΈΠΉ способ ΠΌΠ°ΠΊΠ΅Ρ‚ΠΈΡ€ΠΎΠ²Π°Ρ‚ΡŒ схСму для измСрСния этих свСрхнизких СмкостСй?

    Π£ мСня Π΅ΡΡ‚ΡŒ нСбольшой концСнтричСский Ρ†ΠΈΠ»ΠΈΠ½Π΄Ρ€, Π² ΠΊΠΎΡ‚ΠΎΡ€ΠΎΠΌ внСшний Ρ†ΠΈΠ»ΠΈΠ½Π΄Ρ€ физичСски зафиксирован (ΠΈ связан с Π½Π΅ΠΊΠΎΡ‚ΠΎΡ€Ρ‹ΠΌ ΠΏΠΎΡ‚Π΅Π½Ρ†ΠΈΠ°Π»ΠΎΠΌ), Π° Π²Π½ΡƒΡ‚Ρ€Π΅Π½Π½ΠΈΠΉ двиТСтся Π²Π½ΡƒΡ‚Ρ€ΡŒ ΠΈ Π½Π°Ρ€ΡƒΠΆΡƒ.Π­Ρ‚ΠΎ Π΄Π²ΠΈΠΆΠ΅Π½ΠΈΠ΅ ΠΌΠ½Π΅ Π½ΡƒΠΆΠ½ΠΎ ΠΎΡ‚ΡΠ»Π΅ΠΆΠΈΠ²Π°Ρ‚ΡŒ. ИзмСнСниС ΠΏΠ»ΠΎΡ‰Π°Π΄ΠΈ пСрСкрытия ΠΌΠ΅ΠΆΠ΄Ρƒ двумя Ρ†ΠΈΠ»ΠΈΠ½Π΄Ρ€Π°ΠΌΠΈ ΠΏΡ€ΠΈΠ²ΠΎΠ΄ΠΈΡ‚ ΠΊ измСнСнию Смкости. Π’Π°ΠΊΠΈΠΌ ΠΎΠ±Ρ€Π°Π·ΠΎΠΌ, я постоянно ΠΎΡ‚ΡΠ»Π΅ΠΆΠΈΠ²Π°ΡŽ Π΅Π³ΠΎ ΠΏΠΎΠ»ΠΎΠΆΠ΅Π½ΠΈΠ΅, отслСТивая ΠΈΠ·ΠΌΠ΅Π½Π΅Π½ΠΈΠ΅ Смкости.

    1 ΠžΡ‚Π²Π΅Ρ‚ 1

    Π”ΠΎΠ²ΠΎΠ»ΡŒΠ½ΠΎ просто ΠΎΠ±Π½Π°Ρ€ΡƒΠΆΠΈΡ‚ΡŒ ΠΈΠ·ΠΌΠ΅Π½Π΅Π½ΠΈΠ΅ Смкости Π½Π° 0,1 ΠΏΠ€ Π² качСствС ΡΠΎΠΎΡ‚Π½ΠΎΡˆΠ΅Π½ΠΈΡ. Π’ΠΎΠ·ΠΌΠΎΠΆΠ½ΠΎ, самым простым являСтся созданиС рСлаксационного Π³Π΅Π½Π΅Ρ€Π°Ρ‚ΠΎΡ€Π° ΠΈ Ρ†ΠΈΡ„Ρ€ΠΎΠ²ΠΎΠ΅ ΠΈΠ·ΠΌΠ΅Ρ€Π΅Π½ΠΈΠ΅ частоты ΠΈ измСнСния частоты ΠΏΡ€ΠΈ ΠΏΠΎΠ΄ΠΊΠ»ΡŽΡ‡Π΅Π½ΠΈΠΈ тСстового кондСнсатора.

    ΠžΡ‡Π΅Π½ΡŒ Ρ‚Ρ€ΡƒΠ΄Π½ΠΎ Ρ‚ΠΎΡ‡Π½ΠΎ ΡƒΠ·Π½Π°Ρ‚ΡŒ, сколько эффСктивной Смкости имССтся Π² ΠΎΡΡ‚Π°Π»ΡŒΠ½ΠΎΠΉ части Ρ†Π΅ΠΏΠΈ, Π° Ρ‚Π°ΠΊΠΆΠ΅ Π² Π»ΡŽΠ±Ρ‹Ρ… ΡΠΎΠ΅Π΄ΠΈΠ½ΠΈΡ‚Π΅Π»ΡŒΠ½Ρ‹Ρ… приспособлСниях, ΠΎΡ‚Π²ΠΎΠ΄Π°Ρ…, ΠΊΠ»Π΅ΠΌΠΌΠ°Ρ…, ΠΏΡ€ΠΎΠ²ΠΎΠ΄Π°Ρ…, ΠΎΡ‚Π½ΠΎΡΠΈΡ‚Π΅Π»ΡŒΠ½ΠΎ ΠΊΠΎΡ‚ΠΎΡ€Ρ‹Ρ… измСряСтся ΠΎΡ‚Π½ΠΎΡˆΠ΅Π½ΠΈΠ΅.

    ΠŸΡ€Π΅ΠΈΠΌΡƒΡ‰Π΅ΡΡ‚Π²ΠΎ рСлаксационного Π³Π΅Π½Π΅Ρ€Π°Ρ‚ΠΎΡ€Π° состоит Π² Ρ‚ΠΎΠΌ, Ρ‡Ρ‚ΠΎ ΠΎΠ΄Π½Π° ΠΊΠ»Π΅ΠΌΠΌΠ° кондСнсатора Π·Π°Π·Π΅ΠΌΠ»Π΅Π½Π°, поэтому ΠΏΠ°Ρ€Π°Π·ΠΈΡ‚Π½Ρ‹Π΅ ΠΏΠΎΠΌΠ΅Ρ…ΠΈ ΠΎΡ‚Π½ΠΎΡΠΈΡ‚Π΅Π»ΡŒΠ½ΠΎ ΡΡ‚Π°Π±ΠΈΠ»ΡŒΠ½Ρ‹. НСдостатком являСтся Ρ‚ΠΎ, Ρ‡Ρ‚ΠΎ рассСяниС ΠΌΠΎΠΆΠ΅Ρ‚ Π±Ρ‹Ρ‚ΡŒ большим, довольно Π»Π΅Π³ΠΊΠΎ большим ΠΏΠΎ ΡΡ€Π°Π²Π½Π΅Π½ΠΈΡŽ с 5 ΠΏΠ€.

    ΠΠ»ΡŒΡ‚Π΅Ρ€Π½Π°Ρ‚ΠΈΠ²ΠΎΠΉ являСтся 3-ΠΊΠΎΠ½Ρ‚Π°ΠΊΡ‚Π½ΠΎΠ΅ Π·Π°Ρ‰ΠΈΡ‰Π΅Π½Π½ΠΎΠ΅ ΠΈΠ·ΠΌΠ΅Ρ€Π΅Π½ΠΈΠ΅, ΠΊΠΎΡ‚ΠΎΡ€ΠΎΠ΅ нСвосприимчиво ΠΊ ΠΏΠ°Ρ€Π°Π·ΠΈΡ‚Π½ΠΎΠΉ Смкости Π½Π° любом Π²Ρ‹Π²ΠΎΠ΄Π΅ кондСнсатора ΠΈ Ρ‡ΡƒΠ²ΡΡ‚Π²ΠΈΡ‚Π΅Π»ΡŒΠ½ΠΎ Ρ‚ΠΎΠ»ΡŒΠΊΠΎ ΠΊ ΠΏΠ°Ρ€Π°Π·ΠΈΡ‚Π½Ρ‹ΠΌ ΠΏΠΎΠΌΠ΅Ρ…Π°ΠΌ Π½Π° Π½Π΅ΠΌ. Π’Ρ€Π΅Ρ‚ΠΈΠΉ Π²Ρ‹Π²ΠΎΠ΄ Π·Π°Π·Π΅ΠΌΠ»Π΅Π½. ΠœΠ΅Ρ‚ΠΎΠ΄ Π·Π°ΠΊΠ»ΡŽΡ‡Π°Π΅Ρ‚ΡΡ Π² ΡΠ»Π΅Π΄ΡƒΡŽΡ‰Π΅ΠΌ.

    1) ΠŸΡ€ΠΈΠ»ΠΎΠΆΠΈΡ‚Π΅ ΡΠΈΠ½ΡƒΡΠΎΠΈΠ΄Π°Π»ΡŒΠ½ΠΎΠ΅ напряТСниС ΠΎΡ‚Π½ΠΎΡΠΈΡ‚Π΅Π»ΡŒΠ½ΠΎ Π·Π΅ΠΌΠ»ΠΈ ΠΊ ΠΎΠ΄Π½ΠΎΠΌΡƒ Π²Ρ‹Π²ΠΎΠ΄Ρƒ ΠΈΡΠΏΡ‹Ρ‚Π°Ρ‚Π΅Π»ΡŒΠ½ΠΎΠ³ΠΎ кондСнсатора ΠΎΡ‚ извСстного напряТСния. Бродяги ΠΎΡ‚ этой ΠΊΠ»Π΅ΠΌΠΌΡ‹ ΠΊ Π·Π΅ΠΌΠ»Π΅ ΠΏΠΎΠ΄Π²Π΅Π΄Π΅Π½Ρ‹ ΠΊ Ρ‚ΠΎΡ‡Π½ΠΎ Ρ‚Π°ΠΊΠΎΠΌΡƒ ΠΆΠ΅ Π½Π°ΠΏΡ€ΡΠΆΠ΅Π½ΠΈΡŽ, нас Π½Π΅ интСрСсуСт, ΠΊΠ°ΠΊΠΎΠΉ Ρ‚ΠΎΠΊ трСбуСтся для ΠΈΡ… заряда, достаточно измСрСния напряТСния.

    2) Π—Π°Π·Π΅ΠΌΠ»ΠΈΡ‚Π΅ Π²Ρ‚ΠΎΡ€ΡƒΡŽ ΠΊΠ»Π΅ΠΌΠΌΡƒ ΠΈ ΠΈΠ·ΠΌΠ΅Ρ€ΡŒΡ‚Π΅ Ρ‚ΠΎΠΊ, Π½Π΅ΠΎΠ±Ρ…ΠΎΠ΄ΠΈΠΌΡ‹ΠΉ для этого. НаиболСС распространСнный способ ΡΠ΄Π΅Π»Π°Ρ‚ΡŒ это β€” ΠΈΡΠΏΠΎΠ»ΡŒΠ·ΠΎΠ²Π°Ρ‚ΡŒ ΠΎΠΏΠ΅Ρ€Π°Ρ†ΠΈΠΎΠ½Π½Ρ‹ΠΉ ΡƒΡΠΈΠ»ΠΈΡ‚Π΅Π»ΡŒ с Π²ΠΈΡ€Ρ‚ΡƒΠ°Π»ΡŒΠ½Ρ‹ΠΌ Π·Π°Π·Π΅ΠΌΠ»Π΅Π½ΠΈΠ΅ΠΌ.Π‘Π»ΡƒΠΆΠ΄Π°ΡŽΡ‰ΠΈΠ΅ ΠΎΡ‚ Π²Ρ‚ΠΎΡ€ΠΎΠ³ΠΎ ΠΊΠΎΠ½Ρ‚Π°ΠΊΡ‚Π° ΠΊ Π·Π΅ΠΌΠ»Π΅ ΡƒΠ΄Π΅Ρ€ΠΆΠΈΠ²Π°ΡŽΡ‚ΡΡ Π½Π° ΡƒΡ€ΠΎΠ²Π½Π΅ 0 Π’, поэтому Π² Π½ΠΈΡ… Π½Π΅ ΠΏΡ€ΠΎΡ‚Π΅ΠΊΠ°Π΅Ρ‚ Ρ‚ΠΎΠΊ, поэтому ΠΈΠ·ΠΌΠ΅Ρ€Π΅Π½ΠΈΠ΅ Ρ‚ΠΎΠΊΠ° являСтся Ρ‚ΠΎΡ‡Π½Ρ‹ΠΌ.

    3) Π’Π΅ΠΏΠ΅Ρ€ΡŒ ΠΌΡ‹ Π·Π½Π°Π΅ΠΌ Ρ‚ΠΎΠΊ Ρ‡Π΅Ρ€Π΅Π· кондСнсатор ΠΏΡ€ΠΈ Π·Π°Π΄Π°Π½Π½ΠΎΠΌ напряТСнии Π½Π° Π½Π΅ΠΌ. ВычислитС Π΅ΠΌΠΊΠΎΡΡ‚ΡŒ ΠΏΠΎ импСдансу ΠΈ частотС. Емкостная обратная связь, Π° Π½Π΅ рСзистивная Π½Π° Π²ΠΈΡ€Ρ‚ΡƒΠ°Π»ΡŒΠ½ΠΎΠΌ ΠΎΠΏΠ΅Ρ€Π°Ρ†ΠΈΠΎΠ½Π½ΠΎΠΌ усилитСлС Π·Π΅ΠΌΠ»ΠΈ, позволяСт ΠΈΡΠΊΠ»ΡŽΡ‡ΠΈΡ‚ΡŒ частоту ΠΈΠ· уравнСния.

    НСсмотря Π½Π° Ρ‚ΠΎ, Ρ‡Ρ‚ΠΎ Π·Π°Ρ‰ΠΈΡ‰Π΅Π½Π½ΠΎΠ΅ ΠΈΠ·ΠΌΠ΅Ρ€Π΅Π½ΠΈΠ΅ устраняСт влияниС рассСяния Π½Π° зСмлю, Π»ΡŽΠ±Ρ‹Π΅ рассСяния Π½Π° кондСнсаторС, усилСнныС вашим ΠΈΡΠΏΡ‹Ρ‚Π°Ρ‚Π΅Π»ΡŒΠ½Ρ‹ΠΌ приспособлСниСм, Π½Π°ΠΏΡ€ΠΈΠΌΠ΅Ρ€ пластиковой ΠΏΡ€ΠΈΠΆΠΈΠΌΠ½ΠΎΠΉ ΠΏΡ€ΠΎΠΊΠ»Π°Π΄ΠΊΠΎΠΉ, которая ΡƒΠ΄Π΅Ρ€ΠΆΠΈΠ²Π°Π΅Ρ‚ ΠΊΠΎΠΌΠΏΠΎΠ½Π΅Π½Ρ‚ SMD Π½Π° ΠΎΠΏΠΎΡ€Π΅, измСнят ΠΈΠ·ΠΌΠ΅Ρ€Π΅Π½ΠΈΠ΅ ΠΏΠΎ ΡΡ€Π°Π²Π½Π΅Π½ΠΈΡŽ с Ρ‚Π΅ΠΌ, Ρ‡Ρ‚ΠΎ это Π±Ρ‹Π»ΠΎ Π±Ρ‹ Π² Ρ†Π΅ΠΏΠΈ Π±Π΅Π· этой ΠΏΠ»ΠΎΡ‰Π°Π΄ΠΊΠΈ.

    ΠŸΠ°Ρ€Π°Π»Π»Π΅Π»ΡŒΠ½Ρ‹Π΅ кондСнсаторы

    Π—Π°Π΄Π°Ρ‡Π° 1:

    ΠšΠΎΠ½Π΄Π΅Π½ΡΠ°Ρ‚ΠΎΡ€Ρ‹ сСрии

    Когда кондСнсаторы соСдинСны ΠΎΠ΄ΠΈΠ½ Π·Π° Π΄Ρ€ΡƒΠ³ΠΈΠΌ, это называСтся ΠΏΠΎΡΠ»Π΅Π΄ΠΎΠ²Π°Ρ‚Π΅Π»ΡŒΠ½Ρ‹ΠΌ соСдинСниСм. Π­Ρ‚ΠΎ ΠΏΠΎΠΊΠ°Π·Π°Π½ΠΎ Π½ΠΈΠΆΠ΅. Π§Ρ‚ΠΎΠ±Ρ‹ Ρ€Π°ΡΡΡ‡ΠΈΡ‚Π°Ρ‚ΡŒ ΠΎΠ±Ρ‰ΡƒΡŽ ΠΎΠ±Ρ‰ΡƒΡŽ Π΅ΠΌΠΊΠΎΡΡ‚ΡŒ Π΄Π²ΡƒΡ… кондСнсаторов, соСдинСнных Ρ‚Π°ΠΊΠΈΠΌ ΠΎΠ±Ρ€Π°Π·ΠΎΠΌ, Π²Ρ‹ ΠΌΠΎΠΆΠ΅Ρ‚Π΅ ΠΈΡΠΏΠΎΠ»ΡŒΠ·ΠΎΠ²Π°Ρ‚ΡŒ ΡΠ»Π΅Π΄ΡƒΡŽΡ‰ΡƒΡŽ Ρ„ΠΎΡ€ΠΌΡƒΠ»Ρƒ:

    CΠΎΠ±Ρ‰ = Π‘1 Ρ… Π‘2 ΠΈ Ρ‚Π°ΠΊ Π΄Π°Π»Π΅Π΅
    Π‘1 + Π‘2

    ΠŸΡ€ΠΈΠΌΠ΅Ρ€: Π§Ρ‚ΠΎΠ±Ρ‹ Ρ€Π°ΡΡΡ‡ΠΈΡ‚Π°Ρ‚ΡŒ ΠΎΠ±Ρ‰ΡƒΡŽ Π΅ΠΌΠΊΠΎΡΡ‚ΡŒ для этих Π΄Π²ΡƒΡ… ΠΏΠΎΡΠ»Π΅Π΄ΠΎΠ²Π°Ρ‚Π΅Π»ΡŒΠ½ΠΎ соСдинСнных кондСнсаторов.

    Π—Π°Π΄Π°Ρ‡Π° 2:

    Π’Ρ€ΠΈ ΠΈΠ»ΠΈ Π±ΠΎΠ»Π΅Π΅ ΠΏΠΎΡΠ»Π΅Π΄ΠΎΠ²Π°Ρ‚Π΅Π»ΡŒΠ½ΠΎ соСдинСнных кондСнсатора

    Π—Π°Π΄Π°Ρ‡Π° 3:

    ΠΎΡ‚Π²Π΅Ρ‚Ρ‹

    Π—Π°Π΄Π°Ρ‡Π° 1

    Π—Π°Π΄Π°Ρ‡Π° 2

    Π—Π°Π΄Π°Ρ‡Π° 3

    Β©Kitronik Ltd. Π’Ρ‹ ΠΌΠΎΠΆΠ΅Ρ‚Π΅ Ρ€Π°ΡΠΏΠ΅Ρ‡Π°Ρ‚Π°Ρ‚ΡŒ эту страницу ΠΈ Π΄Π°Ρ‚ΡŒ ссылку Π½Π° Π½Π΅Π΅, Π½ΠΎ Π½Π΅ Π΄ΠΎΠ»ΠΆΠ½Ρ‹ ΠΊΠΎΠΏΠΈΡ€ΠΎΠ²Π°Ρ‚ΡŒ страницу ΠΈΠ»ΠΈ Π΅Π΅ Ρ‡Π°ΡΡ‚ΡŒ Π±Π΅Π· ΠΏΡ€Π΅Π΄Π²Π°Ρ€ΠΈΡ‚Π΅Π»ΡŒΠ½ΠΎΠ³ΠΎ письмСнного согласия ΠΊΠΎΠΌΠΏΠ°Π½ΠΈΠΈ Kitronik.

    22 октября 2019 Π² 07:27

    Π₯ΠΎΡ€ΠΎΡˆΠΈΠ΅ Π·Π°Π΄Π°Ρ‡ΠΈ ΠΌΠ½Π΅ ΠΏΠΎΠΌΠΎΠ³Π»ΠΈ

    03 сСнтября 2019 Π² 10:48

    03 сСнтября 2019 Π² 08:22

    Π­Ρ‚ΠΎΡ‚ сайт оказался ΠΏΠΎΠ»Π΅Π·Π½Ρ‹ΠΌ для ΠΌΠ½ΠΎΠ³ΠΈΡ… ΠΈΠ· нас. ΠšΠΈΡ‚Ρ€ΠΎΠ½ΠΈΠΊ спасибо!

    21 фСвраля 2019 Π³., 14:06

    ΠŸΡ€ΠΈΠ²Π΅Ρ‚, Π½Π° Π΄Π°Π½Π½Ρ‹ΠΉ ΠΌΠΎΠΌΠ΅Π½Ρ‚ Ρƒ нас Π½Π΅Ρ‚ руководства, ΠΏΠΎΠΊΠ°Π·Ρ‹Π²Π°ΡŽΡ‰Π΅Π³ΠΎ это, Π½ΠΎ ΠΌΡ‹ Π΄ΠΎΠ»ΠΆΠ½Ρ‹ Π΅Π³ΠΎ Π΄ΠΎΠ±Π°Π²ΠΈΡ‚ΡŒ. Π’Ρ‹ ΠΏΡ€Π°Π²ΠΈΠ»ΡŒΠ½ΠΎ догадались, ΠΊΠ°ΠΊ Ρ€Π°ΡΡΡ‡ΠΈΡ‚Π°Ρ‚ΡŒ ΠΎΠ±Ρ‰ΡƒΡŽ Π΅ΠΌΠΊΠΎΡΡ‚ΡŒ для вашСй схСмы. НадСюсь, это ΠΏΠΎΠΌΠΎΠ³Π»ΠΎ.

    20 фСвраля 2019 Π³., 18:57

    Π‘Ρ‹Π»ΠΈ Π»ΠΈ Ρƒ вас ΠΏΡ€ΠΎΡ€Π°Π±ΠΎΡ‚Π°Π½Ρ‹ ΠΊΠ°ΠΊΠΈΠ΅-Π»ΠΈΠ±ΠΎ Π·Π°Π΄Π°Ρ‡ΠΈ с ΠΏΠΎΠ΄ΠΎΠ±Π½Ρ‹ΠΌΠΈ изобраТСниями, ΠΊΠ°ΠΊ ΠΏΠΎΠΊΠ°Π·Π°Π½ΠΎ Π²Ρ‹ΡˆΠ΅? Π― ΠΏΡ‹Ρ‚Π°ΡŽΡΡŒ Π²Ρ‹ΡΡΠ½ΠΈΡ‚ΡŒ эту ΠΆΠ΅ ΠΏΡ€ΠΎΠ±Π»Π΅ΠΌΡƒ. Π£ мСня Π΅ΡΡ‚ΡŒ Π΄Π²Π° кондСнсатора ΠΏΠ°Ρ€Π°Π»Π»Π΅Π»ΡŒΠ½ΠΎ Π΄Ρ€ΡƒΠ³ Π΄Ρ€ΡƒΠ³Ρƒ, Π½ΠΎ Ρ‚Π°ΠΊΠΆΠ΅ ΠΏΠΎΡΠ»Π΅Π΄ΠΎΠ²Π°Ρ‚Π΅Π»ΡŒΠ½ΠΎ с ΠΎΠ΄Π½ΠΈΠΌ Π΄Ρ€ΡƒΠ³ΠΈΠΌ кондСнсатором.ΠœΠΎΠ³Ρƒ Π»ΠΈ я Π΄ΠΎΠ±Π°Π²ΠΈΡ‚ΡŒ Π΄Π²Π° ΠΏΠ°Ρ€Π°Π»Π»Π΅Π»ΡŒΠ½ΠΎ, Π° Π·Π°Ρ‚Π΅ΠΌ ΠΈΡΠΏΠΎΠ»ΡŒΠ·ΠΎΠ²Π°Ρ‚ΡŒ это число Π² ΡƒΡ€Π°Π²Π½Π΅Π½ΠΈΠΈ для ряда? Π›ΡŽΠ±ΠΎΠΉ совСт ΠΏΠΎΠ»Π΅Π·Π΅Π½ спасибо!!

    01 фСвраля 2019 Π² 12:16

    Π­Ρ‚ΠΎ Π΄Π΅ΠΉΡΡ‚Π²ΠΈΡ‚Π΅Π»ΡŒΠ½ΠΎ ΠΏΠΎΠΌΠΎΠ³Π»ΠΎ ΠΌΠ½Π΅, большоС спасибо

    04 января 2019 Π² 10:47

    ΠœΠΠ• ΠžΠ§Π•ΠΠ¬ ΠΠ ΠΠ’Π˜Π’Π‘Π― ЭВОВ РАБЧЕВ

    27 октября 2018 Π² 03:49

    ΠžΡ‡Π΅Π½ΡŒ Ρ…ΠΎΡ€ΠΎΡˆΠ°Ρ информация спасибо kitronik

    20 сСнтября 2018 Π³., 10:58

    ΠŸΡ€ΠΈΠ²Π΅Ρ‚, Π­ΠΌΠΌΠ°Π½ΡƒΡΠ»ΡŒ. Π’Ρ‹ ΠΌΠΎΠΆΠ΅Ρ‚Π΅ Ρ€Π°ΡΡΡ‡ΠΈΡ‚Π°Ρ‚ΡŒ Π΅ΠΌΠΊΠΎΡΡ‚ΡŒ ΠΊΠ°ΠΆΠ΄ΠΎΠΉ ΠΈΠ· областСй ΠΏΠΎ ΠΎΡ‚Π΄Π΅Π»ΡŒΠ½ΠΎΡΡ‚ΠΈ, Π° Π·Π°Ρ‚Π΅ΠΌ Π²Ρ‹ΡΡΠ½ΠΈΡ‚ΡŒ, ΠΊΠ°ΠΊ Π½Π°ΠΉΡ‚ΠΈ ΠΎΠ±Ρ‰ΡƒΡŽ Π΅ΠΌΠΊΠΎΡΡ‚ΡŒ, ΠΌΠ΅Ρ‚ΠΎΠ΄ Π±ΡƒΠ΄Π΅Ρ‚ ΠΎΠΏΡ€Π΅Π΄Π΅Π»ΡΡ‚ΡŒΡΡ Ρ‚Π΅ΠΌ, ΠΊΠ°ΠΊ Ρ€Π°Π·Π»ΠΈΡ‡Π½Ρ‹Π΅ области располоТСны ΠΏΠΎ ΠΎΡ‚Π½ΠΎΡˆΠ΅Π½ΠΈΡŽ Π΄Ρ€ΡƒΠ³ ΠΊ Π΄Ρ€ΡƒΠ³Ρƒ. Они ΠΌΠΎΠ³ΡƒΡ‚ Π±Ρ‹Ρ‚ΡŒ ΠΏΠΎΡΠ»Π΅Π΄ΠΎΠ²Π°Ρ‚Π΅Π»ΡŒΠ½ΠΎ Π΄Ρ€ΡƒΠ³ с Π΄Ρ€ΡƒΠ³ΠΎΠΌ ΠΈΠ»ΠΈ ΠΏΠ°Ρ€Π°Π»Π»Π΅Π»ΡŒΠ½ΠΎ. НадСюсь, это ΠΏΠΎΠΌΠΎΠΆΠ΅Ρ‚.

    19 сСнтября 2018 Π² 01:21

    Как Ρ€Π°ΡΡΡ‡ΠΈΡ‚Π°Ρ‚ΡŒ кондСнсаторы, соСдинСнныС ΠΊΠ°ΠΊ ΠΏΠ°Ρ€Π°Π»Π»Π΅Π»ΡŒΠ½ΠΎ, Ρ‚Π°ΠΊ ΠΈ ΠΏΠΎΡΠ»Π΅Π΄ΠΎΠ²Π°Ρ‚Π΅Π»ΡŒΠ½ΠΎ

    16 января 2018 Π³., 18:42

    это ΠΎΡ‡Π΅Π½ΡŒ ΠΏΠΎΠ»Π΅Π·Π½Ρ‹ΠΉ инструмСнт для изучСния основ элСктротСхники. спасибо

    14 июля 2017 г., 10:41

    ΠŸΡ€ΠΈΠ²Π΅Ρ‚, Кин, ΠΎΡΡ‚Π°Π»ΠΎΡΡŒ ΡΠ΄Π΅Π»Π°Ρ‚ΡŒ Π΅Ρ‰Π΅ ΠΎΠ΄ΠΈΠ½ шаг расчСта, Ρ‚Π΅Π±Π΅ Π½ΡƒΠΆΠ½ΠΎ Ρ€Π°Π·Π΄Π΅Π»ΠΈΡ‚ΡŒ 1 Π½Π° 3, ΠΈ Ρ‚ΠΎΠ³Π΄Π° Ρ‚Ρ‹ ΠΏΠΎΠ»ΡƒΡ‡ΠΈΡˆΡŒ ΠΎΡ‚Π²Π΅Ρ‚ для суммы Π‘.НадСюсь это ΠΏΠΎΠΌΠΎΠΆΠ΅Ρ‚.

    13 июля 2017 г., 04:54

    Π€ΠΎΡ€ΠΌΡƒΠ»Π° для ΠΏΠΎΡΠ»Π΅Π΄ΠΎΠ²Π°Ρ‚Π΅Π»ΡŒΠ½ΠΎΠΉ Смкости Π½Π΅ Ρ€Π°Π±ΠΎΡ‚Π°Π΅Ρ‚ для 1F. Если Π²Ρ‹ Π΄ΠΎΠ±Π°Π²ΠΈΡ‚Π΅ 1+1+1, Π²Ρ‹ ΠΏΠΎΠ»ΡƒΡ‡ΠΈΡ‚Π΅ 3F вмСсто .333F. ΠŸΠΎΠΆΠ°Π»ΡƒΠΉΡΡ‚Π°, объясни.

    19 мая 2017 в 08:17

    ΠžΡ‡Π΅Π½ΡŒ ΠΎΡ‡Π΅Π½ΡŒ ΠΏΠΎΠ»Π΅Π·Π½Ρ‹ΠΉ сайт ΠΌΠ½Π΅ нравится.. Π‘ΡƒΠ΄ΡƒΡ‡ΠΈ ΡƒΡ‡Π΅Π½ΠΈΠΊΠΎΠΌ 10 класса я понял ΠΊΠ°ΠΊ Ρ€Π΅ΡˆΠ°Ρ‚ΡŒ вопрос ΠΎ ΠΏΠΎΡΠ»Π΅Π΄ΠΎΠ²Π°Ρ‚Π΅Π»ΡŒΠ½ΠΎΠΌ ΠΈ ΠΏΠ°Ρ€Π°Π»Π»Π΅Π»ΡŒΠ½ΠΎΠΌ соСдинСнии ΠΏΡ€ΠΎΠ²ΠΎΠ΄Π½ΠΈΠΊΠΎΠ²..

    10 мая 2017 г., 11:00

    МнС нравится этот сайт. Π‘ΠΎΠ»ΡŒΡˆΠΎΠ΅ спасибо, я Π±ΡƒΠ΄Ρƒ ΠΏΠΈΡΠ°Ρ‚ΡŒ экзамСны.Ρ‚Π΅ΠΏΠ΅Ρ€ΡŒ моя ΠΏΡ€ΠΎΠ±Π»Π΅ΠΌΠ° с кондСнсаторами Ρ€Π΅ΡˆΠ΅Π½Π° ΠΏΠΎΠ»Π½ΠΎΡΡ‚ΡŒΡŽ. Π΅Ρ‰Π΅ Ρ€Π°Π· спасибо

    19 апрСля 2017 Π³., 09:40

    Π’ΠΎΠ·ΠΌΠΎΠΆΠ½ΠΎ, это Ρ‚ΠΎ, для Ρ‡Π΅Π³ΠΎ ΠΌΡ‹ ΠΊΠΎΠ³Π΄Π°-Π½ΠΈΠ±ΡƒΠ΄ΡŒ создадим рСсурс, Π½ΠΎ сСйчас ΠΏΠΎΠΏΡ€ΠΎΠ±ΡƒΠΉΡ‚Π΅ ΠΏΠΎΠΈΡΠΊΠ°Ρ‚ΡŒ Π² Google, Ρ‚Π°ΠΊ ΠΊΠ°ΠΊ Π² Π˜Π½Ρ‚Π΅Ρ€Π½Π΅Ρ‚Π΅ ΡƒΠΆΠ΅ Π΅ΡΡ‚ΡŒ ΠΌΠ½ΠΎΠ³ΠΎ ΠΈΠ½Ρ„ΠΎΡ€ΠΌΠ°Ρ†ΠΈΠΈ ΠΎΠ± этом.

    16 апрСля 2017 Π³., 14:11

    МнС Π½ΡƒΠΆΠ½ΠΎ большС объяснСний ΠΎ Π—ΠΠšΠžΠΠ• Π­Π›Π•ΠšΠ’Π ΠžΠ‘Π’ΠΠ’Π˜ΠšΠ˜ ΠšΠ£Π›ΠžΠΠ И Π•Π“Πž РАБЧЕВАΠ₯.

    03 апрСля 2017 Π² 06:27

    МнС нравится этот сайт, ΠΎΠ½ ΠΌΠ½Π΅ ΠΎΡ‡Π΅Π½ΡŒ ΠΏΠΎΠΌΠΎΠ³Π°Π΅Ρ‚

    13 дСкабря 2016 Π³., 22:05

    ΠžΠΉΠ½Π½Π½β€¦.это Π΄Π΅ΠΉΡΡ‚Π²ΠΈΡ‚Π΅Π»ΡŒΠ½ΠΎ ΡƒΠ΄ΠΈΠ²ΠΈΡ‚Π΅Π»ΡŒΠ½ΠΎ, это ΠΎΡ‡Π΅Π½ΡŒ ΠΏΠΎΠΌΠΎΠ³Π°Π΅Ρ‚.

    30 ноября 2016 Π³., 09:40

    Π²Π°Ρƒ!! Π― Π΄Π΅ΠΉΡΡ‚Π²ΠΈΡ‚Π΅Π»ΡŒΠ½ΠΎ люблю этот сайт, ΠΎΠ½ ΠΎΡ‡Π΅Π½ΡŒ ΠΏΠΎΠ»Π΅Π·Π΅Π½.

    01 ноября 2016 Π² 20:47

    Моя ΠΏΡ€ΠΎΠ±Π»Π΅ΠΌΠ° с кондСнсатором Ρ€Π΅ΡˆΠ΅Π½Π°

    20 октября 2016 Π³., 16:18

    ΠŸΡ€ΠΈΠ²Π΅Ρ‚, ΠΌΠ½Π΅ нравится этот расчСт ΠΏΠΎΡΠ»Π΅Π΄ΠΎΠ²Π°Ρ‚Π΅Π»ΡŒΠ½ΠΎ-ΠΏΠ°Ρ€Π°Π»Π»Π΅Π»ΡŒΠ½ΠΎ

    03 мая 2016 в 10:02

    ΠŸΡ€ΠΈΠ²Π΅Ρ‚, Дуглас. ΠŸΡ€ΠΈΠΌΠ΅Ρ€Ρ‹ расчСта Π±ΡƒΠ΄ΡƒΡ‚ Ρ€Π°Π±ΠΎΡ‚Π°Ρ‚ΡŒ нСзависимо ΠΎΡ‚ Π½ΠΎΠΌΠΈΠ½Π°Π»ΠΎΠ² ΠΎΡ‚Π΄Π΅Π»ΡŒΠ½Ρ‹Ρ… кондСнсаторов.

    25 апрСля 2016 Π³., 04:55

    ΠŸΡ€ΠΈΠ²Π΅Ρ‚. ΠŸΠΎΡ…ΠΎΠΆΠ΅, Ρ‡Ρ‚ΠΎ Π² Π²Π°ΡˆΠΈΡ… ΠΏΡ€ΠΈΠΌΠ΅Ρ€Π°Ρ… Ρ€Π΅Ρ‡ΡŒ ΠΈΠ΄Π΅Ρ‚ Ρ‚ΠΎΠ»ΡŒΠΊΠΎ ΠΎ кондСнсаторах Ρ€Π°Π·Π½ΠΎΠΉ Смкости.Π‘Ρ‹Π»ΠΎ Π±Ρ‹ ΠΏΠΎΠ»Π΅Π·Π½ΠΎ Π²ΠΊΠ»ΡŽΡ‡ΠΈΡ‚ΡŒ ΠΏΡ€ΠΈΠΌΠ΅Ρ€Ρ‹ ΠΎΠ΄ΠΈΠ½Π°ΠΊΠΎΠ²Ρ‹Ρ… кондСнсаторов, соСдинСнных ΠΏΠΎΡΠ»Π΅Π΄ΠΎΠ²Π°Ρ‚Π΅Π»ΡŒΠ½ΠΎ ΠΈ ΠΏΠ°Ρ€Π°Π»Π»Π΅Π»ΡŒΠ½ΠΎ. Π˜Π΄Π΅Π½Ρ‚ΠΈΡ‡Π½Ρ‹Π΅ кондСнсаторы, соСдинСнныС ΠΏΠΎΡΠ»Π΅Π΄ΠΎΠ²Π°Ρ‚Π΅Π»ΡŒΠ½ΠΎβ€¦.. ΠžΠ±Ρ‰Π°Ρ Π΅ΠΌΠΊΠΎΡΡ‚ΡŒ = номинальная Π΅ΠΌΠΊΠΎΡΡ‚ΡŒ, дСлСнная Π½Π° ΠΎΠ±Ρ‰Π΅Π΅ количСство кондСнсаторов. ΠΠΠŸΠ Π˜ΠœΠ•Π .. 3000 Π€Π°Ρ€Π°Π΄ Γ·(X5 ΠΏΠΎΡΠ»Π΅Π΄ΠΎΠ²Π°Ρ‚Π΅Π»ΡŒΠ½ΠΎ), …3000Π€/5=600Π€. ΠŸΠ°Ρ€Π°Π»Π»Π΅Π»ΡŒΠ½ΠΎ соСдинСнныС ΠΈΠ΄Π΅Π½Ρ‚ΠΈΡ‡Π½Ρ‹Π΅ кондСнсаторы…… ΠžΠ±Ρ‰Π°Ρ Π΅ΠΌΠΊΠΎΡΡ‚ΡŒ = номинальная Π΅ΠΌΠΊΠΎΡΡ‚ΡŒ, умноТСнная Π½Π° ΠΎΠ±Ρ‰Π΅Π΅ количСство кондСнсаторов………. 3000Farad X (X5 ΠΏΠ°Ρ€Π°Π»Π»Π΅Π»ΡŒΠ½ΠΎ)…3000FX5=15,000F УстраняСт всю Β«Ρ‡Π΅ΠΏΡƒΡ…Ρƒ с Π΄Π»ΠΈΠ½Π½Ρ‹ΠΌ Π΄Π΅Π»Π΅Π½ΠΈΠ΅ΠΌΒ»

    01 апрСля 2015 Π² 11:52

    ΠŸΡ€ΠΈΠ²Π΅Ρ‚, НапряТСниС останСтся ΠΏΡ€Π΅ΠΆΠ½ΠΈΠΌ.Π ΠΎΠ±

    12 ΠΌΠ°Ρ€Ρ‚Π° 2015 Π³., 23:36

    ОбъяснСниС понятно, Π° ΠΊΠ°ΠΊ Π±Ρ‹Ρ‚ΡŒ с Ρ€Π°Π±ΠΎΡ‡ΠΈΠΌ напряТСниСм Π΄Π²ΡƒΡ… кондСнсаторов ΠΏΠ°Ρ€Π°Π»Π»Π΅Π»ΡŒΠ½ΠΎ? ΠžΡΡ‚Π°Π΅Ρ‚ΡΡ Π»ΠΈ ΠΎΠ½ΠΎ ΠΏΡ€Π΅ΠΆΠ½ΠΈΠΌ ΠΈΠ»ΠΈ складываСтся номинальноС напряТСниС ΠΎΡ‚Π΄Π΅Π»ΡŒΠ½Ρ‹Ρ… кондСнсаторов. ΠŸΡ€Π΅Π΄ΠΏΠΎΠ»ΠΎΠΆΠΈΠΌ, Ρ‡Ρ‚ΠΎ ΠΎΠ±Π° кондСнсатора ΠΈΠΌΠ΅ΡŽΡ‚ ΠΎΠ΄ΠΈΠ½Π°ΠΊΠΎΠ²ΠΎΠ΅ Ρ€Π°Π±ΠΎΡ‡Π΅Π΅ напряТСниС Π½ΠΎΠΌΠΈΠ½Π°Π»ΠΎΠΌ

    .

    Π‘Π΅Π»ΠΊΠΎ Π¦Π°Ρ€ΡŒ Π‘ΠΎΠ»ΠΎΠΌΠΎΠ½

    24 фСвраля 2015 Π³. , 13:23

    это объяснСниС простоС ΠΈ Π»Π΅Π³ΠΊΠΎΠ΅ для понимания ΠΈ нравится.

    07 дСкабря 2014 Π² 00:57

    Пока это СдинствСнноС объяснСниС, ΠΊΠΎΡ‚ΠΎΡ€ΠΎΠ΅ я смог ΠΏΠΎΠ½ΡΡ‚ΡŒ.Бпасибо

    23 мая 2014 г., 12:17

    Бпасибо, сСйчас исправил!

    21 мая 2014 г., 22:04

    я Π΄ΡƒΠΌΠ°ΡŽ, Ρ‡Ρ‚ΠΎ Π·Π°Π΄Π°Π½ΠΈΠ΅ 3 Π΄ΠΎΠ»ΠΆΠ½ΠΎ Π±Ρ‹Ρ‚ΡŒ 1.167F, Π° Π½Π΅ 1.67F

    ΠšΠΈΡ‚Ρ€ΠΎΠ½ΠΈΠΊ Π˜Π½Ρ„ΠΎΡ€ΠΌΠ°Ρ†ΠΈΠΎΠ½Π½Ρ‹ΠΉ Π±ΡŽΠ»Π»Π΅Ρ‚Π΅Π½ΡŒ

    Π—Π°Ρ€Π΅Π³ΠΈΡΡ‚Ρ€ΠΈΡ€ΡƒΠΉΡ‚Π΅ΡΡŒ сСйчас, Ρ‡Ρ‚ΠΎΠ±Ρ‹ ΠΏΠ΅Ρ€Π²Ρ‹ΠΌΠΈ ΡƒΠ·Π½Π°Π²Π°Ρ‚ΡŒ ΠΎ Π½ΠΎΠ²Π΅ΠΉΡˆΠΈΡ… ΠΏΡ€ΠΎΠ΄ΡƒΠΊΡ‚Π°Ρ… ΠΈ ​​рСсурсах!

    ΠœΡƒΠ»ΡŒΡ‚ΠΈΠΌΠ΅Ρ‚Ρ€ опрСдСляСт Π΅ΠΌΠΊΠΎΡΡ‚ΡŒ, заряТая кондСнсатор извСстным Ρ‚ΠΎΠΊΠΎΠΌ, измСряя ΠΏΠΎΠ»ΡƒΡ‡Π΅Π½Π½ΠΎΠ΅ напряТСниС ΠΈ Π·Π°Ρ‚Π΅ΠΌ вычисляя Π΅ΠΌΠΊΠΎΡΡ‚ΡŒ.

    ΠŸΡ€Π΅Π΄ΡƒΠΏΡ€Π΅ΠΆΠ΄Π΅Π½ΠΈΠ΅: Π₯ΠΎΡ€ΠΎΡˆΠΈΠΉ кондСнсатор Π½Π°ΠΊΠ°ΠΏΠ»ΠΈΠ²Π°Π΅Ρ‚ элСктричСский заряд ΠΈ ΠΌΠΎΠΆΠ΅Ρ‚ ΠΎΡΡ‚Π°Π²Π°Ρ‚ΡŒΡΡ ΠΏΠΎΠ΄ напряТСниСм послС ΠΎΡ‚ΠΊΠ»ΡŽΡ‡Π΅Π½ΠΈΡ питания.ΠŸΡ€Π΅ΠΆΠ΄Π΅ Ρ‡Π΅ΠΌ ΠΏΡ€ΠΈΠΊΠ°ΡΠ°Ρ‚ΡŒΡΡ ΠΊ Π½Π΅ΠΌΡƒ ΠΈΠ»ΠΈ ΠΏΡ€ΠΎΠ²ΠΎΠ΄ΠΈΡ‚ΡŒ измСрСния, Π°) Π²Ρ‹ΠΊΠ»ΡŽΡ‡ΠΈΡ‚Π΅ всС ΠΏΠΈΡ‚Π°Π½ΠΈΠ΅, Π±) ΠΈΡΠΏΠΎΠ»ΡŒΠ·ΡƒΠΉΡ‚Π΅ ΠΌΡƒΠ»ΡŒΡ‚ΠΈΠΌΠ΅Ρ‚Ρ€, Ρ‡Ρ‚ΠΎΠ±Ρ‹ ΡƒΠ±Π΅Π΄ΠΈΡ‚ΡŒΡΡ, Ρ‡Ρ‚ΠΎ ΠΏΠΈΡ‚Π°Π½ΠΈΠ΅ ΠΎΡ‚ΠΊΠ»ΡŽΡ‡Π΅Π½ΠΎ, ΠΈ Π²) остороТно разрядитС кондСнсатор, ΠΏΠΎΠ΄ΠΊΠ»ΡŽΡ‡ΠΈΠ² рСзистор ΠΊ ΠΏΡ€ΠΎΠ²ΠΎΠ΄Π°ΠΌ (ΠΊΠ°ΠΊ ΡƒΠΊΠ°Π·Π°Π½ΠΎ Π² ΡΠ»Π΅Π΄ΡƒΡŽΡ‰Π΅ΠΌ Π°Π±Π·Π°Ρ†Π΅). ΠžΠ±ΡΠ·Π°Ρ‚Π΅Π»ΡŒΠ½ΠΎ ΠΈΡΠΏΠΎΠ»ΡŒΠ·ΡƒΠΉΡ‚Π΅ ΡΠΎΠΎΡ‚Π²Π΅Ρ‚ΡΡ‚Π²ΡƒΡŽΡ‰ΠΈΠ΅ срСдства ΠΈΠ½Π΄ΠΈΠ²ΠΈΠ΄ΡƒΠ°Π»ΡŒΠ½ΠΎΠΉ Π·Π°Ρ‰ΠΈΡ‚Ρ‹.

    Для бСзопасного разряда кондСнсатора: ПослС ΠΎΡ‚ΠΊΠ»ΡŽΡ‡Π΅Π½ΠΈΡ питания ΠΏΠΎΠ΄ΠΊΠ»ΡŽΡ‡ΠΈΡ‚Π΅ рСзистор 20 000 Ом, 5 Π’Ρ‚ ΠΊ ΠΊΠ»Π΅ΠΌΠΌΠ°ΠΌ кондСнсатора Π½Π° ΠΏΡΡ‚ΡŒ сСкунд. Π‘ ΠΏΠΎΠΌΠΎΡ‰ΡŒΡŽ ΠΌΡƒΠ»ΡŒΡ‚ΠΈΠΌΠ΅Ρ‚Ρ€Π° ΡƒΠ±Π΅Π΄ΠΈΡ‚Π΅ΡΡŒ, Ρ‡Ρ‚ΠΎ кондСнсатор ΠΏΠΎΠ»Π½ΠΎΡΡ‚ΡŒΡŽ разряТСн.

    1. Π‘ ΠΏΠΎΠΌΠΎΡ‰ΡŒΡŽ Ρ†ΠΈΡ„Ρ€ΠΎΠ²ΠΎΠ³ΠΎ ΠΌΡƒΠ»ΡŒΡ‚ΠΈΠΌΠ΅Ρ‚Ρ€Π° (DMM) ΡƒΠ±Π΅Π΄ΠΈΡ‚Π΅ΡΡŒ, Ρ‡Ρ‚ΠΎ ΠΏΠΈΡ‚Π°Π½ΠΈΠ΅ Ρ†Π΅ΠΏΠΈ ΠΎΡ‚ΠΊΠ»ΡŽΡ‡Π΅Π½ΠΎ. Если кондСнсатор ΠΈΡΠΏΠΎΠ»ΡŒΠ·ΡƒΠ΅Ρ‚ΡΡ Π² Ρ†Π΅ΠΏΠΈ ΠΏΠ΅Ρ€Π΅ΠΌΠ΅Π½Π½ΠΎΠ³ΠΎ Ρ‚ΠΎΠΊΠ°, установитС ΠΌΡƒΠ»ΡŒΡ‚ΠΈΠΌΠ΅Ρ‚Ρ€ Π½Π° ΠΈΠ·ΠΌΠ΅Ρ€Π΅Π½ΠΈΠ΅ напряТСния ΠΏΠ΅Ρ€Π΅ΠΌΠ΅Π½Π½ΠΎΠ³ΠΎ Ρ‚ΠΎΠΊΠ°. Если ΠΎΠ½ ΠΈΡΠΏΠΎΠ»ΡŒΠ·ΡƒΠ΅Ρ‚ΡΡ Π² Ρ†Π΅ΠΏΠΈ постоянного Ρ‚ΠΎΠΊΠ°, настройтС Ρ†ΠΈΡ„Ρ€ΠΎΠ²ΠΎΠΉ ΠΌΡƒΠ»ΡŒΡ‚ΠΈΠΌΠ΅Ρ‚Ρ€ Π½Π° ΠΈΠ·ΠΌΠ΅Ρ€Π΅Π½ΠΈΠ΅ напряТСния постоянного Ρ‚ΠΎΠΊΠ°.
    2. ΠžΡΠΌΠΎΡ‚Ρ€ΠΈΡ‚Π΅ кондСнсатор. Если ΡƒΡ‚Π΅Ρ‡ΠΊΠΈ, Ρ‚Ρ€Π΅Ρ‰ΠΈΠ½Ρ‹, выпуклости ΠΈΠ»ΠΈ Π΄Ρ€ΡƒΠ³ΠΈΠ΅ ΠΏΡ€ΠΈΠ·Π½Π°ΠΊΠΈ износа ΠΎΡ‡Π΅Π²ΠΈΠ΄Π½Ρ‹, Π·Π°ΠΌΠ΅Π½ΠΈΡ‚Π΅ кондСнсатор.
    3. ΠŸΠΎΠ²Π΅Ρ€Π½ΠΈΡ‚Π΅ Ρ†ΠΈΡ„Π΅Ρ€Π±Π»Π°Ρ‚ Π² Ρ€Π΅ΠΆΠΈΠΌ измСрСния Смкости (

    ΠžΠ±Π·ΠΎΡ€ измСрСния Смкости

    Поиск ΠΈ устранСниС нСисправностСй ΠΎΠ΄Π½ΠΎΡ„Π°Π·Π½Ρ‹Ρ… Π΄Π²ΠΈΠ³Π°Ρ‚Π΅Π»Π΅ΠΉ β€” ΠΎΠ΄Π½ΠΎ ΠΈΠ· Π½Π°ΠΈΠ±ΠΎΠ»Π΅Π΅ ΠΏΡ€Π°ΠΊΡ‚ΠΈΡ‡Π½Ρ‹Ρ… ΠΏΡ€ΠΈΠΌΠ΅Π½Π΅Π½ΠΈΠΉ Ρ„ΡƒΠ½ΠΊΡ†ΠΈΠΈ измСрСния Смкости Ρ†ΠΈΡ„Ρ€ΠΎΠ²ΠΎΠ³ΠΎ ΠΌΡƒΠ»ΡŒΡ‚ΠΈΠΌΠ΅Ρ‚Ρ€Π°.

    ΠžΠ΄Π½ΠΎΡ„Π°Π·Π½Ρ‹ΠΉ Π΄Π²ΠΈΠ³Π°Ρ‚Π΅Π»ΡŒ с кондСнсаторным пуском, ΠΊΠΎΡ‚ΠΎΡ€Ρ‹ΠΉ Π½Π΅ запускаСтся, являСтся ΠΏΡ€ΠΈΠ·Π½Π°ΠΊΠΎΠΌ нСисправного кондСнсатора. Π’Π°ΠΊΠΈΠ΅ Π΄Π²ΠΈΠ³Π°Ρ‚Π΅Π»ΠΈ Π±ΡƒΠ΄ΡƒΡ‚ ΠΏΡ€ΠΎΠ΄ΠΎΠ»ΠΆΠ°Ρ‚ΡŒ Ρ€Π°Π±ΠΎΡ‚Π°Ρ‚ΡŒ послС запуска, Ρ‡Ρ‚ΠΎ услоТняСт поиск ΠΈ устранСниС нСисправностСй. Π’Ρ‹Ρ…ΠΎΠ΄ ΠΈΠ· строя кондСнсатора ТСсткого пуска компрСссоров HVAC являСтся Ρ…ΠΎΡ€ΠΎΡˆΠΈΠΌ ΠΏΡ€ΠΈΠΌΠ΅Ρ€ΠΎΠΌ этой ΠΏΡ€ΠΎΠ±Π»Π΅ΠΌΡ‹. Π”Π²ΠΈΠ³Π°Ρ‚Π΅Π»ΡŒ компрСссора ΠΌΠΎΠΆΠ΅Ρ‚ Π·Π°ΠΏΡƒΡΡ‚ΠΈΡ‚ΡŒΡΡ, Π½ΠΎ вскорС пСрСгрССтся, Ρ‡Ρ‚ΠΎ ΠΏΡ€ΠΈΠ²Π΅Π΄Π΅Ρ‚ ΠΊ ΠΎΡ‚ΠΊΠ»ΡŽΡ‡Π΅Π½ΠΈΡŽ Π²Ρ‹ΠΊΠ»ΡŽΡ‡Π°Ρ‚Π΅Π»Ρ.

    ΠžΠ΄Π½ΠΎΡ„Π°Π·Π½Ρ‹Π΅ Π΄Π²ΠΈΠ³Π°Ρ‚Π΅Π»ΠΈ с Ρ‚Π°ΠΊΠΈΠΌΠΈ ΠΏΡ€ΠΎΠ±Π»Π΅ΠΌΠ°ΠΌΠΈ ΠΈ ΡˆΡƒΠΌΠ½Ρ‹Π΅ ΠΎΠ΄Π½ΠΎΡ„Π°Π·Π½Ρ‹Π΅ Π΄Π²ΠΈΠ³Π°Ρ‚Π΅Π»ΠΈ с кондСнсаторами Ρ‚Ρ€Π΅Π±ΡƒΡŽΡ‚ ΠΌΡƒΠ»ΡŒΡ‚ΠΈΠΌΠ΅Ρ‚Ρ€Π° для ΠΏΡ€ΠΎΠ²Π΅Ρ€ΠΊΠΈ исправности кондСнсаторов.ΠŸΠΎΡ‡Ρ‚ΠΈ всС кондСнсаторы Π΄Π²ΠΈΠ³Π°Ρ‚Π΅Π»Π΅ΠΉ ΠΈΠΌΠ΅ΡŽΡ‚ Π½ΠΎΠΌΠΈΠ½Π°Π» Π² ΠΌΠΈΠΊΡ€ΠΎΡ„Π°Ρ€Π°Π΄Π°Ρ…, ΡƒΠΊΠ°Π·Π°Π½Π½Ρ‹ΠΉ Π½Π° кондСнсаторС.

    Π’Ρ€Π΅Ρ…Ρ„Π°Π·Π½Ρ‹Π΅ кондСнсаторы для ΠΊΠΎΡ€Ρ€Π΅ΠΊΡ†ΠΈΠΈ коэффициСнта мощности ΠΎΠ±Ρ‹Ρ‡Π½ΠΎ Π·Π°Ρ‰ΠΈΡ‰Π΅Π½Ρ‹ прСдохранитСлями. Если ΠΎΠ΄ΠΈΠ½ ΠΈΠ»ΠΈ нСсколько ΠΈΠ· этих кондСнсаторов Π²Ρ‹ΠΉΠ΄ΡƒΡ‚ ΠΈΠ· строя, это ΠΏΡ€ΠΈΠ²Π΅Π΄Π΅Ρ‚ ΠΊ нСэффСктивности систСмы, скорСС всСго, увСличатся счСта Π·Π° ΠΊΠΎΠΌΠΌΡƒΠ½Π°Π»ΡŒΠ½Ρ‹Π΅ услуги ΠΈ ΠΌΠΎΠ³ΡƒΡ‚ ΠΏΡ€ΠΎΠΈΠ·ΠΎΠΉΡ‚ΠΈ Π½Π΅ΠΏΡ€Π΅Π΄Π½Π°ΠΌΠ΅Ρ€Π΅Π½Π½Ρ‹Π΅ ΠΎΡ‚ΠΊΠ»ΡŽΡ‡Π΅Π½ΠΈΡ оборудования. Π’ случаС пСрСгорания прСдохранитСля кондСнсатора Π½Π΅ΠΎΠ±Ρ…ΠΎΠ΄ΠΈΠΌΠΎ ΠΈΠ·ΠΌΠ΅Ρ€ΠΈΡ‚ΡŒ ΠΏΡ€Π΅Π΄ΠΏΠΎΠ»Π°Π³Π°Π΅ΠΌΠΎΠ΅ Π·Π½Π°Ρ‡Π΅Π½ΠΈΠ΅ нСисправного кондСнсатора Π² ΠΌΠΈΠΊΡ€ΠΎΡ„Π°Ρ€Π°Π΄Π°Ρ… ΠΈ ΡƒΠ±Π΅Π΄ΠΈΡ‚ΡŒΡΡ, Ρ‡Ρ‚ΠΎ ΠΎΠ½ΠΎ находится Π² ΠΏΡ€Π΅Π΄Π΅Π»Π°Ρ… Π΄ΠΈΠ°ΠΏΠ°Π·ΠΎΠ½Π°, ΡƒΠΊΠ°Π·Π°Π½Π½ΠΎΠ³ΠΎ Π½Π° кондСнсаторС.

    Π‘Ρ‚ΠΎΠΈΡ‚ Π·Π½Π°Ρ‚ΡŒ Π½Π΅ΠΊΠΎΡ‚ΠΎΡ€Ρ‹Π΅ Π΄ΠΎΠΏΠΎΠ»Π½ΠΈΡ‚Π΅Π»ΡŒΠ½Ρ‹Π΅ Ρ„Π°ΠΊΡ‚ΠΎΡ€Ρ‹, связанныС с Π΅ΠΌΠΊΠΎΡΡ‚ΡŒΡŽ:

    • ΠšΠΎΠ½Π΄Π΅Π½ΡΠ°Ρ‚ΠΎΡ€Ρ‹ ΠΈΠΌΠ΅ΡŽΡ‚ ΠΎΠ³Ρ€Π°Π½ΠΈΡ‡Π΅Π½Π½Ρ‹ΠΉ срок слуТбы ΠΈ часто ΡΠ²Π»ΡΡŽΡ‚ΡΡ ΠΏΡ€ΠΈΡ‡ΠΈΠ½ΠΎΠΉ нСисправности.
    • НСисправныС кондСнсаторы ΠΌΠΎΠ³ΡƒΡ‚ ΠΈΠΌΠ΅Ρ‚ΡŒ ΠΊΠΎΡ€ΠΎΡ‚ΠΊΠΎΠ΅ Π·Π°ΠΌΡ‹ΠΊΠ°Π½ΠΈΠ΅, ΠΎΠ±Ρ€Ρ‹Π² Ρ†Π΅ΠΏΠΈ ΠΈΠ»ΠΈ ΠΌΠΎΠ³ΡƒΡ‚ физичСски ΠΈΡΠΏΠΎΡ€Ρ‚ΠΈΡ‚ΡŒΡΡ Π΄ΠΎ Ρ‚ΠΎΡ‡ΠΊΠΈ ΠΎΡ‚ΠΊΠ°Π·Π°.
    • ΠŸΡ€ΠΈ ΠΊΠΎΡ€ΠΎΡ‚ΠΊΠΎΠΌ Π·Π°ΠΌΡ‹ΠΊΠ°Π½ΠΈΠΈ кондСнсатора ΠΌΠΎΠΆΠ΅Ρ‚ ΠΏΠ΅Ρ€Π΅Π³ΠΎΡ€Π΅Ρ‚ΡŒ ΠΏΡ€Π΅Π΄ΠΎΡ…Ρ€Π°Π½ΠΈΡ‚Π΅Π»ΡŒ ΠΈΠ»ΠΈ ΠΏΠΎΠ²Ρ€Π΅Π΄ΠΈΡ‚ΡŒΡΡ Π΄Ρ€ΡƒΠ³ΠΈΠ΅ ΠΊΠΎΠΌΠΏΠΎΠ½Π΅Π½Ρ‚Ρ‹.
    • Когда кондСнсатор открываСтся ΠΈΠ»ΠΈ ΠΈΠ·Π½Π°ΡˆΠΈΠ²Π°Π΅Ρ‚ΡΡ, Ρ†Π΅ΠΏΡŒ ΠΈΠ»ΠΈ Π΅Π΅ ΠΊΠΎΠΌΠΏΠΎΠ½Π΅Π½Ρ‚Ρ‹ ΠΌΠΎΠ³ΡƒΡ‚ Π½Π΅ Ρ€Π°Π±ΠΎΡ‚Π°Ρ‚ΡŒ.
    • Износ Ρ‚Π°ΠΊΠΆΠ΅ ΠΌΠΎΠΆΠ΅Ρ‚ ΠΈΠ·ΠΌΠ΅Π½ΠΈΡ‚ΡŒ Π·Π½Π°Ρ‡Π΅Π½ΠΈΠ΅ Смкости кондСнсатора, Ρ‡Ρ‚ΠΎ ΠΌΠΎΠΆΠ΅Ρ‚ Π²Ρ‹Π·Π²Π°Ρ‚ΡŒ ΠΏΡ€ΠΎΠ±Π»Π΅ΠΌΡ‹.

    ΠœΡƒΠ»ΡŒΡ‚ΠΈΠΌΠ΅Ρ‚Ρ€ опрСдСляСт Π΅ΠΌΠΊΠΎΡΡ‚ΡŒ ΠΏΡƒΡ‚Π΅ΠΌ зарядки кондСнсатора извСстным Ρ‚ΠΎΠΊΠΎΠΌ, измСрСния ΠΏΠΎΠ»ΡƒΡ‡Π΅Π½Π½ΠΎΠ³ΠΎ напряТСния ΠΈ ΠΏΠΎΡΠ»Π΅Π΄ΡƒΡŽΡ‰Π΅Π³ΠΎ расчСта Смкости.

    ΠŸΡ€Π΅Π΄ΡƒΠΏΡ€Π΅ΠΆΠ΄Π΅Π½ΠΈΠ΅. Π˜ΡΠΏΡ€Π°Π²Π½Ρ‹ΠΉ кондСнсатор Π½Π°ΠΊΠ°ΠΏΠ»ΠΈΠ²Π°Π΅Ρ‚ элСктричСский заряд ΠΈ ΠΌΠΎΠΆΠ΅Ρ‚ ΠΎΡΡ‚Π°Π²Π°Ρ‚ΡŒΡΡ ΠΏΠΎΠ΄ напряТСниСм послС ΠΎΡ‚ΠΊΠ»ΡŽΡ‡Π΅Π½ΠΈΡ питания. ΠŸΡ€Π΅ΠΆΠ΄Π΅ Ρ‡Π΅ΠΌ ΠΏΡ€ΠΈΠΊΠ°ΡΠ°Ρ‚ΡŒΡΡ ΠΊ Π½Π΅ΠΌΡƒ ΠΈΠ»ΠΈ ΠΏΡ€ΠΎΠ²ΠΎΠ΄ΠΈΡ‚ΡŒ измСрСния, Π°) Π²Ρ‹ΠΊΠ»ΡŽΡ‡ΠΈΡ‚Π΅ всС ΠΏΠΈΡ‚Π°Π½ΠΈΠ΅, Π±) ΠΈΡΠΏΠΎΠ»ΡŒΠ·ΡƒΠΉΡ‚Π΅ ΠΌΡƒΠ»ΡŒΡ‚ΠΈΠΌΠ΅Ρ‚Ρ€, Ρ‡Ρ‚ΠΎΠ±Ρ‹ ΡƒΠ±Π΅Π΄ΠΈΡ‚ΡŒΡΡ, Ρ‡Ρ‚ΠΎ ΠΏΠΈΡ‚Π°Π½ΠΈΠ΅ ΠΎΡ‚ΠΊΠ»ΡŽΡ‡Π΅Π½ΠΎ, ΠΈ Π²) остороТно разрядитС кондСнсатор, ΠΏΠΎΠ΄ΠΊΠ»ΡŽΡ‡ΠΈΠ² рСзистор ΠΊ ΠΏΡ€ΠΎΠ²ΠΎΠ΄Π°ΠΌ (ΠΊΠ°ΠΊ ΡƒΠΊΠ°Π·Π°Π½ΠΎ Π² ΡΠ»Π΅Π΄ΡƒΡŽΡ‰Π΅ΠΌ Π°Π±Π·Π°Ρ†Π΅). ΠžΠ±ΡΠ·Π°Ρ‚Π΅Π»ΡŒΠ½ΠΎ ΠΈΡΠΏΠΎΠ»ΡŒΠ·ΡƒΠΉΡ‚Π΅ ΡΠΎΠΎΡ‚Π²Π΅Ρ‚ΡΡ‚Π²ΡƒΡŽΡ‰ΠΈΠ΅ срСдства ΠΈΠ½Π΄ΠΈΠ²ΠΈΠ΄ΡƒΠ°Π»ΡŒΠ½ΠΎΠΉ Π·Π°Ρ‰ΠΈΡ‚Ρ‹.

    Π§Ρ‚ΠΎΠ±Ρ‹ бСзопасно Ρ€Π°Π·Ρ€ΡΠ΄ΠΈΡ‚ΡŒ кондСнсатор: послС ΠΎΡ‚ΠΊΠ»ΡŽΡ‡Π΅Π½ΠΈΡ питания ΠΏΠΎΠ΄ΠΊΠ»ΡŽΡ‡ΠΈΡ‚Π΅ рСзистор 20 000 Ом, 5 Π’Ρ‚ ΠΊ ΠΊΠ»Π΅ΠΌΠΌΠ°ΠΌ кондСнсатора Π½Π° ΠΏΡΡ‚ΡŒ сСкунд.Π‘ ΠΏΠΎΠΌΠΎΡ‰ΡŒΡŽ ΠΌΡƒΠ»ΡŒΡ‚ΠΈΠΌΠ΅Ρ‚Ρ€Π° ΡƒΠ±Π΅Π΄ΠΈΡ‚Π΅ΡΡŒ, Ρ‡Ρ‚ΠΎ кондСнсатор ΠΏΠΎΠ»Π½ΠΎΡΡ‚ΡŒΡŽ разряТСн.

    1. Π‘ ΠΏΠΎΠΌΠΎΡ‰ΡŒΡŽ Ρ†ΠΈΡ„Ρ€ΠΎΠ²ΠΎΠ³ΠΎ ΠΌΡƒΠ»ΡŒΡ‚ΠΈΠΌΠ΅Ρ‚Ρ€Π° (DMM) ΡƒΠ±Π΅Π΄ΠΈΡ‚Π΅ΡΡŒ, Ρ‡Ρ‚ΠΎ ΠΏΠΈΡ‚Π°Π½ΠΈΠ΅ Ρ†Π΅ΠΏΠΈ ΠΎΡ‚ΠΊΠ»ΡŽΡ‡Π΅Π½ΠΎ. Если кондСнсатор ΠΈΡΠΏΠΎΠ»ΡŒΠ·ΡƒΠ΅Ρ‚ΡΡ Π² Ρ†Π΅ΠΏΠΈ ΠΏΠ΅Ρ€Π΅ΠΌΠ΅Π½Π½ΠΎΠ³ΠΎ Ρ‚ΠΎΠΊΠ°, установитС ΠΌΡƒΠ»ΡŒΡ‚ΠΈΠΌΠ΅Ρ‚Ρ€ Π½Π° ΠΈΠ·ΠΌΠ΅Ρ€Π΅Π½ΠΈΠ΅ напряТСния ΠΏΠ΅Ρ€Π΅ΠΌΠ΅Π½Π½ΠΎΠ³ΠΎ Ρ‚ΠΎΠΊΠ°. Если ΠΎΠ½ ΠΈΡΠΏΠΎΠ»ΡŒΠ·ΡƒΠ΅Ρ‚ΡΡ Π² Ρ†Π΅ΠΏΠΈ постоянного Ρ‚ΠΎΠΊΠ°, настройтС Ρ†ΠΈΡ„Ρ€ΠΎΠ²ΠΎΠΉ ΠΌΡƒΠ»ΡŒΡ‚ΠΈΠΌΠ΅Ρ‚Ρ€ Π½Π° ΠΈΠ·ΠΌΠ΅Ρ€Π΅Π½ΠΈΠ΅ напряТСния постоянного Ρ‚ΠΎΠΊΠ°.
    2. ΠžΡΠΌΠΎΡ‚Ρ€ΠΈΡ‚Π΅ кондСнсатор. Если ΡƒΡ‚Π΅Ρ‡ΠΊΠΈ, Ρ‚Ρ€Π΅Ρ‰ΠΈΠ½Ρ‹, выпуклости ΠΈΠ»ΠΈ Π΄Ρ€ΡƒΠ³ΠΈΠ΅ ΠΏΡ€ΠΈΠ·Π½Π°ΠΊΠΈ износа ΠΎΡ‡Π΅Π²ΠΈΠ΄Π½Ρ‹, Π·Π°ΠΌΠ΅Π½ΠΈΡ‚Π΅ кондСнсатор.
    3. ΠŸΠΎΠ²Π΅Ρ€Π½ΠΈΡ‚Π΅ Ρ†ΠΈΡ„Π΅Ρ€Π±Π»Π°Ρ‚ Π² Ρ€Π΅ΠΆΠΈΠΌ измСрСния Смкости ( ).Π‘ΠΈΠΌΠ²ΠΎΠ» часто Π΄Π΅Π»ΠΈΡ‚ мСсто Π½Π° Ρ†ΠΈΡ„Π΅Ρ€Π±Π»Π°Ρ‚Π΅ с Π΄Ρ€ΡƒΠ³ΠΎΠΉ Ρ„ΡƒΠ½ΠΊΡ†ΠΈΠ΅ΠΉ. Π’ Π΄ΠΎΠΏΠΎΠ»Π½Π΅Π½ΠΈΠ΅ ΠΊ Ρ€Π΅Π³ΡƒΠ»ΠΈΡ€ΠΎΠ²ΠΊΠ΅ диска ΠΎΠ±Ρ‹Ρ‡Π½ΠΎ Π½Π΅ΠΎΠ±Ρ…ΠΎΠ΄ΠΈΠΌΠΎ Π½Π°ΠΆΠ°Ρ‚ΡŒ Ρ„ΡƒΠ½ΠΊΡ†ΠΈΠΎΠ½Π°Π»ΡŒΠ½ΡƒΡŽ ΠΊΠ½ΠΎΠΏΠΊΡƒ, Ρ‡Ρ‚ΠΎΠ±Ρ‹ Π°ΠΊΡ‚ΠΈΠ²ΠΈΡ€ΠΎΠ²Π°Ρ‚ΡŒ ΠΈΠ·ΠΌΠ΅Ρ€Π΅Π½ΠΈΠ΅. Π˜Π½ΡΡ‚Ρ€ΡƒΠΊΡ†ΠΈΠΈ см. Π² руководствС ΠΏΠΎΠ»ΡŒΠ·ΠΎΠ²Π°Ρ‚Π΅Π»Ρ вашСго ΠΌΡƒΠ»ΡŒΡ‚ΠΈΠΌΠ΅Ρ‚Ρ€Π°.

    Для ΠΏΡ€Π°Π²ΠΈΠ»ΡŒΠ½ΠΎΠ³ΠΎ измСрСния кондСнсатор Π½Π΅ΠΎΠ±Ρ…ΠΎΠ΄ΠΈΠΌΠΎ ΡƒΠ΄Π°Π»ΠΈΡ‚ΡŒ ΠΈΠ· Ρ†Π΅ΠΏΠΈ. РазрядитС кондСнсатор, ΠΊΠ°ΠΊ описано Π² ΠΏΡ€Π΅Π΄ΡƒΠΏΡ€Π΅ΠΆΠ΄Π΅Π½ΠΈΠΈ Π²Ρ‹ΡˆΠ΅.

    ΠŸΡ€ΠΈΠΌΠ΅Ρ‡Π°Π½ΠΈΠ΅. НСкоторыС ΠΌΡƒΠ»ΡŒΡ‚ΠΈΠΌΠ΅Ρ‚Ρ€Ρ‹ ΠΏΠΎΠ΄Π΄Π΅Ρ€ΠΆΠΈΠ²Π°ΡŽΡ‚ ΠΎΡ‚Π½ΠΎΡΠΈΡ‚Π΅Π»ΡŒΠ½Ρ‹ΠΉ (REL) Ρ€Π΅ΠΆΠΈΠΌ. ΠŸΡ€ΠΈ ΠΈΠ·ΠΌΠ΅Ρ€Π΅Π½ΠΈΠΈ ΠΌΠ°Π»Ρ‹Ρ… Π·Π½Π°Ρ‡Π΅Π½ΠΈΠΉ Смкости ΠΌΠΎΠΆΠ½ΠΎ ΠΈΡΠΏΠΎΠ»ΡŒΠ·ΠΎΠ²Π°Ρ‚ΡŒ ΠΎΡ‚Π½ΠΎΡΠΈΡ‚Π΅Π»ΡŒΠ½Ρ‹ΠΉ Ρ€Π΅ΠΆΠΈΠΌ для удалСния Смкости ΠΈΠ·ΠΌΠ΅Ρ€ΠΈΡ‚Π΅Π»ΡŒΠ½Ρ‹Ρ… ΠΏΡ€ΠΎΠ²ΠΎΠ΄ΠΎΠ².Π§Ρ‚ΠΎΠ±Ρ‹ пСрСвСсти ΠΌΡƒΠ»ΡŒΡ‚ΠΈΠΌΠ΅Ρ‚Ρ€ Π² ΠΎΡ‚Π½ΠΎΡΠΈΡ‚Π΅Π»ΡŒΠ½Ρ‹ΠΉ Ρ€Π΅ΠΆΠΈΠΌ измСрСния Смкости, ΠΎΡΡ‚Π°Π²ΡŒΡ‚Π΅ ΠΈΠ·ΠΌΠ΅Ρ€ΠΈΡ‚Π΅Π»ΡŒΠ½Ρ‹Π΅ ΠΏΡ€ΠΎΠ²ΠΎΠ΄Π° ΠΎΡ‚ΠΊΡ€Ρ‹Ρ‚Ρ‹ΠΌΠΈ ΠΈ Π½Π°ΠΆΠΌΠΈΡ‚Π΅ ΠΊΠ½ΠΎΠΏΠΊΡƒ REL. Π­Ρ‚ΠΎ удаляСт Π·Π½Π°Ρ‡Π΅Π½ΠΈΠ΅ остаточной Смкости ΠΈΠ·ΠΌΠ΅Ρ€ΠΈΡ‚Π΅Π»ΡŒΠ½Ρ‹Ρ… ΠΏΡ€ΠΎΠ²ΠΎΠ΄ΠΎΠ².

  • ΠŸΠΎΠ΄ΡΠΎΠ΅Π΄ΠΈΠ½ΠΈΡ‚Π΅ ΠΈΠ·ΠΌΠ΅Ρ€ΠΈΡ‚Π΅Π»ΡŒΠ½Ρ‹Π΅ ΠΏΡ€ΠΎΠ²ΠΎΠ΄Π° ΠΊ ΠΊΠ»Π΅ΠΌΠΌΠ°ΠΌ кондСнсатора. Π”Π΅Ρ€ΠΆΠΈΡ‚Π΅ ΠΈΠ·ΠΌΠ΅Ρ€ΠΈΡ‚Π΅Π»ΡŒΠ½Ρ‹Π΅ ΠΏΡ€ΠΎΠ²ΠΎΠ΄Π° ΠΏΠΎΠ΄ΠΊΠ»ΡŽΡ‡Π΅Π½Π½Ρ‹ΠΌΠΈ Π² Ρ‚Π΅Ρ‡Π΅Π½ΠΈΠ΅ Π½Π΅ΡΠΊΠΎΠ»ΡŒΠΊΠΈΡ… сСкунд, Ρ‡Ρ‚ΠΎΠ±Ρ‹ ΠΌΡƒΠ»ΡŒΡ‚ΠΈΠΌΠ΅Ρ‚Ρ€ автоматичСски Π²Ρ‹Π±Ρ€Π°Π» Π½ΡƒΠΆΠ½Ρ‹ΠΉ Π΄ΠΈΠ°ΠΏΠ°Π·ΠΎΠ½.
  • ΠŸΡ€ΠΎΡ‡ΠΈΡ‚Π°ΠΉΡ‚Π΅ ΠΎΡ‚ΠΎΠ±Ρ€Π°ΠΆΠ°Π΅ΠΌΠΎΠ΅ ΠΈΠ·ΠΌΠ΅Ρ€Π΅Π½ΠΈΠ΅. Если Π·Π½Π°Ρ‡Π΅Π½ΠΈΠ΅ Смкости находится Π² ΠΏΡ€Π΅Π΄Π΅Π»Π°Ρ… Π΄ΠΈΠ°ΠΏΠ°Π·ΠΎΠ½Π° измСрСния, ΠΌΡƒΠ»ΡŒΡ‚ΠΈΠΌΠ΅Ρ‚Ρ€ ΠΎΡ‚ΠΎΠ±Ρ€Π°Π·ΠΈΡ‚ Π·Π½Π°Ρ‡Π΅Π½ΠΈΠ΅ Смкости.Он Π±ΡƒΠ΄Π΅Ρ‚ ΠΎΡ‚ΠΎΠ±Ρ€Π°ΠΆΠ°Ρ‚ΡŒ OL, Ссли Π°) Π·Π½Π°Ρ‡Π΅Π½ΠΈΠ΅ Смкости Π²Ρ‹ΡˆΠ΅ Π΄ΠΈΠ°ΠΏΠ°Π·ΠΎΠ½Π° измСрСния ΠΈΠ»ΠΈ Π±) кондСнсатор нСисправСн.
  • ΠžΠ±Π·ΠΎΡ€ измСрСния Смкости

    Поиск ΠΈ устранСниС нСисправностСй ΠΎΠ΄Π½ΠΎΡ„Π°Π·Π½Ρ‹Ρ… Π΄Π²ΠΈΠ³Π°Ρ‚Π΅Π»Π΅ΠΉ β€” ΠΎΠ΄Π½ΠΎ ΠΈΠ· Π½Π°ΠΈΠ±ΠΎΠ»Π΅Π΅ ΠΏΡ€Π°ΠΊΡ‚ΠΈΡ‡Π½Ρ‹Ρ… ΠΏΡ€ΠΈΠΌΠ΅Π½Π΅Π½ΠΈΠΉ Ρ„ΡƒΠ½ΠΊΡ†ΠΈΠΈ измСрСния Смкости Ρ†ΠΈΡ„Ρ€ΠΎΠ²ΠΎΠ³ΠΎ ΠΌΡƒΠ»ΡŒΡ‚ΠΈΠΌΠ΅Ρ‚Ρ€Π°.

    ΠžΠ΄Π½ΠΎΡ„Π°Π·Π½Ρ‹ΠΉ Π΄Π²ΠΈΠ³Π°Ρ‚Π΅Π»ΡŒ с кондСнсаторным пуском, ΠΊΠΎΡ‚ΠΎΡ€Ρ‹ΠΉ Π½Π΅ запускаСтся, являСтся ΠΏΡ€ΠΈΠ·Π½Π°ΠΊΠΎΠΌ нСисправного кондСнсатора. Π’Π°ΠΊΠΈΠ΅ Π΄Π²ΠΈΠ³Π°Ρ‚Π΅Π»ΠΈ Π±ΡƒΠ΄ΡƒΡ‚ ΠΏΡ€ΠΎΠ΄ΠΎΠ»ΠΆΠ°Ρ‚ΡŒ Ρ€Π°Π±ΠΎΡ‚Π°Ρ‚ΡŒ послС запуска, Ρ‡Ρ‚ΠΎ услоТняСт поиск ΠΈ устранСниС нСисправностСй. Π’Ρ‹Ρ…ΠΎΠ΄ ΠΈΠ· строя кондСнсатора ТСсткого пуска компрСссоров HVAC являСтся Ρ…ΠΎΡ€ΠΎΡˆΠΈΠΌ ΠΏΡ€ΠΈΠΌΠ΅Ρ€ΠΎΠΌ этой ΠΏΡ€ΠΎΠ±Π»Π΅ΠΌΡ‹.Π”Π²ΠΈΠ³Π°Ρ‚Π΅Π»ΡŒ компрСссора ΠΌΠΎΠΆΠ΅Ρ‚ Π·Π°ΠΏΡƒΡΡ‚ΠΈΡ‚ΡŒΡΡ, Π½ΠΎ вскорС пСрСгрССтся, Ρ‡Ρ‚ΠΎ ΠΏΡ€ΠΈΠ²Π΅Π΄Π΅Ρ‚ ΠΊ ΠΎΡ‚ΠΊΠ»ΡŽΡ‡Π΅Π½ΠΈΡŽ Π²Ρ‹ΠΊΠ»ΡŽΡ‡Π°Ρ‚Π΅Π»Ρ.

    ΠžΠ΄Π½ΠΎΡ„Π°Π·Π½Ρ‹Π΅ Π΄Π²ΠΈΠ³Π°Ρ‚Π΅Π»ΠΈ с Ρ‚Π°ΠΊΠΈΠΌΠΈ ΠΏΡ€ΠΎΠ±Π»Π΅ΠΌΠ°ΠΌΠΈ ΠΈ ΡˆΡƒΠΌΠ½Ρ‹Π΅ ΠΎΠ΄Π½ΠΎΡ„Π°Π·Π½Ρ‹Π΅ Π΄Π²ΠΈΠ³Π°Ρ‚Π΅Π»ΠΈ с кондСнсаторами Ρ‚Ρ€Π΅Π±ΡƒΡŽΡ‚ ΠΌΡƒΠ»ΡŒΡ‚ΠΈΠΌΠ΅Ρ‚Ρ€Π° для ΠΏΡ€ΠΎΠ²Π΅Ρ€ΠΊΠΈ исправности кондСнсаторов. ΠŸΠΎΡ‡Ρ‚ΠΈ всС кондСнсаторы Π΄Π²ΠΈΠ³Π°Ρ‚Π΅Π»Π΅ΠΉ ΠΈΠΌΠ΅ΡŽΡ‚ Π½ΠΎΠΌΠΈΠ½Π°Π» Π² ΠΌΠΈΠΊΡ€ΠΎΡ„Π°Ρ€Π°Π΄Π°Ρ…, ΡƒΠΊΠ°Π·Π°Π½Π½Ρ‹ΠΉ Π½Π° кондСнсаторС.

    Π’Ρ€Π΅Ρ…Ρ„Π°Π·Π½Ρ‹Π΅ кондСнсаторы для ΠΊΠΎΡ€Ρ€Π΅ΠΊΡ†ΠΈΠΈ коэффициСнта мощности ΠΎΠ±Ρ‹Ρ‡Π½ΠΎ Π·Π°Ρ‰ΠΈΡ‰Π΅Π½Ρ‹ прСдохранитСлями. Если ΠΎΠ΄ΠΈΠ½ ΠΈΠ»ΠΈ нСсколько ΠΈΠ· этих кондСнсаторов Π²Ρ‹ΠΉΠ΄ΡƒΡ‚ ΠΈΠ· строя, это ΠΏΡ€ΠΈΠ²Π΅Π΄Π΅Ρ‚ ΠΊ нСэффСктивности систСмы, скорСС всСго, увСличатся счСта Π·Π° ΠΊΠΎΠΌΠΌΡƒΠ½Π°Π»ΡŒΠ½Ρ‹Π΅ услуги ΠΈ ΠΌΠΎΠ³ΡƒΡ‚ ΠΏΡ€ΠΎΠΈΠ·ΠΎΠΉΡ‚ΠΈ Π½Π΅ΠΏΡ€Π΅Π΄Π½Π°ΠΌΠ΅Ρ€Π΅Π½Π½Ρ‹Π΅ ΠΎΡ‚ΠΊΠ»ΡŽΡ‡Π΅Π½ΠΈΡ оборудования.Π’ случаС пСрСгорания прСдохранитСля кондСнсатора Π½Π΅ΠΎΠ±Ρ…ΠΎΠ΄ΠΈΠΌΠΎ ΠΈΠ·ΠΌΠ΅Ρ€ΠΈΡ‚ΡŒ ΠΏΡ€Π΅Π΄ΠΏΠΎΠ»Π°Π³Π°Π΅ΠΌΠΎΠ΅ Π·Π½Π°Ρ‡Π΅Π½ΠΈΠ΅ нСисправного кондСнсатора Π² ΠΌΠΈΠΊΡ€ΠΎΡ„Π°Ρ€Π°Π΄Π°Ρ… ΠΈ ΡƒΠ±Π΅Π΄ΠΈΡ‚ΡŒΡΡ, Ρ‡Ρ‚ΠΎ ΠΎΠ½ΠΎ находится Π² ΠΏΡ€Π΅Π΄Π΅Π»Π°Ρ… Π΄ΠΈΠ°ΠΏΠ°Π·ΠΎΠ½Π°, ΡƒΠΊΠ°Π·Π°Π½Π½ΠΎΠ³ΠΎ Π½Π° кондСнсаторС.

    Π‘Ρ‚ΠΎΠΈΡ‚ Π·Π½Π°Ρ‚ΡŒ Π½Π΅ΠΊΠΎΡ‚ΠΎΡ€Ρ‹Π΅ Π΄ΠΎΠΏΠΎΠ»Π½ΠΈΡ‚Π΅Π»ΡŒΠ½Ρ‹Π΅ Ρ„Π°ΠΊΡ‚ΠΎΡ€Ρ‹, связанныС с Π΅ΠΌΠΊΠΎΡΡ‚ΡŒΡŽ:

    • ΠšΠΎΠ½Π΄Π΅Π½ΡΠ°Ρ‚ΠΎΡ€Ρ‹ ΠΈΠΌΠ΅ΡŽΡ‚ ΠΎΠ³Ρ€Π°Π½ΠΈΡ‡Π΅Π½Π½Ρ‹ΠΉ срок слуТбы ΠΈ часто ΡΠ²Π»ΡΡŽΡ‚ΡΡ ΠΏΡ€ΠΈΡ‡ΠΈΠ½ΠΎΠΉ нСисправности.
    • НСисправныС кондСнсаторы ΠΌΠΎΠ³ΡƒΡ‚ ΠΈΠΌΠ΅Ρ‚ΡŒ ΠΊΠΎΡ€ΠΎΡ‚ΠΊΠΎΠ΅ Π·Π°ΠΌΡ‹ΠΊΠ°Π½ΠΈΠ΅, ΠΎΠ±Ρ€Ρ‹Π² Ρ†Π΅ΠΏΠΈ ΠΈΠ»ΠΈ ΠΌΠΎΠ³ΡƒΡ‚ физичСски ΠΈΡΠΏΠΎΡ€Ρ‚ΠΈΡ‚ΡŒΡΡ Π΄ΠΎ Ρ‚ΠΎΡ‡ΠΊΠΈ ΠΎΡ‚ΠΊΠ°Π·Π°.
    • ΠŸΡ€ΠΈ ΠΊΠΎΡ€ΠΎΡ‚ΠΊΠΎΠΌ Π·Π°ΠΌΡ‹ΠΊΠ°Π½ΠΈΠΈ кондСнсатора ΠΌΠΎΠΆΠ΅Ρ‚ ΠΏΠ΅Ρ€Π΅Π³ΠΎΡ€Π΅Ρ‚ΡŒ ΠΏΡ€Π΅Π΄ΠΎΡ…Ρ€Π°Π½ΠΈΡ‚Π΅Π»ΡŒ ΠΈΠ»ΠΈ ΠΏΠΎΠ²Ρ€Π΅Π΄ΠΈΡ‚ΡŒΡΡ Π΄Ρ€ΡƒΠ³ΠΈΠ΅ ΠΊΠΎΠΌΠΏΠΎΠ½Π΅Π½Ρ‚Ρ‹.
    • Когда кондСнсатор открываСтся ΠΈΠ»ΠΈ ΠΈΠ·Π½Π°ΡˆΠΈΠ²Π°Π΅Ρ‚ΡΡ, Ρ†Π΅ΠΏΡŒ ΠΈΠ»ΠΈ Π΅Π΅ ΠΊΠΎΠΌΠΏΠΎΠ½Π΅Π½Ρ‚Ρ‹ ΠΌΠΎΠ³ΡƒΡ‚ Π½Π΅ Ρ€Π°Π±ΠΎΡ‚Π°Ρ‚ΡŒ.
    • Износ Ρ‚Π°ΠΊΠΆΠ΅ ΠΌΠΎΠΆΠ΅Ρ‚ ΠΈΠ·ΠΌΠ΅Π½ΠΈΡ‚ΡŒ Π·Π½Π°Ρ‡Π΅Π½ΠΈΠ΅ Смкости кондСнсатора, Ρ‡Ρ‚ΠΎ ΠΌΠΎΠΆΠ΅Ρ‚ Π²Ρ‹Π·Π²Π°Ρ‚ΡŒ ΠΏΡ€ΠΎΠ±Π»Π΅ΠΌΡ‹

    Как ΠΏΡ€ΠΎΠ²Π΅Ρ€ΠΈΡ‚ΡŒ кондСнсатор? ИспользованиС Ρ€Π°Π·Π»ΠΈΡ‡Π½Ρ‹Ρ… ΠΌΠ΅Ρ‚ΠΎΠ΄ΠΎΠ² β€” всС ΠΎ Ρ‚Π΅Ρ…Π½ΠΈΠΊΠ΅

    Как ΠΏΡ€ΠΎΠ²Π΅Ρ€ΠΈΡ‚ΡŒ кондСнсатор с ΠΏΠΎΠΌΠΎΡ‰ΡŒΡŽ ΠΌΡƒΠ»ΡŒΡ‚ΠΈΠΌΠ΅Ρ‚Ρ€Π°? Π Π°Π·Π»ΠΈΡ‡Π½Ρ‹Π΅ ΠΌΠ΅Ρ‚ΠΎΠ΄Ρ‹ ΠΏΡ€ΠΎΠ²Π΅Ρ€ΠΊΠΈ кондСнсаторов

    Π’ элСктронных схСмах кондСнсатор являСтся ΠΎΠ΄Π½ΠΈΠΌ ΠΈΠ· Π½Π°ΠΈΠ±ΠΎΠ»Π΅Π΅ часто ΠΈΡΠΏΠΎΠ»ΡŒΠ·ΡƒΠ΅ΠΌΡ‹Ρ… ΠΊΠΎΠΌΠΏΠΎΠ½Π΅Π½Ρ‚ΠΎΠ².ΠŸΡ€ΠΈ устранСнии Π½Π΅ΠΏΠΎΠ»Π°Π΄ΠΎΠΊ Π² Ρ‚Π°ΠΊΠΈΡ… цСпях Π½Π΅ΠΎΠ±Ρ…ΠΎΠ΄ΠΈΠΌΠΎ Π·Π½Π°Ρ‚ΡŒ , ΠΊΠ°ΠΊ ΠΏΡ€ΠΎΠ²Π΅Ρ€ΠΈΡ‚ΡŒ кондСнсатор .

    Π’ этой ΡΡ‚Π°Ρ‚ΡŒΠ΅ ΠΌΡ‹ обсудим, ΠΊΠ°ΠΊ ΠΏΡ€ΠΎΠ²Π΅Ρ€ΠΈΡ‚ΡŒ кондСнсатор Π½Π° исправный, ΠΊΠΎΡ€ΠΎΡ‚ΠΊΠΎΠ·Π°ΠΌΠΊΠ½ΡƒΡ‚Ρ‹ΠΉ ΠΈΠ»ΠΈ Ρ€Π°Π·ΠΎΠΌΠΊΠ½ΡƒΡ‚Ρ‹ΠΉ состояниС Ρ€Π°Π·Π½Ρ‹ΠΌΠΈ ΠΌΠ΅Ρ‚ΠΎΠ΄Π°ΠΌΠΈ.

    ΠŸΠ΅Ρ€Π΅Π΄ ΠΏΡ€ΠΎΠ²Π΅Ρ€ΠΊΠΎΠΉ кондСнсатора Π½Π΅ΠΎΠ±Ρ…ΠΎΠ΄ΠΈΠΌΠΎ ΡƒΠ·Π½Π°Ρ‚ΡŒ ΠΎ самом кондСнсаторС.

    ΠšΠΎΠ½Π΄Π΅Π½ΡΠ°Ρ‚ΠΎΡ€

    ΠšΠΎΠ½Π΄Π΅Π½ΡΠ°Ρ‚ΠΎΡ€ прСдставляСт собой элСктронный ΠΊΠΎΠΌΠΏΠΎΠ½Π΅Π½Ρ‚ с двумя Π²Ρ‹Π²ΠΎΠ΄Π°ΠΌΠΈ, способный Π½Π°ΠΊΠ°ΠΏΠ»ΠΈΠ²Π°Ρ‚ΡŒ заряд Π² элСктричСском ΠΏΠΎΠ»Π΅. Он состоит ΠΈΠ· Π΄Π²ΡƒΡ… мСталличСских пластин, Ρ€Π°Π·Π΄Π΅Π»Π΅Π½Π½Ρ‹Ρ… срСдой, извСстной ΠΊΠ°ΠΊ диэлСктрик .

    Когда кондСнсатор ΠΏΠΎΠ΄ΠΊΠ»ΡŽΡ‡Π΅Π½ ΠΊ Π±Π°Ρ‚Π°Ρ€Π΅Π΅, ΠΌΠ΅ΠΆΠ΄Ρƒ мСталличСскими пластинами Π²ΠΎΠ·Π½ΠΈΠΊΠ°Π΅Ρ‚ элСктричСскоС ΠΏΠΎΠ»Π΅. Благодаря этому элСктричСскому полю мСталличСскиС пластины Π½Π°ΠΊΠ°ΠΏΠ»ΠΈΠ²Π°ΡŽΡ‚ заряд.

    Π‘ΠΏΠΎΡΠΎΠ±Π½ΠΎΡΡ‚ΡŒ кондСнсатора Π½Π°ΠΊΠ°ΠΏΠ»ΠΈΠ²Π°Ρ‚ΡŒ заряд извСстна ΠΊΠ°ΠΊ Π΅ΠΌΠΊΠΎΡΡ‚ΡŒ . Π˜Π·ΠΌΠ΅Ρ€ΡΠ΅Ρ‚ΡΡ Π² Ρ„Π°Ρ€Π°Π΄ ΠΈ обозначаСтся ΠΊΠ°ΠΊ F ​​ .

    ΠšΠ»Π΅ΠΌΠΌΡ‹ кондСнсатора

    Π•ΡΡ‚ΡŒ Π΄Π²Π° Π²Ρ‹Π²ΠΎΠ΄Π° кондСнсатора i.Π΅. ΠΏΠΎΠ»ΠΎΠΆΠΈΡ‚Π΅Π»ΡŒΠ½Π°Ρ ΠΈ ΠΎΡ‚Ρ€ΠΈΡ†Π°Ρ‚Π΅Π»ΡŒΠ½Π°Ρ ΠΊΠ»Π΅ΠΌΠΌΡ‹, Ρ‚Π°ΠΊΠΆΠ΅ извСстныС ΠΊΠ°ΠΊ Π°Π½ΠΎΠ΄ ΠΈ ΠΊΠ°Ρ‚ΠΎΠ΄ соотвСтствСнно.

    Π’ зависимости ΠΎΡ‚ полярности Π²Ρ‹Π²ΠΎΠ΄ΠΎΠ² Π΅ΡΡ‚ΡŒ Π΄Π²Π° Ρ‚ΠΈΠΏΠ° кондСнсаторов.

    ΠŸΠΎΠ»ΡΡ€Π½Ρ‹Π΅ кондСнсаторы ΠšΠΎΠ½Π΄Π΅Π½ΡΠ°Ρ‚ΠΎΡ€Ρ‹ Polar

    , Ρ‚Π°ΠΊΠΆΠ΅ извСстныС ΠΊΠ°ΠΊ элСктролитичСскиС кондСнсаторы , ΠΈΡΠΏΠΎΠ»ΡŒΠ·ΡƒΡŽΡ‚ элСктролит Π² качСствС ΠΎΠ΄Π½ΠΎΠ³ΠΎ ΠΈΠ· Π²Ρ‹Π²ΠΎΠ΄ΠΎΠ² для увСличСния Смкости накоплСния заряда. Он ΠΈΠΌΠ΅Π΅Ρ‚ Π±ΠΎΠ»ΡŒΡˆΡƒΡŽ Π΅ΠΌΠΊΠΎΡΡ‚ΡŒ ΠΏΠΎ ΡΡ€Π°Π²Π½Π΅Π½ΠΈΡŽ с нСполярными кондСнсаторами.

    Π•Π³ΠΎ пластины поляризованы i.Π΅. Π΄Π²Π΅ ΡƒΠ½ΠΈΠΊΠ°Π»ΡŒΠ½Ρ‹Π΅ ΠΊΠ»Π΅ΠΌΠΌΡ‹, извСстныС ΠΊΠ°ΠΊ Π°Π½ΠΎΠ΄ (ΠΏΠΎΠ»ΠΎΠΆΠΈΡ‚Π΅Π»ΡŒΠ½Ρ‹ΠΉ) ΠΈ ΠΊΠ°Ρ‚ΠΎΠ΄ (ΠΎΡ‚Ρ€ΠΈΡ†Π°Ρ‚Π΅Π»ΡŒΠ½Ρ‹ΠΉ).

    ΠŸΡ€ΠΈ использовании полярного кондСнсатора ΠΊΡ€Π°ΠΉΠ½Π΅ Π²Π°ΠΆΠ½ΠΎ ΠΏΡ€ΠΎΠ²Π΅Ρ€ΠΈΡ‚ΡŒ ΠΏΠΎΠ»ΡΡ€Π½ΠΎΡΡ‚ΡŒ Π΅Π³ΠΎ ΠΊΠ»Π΅ΠΌΠΌΡ‹ . КлСмма Π°Π½ΠΎΠ΄Π° всСгда Π΄ΠΎΠ»ΠΆΠ½Π° ΠΏΠΎΠ΄Π΄Π΅Ρ€ΠΆΠΈΠ²Π°Ρ‚ΡŒ Π±ΠΎΠ»Π΅Π΅ высокоС напряТСниС , Ρ‡Π΅ΠΌ Π΅Π΅ ΠΊΠ»Π΅ΠΌΠΌΡ‹ ΠΊΠ°Ρ‚ΠΎΠ΄ . ИзмСнСниС полярности ΠΌΠΎΠΆΠ΅Ρ‚ ΠΏΠΎΠ²Ρ€Π΅Π΄ΠΈΡ‚ΡŒ кондСнсатор ΠΈ Π΄Π°ΠΆΠ΅ Ρ€Π°Π·Ρ€ΡƒΡˆΠΈΡ‚ΡŒ Π΅Π³ΠΎ.

    ΠŸΡ€ΠΎΡ‰Π΅ говоря, всСгда ΠΏΠΎΠ΄ΠΊΠ»ΡŽΡ‡Π°ΠΉΡ‚Π΅ плюс ΠΊ ΠΏΠ»ΡŽΡΡƒ, Π° минус ΠΊ минусу аккумулятора.

    НСполярный кондСнсатор

    НСполярный кондСнсатор ΠΈΠ»ΠΈ нСполяризованный кондСнсатор Π½Π΅ ΠΈΠΌΠ΅Π΅Ρ‚ полярности . ΠœΠ΅ΠΆΠ΄Ρƒ Π΅Π³ΠΎ Ρ‚Π΅Ρ€ΠΌΠΈΠ½Π°Π»Π°ΠΌΠΈ Π½Π΅Ρ‚ Π½ΠΈΠΊΠ°ΠΊΠΎΠΉ Ρ€Π°Π·Π½ΠΈΡ†Ρ‹. Оба Π²Ρ‹Π²ΠΎΠ΄Π° ΠΌΠΎΠ³ΡƒΡ‚ Π΄Π΅ΠΉΡΡ‚Π²ΠΎΠ²Π°Ρ‚ΡŒ ΠΊΠ°ΠΊ ΠΊΠ°Ρ‚ΠΎΠ΄ ΠΈ Π°Π½ΠΎΠ΄.

    НСполярныС кондСнсаторы ΠΈΠΌΠ΅ΡŽΡ‚ ΠΎΡ‡Π΅Π½ΡŒ Π½ΠΈΠ·ΠΊΡƒΡŽ Π΅ΠΌΠΊΠΎΡΡ‚ΡŒ Π² Π΄ΠΈΠ°ΠΏΠ°Π·ΠΎΠ½Π΅ ΠΎΡ‚ Π½Π΅ΡΠΊΠΎΠ»ΡŒΠΊΠΈΡ… ΠΏΠΈΠΊΠΎΡ„Π°Ρ€Π°Π΄ Π΄ΠΎ Π½Π΅ΡΠΊΠΎΠ»ΡŒΠΊΠΈΡ… ΠΌΠΈΠΊΡ€ΠΎΡ„Π°Ρ€Π°Π΄.

    Π§ΠΈΡ‚Π°ΠΉΡ‚Π΅ Ρ‚Π°ΠΊΠΆΠ΅: ΠŸΡ€ΠΎΠ²Π΅Ρ€ΠΊΠ° транзистора для ΠΈΠ΄Π΅Π½Ρ‚ΠΈΡ„ΠΈΠΊΠ°Ρ†ΠΈΠΈ ΠΊΠ»Π΅ΠΌΠΌ, Ρ‚ΠΈΠΏΠ° ΠΈ состояния.

    НСт ΠΏΠΎΠ»ΠΎΠΆΠΈΡ‚Π΅Π»ΡŒΠ½Ρ‹Ρ… ΠΈ ΠΎΡ‚Ρ€ΠΈΡ†Π°Ρ‚Π΅Π»ΡŒΠ½Ρ‹Ρ… ΠΊΠ»Π΅ΠΌΠΌ.КлСмма, ΠΏΠΎΠ΄ΠΊΠ»ΡŽΡ‡Π΅Π½Π½Π°Ρ ΠΊ ΠΏΠΎΠ»ΠΎΠΆΠΈΡ‚Π΅Π»ΡŒΠ½ΠΎΠΉ ΠΊΠ»Π΅ΠΌΠΌΠ΅ Π±Π°Ρ‚Π°Ρ€Π΅ΠΈ, дСйствуСт ΠΊΠ°ΠΊ Π°Π½ΠΎΠ΄. Π’ Ρ‚ΠΎ врСмя ΠΊΠ°ΠΊ ΠΊΠ»Π΅ΠΌΠΌΠ°, ΠΏΠΎΠ΄ΠΊΠ»ΡŽΡ‡Π΅Π½Π½Π°Ρ ΠΊ ΠΎΡ‚Ρ€ΠΈΡ†Π°Ρ‚Π΅Π»ΡŒΠ½ΠΎΠΉ ΠΊΠ»Π΅ΠΌΠΌΠ΅ аккумулятора, дСйствуСт ΠΊΠ°ΠΊ ΠΊΠ°Ρ‚ΠΎΠ΄. ИзмСнСниС полярности Π±Π°Ρ‚Π°Ρ€Π΅ΠΈ Π½Π΅ влияСт Π½Π° кондСнсатор.

    Π’ΠΈΠ·ΡƒΠ°Π»ΡŒΠ½Π°Ρ идСнтификация ΠΊΠ»Π΅ΠΌΠΌ

    Как извСстно, нСполярныС кондСнсаторы Π½Π΅ ΠΈΠΌΠ΅ΡŽΡ‚ Ρ€Π°Π·Π½Ρ‹Ρ… Π²Ρ‹Π²ΠΎΠ΄ΠΎΠ². Π’Π°ΠΊΠΈΠΌ ΠΎΠ±Ρ€Π°Π·ΠΎΠΌ, Π½Π΅Ρ‚ нСобходимости Π² ΠΈΠ΄Π΅Π½Ρ‚ΠΈΡ„ΠΈΠΊΠ°Ρ†ΠΈΠΈ Π΅Π³ΠΎ Ρ‚Π΅Ρ€ΠΌΠΈΠ½Π°Π»ΠΎΠ².

    Однако ΠΊΡ€Π°ΠΉΠ½Π΅ Π²Π°ΠΆΠ½ΠΎ ΠΈΠ΄Π΅Π½Ρ‚ΠΈΡ„ΠΈΡ†ΠΈΡ€ΠΎΠ²Π°Ρ‚ΡŒ ΠΊΠ»Π΅ΠΌΠΌΡ‹ полярного элСктролитичСского кондСнсатора.

    ΠŸΠ΅Ρ€Π²Ρ‹ΠΉ ΠΌΠ΅Ρ‚ΠΎΠ΄

    ΠŸΡ€ΠΈ ΠΈΠ·Π³ΠΎΡ‚ΠΎΠ²Π»Π΅Π½ΠΈΠΈ Анодная Π²Π΅Ρ‚Π²ΡŒ полярного кондСнсатора дСлаСтся Π½Π° Π΄Π»ΠΈΠ½Π½Π΅Π΅ ΠΏΠΎ ΡΡ€Π°Π²Π½Π΅Π½ΠΈΡŽ с ΠΊΠ°Ρ‚ΠΎΠ΄Π½ΠΎΠΉ Π²Π΅Ρ‚Π²ΡŒΡŽ. Π­Ρ‚ΠΎΡ‚ ΠΌΠ΅Ρ‚ΠΎΠ΄ Ρ€Π°Π±ΠΎΡ‚Π°Π΅Ρ‚ Ρ‚ΠΎΠ»ΡŒΠΊΠΎ Ρ‚ΠΎΠ³Π΄Π°, ΠΊΠΎΠ³Π΄Π° кондСнсатор Π½Π΅ ΠΈΡΠΏΠΎΠ»ΡŒΠ·ΡƒΠ΅Ρ‚ΡΡ. Π’Ρ‚ΠΎΡ€ΠΎΠΉ ΠΌΠ΅Ρ‚ΠΎΠ΄ Ρ€Π°Π±ΠΎΡ‚Π°Π΅Ρ‚ ΠΊΠ°ΠΊ для Π½ΠΎΠ²Ρ‹Ρ…, Ρ‚Π°ΠΊ ΠΈ для Π±Ρ‹Π²ΡˆΠΈΡ… Π² ΡƒΠΏΠΎΡ‚Ρ€Π΅Π±Π»Π΅Π½ΠΈΠΈ кондСнсаторов.

    Π’Ρ‚ΠΎΡ€ΠΎΠΉ ΠΌΠ΅Ρ‚ΠΎΠ΄

    ΠžΡ‚Ρ€ΠΈΡ†Π°Ρ‚Π΅Π»ΡŒΠ½Π°Ρ ΠΊΠ»Π΅ΠΌΠΌΠ° кондСнсатора ΡƒΠΊΠ°Π·Π°Π½Π° Π½Π° Π΅Π³ΠΎ корпусС с ΠΌΠ°Ρ€ΠΊΠΈΡ€ΠΎΠ²ΠΊΠΎΠΉ Β« – Β», ΡƒΠΊΠ°Π·Ρ‹Π²Π°ΡŽΡ‰Π΅ΠΉ Π½Π° ΠΊΠ°Ρ‚ΠΎΠ΄Π½ΡƒΡŽ Π²Π΅Ρ‚Π²ΡŒ .

    Однако кондСнсаторы Polar SMD ΠΈΠΌΠ΅ΡŽΡ‚ ΠΌΠ°Ρ€ΠΊΠΈΡ€ΠΎΠ²ΠΊΡƒ Π½Π° ΠΏΠΎΠ»ΠΎΠΆΠΈΡ‚Π΅Π»ΡŒΠ½ΠΎΠΉ ΠΊΠ»Π΅ΠΌΠΌΠ΅ (Π°Π½ΠΎΠ΄).

    Π Π°Π·Π»ΠΈΡ‡Π½Ρ‹Π΅ ΠΌΠ΅Ρ‚ΠΎΠ΄Ρ‹ ΠΏΡ€ΠΎΠ²Π΅Ρ€ΠΊΠΈ кондСнсаторов

    Для ΠΏΡ€ΠΎΠ²Π΅Ρ€ΠΊΠΈ кондСнсатора Π½Π΅ΠΎΠ±Ρ…ΠΎΠ΄ΠΈΠΌΠΎ ΡƒΠ΄Π°Π»ΠΈΡ‚ΡŒ кондСнсатор ΠΈΠ· Π΅Π³ΠΎ Ρ†Π΅ΠΏΠΈ, Ссли ΠΎΠ½ Π΅ΡΡ‚ΡŒ Π² ΠΊΠ°ΠΊΠΎΠΉ-Π»ΠΈΠ±ΠΎ Ρ†Π΅ΠΏΠΈ. Π—Π°Ρ‚Π΅ΠΌ разрядитС кондСнсатор, Ρ‚Π°ΠΊ ΠΊΠ°ΠΊ ΠΎΠ½ ΠΌΠΎΠΆΠ΅Ρ‚ ΠΈΠΌΠ΅Ρ‚ΡŒ Π½Π΅ΠΊΠΎΡ‚ΠΎΡ€Ρ‹ΠΉ Π½Π°ΠΊΠΎΠΏΠ»Π΅Π½Π½Ρ‹ΠΉ заряд. Π­Ρ‚ΠΎ ΠΌΠΎΠΆΠ΅Ρ‚ ΠΏΠΎΠ²Ρ€Π΅Π΄ΠΈΡ‚ΡŒ вашС ΠΈΡΠΏΡ‹Ρ‚Π°Ρ‚Π΅Π»ΡŒΠ½ΠΎΠ΅ ΠΎΠ±ΠΎΡ€ΡƒΠ΄ΠΎΠ²Π°Π½ΠΈΠ΅.

    Π§Ρ‚ΠΎΠ±Ρ‹ ΠΏΡ€Π°Π²ΠΈΠ»ΡŒΠ½ΠΎ Ρ€Π°Π·Ρ€ΡΠ΄ΠΈΡ‚ΡŒ кондСнсатор, ΠΏΠΎΠ΄ΠΊΠ»ΡŽΡ‡ΠΈΡ‚Π΅ рСзистор ΠΌΠ΅ΠΆΠ΄Ρƒ Π΅Π³ΠΎ Π²Ρ‹Π²ΠΎΠ΄Π°ΠΌΠΈ.Заряд рассССтся Ρ‡Π΅Ρ€Π΅Π· рСзистор.

    ΠœΡƒΠ»ΡŒΡ‚ΠΈΠΌΠ΅Ρ‚Ρ€ β€” это ваТный инструмСнт, Π½Π΅ΠΎΠ±Ρ…ΠΎΠ΄ΠΈΠΌΡ‹ΠΉ для ΠΏΡ€ΠΎΠ²Π΅Ρ€ΠΊΠΈ кондСнсатора . НиТС ΠΎΠ±ΡΡƒΠΆΠ΄Π°ΡŽΡ‚ΡΡ Ρ€Π°Π·Π»ΠΈΡ‡Π½Ρ‹Π΅ ΠΌΠ΅Ρ‚ΠΎΠ΄Ρ‹ ΠΏΡ€ΠΎΠ²Π΅Ρ€ΠΊΠΈ кондСнсаторов с ΠΏΠΎΠΌΠΎΡ‰ΡŒΡŽ ΠΌΡƒΠ»ΡŒΡ‚ΠΈΠΌΠ΅Ρ‚Ρ€Π°.

    ΠŸΡ€ΠΎΠ²Π΅Ρ€ΠΊΠ° кондСнсатора с ΠΏΠΎΠΌΠΎΡ‰ΡŒΡŽ ΠΏΡ€ΠΎΠ²Π΅Ρ€ΠΊΠΈ цСлостности Ρ†Π΅ΠΏΠΈ

    ΠœΠ΅Ρ‚ΠΎΠ΄ ΠΏΡ€ΠΎΠ²Π΅Ρ€ΠΊΠΈ цСлостности кондСнсатора ΠΏΠΎΠΊΠ°Π·Ρ‹Π²Π°Π΅Ρ‚, являСтся Π»ΠΈ ΠΎΠ½ Ρ€Π°Π·ΠΎΠΌΠΊΠ½ΡƒΡ‚Ρ‹ΠΌ, ΠΊΠΎΡ€ΠΎΡ‚ΠΊΠΈΠΌ ΠΈΠ»ΠΈ исправным .

    • Π£Π΄Π°Π»ΠΈΡ‚Π΅ ΠΏΠΎΠ΄ΠΎΠ·Ρ€ΠΈΡ‚Π΅Π»ΡŒΠ½Ρ‹ΠΉ кондСнсатор ΠΈΠ· Π΅Π³ΠΎ Ρ†Π΅ΠΏΠΈ.
    • РазрядитС с ΠΏΠΎΠΌΠΎΡ‰ΡŒΡŽ рСзистора.
    • УстановитС ΠΌΡƒΠ»ΡŒΡ‚ΠΈΠΌΠ΅Ρ‚Ρ€ Π² Ρ€Π΅ΠΆΠΈΠΌ нСпрСрывности .
    • ΠŸΠΎΠΌΠ΅ΡΡ‚ΠΈΡ‚Π΅ красный Ρ‰ΡƒΠΏ ΠΌΡƒΠ»ΡŒΡ‚ΠΈΠΌΠ΅Ρ‚Ρ€Π° Π½Π° Π°Π½ΠΎΠ΄, Π° Ρ‡Π΅Ρ€Π½Ρ‹ΠΉ (ΠΎΠ±Ρ‰ΠΈΠΉ) Ρ‰ΡƒΠΏ Π½Π° ΠΊΠ°Ρ‚ΠΎΠ΄ кондСнсатора.
    • Если ΠΌΡƒΠ»ΡŒΡ‚ΠΈΠΌΠ΅Ρ‚Ρ€ ΠΏΠΎΠΊΠ°Π·Ρ‹Π²Π°Π΅Ρ‚ Π·Π½Π°ΠΊ нСпрСрывности ( Π·Π²ΡƒΠΊΠΎΠ²ΠΎΠΉ сигнал ΠΈΠ»ΠΈ свСтодиод ), Π° Π·Π°Ρ‚Π΅ΠΌ ΠΎΠ½ останавливаСтся (ΠΏΠΎΠΊΠ°Π·Ρ‹Π²Π°Π΅Ρ‚ OL ). Π­Ρ‚ΠΎ ΠΎΠ·Π½Π°Ρ‡Π°Π΅Ρ‚, Ρ‡Ρ‚ΠΎ кондСнсатор Ρ…ΠΎΡ€ΠΎΡˆΠΈΠΉ .

    Π’Π°ΠΊΠΆΠ΅ Ρ‡ΠΈΡ‚Π°ΠΉΡ‚Π΅: Различия ΠΌΠ΅ΠΆΠ΄Ρƒ кондСнсатором ΠΈ Π±Π°Ρ‚Π°Ρ€Π΅Π΅ΠΉ

    • Если кондСнсатор Π½Π΅ ΠΏΠΎΠΊΠ°Π·Ρ‹Π²Π°Π΅Ρ‚ Π½ΠΈΠΊΠ°ΠΊΠΈΡ… ΠΏΡ€ΠΈΠ·Π½Π°ΠΊΠΎΠ² нСпрСрывности, кондСнсатор Ρ€Π°Π·ΠΎΠΌΠΊΠ½ΡƒΡ‚ .
    • Если ΠΌΡƒΠ»ΡŒΡ‚ΠΈΠΌΠ΅Ρ‚Ρ€ ΠΈΠ·Π΄Π°Π΅Ρ‚ Π½Π΅ΠΏΡ€Π΅Ρ€Ρ‹Π²Π½Ρ‹ΠΉ Π·Π²ΡƒΠΊΠΎΠ²ΠΎΠΉ сигнал, кондСнсатор ΠΊΠΎΡ€ΠΎΡ‚ΠΈΡ‚ Β ΠΈ Ρ‚Ρ€Π΅Π±ΡƒΠ΅Ρ‚ Π·Π°ΠΌΠ΅Π½Ρ‹.
    ΠŸΡ€ΠΎΠ²Π΅Ρ€ΠΊΠ° кондСнсатора с ΠΏΠΎΠΌΠΎΡ‰ΡŒΡŽ тСста сопротивлСния

    ΠŸΡ€ΠΎΠ²Π΅Ρ€ΠΊΠ° сопротивлСния Ρ‚Π°ΠΊΠΆΠ΅ ΠΈΡΠΏΠΎΠ»ΡŒΠ·ΡƒΠ΅Ρ‚ΡΡ для ΠΏΡ€ΠΎΠ²Π΅Ρ€ΠΊΠΈ кондСнсатора. Π­Ρ‚ΠΎΡ‚ тСст ΠΌΠΎΠΆΠ΅Ρ‚ Π²Ρ‹ΠΏΠΎΠ»Π½ΡΡ‚ΡŒ ΠΊΠ°ΠΊ Ρ†ΠΈΡ„Ρ€ΠΎΠ²ΠΎΠΉ, Ρ‚Π°ΠΊ ΠΈ Π°Π½Π°Π»ΠΎΠ³ΠΎΠ²Ρ‹ΠΉ ΠΌΡƒΠ»ΡŒΡ‚ΠΈΠΌΠ΅Ρ‚Ρ€. ΠœΠ΅Ρ‚ΠΎΠ΄ остаСтся ΠΎΠ΄ΠΈΠ½Π°ΠΊΠΎΠ²Ρ‹ΠΌ для ΠΎΠ±ΠΎΠΈΡ… ΠΌΡƒΠ»ΡŒΡ‚ΠΈΠΌΠ΅Ρ‚Ρ€ΠΎΠ².

    • Π£Π΄Π°Π»ΠΈΡ‚Π΅ кондСнсатор ΠΈΠ· Ρ†Π΅ΠΏΠΈ.
    • РазрядитС кондСнсатор с ΠΏΠΎΠΌΠΎΡ‰ΡŒΡŽ рСзистора.
    • УстановитС Ρ€ΡƒΡ‡ΠΊΡƒ ΠΌΡƒΠ»ΡŒΡ‚ΠΈΠΌΠ΅Ρ‚Ρ€Π° Π² Ρ€Π΅ΠΆΠΈΠΌ высокого сопротивлСния (Π²Ρ‹ΡˆΠ΅ 10 кОм).
    • ΠŸΠΎΠΌΠ΅ΡΡ‚ΠΈΡ‚Π΅ красный Ρ‰ΡƒΠΏ Π½Π° Π°Π½ΠΎΠ΄Π½ΡƒΡŽ ΠΊΠ»Π΅ΠΌΠΌΡƒ ΠΈ Ρ‡Π΅Ρ€Π½Ρ‹ΠΉ Ρ‰ΡƒΠΏ Π½Π° ΠΊΠ°Ρ‚ΠΎΠ΄Π½ΡƒΡŽ ΠΊΠ»Π΅ΠΌΠΌΡƒ кондСнсатора.
    • Показания сопротивлСния Π΄ΠΎΠ»ΠΆΠ½Ρ‹ Π½Π°Ρ‡ΠΈΠ½Π°Ρ‚ΡŒΡΡ с Π½Π΅ΠΊΠΎΡ‚ΠΎΡ€ΠΎΠΉ Ρ‚ΠΎΡ‡ΠΊΠΈ посСрСдинС ΠΈ Π½Π°Ρ‡ΠΈΠ½Π°Ρ‚ΡŒΡΡ с , увСличивая Π΄ΠΎ бСсконСчности . Π­Ρ‚ΠΎ ΠΏΠΎΠΊΠ°Π·Ρ‹Π²Π°Π΅Ρ‚, Ρ‡Ρ‚ΠΎ кондСнсатор Ρ…ΠΎΡ€ΠΎΡˆΠΈΠΉ .

    Π’Π°ΠΊΠΆΠ΅ Ρ‡ΠΈΡ‚Π°ΠΉΡ‚Π΅: Как ΠΏΡ€ΠΎΠ²Π΅Ρ€ΠΈΡ‚ΡŒ Π΄ΠΈΠΎΠ΄ ΠΈ ΠΌΠ΅Ρ‚ΠΎΠ΄Ρ‹ тСстирования Π΄ΠΈΠΎΠ΄Π°, свСтодиода ΠΈ стабилитрона

    • Если кондСнсатор ΠΏΠΎΠΊΠ°Π·Ρ‹Π²Π°Π΅Ρ‚ высокоС сопротивлСниС Π΄Π°ΠΆΠ΅ послС разряда, кондСнсатор Ρ€Π°Π·ΠΎΠΌΠΊΠ½ΡƒΡ‚ .
    • Если кондСнсатор ΠΏΠΎΠΊΠ°Π·Ρ‹Π²Π°Π΅Ρ‚ 0 ΠΈΠ»ΠΈ ΠΎΡ‡Π΅Π½ΡŒ Π½ΠΈΠ·ΠΊΠΎΠ΅ сопротивлСниС, это ΠΊΠΎΡ€ΠΎΡ‚ΠΊΠΎΠ΅ Π·Π°ΠΌΡ‹ΠΊΠ°Π½ΠΈΠ΅ .

    ΠŸΡ€ΠΈΡ‡ΠΈΠ½Π° увСличСния сопротивлСния Π² Ρ‚ΠΎΠΌ, Ρ‡Ρ‚ΠΎ ΠΈΠ·Π½Π°Ρ‡Π°Π»ΡŒΠ½ΠΎ кондСнсатор заряТал ΠΎΡ‚ ΠΌΡƒΠ»ΡŒΡ‚ΠΈΠΌΠ΅Ρ‚Ρ€Π°. Π’Π°ΠΊΠΈΠΌ ΠΎΠ±Ρ€Π°Π·ΠΎΠΌ, ΠΎΠ½ позволяСт Ρ‚ΠΎΠΊΡƒ ΠΏΡ€ΠΎΡ‚Π΅ΠΊΠ°Ρ‚ΡŒ (Π² этом случаС ΠΎΠΌΠΌΠ΅Ρ‚Ρ€ измСряСт сопротивлСниС ). Когда кондСнсатор ΠΏΠΎΠ»ΡƒΡ‡ΠΈΠ» ΠΏΠΎΠ»Π½ΠΎΡΡ‚ΡŒΡŽ заряТСнный , ΠΎΠ½ большС Π½Π΅ пропускал Ρ‚ΠΎΠΊ. Из-Π·Π° Ρ‡Π΅Π³ΠΎ ΠΎΠ½ выглядит ΠΊΠ°ΠΊ ΠΎΡ‚ΠΊΡ€Ρ‹Ρ‚Ρ‹ΠΉ ΠΏΡƒΡ‚ΡŒ ( бСсконСчноС сопротивлСниС )

    ΠŸΡ€ΠΎΠ²Π΅Ρ€ΠΊΠ° кондСнсатора Π² Смкостном Ρ€Π΅ΠΆΠΈΠΌΠ΅

    Π Π΅ΠΆΠΈΠΌ Смкости β€” это ΡƒΠ½ΠΈΠΊΠ°Π»ΡŒΠ½Ρ‹ΠΉ Ρ€Π΅ΠΆΠΈΠΌ Ρ†ΠΈΡ„Ρ€ΠΎΠ²Ρ‹Ρ… ΠΌΡƒΠ»ΡŒΡ‚ΠΈΠΌΠ΅Ρ‚Ρ€ΠΎΠ², ΠΈΡΠΏΠΎΠ»ΡŒΠ·ΡƒΠ΅ΠΌΡ‹ΠΉ для измСрСния Смкости.Если Π²Ρ‹ Ρ…ΠΎΡ‚ΠΈΡ‚Π΅ ΠΏΡ€ΠΎΠ²Π΅Ρ€ΠΈΡ‚ΡŒ кондСнсатор с ΠΏΠΎΠΌΠΎΡ‰ΡŒΡŽ этого ΠΌΠ΅Ρ‚ΠΎΠ΄Π°, Π²Π°ΠΌ Π½ΡƒΠΆΠ½ΠΎ Π·Π½Π°Ρ‚ΡŒ, ΠΊΠ°ΠΊ ΡΡ‡ΠΈΡ‚Ρ‹Π²Π°Ρ‚ΡŒ Π·Π½Π°Ρ‡Π΅Π½ΠΈΠ΅ кондСнсатора.

    Как ΠΏΡ€ΠΎΡ‡ΠΈΡ‚Π°Ρ‚ΡŒ Π·Π½Π°Ρ‡Π΅Π½ΠΈΠ΅ кондСнсатора:

    ЭлСктролитичСский кондСнсатор ΠΎΠ±Ρ‹Ρ‡Π½ΠΎ ΡƒΠΊΠ°Π·Ρ‹Π²Π°Π΅Ρ‚ ΠΏΠΎΠ»Π½ΠΎΠ΅ Π·Π½Π°Ρ‡Π΅Π½ΠΈΠ΅, ΠΊΠ°ΠΊ ΠΏΠΎΠΊΠ°Π·Π°Π½ΠΎ Π½Π° рисункС Π½ΠΈΠΆΠ΅.

    Однако Π·Π½Π°Ρ‡Π΅Π½ΠΈΠ΅ кСрамичСского кондСнсатора записано Π² ΠΊΠΎΠ΄Π΅. Π’Ρ‹ ΠΌΠΎΠΆΠ΅Ρ‚Π΅ ΠΏΡ€Π΅ΠΎΠ±Ρ€Π°Π·ΠΎΠ²Π°Ρ‚ΡŒ/Ρ€Π°ΡΡˆΠΈΡ„Ρ€ΠΎΠ²Π°Ρ‚ΡŒ Π΅Π³ΠΎ, ΠΈΡΠΏΠΎΠ»ΡŒΠ·ΡƒΡ Π΅Π³ΠΎ особый ΠΌΠ΅Ρ‚ΠΎΠ΄. ΠŸΡ€ΠΈΠΌΠ΅Ρ€ чтСния кСрамичСского кондСнсатора ΠΏΡ€ΠΈΠ²Π΅Π΄Π΅Π½ Π½ΠΈΠΆΠ΅.

    ΠšΠ΅Ρ€Π°ΠΌΠΈΡ‡Π΅ΡΠΊΠΈΠΉ кондСнсатор ΠΈΠΌΠ΅Π΅Ρ‚ Π½ΠΎΠΌΠ΅Ρ€ 103 .

    • ΠŸΠ΅Ρ€Π²Ρ‹Π΅ Π΄Π²Π΅ Ρ†ΠΈΡ„Ρ€Ρ‹ ΡΠ²Π»ΡΡŽΡ‚ΡΡ Π·Π½Π°Ρ‡Π°Ρ‰ΠΈΠΌΠΈ Ρ†ΠΈΡ„Ρ€Π°ΠΌΠΈ ΠΈ Π·Π°ΠΏΠΈΡΡ‹Π²Π°ΡŽΡ‚ΡΡ ΠΊΠ°ΠΊ Π΅ΡΡ‚ΡŒ. НапримСр, 10 .
    • Π’Ρ€Π΅Ρ‚ΡŒΡ Ρ†ΠΈΡ„Ρ€Π° β€˜ 3 ’ ΠΏΠΎΠΊΠ°Π·Ρ‹Π²Π°Π΅Ρ‚ ΠΌΠ½ΠΎΠΆΠΈΡ‚Π΅Π»ΡŒ 10 3 . Π’Π°ΠΊΠΈΠΌ ΠΎΠ±Ρ€Π°Π·ΠΎΠΌ, общая Π΅ΠΌΠΊΠΎΡΡ‚ΡŒ Ρ€Π°Π²Π½Π° 10*10 3 , Ρ‡Ρ‚ΠΎ Ρ€Π°Π²Π½ΠΎ 10000 ΠΏΠ€ .
    • ΠšΠ΅Ρ€Π°ΠΌΠΈΡ‡Π΅ΡΠΊΠΈΠ΅ кондСнсаторы ΠΈΠ·ΠΌΠ΅Ρ€ΡΡŽΡ‚ΡΡ Π² ΠΏΠΈΠΊΠΎΡ„Π°Ρ€Π°Π΄Π°Ρ… 10 -12 F .
    • Π˜Ρ‚Π°ΠΊ, Π΅ΠΌΠΊΠΎΡΡ‚ΡŒ этого кондСнсатора Ρ€Π°Π²Π½Π° 10 Π½Π€ .

    Π‘Π»Π΅Π΄ΡƒΡŽΡ‰ΠΈΠΌ шагом Π±ΡƒΠ΄Π΅Ρ‚ Π½Π°ΠΉΡ‚ΠΈ допуск . Π­Ρ‚ΠΎ Π΄Π°Π΅Ρ‚ ΠΌΠΈΠ½ΠΈΠΌΠ°Π»ΡŒΠ½Ρ‹ΠΉ ΠΈ ΠΌΠ°ΠΊΡΠΈΠΌΠ°Π»ΡŒΠ½Ρ‹ΠΉ Π΄ΠΈΠ°ΠΏΠ°Π·ΠΎΠ½, Π² ΠΊΠΎΡ‚ΠΎΡ€ΠΎΠΌ Π΅ΠΌΠΊΠΎΡΡ‚ΡŒ ΠΌΠΎΠΆΠ΅Ρ‚ ΠΎΡ‚Π»ΠΈΡ‡Π°Ρ‚ΡŒΡΡ ΠΎΡ‚ своСго номинального значСния.

    НСкоторыС ΠΈΠ· стандартных Π·Π½Π°Ρ‡Π΅Π½ΠΈΠΉ допуска ΡƒΠΊΠ°Π·Ρ‹Π²Π°ΡŽΡ‚ΡΡ Π±ΡƒΠΊΠ²Π°ΠΌΠΈ j, k, l, m ΠΈ n для добавлСния/вычитания ΠΏΡ€ΠΎΡ†Π΅Π½Ρ‚Π° ΠΎΡ‚ 5,10,15,20 ΠΈ 30 соотвСтствСнно.

    Π’Π΅ΠΏΠ΅Ρ€ΡŒ Π΄Π°Π²Π°ΠΉΡ‚Π΅ ΠΏΠ΅Ρ€Π΅ΠΉΠ΄Π΅ΠΌ ΠΊ тСсту измСрСния Смкости.

    • Π£Π΄Π°Π»ΠΈΡ‚Π΅ кондСнсатор ΠΈΠ· Ρ†Π΅ΠΏΠΈ.
    • РазрядитС кондСнсатор с ΠΏΠΎΠΌΠΎΡ‰ΡŒΡŽ рСзистора.
    • УстановитС ΠΌΡƒΠ»ΡŒΡ‚ΠΈΠΌΠ΅Ρ‚Ρ€ Π² Ρ€Π΅ΠΆΠΈΠΌ измСрСния Смкости .
    • НСкоторыС ΠΌΠΎΠ΄Π΅Π»ΠΈ ΠΌΡƒΠ»ΡŒΡ‚ΠΈΠΌΠ΅Ρ‚Ρ€ΠΎΠ² ΠΈΠΌΠ΅ΡŽΡ‚ ΡΠΏΠ΅Ρ†ΠΈΠ°Π»ΡŒΠ½Ρ‹Π΅ ΠΊΠ»Π΅ΠΌΠΌΡ‹ для измСрСния Смкости.

    Β 

    • ΠŸΠΎΠΌΠ΅ΡΡ‚ΠΈΡ‚Π΅ Ρ‰ΡƒΠΏΡ‹ ΠΌΡƒΠ»ΡŒΡ‚ΠΈΠΌΠ΅Ρ‚Ρ€Π° Π½Π° кондСнсатор.
    • Если измСрСнная Π΅ΠΌΠΊΠΎΡΡ‚ΡŒ соотвСтствуСт записанному Π·Π½Π°Ρ‡Π΅Π½ΠΈΡŽ (Π²ΠΊΠ»ΡŽΡ‡Π°Ρ допуск) кондСнсатора, кондСнсатор исправСн .
    ΠŸΡ€ΠΎΠ²Π΅Ρ€ΠΊΠ° кондСнсатора ΠΏΠΎ Π½Π°ΠΏΡ€ΡΠΆΠ΅Π½ΠΈΡŽ:

    Π‘ΠΏΠΎΡΠΎΠ±Π½ΠΎΡΡ‚ΡŒ кондСнсатора Π½Π°ΠΊΠ°ΠΏΠ»ΠΈΠ²Π°Ρ‚ΡŒ заряд, ΠΊΠΎΡ‚ΠΎΡ€Ρ‹ΠΉ отраТаСтся Π² Π²ΠΈΠ΄Π΅ напряТСния Π½Π° Π΅Π³ΠΎ Π²Ρ‹Π²ΠΎΠ΄Π°Ρ….

    Π­Ρ‚ΠΎΡ‚ тСст ΠΏΠΎΠΊΠ°Π·Ρ‹Π²Π°Π΅Ρ‚, ΠΌΠΎΠΆΠ΅Ρ‚ кондСнсатор ΡƒΠ΄Π΅Ρ€ΠΆΠΈΠ²Π°Ρ‚ΡŒ заряд ΠΈΠ»ΠΈ Π½Π΅Ρ‚. Если кондСнсатор Ρ…ΠΎΡ€ΠΎΡˆΠΈΠΉ , ΠΎΠ½ сохранит Π½Π΅ΠΊΠΎΡ‚ΠΎΡ€Ρ‹ΠΉ заряд. ΠΊΠΎΡ‚ΠΎΡ€Ρ‹ΠΉ Π±ΡƒΠ΄Π΅Ρ‚ ΠΎΡ‚ΠΎΠ±Ρ€Π°ΠΆΠ°Ρ‚ΡŒΡΡ ΠΊΠ°ΠΊ напряТСниС Π½Π° Π΅Π³ΠΎ ΠΊΠ»Π΅ΠΌΠΌΠ΅, ΠΈ ΠΌΡ‹ ΠΌΠΎΠΆΠ΅ΠΌ ΠΈΠ·ΠΌΠ΅Ρ€ΠΈΡ‚ΡŒ Π΅Π³ΠΎ с ΠΏΠΎΠΌΠΎΡ‰ΡŒΡŽ Π²ΠΎΠ»ΡŒΡ‚ΠΌΠ΅Ρ‚Ρ€Π° .

    ΠŸΠ΅Ρ€Π΅Π΄ ΠΏΡ€ΠΎΠ²Π΅Ρ€ΠΊΠΎΠΉ кондСнсатора Π½Π° напряТСниС Π½Π΅ΠΎΠ±Ρ…ΠΎΠ΄ΠΈΠΌΠΎ ΡƒΠ·Π½Π°Ρ‚ΡŒ ΠΎ номинальном напряТСнии кондСнсатора.

    НоминальноС напряТСниС кондСнсатора всСгда указываСтся рядом с Π΅Π³ΠΎ Π·Π½Π°Ρ‡Π΅Π½ΠΈΠ΅ΠΌ Смкости, ΠΊΠ°ΠΊ ΠΏΠΎΠΊΠ°Π·Π°Π½ΠΎ Π½Π° рисункС Π½ΠΈΠΆΠ΅.

    ΠŸΡ€ΠΈ зарядкС кондСнсатора аккумулятором напряТСниС аккумулятора Π΄ΠΎΠ»ΠΆΠ½ΠΎ Π±Ρ‹Ρ‚ΡŒ Π½ΠΈΠΆΠ΅ номинального напряТСния кондСнсатора. Π’ ΠΏΡ€ΠΎΡ‚ΠΈΠ²Π½ΠΎΠΌ случаС кондСнсатор ΠΏΠ΅Ρ€Π΅Π³ΠΎΡ€ΠΈΡ‚ .

    Π’ этом тСстС ΠΌΡ‹ ΠΈΡΠΏΠΎΠ»ΡŒΠ·ΡƒΠ΅ΠΌ кондСнсатор Π½ΠΎΠΌΠΈΠ½Π°Π»ΠΎΠΌ 63 Π’ с 12-Π²ΠΎΠ»ΡŒΡ‚ΠΎΠ²ΠΎΠΉ Π±Π°Ρ‚Π°Ρ€Π΅Π΅ΠΉ.

    • Π£Π΄Π°Π»ΠΈΡ‚Π΅ кондСнсатор ΠΈΠ· Ρ†Π΅ΠΏΠΈ.
    • ΠžΠΏΡ€Π΅Π΄Π΅Π»ΠΈΡ‚Π΅ ΠΊΠ»Π΅ΠΌΠΌΡ‹ ΠΈ разрядитС кондСнсатор с ΠΏΠΎΠΌΠΎΡ‰ΡŒΡŽ рСзистора.
    • ΠŸΠΎΠ΄ΡΠΎΠ΅Π΄ΠΈΠ½ΠΈΡ‚Π΅ ΠΏΠΎΠ»ΠΎΠΆΠΈΡ‚Π΅Π»ΡŒΠ½ΡƒΡŽ ΠΊΠ»Π΅ΠΌΠΌΡƒ аккумулятора ΠΊ ΠΏΠΎΠ»ΠΎΠΆΠΈΡ‚Π΅Π»ΡŒΠ½ΠΎΠΉ, Π° ΠΎΡ‚Ρ€ΠΈΡ†Π°Ρ‚Π΅Π»ΡŒΠ½ΡƒΡŽ β€” ΠΊ ΠΎΡ‚Ρ€ΠΈΡ†Π°Ρ‚Π΅Π»ΡŒΠ½ΠΎΠΉ ΠΊΠ»Π΅ΠΌΠΌΠ΅ кондСнсатора.( Π±ΡƒΠ΄ΡŒΡ‚Π΅ остороТны Β Π½Π΅ ΠΏΡ€ΠΈΠΊΠ°ΡΠ°ΠΉΡ‚Π΅ΡΡŒ Π΄Ρ€ΡƒΠ³ ΠΊ Π΄Ρ€ΡƒΠ³Ρƒ ΠΊΠ»Π΅ΠΌΠΌΠ°ΠΌΠΈ аккумулятора)

    • ΠŸΡƒΡΡ‚ΡŒ зарядит Π½Π° нСсколько сСкунд.
    • Π˜Π·Π²Π»Π΅ΠΊΠΈΡ‚Π΅ аккумулятор.
    • УстановитС ΠΌΡƒΠ»ΡŒΡ‚ΠΈΠΌΠ΅Ρ‚Ρ€ Π² Π΄ΠΈΠ°ΠΏΠ°Π·ΠΎΠ½ настройки Π²ΠΎΠ»ΡŒΡ‚ΠΌΠ΅Ρ‚Ρ€Π° постоянного Ρ‚ΠΎΠΊΠ° Π²Ρ‹ΡˆΠ΅ 12 Π²ΠΎΠ»ΡŒΡ‚.
    • Π—Π°ΠΏΠΈΡˆΠΈΡ‚Π΅ Π½Π°Ρ‡Π°Π»ΡŒΠ½ΠΎΠ΅ ΠΌΠ³Π½ΠΎΠ²Π΅Π½Π½ΠΎΠ΅ Π·Π½Π°Ρ‡Π΅Π½ΠΈΠ΅ напряТСния кондСнсатора.

    • Ссли показания близятся ΠΊ 12 Π²ΠΎΠ»ΡŒΡ‚Π°ΠΌ, кондСнсатор исправСн .
    • Если показания напряТСния Π½Π°ΠΌΠ½ΠΎΠ³ΠΎ Π½ΠΈΠΆΠ΅ 12 Π²ΠΎΠ»ΡŒΡ‚, кондСнсатор нСисправСн ΠΈ Π½Π΅ ΠΌΠΎΠΆΠ΅Ρ‚ Π½Π°ΠΊΠΎΠΏΠΈΡ‚ΡŒ достаточный заряд.
    Как ΠΏΡ€ΠΎΠ²Π΅Ρ€ΠΈΡ‚ΡŒ кондСнсатор, рассчитав Π΅Π³ΠΎ ΠΏΠΎΡΡ‚ΠΎΡΠ½Π½ΡƒΡŽ Π²Ρ€Π΅ΠΌΠ΅Π½ΠΈ RC

    ΠŸΠΎΡΡ‚ΠΎΡΠ½Π½Π°Ρ Π²Ρ€Π΅ΠΌΠ΅Π½ΠΈ RC (обозначаСтся грСчСским словом Ρ‚Π°Ρƒ «τ» ) β€” это врСмя, Π² Ρ‚Π΅Ρ‡Π΅Π½ΠΈΠ΅ ΠΊΠΎΡ‚ΠΎΡ€ΠΎΠ³ΠΎ кондСнсатор заряТаСтся Π΄ΠΎ 63,2% ΠΏΡ€ΠΈΠ»ΠΎΠΆΠ΅Π½Π½ΠΎΠ³ΠΎ напряТСния.

    ΠŸΠΎΡΡ‚ΠΎΡΠ½Π½Π°Ρ Π²Ρ€Π΅ΠΌΠ΅Π½ΠΈ Ο„ рассчитываСтся ΠΊΠ°ΠΊ сопротивлСниС ΡƒΠΌΠ½ΠΎΠΆΠΈΡ‚ΡŒ Π½Π° Π΅ΠΌΠΊΠΎΡΡ‚ΡŒ :

    Ο„ = RC

    Π’ этом ΡƒΡ€Π°Π²Π½Π΅Π½ΠΈΠΈ рСзистор R ΠΈΠΌΠ΅Π΅Ρ‚ извСстноС Π·Π½Π°Ρ‡Π΅Π½ΠΈΠ΅, ΠΈ ΠΌΡ‹ ΠΈΠ·ΠΌΠ΅Ρ€ΠΈΠΌ Ο„ Π²ΠΎ врСмя этого тСста.

    Π’ этом тСстС ΠΌΡ‹ ΠΈΡΠΏΠΎΠ»ΡŒΠ·ΡƒΠ΅ΠΌ Π±Π°Ρ‚Π°Ρ€Π΅ΡŽ 12 Π’ с рСзистором 10 кОм . ΠœΡ‹ соСдинили ΠΈΡ… ΠΏΠΎΡΠ»Π΅Π΄ΠΎΠ²Π°Ρ‚Π΅Π»ΡŒΠ½ΠΎ с кондСнсатором. ΠœΡ‹ ΠΈΡΠΏΠΎΠ»ΡŒΠ·ΡƒΠ΅ΠΌ Π²ΠΎΠ»ΡŒΡ‚ΠΌΠ΅Ρ‚Ρ€ для измСрСния напряТСния Π½Π° кондСнсаторС ΠΈ сСкундомСр для измСрСния Π²Ρ€Π΅ΠΌΠ΅Π½ΠΈ.

    • НастройтС Ρ†Π΅ΠΏΡŒ , ΠΊΠ°ΠΊ ΡƒΠΊΠ°Π·Π°Π½ΠΎ Π½ΠΈΠΆΠ΅.
    • ΠŸΠΎΠ΄ΡΠΎΠ΅Π΄ΠΈΠ½ΠΈΡ‚Π΅ ΠΊΠ»Π΅ΠΌΠΌΡ‹ аккумулятора, Ρ‡Ρ‚ΠΎΠ±Ρ‹ Π½Π°Ρ‡Π°Ρ‚ΡŒ зарядку кондСнсатора.
    • ЗапуститС сСкундомСр, ΠΊΠ°ΠΊ Ρ‚ΠΎΠ»ΡŒΠΊΠΎ Π²Ρ‹ ΠΏΠΎΠ΄ΠΊΠ»ΡŽΡ‡ΠΈΡ‚Π΅ ΠΊΠ»Π΅ΠΌΠΌΡ‹ аккумулятора.
    • ΠŸΡ€ΠΎΠ²Π΅Ρ€ΡŒΡ‚Π΅ показания напряТСния с ΠΏΠΎΠΌΠΎΡ‰ΡŒΡŽ Π²ΠΎΠ»ΡŒΡ‚ΠΌΠ΅Ρ‚Ρ€Π°.
    • Как Ρ‚ΠΎΠ»ΡŒΠΊΠΎ ΠΎΠ½ достигаСт 63,2% ΠΈΠ· 12v (Ρ‡Ρ‚ΠΎ составляСт 7,5v ). ЗасСкитС врСмя Π½Π° сСкундомСрС.

    Π’Π°ΠΊΠΆΠ΅ Ρ‡ΠΈΡ‚Π°ΠΉΡ‚Π΅: Π¦ΠΈΡ„Ρ€ΠΎΠ²ΠΎΠΉ логичСский Π²Π΅Π½Ρ‚ΠΈΠ»ΡŒ И-НЕ (ΡƒΠ½ΠΈΠ²Π΅Ρ€ΡΠ°Π»ΡŒΠ½Ρ‹ΠΉ Π²Π΅Π½Ρ‚ΠΈΠ»ΡŒ), Π΅Π³ΠΎ символы, схСмы ΠΈ Π΄Π΅Ρ‚Π°Π»ΠΈ ИБ

    ΠŸΡ€Π΅Π΄ΠΏΠΎΠ»ΠΎΠΆΠΈΠΌ, сСкундомСр ΠΏΠΎΠΊΠ°Π·Ρ‹Π²Π°Π΅Ρ‚ 9 сСкунд .

    • Β Β Π˜ΡΠΏΠΎΠ»ΡŒΠ·ΡƒΠΉΡ‚Π΅ ΡƒΡ€Π°Π²Π½Π΅Π½ΠΈΠ΅ постоянной Π²Ρ€Π΅ΠΌΠ΅Π½ΠΈ RC для расчСта Смкости.

    Π‘ = Ο„/R

    Π‘ = 9/10 3

    Π‘ = 0,9 ΠΌΠ€ = 900 ΠΌΠΊΠ€

    • Π‘Ρ€Π°Π²Π½ΠΈΡ‚Π΅ это рассчитанноС Π·Π½Π°Ρ‡Π΅Π½ΠΈΠ΅ Смкости с ΡƒΠΊΠ°Π·Π°Π½Π½Ρ‹ΠΌ Π·Π½Π°Ρ‡Π΅Π½ΠΈΠ΅ΠΌ Смкости.
    • Если Ρ€Π°Π·Π½ΠΈΡ†Π° ΠΎΡ‡Π΅Π½ΡŒ ΠΌΠ°Π»Π°, Π²ΠΊΠ»ΡŽΡ‡Π°Ρ Π΄ΠΈΠ°ΠΏΠ°Π·ΠΎΠ½ допуска ΠΎΡ‚ 10% Π΄ΠΎ 20%. ΠšΠΎΠ½Π΄Π΅Π½ΡΠ°Ρ‚ΠΎΡ€ Ρ…ΠΎΡ€ΠΎΡˆΠΈΠΉ .
    • Если рассчитанноС Π·Π½Π°Ρ‡Π΅Π½ΠΈΠ΅ Смкости слишком ΠΌΠ°Π»ΠΎ, Ρ‡Π΅ΠΌ Π·Π°Π΄Π°Π½Π½ΠΎΠ΅ Π·Π½Π°Ρ‡Π΅Π½ΠΈΠ΅. кондСнсатор ΠΏΠ»ΠΎΡ…ΠΎΠΉ .
    Π’ΠΈΠ·ΡƒΠ°Π»ΡŒΠ½Π°Ρ ΠΏΡ€ΠΎΠ²Π΅Ρ€ΠΊΠ° кондСнсатора

    Π’Ρ‹ ΠΌΠΎΠΆΠ΅Ρ‚Π΅ ΠΎΠΏΡ€Π΅Π΄Π΅Π»ΠΈΡ‚ΡŒ нСисправный кондСнсатор, просто наблюдая Π·Π° Π΅Π³ΠΎ ΠΏΡ€ΠΈΠ·Π½Π°ΠΊΠ°ΠΌΠΈ.

    НСисправный ΠΈΠ»ΠΈ ΠΏΠΎΠ²Ρ€Π΅ΠΆΠ΄Π΅Π½Π½Ρ‹ΠΉ кондСнсатор Π±ΡƒΠ΄Π΅Ρ‚ ΠΈΠΌΠ΅Ρ‚ΡŒ любой ΠΈΠ· ΡΠ»Π΅Π΄ΡƒΡŽΡ‰ΠΈΡ… ΠΏΡ€ΠΈΠ·Π½Π°ΠΊΠΎΠ².

    Π’Π΅Ρ€Ρ…Π½Π΅Π΅ Π²Ρ‹ΠΏΡƒΠΊΠ»ΠΎΠ΅ вСнтиляционноС отвСрстиС:

    Π’ элСктролитичСских кондСнсаторах Π΅ΡΡ‚ΡŒ вСнтиляционноС отвСрстиС (Π½Π΅ вСнтиляционноС отвСрстиС, Π° слабыС мСста) Π² Ρ„ΠΎΡ€ΠΌΠ΅ X, K, T свСрху. Π­Ρ‚ΠΎ сдСлано для сброса давлСния Π²ΠΎ врСмя ΠΎΡ‚ΠΊΠ°Π·Π° кондСнсатора, Ρ‡Ρ‚ΠΎΠ±Ρ‹ ΠΈΠ·Π±Π΅ΠΆΠ°Ρ‚ΡŒ поврСТдСния (Π²Π·Ρ€Ρ‹Π²Π°) Π»ΡŽΠ±Ρ‹Ρ… Π΄Ρ€ΡƒΠ³ΠΈΡ… ΠΊΠΎΠΌΠΏΠΎΠ½Π΅Π½Ρ‚ΠΎΠ².

    ΠŸΡ€ΠΈ ΠΎΡ‚ΠΊΠ°Π·Π΅ элСктролит Π²Π½ΡƒΡ‚Ρ€ΠΈ кондСнсатора выдСляСт Π³Π°Π·. Π­Ρ‚ΠΎΡ‚ Π³Π°Π· создаСт Π΄Π°Π²Π»Π΅Π½ΠΈΠ΅ ΠΈ Π»ΠΎΠΌΠ°Π΅Ρ‚ Π²Π΅Ρ€Ρ…Π½ΠΈΠΉ Π²Π΅Π½Ρ‚ΠΈΠ»ΡŒ. Π’ Ρ€Π΅Π·ΡƒΠ»ΡŒΡ‚Π°Ρ‚Π΅ ΠΈΠ½ΠΎΠ³Π΄Π° выпуклая Π²Π΅Ρ€ΡˆΠΈΠ½Π° ΠΈΠ»ΠΈ элСктролитичСский разряд . ВыдСлСния ΠΈΠΌΠ΅ΡŽΡ‚ Ρ‡Π΅Ρ€Π½Ρ‹ΠΉ, ΠΎΡ€Π°Π½ΠΆΠ΅Π²Ρ‹ΠΉ ΠΈΠ»ΠΈ Π±Π΅Π»Ρ‹ΠΉ Ρ†Π²Π΅Ρ‚ Π² зависимости ΠΎΡ‚ элСктролитичСских Ρ…ΠΈΠΌΠΈΠΊΠ°Ρ‚ΠΎΠ².

    Π’Ρ‹ΠΏΡƒΠΊΠ»ΠΎΠ΅ Π΄Π½ΠΎ ΠΈ поднятый футляр

    Иногда ΠΏΡ€ΠΈ Π²Ρ‹Ρ…ΠΎΠ΄Π΅ ΠΈΠ· строя кондСнсатора Π²Π΅Ρ€Ρ…Π½ΠΈΠΉ Π²Π΅Π½Ρ‚ΠΈΠ»ΡŒ Π½Π΅ ломаСтся. Π² Ρ‚Π°ΠΊΠΎΠΌ случаС Π²Π½ΡƒΡ‚Ρ€Π΅Π½Π½Π΅Π΅ Π΄Π°Π²Π»Π΅Π½ΠΈΠ΅ ΠΏΡ€ΠΎΡ…ΠΎΠ΄ΠΈΡ‚ Ρ‡Π΅Ρ€Π΅Π· Π΄Π½ΠΎ .НиТняя Ρ‡Π°ΡΡ‚ΡŒ элСктролитичСского кондСнсатора ΠΏΠΎΠΊΡ€Ρ‹Ρ‚Π° Ρ€Π΅Π·ΠΈΠ½ΠΎΠΉ . Π“Π°Π· Π²Π½ΡƒΡ‚Ρ€ΠΈ Π²Ρ‹Ρ‚Π°Π»ΠΊΠΈΠ²Π°Π΅Ρ‚ эту Ρ€Π΅Π·ΠΈΠ½Ρƒ, Π² Ρ€Π΅Π·ΡƒΠ»ΡŒΡ‚Π°Ρ‚Π΅ Ρ‡Π΅Π³ΠΎ Π΄Π½ΠΎ Π²Ρ‹ΠΏΠΈΡ€Π°Π΅Ρ‚ , Π° Ρ‚Π°ΠΊΠΆΠ΅ ΠΏΠΎΠ΄Π½ΠΈΠΌΠ°Π΅Ρ‚ корпус Π½Π°Π΄ ΠΏΠ΅Ρ‡Π°Ρ‚Π½ΠΎΠΉ ΠΏΠ»Π°Ρ‚ΠΎΠΉ.

    ΠšΠ΅Ρ€Π°ΠΌΠΈΡ‡Π΅ΡΠΊΠΈΠ΅ ΠΈ повСрхностныС кондСнсаторы

    Π’Ρ‹ ΠΌΠΎΠΆΠ΅Ρ‚Π΅ ΠΎΠΏΡ€Π΅Π΄Π΅Π»ΠΈΡ‚ΡŒ нСисправный кСрамичСский кондСнсатор ΠΏΠΎ ΡΠ»Π΅Π΄ΡƒΡŽΡ‰ΠΈΠΌ ΠΏΡ€ΠΈΠ·Π½Π°ΠΊΠ°ΠΌ.

    • ΠΈΠΌΠ΅Π΅Ρ‚ ΠΏΠΎΠ²Ρ€Π΅ΠΆΠ΄Π΅Π½Π½Ρ‹ΠΉ корпус ΠΈΠ»ΠΈ отвСрстиС Π² корпусС.
    • Π›ΡŽΠ±Π°Ρ ΠΈΠ· Π΅Π³ΠΎ Π½ΠΎΠΆΠ΅ΠΊ ΠΏΠΎΠ²Ρ€Π΅ΠΆΠ΄Π΅Π½Π° рядом с корпусом.
    • Π’Ρ€Π΅Ρ‰ΠΈΠ½Ρ‹ Π² корпусС.

    Π’Ρ‹ Ρ‚Π°ΠΊΠΆΠ΅ ΠΌΠΎΠΆΠ΅Ρ‚Π΅ ΠΏΡ€ΠΎΡ‡ΠΈΡ‚Π°Ρ‚ΡŒ:

    Как ΠΏΡ€ΠΎΠ²Π΅Ρ€ΠΈΡ‚ΡŒ кондСнсатор ΠΌΡƒΠ»ΡŒΡ‚ΠΈΠΌΠ΅Ρ‚Ρ€ΠΎΠΌ? – АйкСн Колон

    Π’Ρ‹ ΠΌΠΎΠΆΠ΅Ρ‚Π΅ ΠΈΡΠΏΠΎΠ»ΡŒΠ·ΠΎΠ²Π°Ρ‚ΡŒ Ρ€Π°Π·Π»ΠΈΡ‡Π½Ρ‹Π΅ ΠΌΠ΅Ρ‚ΠΎΠ΄Ρ‹ ΠΏΡ€ΠΎΠ²Π΅Ρ€ΠΊΠΈ кондСнсатора с ΠΏΠΎΠΌΠΎΡ‰ΡŒΡŽ Ρ†ΠΈΡ„Ρ€ΠΎΠ²ΠΎΠ³ΠΎ/Π°Π½Π°Π»ΠΎΠ³ΠΎΠ²ΠΎΠ³ΠΎ ΠΌΡƒΠ»ΡŒΡ‚ΠΈΠΌΠ΅Ρ‚Ρ€Π°. Для Π»Π°ΠΌΠΏΠΎΠ²Ρ‹Ρ… ΠΈ транзисторных источников питания ΠΈΡΠΏΠΎΠ»ΡŒΠ·ΡƒΠ΅Ρ‚ΡΡ элСктролитичСская модСль, Π° нСэлСктролитичСская модСль ΠΈΡΠΏΠΎΠ»ΡŒΠ·ΡƒΠ΅Ρ‚ΡΡ для контроля скачков постоянного Ρ‚ΠΎΠΊΠ°.

    Π€ΠΎΡ€ΠΌΠ° элСктролита ΠΌΠΎΠΆΠ΅Ρ‚ ΡƒΡ…ΡƒΠ΄ΡˆΠΈΡ‚ΡŒΡΡ ΠΈΠ·-Π·Π° разряда Π΄ΠΎΠΏΠΎΠ»Π½ΠΈΡ‚Π΅Π»ΡŒΠ½ΠΎΠ³ΠΎ Ρ‚ΠΎΠΊΠ° ΠΈΠ·-Π·Π° ΠΊΠΎΡ€ΠΎΡ‚ΠΊΠΎΠ³ΠΎ замыкания.НаиболСС распространСнная потСря нСэлСктролитичСских Ρ„ΠΎΡ€ΠΌ связана с ΡƒΡ‚Π΅Ρ‡ΠΊΠΎΠΉ Π½Π°ΠΊΠΎΠΏΠ»Π΅Π½Π½ΠΎΠ³ΠΎ заряда.

    БущСствуСт мноТСство ΠΏΠΎΠ΄Ρ…ΠΎΠ΄ΠΎΠ² ΠΊ ΠΏΡ€ΠΎΠ²Π΅Ρ€ΠΊΠ΅ кондСнсатора. Π’Π΅ΠΌ Π½Π΅ ΠΌΠ΅Π½Π΅Π΅, ΠΌΡ‹ рассмотрим , ΠΊΠ°ΠΊ ΠΏΡ€ΠΎΠ²Π΅Ρ€ΠΈΡ‚ΡŒ кондСнсатор с ΠΏΠΎΠΌΠΎΡ‰ΡŒΡŽ ΠΌΡƒΠ»ΡŒΡ‚ΠΈΠΌΠ΅Ρ‚Ρ€Π° Π² нашСй ΡΡ‚Π°Ρ‚ΡŒΠ΅. Π’Π°ΠΊ Ρ‡Ρ‚ΠΎ слСдитС Π·Π° обновлСниями Π΄ΠΎ ΠΊΠΎΠ½Ρ†Π°, Ρ‡Ρ‚ΠΎΠ±Ρ‹ ΡƒΠ·Π½Π°Ρ‚ΡŒ всС ΠΎΠ± этом.

    Π§Ρ‚ΠΎ Ρ‚Π°ΠΊΠΎΠ΅ кондСнсатор?

    Одним ΠΈΠ· Π²ΠΈΠ΄ΠΎΠ² элСктричСских Π΄Π΅Ρ‚Π°Π»Π΅ΠΉ являСтся кондСнсатор, ΠΈΡΠΏΠΎΠ»ΡŒΠ·ΡƒΠ΅ΠΌΡ‹ΠΉ для накоплСния энСргии Π² элСктричСском зарядС. Они ΠΈΡΠΏΠΎΠ»ΡŒΠ·ΡƒΡŽΡ‚ΡΡ для выполнСния Ρ€Π°Π·Π»ΠΈΡ‡Π½Ρ‹Ρ… Ρ„ΡƒΠ½ΠΊΡ†ΠΈΠΉ Π² Ρ€Π°Π·Π»ΠΈΡ‡Π½Ρ‹Ρ… элСктричСских ΠΈ ΠΊΠΎΠΌΠΏΡŒΡŽΡ‚Π΅Ρ€Π½Ρ‹Ρ… цСпях.

    МоТно ΠΏΠΎΡΡ‚Π°Π²ΠΈΡ‚ΡŒ кондСнсатор Π² Π°ΠΊΡ‚ΠΈΠ²Π½ΡƒΡŽ Ρ†Π΅ΠΏΡŒ. Π’Π°ΠΊΠΈΠΌ ΠΎΠ±Ρ€Π°Π·ΠΎΠΌ ΠΌΠΎΠΆΠ½ΠΎ ΠΎΡΡƒΡ‰Π΅ΡΡ‚Π²ΠΈΡ‚ΡŒ зарядку кондСнсатора. ЭлСктричСский заряд Π½Π°Ρ‡Π½Π΅Ρ‚ Ρ‚Π΅Ρ‡ΡŒ Ρ‡Π΅Ρ€Π΅Π· кондСнсатор, ΠΏΠΎΠΊΠ° ΠΎΠ½ Π½Π΅ Π±ΡƒΠ΄Π΅Ρ‚ присоСдинСн.

    Если пСрвичная пластина кондСнсатора Π½Π΅ ΡƒΠ΄Π΅Ρ€ΠΆΠΈΠ²Π°Π΅Ρ‚ элСктричСский заряд, вторая пластина возвращаСтся Π² Ρ†Π΅ΠΏΡŒ. ΠŸΠΎΡΡ‚ΠΎΠΌΡƒ этот ΠΌΠ΅Ρ‚ΠΎΠ΄ рассматриваСтся ΠΊΠ°ΠΊ зарядка ΠΈ разрядка Π² кондСнсаторС.

    Как ΠΏΡ€ΠΎΠ²Π΅Ρ€ΠΈΡ‚ΡŒ кондСнсатор?

    ΠœΠ½ΠΎΠ³ΠΎΡ‡ΠΈΡΠ»Π΅Π½Π½Ρ‹Π΅ Ρ„ΠΎΡ€ΠΌΡ‹ элСктротСхничСской ΠΈ ΠΊΠΎΠΌΠΏΡŒΡŽΡ‚Π΅Ρ€Π½ΠΎΠΉ ΠΏΡ€ΠΎΠ΄ΡƒΠΊΡ†ΠΈΠΈ, ΠΏΡ€ΠΈΡΡƒΡ‚ΡΡ‚Π²ΡƒΡŽΡ‰ΠΈΠ΅ Π½Π° Ρ€Ρ‹Π½ΠΊΠ΅, ΠΎΡ‚Π»ΠΈΡ‡Π°ΡŽΡ‚ΡΡ Π΄Ρ€ΡƒΠ³ ΠΎΡ‚ Π΄Ρ€ΡƒΠ³Π°.Π›ΡŽΠ±ΠΎΠΉ ΠΈΠ· Π½ΠΈΡ… ΠΎΡ‡Π΅Π½ΡŒ чувствитСлСн ΠΊ колСбаниям напряТСния. Π’ΠΎΡ‡Π½ΠΎ Ρ‚Π°ΠΊ ΠΆΠ΅ кондСнсатор часто уязвим ΠΊ колСбаниям напряТСния, поэтому Π΅ΡΡ‚ΡŒ способы ΠΏΡ€ΠΎΠ²Π΅Ρ€ΠΈΡ‚ΡŒ кондСнсатор, ΠΊΠΎΡ‚ΠΎΡ€Ρ‹Π΅ ΠΌΡ‹ обсудим.

    ΠŸΡ€ΠΎΠ²Π΅Ρ€ΠΊΠ° кондСнсатора ΠΈΠ³Ρ€Π°Π΅Ρ‚ Π²Π°ΠΆΠ½ΡƒΡŽ Ρ€ΠΎΠ»ΡŒ Π² ΠΏΡ€ΠΎΠ²Π΅Ρ€ΠΊΠ΅ работоспособности кондСнсатора для Ρ€Π΅ΡˆΠ΅Π½ΠΈΡ ΠΏΡ€ΠΎΠ±Π»Π΅ΠΌ, связанных с ΠΎΡ‚ΠΊΠ°Π·ΠΎΠΌ кондСнсатора. Π”Π°Π²Π°ΠΉΡ‚Π΅ ΡƒΠ·Π½Π°Π΅ΠΌ большС ΠΎ Ρ‚ΠΎΠΌ, ΠΊΠ°ΠΊ ΠΌΡ‹ ΠΌΠΎΠΆΠ΅ΠΌ ΠΈΠ·ΠΌΠ΅Ρ€ΠΈΡ‚ΡŒ Π΅ΠΌΠΊΠΎΡΡ‚ΡŒ с ΠΏΠΎΠΌΠΎΡ‰ΡŒΡŽ Π»ΡƒΡ‡ΡˆΠ΅Π³ΠΎ измСритСля Смкости.

    Как ΠΈΠ·ΠΌΠ΅Ρ€ΠΈΡ‚ΡŒ Π΅ΠΌΠΊΠΎΡΡ‚ΡŒ?

    Для ΠΏΡ€ΠΎΠ²Π΅Ρ€ΠΊΠΈ Ρ€Π΅Π·ΡƒΠ»ΡŒΡ‚ΠΈΡ€ΡƒΡŽΡ‰Π΅Π³ΠΎ напряТСния ΠΈΡΠΏΠΎΠ»ΡŒΠ·ΡƒΠ΅Ρ‚ΡΡ ΠΌΡƒΠ»ΡŒΡ‚ΠΈΠΌΠ΅Ρ‚Ρ€ для ΠΎΡ†Π΅Π½ΠΊΠΈ Смкости.Π˜Π·ΠΌΠ΅Ρ€ΠΈΡ‚ΡŒ ΠΌΠΎΠΆΠ½ΠΎ Ρ‡Π΅Ρ€Π΅Π· зарядный кондСнсатор. ПослС этого Π²Ρ‹ ΠΌΠΎΠΆΠ΅Ρ‚Π΅ ΠΈΡΠΏΠΎΠ»ΡŒΠ·ΠΎΠ²Π°Ρ‚ΡŒ свою Π΅ΠΌΠΊΠΎΡΡ‚ΡŒ для измСрСния с ΠΏΠΎΠΌΠΎΡ‰ΡŒΡŽ ΠΌΡƒΠ»ΡŒΡ‚ΠΈΠΌΠ΅Ρ‚Ρ€Π°.

    Π—Π΄Π΅ΡΡŒ ΠΌΡ‹ рассмотрСли, ΠΊΠ°ΠΊ ΠΈΡΠΏΠΎΠ»ΡŒΠ·ΠΎΠ²Π°Ρ‚ΡŒ ΠΌΡƒΠ»ΡŒΡ‚ΠΈΠΌΠ΅Ρ‚Ρ€ для измСрСния кондСнсатора. НачнитС с Ρ‚ΠΎΠ³ΠΎ, Ρ‡Ρ‚ΠΎ Π²ΠΎΠ·ΡŒΠΌΠΈΡ‚Π΅ Ρ†ΠΈΡ„Ρ€ΠΎΠ²ΠΎΠΉ ΠΌΡƒΠ»ΡŒΡ‚ΠΈΠΌΠ΅Ρ‚Ρ€ (Ρ†ΠΈΡ„Ρ€ΠΎΠ²ΠΎΠΉ ΠΌΡƒΠ»ΡŒΡ‚ΠΈΠΌΠ΅Ρ‚Ρ€), Ρ‡Ρ‚ΠΎΠ±Ρ‹ ΡƒΠ±Π΅Π΄ΠΈΡ‚ΡŒΡΡ, Ρ‡Ρ‚ΠΎ источник питания схСмы Π²Ρ‹ΠΊΠ»ΡŽΡ‡Π΅Π½.

    Π•ΡΡ‚ΡŒ Π΄Π²Π΅ Π²Π°ΠΆΠ½Ρ‹Π΅ Π²Π΅Ρ‰ΠΈ, ΠΊΠΎΡ‚ΠΎΡ€Ρ‹Π΅ Π²Ρ‹ Π΄ΠΎΠ»ΠΆΠ½Ρ‹ ΠΏΠΎΠΌΠ½ΠΈΡ‚ΡŒ. Если кондСнсатор ΠΈΡΠΏΠΎΠ»ΡŒΠ·ΡƒΠ΅Ρ‚ΡΡ Π² Ρ†Π΅ΠΏΠΈ ΠΏΠ΅Ρ€Π΅ΠΌΠ΅Π½Π½ΠΎΠ³ΠΎ Ρ‚ΠΎΠΊΠ°, установитС ΠΌΡƒΠ»ΡŒΡ‚ΠΈΠΌΠ΅Ρ‚Ρ€ для измСрСния напряТСния ΠΏΠ΅Ρ€Π΅ΠΌΠ΅Π½Π½ΠΎΠ³ΠΎ Ρ‚ΠΎΠΊΠ°, ΠΈΠ½Π°Ρ‡Π΅ ΠΎΠ½ Π½Π΅ Π±ΡƒΠ΄Π΅Ρ‚ Π·Π½Π°Ρ‚ΡŒ Ρ‚ΠΎΡ‡Π½Ρ‹Ρ… Ρ€Π΅Π·ΡƒΠ»ΡŒΡ‚Π°Ρ‚ΠΎΠ².

    Аналогичным ΠΎΠ±Ρ€Π°Π·ΠΎΠΌ, Ссли кондСнсатор ΠΏΠΎΠ΄ΠΊΠ»ΡŽΡ‡Π΅Π½ ΠΊ Ρ†Π΅ΠΏΠΈ постоянного Ρ‚ΠΎΠΊΠ°, установитС Ρ†ΠΈΡ„Ρ€ΠΎΠ²ΠΎΠΉ ΠΌΡƒΠ»ΡŒΡ‚ΠΈΠΌΠ΅Ρ‚Ρ€ для измСрСния напряТСния постоянного Ρ‚ΠΎΠΊΠ°. Если ΠΎΠ½ ΠΏΡ€ΠΎΠ»ΡŒΠ΅Ρ‚ΡΡ, Ρ€Π°Π·ΠΎΠ±ΡŒΠ΅Ρ‚ΡΡ ΠΈΠ»ΠΈ порвСтся, осмотритС кондСнсатор ΠΎΠ΄ΠΈΠ½ Ρ€Π°Π·, Π° Π·Π°Ρ‚Π΅ΠΌ ΠΎΡ‚Ρ€Π΅ΠΌΠΎΠ½Ρ‚ΠΈΡ€ΡƒΠΉΡ‚Π΅ кондСнсатор. Π—Π°ΠΊΡ€Π΅ΠΏΠΈΡ‚Π΅ Ρ†ΠΈΡ„Π΅Ρ€Π±Π»Π°Ρ‚ Π½Π° Π·Π½Π°ΠΊΠ΅ Смкости, рассматриваСмом ΠΊΠ°ΠΊ способ расчСта Смкости.

    Π’ качСствС Π΄ΠΎΠΏΠΎΠ»Π½ΠΈΡ‚Π΅Π»ΡŒΠ½ΠΎΠΉ Ρ„ΡƒΠ½ΠΊΡ†ΠΈΠΈ эмблСма Ρ‚Π°ΠΊΠΆΠ΅ ΠΈΠΌΠ΅Π΅Ρ‚ ΠΌΠ΅Ρ‚ΠΊΡƒ Π½Π°Π΄ Ρ†ΠΈΡ„Π΅Ρ€Π±Π»Π°Ρ‚ΠΎΠΌ. Как ΠΏΡ€Π°Π²ΠΈΠ»ΠΎ, Ρ„ΡƒΠ½ΠΊΡ†ΠΈΠΎΠ½Π°Π»ΡŒΠ½Π°Ρ ΠΊΠ½ΠΎΠΏΠΊΠ° наТимаСтся, Ρ‡Ρ‚ΠΎΠ±Ρ‹ Π²ΠΊΠ»ΡŽΡ‡ΠΈΡ‚ΡŒ ΠΈΠ·ΠΌΠ΅Ρ€Π΅Π½ΠΈΠ΅ для Ρ€Π΅Π³ΡƒΠ»ΠΈΡ€ΠΎΠ²ΠΊΠΈ Ρ†ΠΈΡ„Π΅Ρ€Π±Π»Π°Ρ‚Π°.

    ΠœΡ‹ ΠΏΡ€Π΅Π΄ΠΏΠΎΡ‡Π»ΠΈ Ρ‚Ρ€ΠΈ способа ΠΏΡ€ΠΎΠ²Π΅Ρ€ΠΊΠΈ Смкости, ΠΈ ΠΎΠ½ΠΈ ΠΎΠ±ΡΡƒΠΆΠ΄Π°ΡŽΡ‚ΡΡ Π½ΠΈΠΆΠ΅.

    Как ΠΏΡ€ΠΎΠ²Π΅Ρ€ΠΈΡ‚ΡŒ кондСнсатор с ΠΏΠΎΠΌΠΎΡ‰ΡŒΡŽ ΠΌΡƒΠ»ΡŒΡ‚ΠΈΠΌΠ΅Ρ‚Ρ€Π°:

    Π’ΠΎ врСмя тСстирования кондСнсатора ΠΏΡ€ΠΈ поискС ΠΈ устранСнии нСисправностСй элСктричСских ΠΈ элСктронных ΠΊΠΎΠΌΠΏΠΎΠ½Π΅Π½Ρ‚ΠΎΠ² ΠΌΠΎΠΆΠ΅Ρ‚ Π²ΠΎΠ·Π½ΠΈΠΊΠ½ΡƒΡ‚ΡŒ нСсколько ΠΏΡ€ΠΎΠ±Π»Π΅ΠΌ. Π—Π΄Π΅ΡΡŒ, ΠΈΡΠΏΠΎΠ»ΡŒΠ·ΡƒΡ Π°Π½Π°Π»ΠΎΠ³ΠΎΠ²Ρ‹Π΅ ΠΈ оптичСскиС ΠΌΡƒΠ»ΡŒΡ‚ΠΈΠΌΠ΅Ρ‚Ρ€Ρ‹, ΠΌΡƒΠ»ΡŒΡ‚ΠΈΠΌΠ΅Ρ‚Ρ€ ΠΌΠΎΠΆΠ΅Ρ‚ ΠΏΡ€ΠΎΠ²Π΅Ρ€ΠΈΡ‚ΡŒ кондСнсатор.

    Но ΠΌΠΎΠΆΠ½ΠΎ ΠΏΡ€ΠΎΠ²Π΅Ρ€ΠΈΡ‚ΡŒ кондСнсатор, исправСн ΠΎΠ½ ΠΈΠ»ΠΈ Π½Π΅Ρ‚. ΠŸΡ€ΠΈ использовании Ρ‚Π°ΠΊΠΎΠΉ Ρ„ΡƒΠ½ΠΊΡ†ΠΈΠΈ, ΠΊΠ°ΠΊ ΠΈΠ·ΠΌΠ΅Ρ€Π΅Π½ΠΈΠ΅ Смкости, Π·Π½Π°Ρ‡Π΅Π½ΠΈΠ΅ Смкости ΠΌΠΎΠΆΠ½ΠΎ ΠΏΡ€ΠΎΠ²Π΅Ρ€ΠΈΡ‚ΡŒ с ΠΏΠΎΠΌΠΎΡ‰ΡŒΡŽ Ρ†ΠΈΡ„Ρ€ΠΎΠ²ΠΎΠ³ΠΎ ΠΌΡƒΠ»ΡŒΡ‚ΠΈΠΌΠ΅Ρ‚Ρ€Π°.

    Как ΠΏΡ€Π°Π²ΠΈΠ»ΠΎ, для измСрСния кондСнсатора Ρ‚Ρ€Π΅Π±ΡƒΡŽΡ‚ΡΡ Ρ€Π°Π·Π»ΠΈΡ‡Π½Ρ‹Π΅ ΠΌΠ΅Ρ‚ΠΎΠ΄Ρ‹, Ρ‚Π°ΠΊΠΈΠ΅ ΠΊΠ°ΠΊ Π°Π½Π°Π»ΠΎΠ³ΠΎΠ²Ρ‹ΠΉ, ΠΈΠ½Ρ‚Π΅Ρ€Π°ΠΊΡ‚ΠΈΠ²Π½Ρ‹ΠΉ, Π²ΠΎΠ»ΡŒΡ‚ΠΌΠ΅Ρ‚Ρ€, ΠΌΡƒΠ»ΡŒΡ‚ΠΈΠΌΠ΅Ρ‚Ρ€ с двумя Ρ€Π΅ΠΆΠΈΠΌΠ°ΠΌΠΈ, Ρ‚Π°ΠΊΠΈΠΌΠΈ ΠΊΠ°ΠΊ Ρ€Π΅ΠΆΠΈΠΌ измСрСния Смкости, Ρ€Π΅ΠΆΠΈΠΌ ΠΎΠΌΠΌΠ΅Ρ‚Ρ€Π° ΠΈ обычная искровая систСма.

    ΠŸΡ€ΠΈ ΠΏΡ€ΠΎΠ²Π΅Ρ€ΠΊΠ΅ кондСнсатора эти ΠΏΠΎΠ΄Ρ…ΠΎΠ΄Ρ‹ ΠΈΠ³Ρ€Π°ΡŽΡ‚ Π²Π°ΠΆΠ½ΡƒΡŽ Ρ€ΠΎΠ»ΡŒ Π² ΠΎΠΏΡ€Π΅Π΄Π΅Π»Π΅Π½ΠΈΠΈ Ρ‚ΠΎΠ³ΠΎ, исправСн Π»ΠΈ кондСнсатор, доступСн Π»ΠΈ ΠΎΠ½, нСисправСн, ΠΊΠΎΡ€ΠΎΡ‚ΠΊΠΎΠ΅ Π·Π°ΠΌΡ‹ΠΊΠ°Π½ΠΈΠ΅ ΠΈΠ»ΠΈ разряТСн.

    Но, ΠΏΡ€Π΅ΠΆΠ΄Π΅ Ρ‡Π΅ΠΌ ΠΏΡ€ΠΈΡΡ‚ΡƒΠΏΠΈΡ‚ΡŒ ΠΊ ΠΈΠ·ΠΌΠ΅Ρ€Π΅Π½ΠΈΡŽ Смкости, Π½Π΅ΠΎΠ±Ρ…ΠΎΠ΄ΠΈΠΌΠΎ ΡƒΠ±Π΅Π΄ΠΈΡ‚ΡŒΡΡ, Ρ‡Ρ‚ΠΎ Ρ†Π΅ΠΏΠΈ питания ΠΎΡ‚ΠΊΠ»ΡŽΡ‡Π΅Π½Ρ‹.Π§Ρ‚ΠΎΠ±Ρ‹ ΡƒΠ±Π΅Π΄ΠΈΡ‚ΡŒΡΡ, Ρ‡Ρ‚ΠΎ всС ΠΏΠΈΡ‚Π°Π½ΠΈΠ΅ Ρ†Π΅ΠΏΠΈ ΠΎΡ‚ΠΊΠ»ΡŽΡ‡Π΅Π½ΠΎ:

    1. Π˜ΡΠΏΠΎΠ»ΡŒΠ·ΡƒΠΉΡ‚Π΅ оптичСский ΠΌΡƒΠ»ΡŒΡ‚ΠΈΠΌΠ΅Ρ‚Ρ€ (DMM).
    2. НастройтС ΠΌΡƒΠ»ΡŒΡ‚ΠΈΠΌΠ΅Ρ‚Ρ€ Π½Π° расчСт напряТСния ΠΏΠ΅Ρ€Π΅ΠΌΠ΅Π½Π½ΠΎΠ³ΠΎ Ρ‚ΠΎΠΊΠ°, нСзависимо ΠΎΡ‚ Ρ‚ΠΎΠ³ΠΎ, ΠΈΡΠΏΠΎΠ»ΡŒΠ·ΡƒΠ΅Ρ‚ΡΡ Π»ΠΈ кондСнсатор Π² Ρ†Π΅ΠΏΠΈ ΠΏΠ΅Ρ€Π΅ΠΌΠ΅Π½Π½ΠΎΠ³ΠΎ Ρ‚ΠΎΠΊΠ°.
    3. ΠŸΡ€Π΅Π΄ΠΏΠΎΠ»ΠΎΠΆΠΈΠΌ, ΠΈΡΠΏΠΎΠ»ΡŒΠ·ΡƒΠ΅Ρ‚ΡΡ Ρ†Π΅ΠΏΡŒ постоянного Ρ‚ΠΎΠΊΠ°, настройтС Ρ†ΠΈΡ„Ρ€ΠΎΠ²ΠΎΠΉ ΠΌΡƒΠ»ΡŒΡ‚ΠΈΠΌΠ΅Ρ‚Ρ€ для расчСта напряТСния постоянного Ρ‚ΠΎΠΊΠ°.

    ΠžΠ±ΡΠ·Π°Ρ‚Π΅Π»ΡŒΠ½ΠΎ осмотритС кондСнсатор физичСски. Π—Π°ΠΌΠ΅Π½ΠΈΡ‚Π΅ кондСнсатор ΠΏΡ€ΠΈ Π½Π°Π»ΠΈΡ‡ΠΈΠΈ ΡƒΡ‚Π΅Ρ‡Π΅ΠΊ, Ρ€Π°Π·Ρ€Ρ‹Π²ΠΎΠ², Π²Π·Π΄ΡƒΡ‚ΠΈΠΉ ΠΈΠ»ΠΈ Π»ΡŽΠ±Ρ‹Ρ… Π΄Ρ€ΡƒΠ³ΠΈΡ… ΠΏΡ€ΠΈΠ·Π½Π°ΠΊΠΎΠ² ΠΊΠΎΡ€Ρ€ΠΎΠ·ΠΈΠΈ.

    ΠŸΠ΅Ρ€Π΅ΠΊΠ»ΡŽΡ‡ΠΈΡ‚Π΅ Ρ†ΠΈΡ„Π΅Ρ€Π±Π»Π°Ρ‚ Π² Ρ€Π΅ΠΆΠΈΠΌ расчСта Смкости. Иногда этот символ Π·Π°Π½ΠΈΠΌΠ°Π΅Ρ‚ ΠΎΠ΄Π½ΠΎ ΠΈ Ρ‚ΠΎ ΠΆΠ΅ ΠΏΠΎΠ»ΠΎΠΆΠ΅Π½ΠΈΠ΅ с Π΄Ρ€ΡƒΠ³ΠΎΠΉ Ρ„ΡƒΠ½ΠΊΡ†ΠΈΠ΅ΠΉ Π½Π° Ρ†ΠΈΡ„Π΅Ρ€Π±Π»Π°Ρ‚Π΅.

    Помимо ΠΏΠ΅Ρ€Π΅ΠΊΠ»ΡŽΡ‡Π΅Π½ΠΈΡ Π½Π°Π±ΠΎΡ€Π°, для запуска измСрСния ΠΎΠ±Ρ‹Ρ‡Π½ΠΎ Π½Π΅ΠΎΠ±Ρ…ΠΎΠ΄ΠΈΠΌΠΎ Π½Π°ΠΆΠ°Ρ‚ΡŒ Ρ„ΡƒΠ½ΠΊΡ†ΠΈΠΎΠ½Π°Π»ΡŒΠ½ΡƒΡŽ ΠΊΠ½ΠΎΠΏΠΊΡƒ. ΠžΠ±Ρ€Π°Ρ‚ΠΈΡ‚Π΅ΡΡŒ ΠΊ руководству ΠΏΠΎΠ»ΡŒΠ·ΠΎΠ²Π°Ρ‚Π΅Π»Ρ вашСго ΠΌΡƒΠ»ΡŒΡ‚ΠΈΠΌΠ΅Ρ‚Ρ€Π° для получСния инструкций.

    Как ΠΏΡ€ΠΎΠ²Π΅Ρ€ΠΈΡ‚ΡŒ кондСнсатор PCBWay

    Из мноТСства доступных способов ΠΏΡ€ΠΎΠ²Π΅Ρ€ΠΊΠΈ кондСнсатора Π»ΡƒΡ‡ΡˆΠ΅ всСго ΠΏΠΎΠ΄Ρ…ΠΎΠ΄ΠΈΡ‚ PCBway. Π‘Π»Π΅Π΄ΠΎΠ²Π°Ρ‚Π΅Π»ΡŒΠ½ΠΎ, ΠΌΡ‹ Π½Π°Ρ‡Π½Π΅ΠΌ с PCBWay Π½ΠΈΠΆΠ΅ β€”

    .

    Π¨Π°Π³ 1. ΠžΡ‚ΡΠΎΠ΅Π΄ΠΈΠ½ΠΈΡ‚Π΅ кондСнсатор ΠΎΡ‚ любого источника элСктропитания

    Π’Ρ‹ Π΄ΠΎΠ»ΠΆΠ½Ρ‹ ΡƒΠ΄Π°Π»ΠΈΡ‚ΡŒ кондСнсатор ΠΈΠ· элСктричСской Ρ†Π΅ΠΏΠΈ для Ρ‚ΠΎΡ‡Π½ΠΎΠ³ΠΎ расчСта.НСкоторыС ΠΌΡƒΠ»ΡŒΡ‚ΠΈΠΌΠ΅Ρ‚Ρ€Ρ‹ Π΄Π°ΡŽΡ‚ ΠΎΡ‚Π½ΠΎΡΠΈΡ‚Π΅Π»ΡŒΠ½Ρ‹ΠΉ Ρ€Π΅ΠΆΠΈΠΌ.

    Π­Ρ‚ΠΎΡ‚ Ρ€Π΅ΠΆΠΈΠΌ ΠΈΡΠΏΠΎΠ»ΡŒΠ·ΡƒΠ΅Ρ‚ΡΡ для удалСния Π²Ρ‹Π²ΠΎΠ΄ΠΎΠ² Смкости ΠΈΠ· ΠΈΠ·ΠΌΠ΅Ρ€Π΅Π½ΠΈΠΉ всякий Ρ€Π°Π·, ΠΊΠΎΠ³Π΄Π° Ρ€Π°ΡΡΡ‡ΠΈΡ‚Ρ‹Π²Π°ΡŽΡ‚ΡΡ значСния Π±Π°Π·ΠΎΠ²ΠΎΠΉ Смкости. Π£Π±Π΅Π΄ΠΈΡ‚Π΅ΡΡŒ, Ρ‡Ρ‚ΠΎ Π½Π° ΡΠ»Π΅Π΄ΡƒΡŽΡ‰Π΅ΠΌ этапС Ρƒ вас Π²ΠΊΠ»ΡŽΡ‡Π΅Π½ Ρ€Π΅ΠΆΠΈΠΌ REL, Ρ‚Π°ΠΊ ΠΊΠ°ΠΊ ΠΎΠ½ Π²Π°ΠΌ понадобится.

    Π¨Π°Π³ 2: НаТмитС ΠΎΠΏΡ†ΠΈΠΈ .

    ΠŸΡ€ΠΈ использовании ΠΌΡƒΠ»ΡŒΡ‚ΠΈΠΌΠ΅Ρ‚Ρ€Π° для измСрСния Смкости Π² ΠΎΡ‚Π½ΠΎΡΠΈΡ‚Π΅Π»ΡŒΠ½ΠΎΠΌ Ρ€Π΅ΠΆΠΈΠΌΠ΅ Ρ‰ΡƒΠΏΡ‹ Π΄ΠΎΠ»ΠΆΠ½Ρ‹ Π±Ρ‹Ρ‚ΡŒ Ρ€Π°Π·ΠΎΠΌΠΊΠ½ΡƒΡ‚Ρ‹. Π”Π°Π»Π΅Π΅ Π²Π°ΠΌ просто Π½ΡƒΠΆΠ½ΠΎ Π±ΡƒΠ΄Π΅Ρ‚ Π½Π°ΠΆΠΈΠΌΠ°Ρ‚ΡŒ ΠΊΠ½ΠΎΠΏΠΊΠΈ REL. Π’Π΅ΠΌ Π½Π΅ ΠΌΠ΅Π½Π΅Π΅, Ρ‚Π°ΠΊΠΈΠΌ ΠΎΠ±Ρ€Π°Π·ΠΎΠΌ, испытаниС Π±ΡƒΠ΄Π΅Ρ‚ ΡΠΏΠΎΡΠΎΠ±ΡΡ‚Π²ΠΎΠ²Π°Ρ‚ΡŒ остаточной Смкости, ΠΊΠΎΡ‚ΠΎΡ€ΡƒΡŽ ΠΌΠΎΠΆΠ½ΠΎ ΠΎΡ‚Π±Ρ€ΠΎΡΠΈΡ‚ΡŒ.

    Π¨Π°Π³ 3: ЗатянитС ΠΊΠ»Π΅ΠΌΠΌΡ‹ Смкости .

    Π§Ρ‚ΠΎΠ±Ρ‹ ΠΌΡƒΠ»ΡŒΡ‚ΠΈΠΌΠ΅Ρ‚Ρ€ ΠΌΠΎΠ³ Π²Ρ‹Π±Ρ€Π°Ρ‚ΡŒ ΠΏΡ€Π°Π²ΠΈΠ»ΡŒΠ½Ρ‹ΠΉ Π΄ΠΈΠ°ΠΏΠ°Π·ΠΎΠ½, подсоСдинитС ΠΊΠ»Π΅ΠΌΠΌΡ‹ кондСнсатора ΠΊ ΠΈΠ·ΠΌΠ΅Ρ€ΠΈΡ‚Π΅Π»ΡŒΠ½Ρ‹ΠΌ ΠΏΡ€ΠΎΠ²ΠΎΠ΄Π°ΠΌ Π½Π° нСсколько сСкунд. Π’Ρ‹ Π΄ΠΎΠ»ΠΆΠ½Ρ‹ ΡƒΠ±Π΅Π΄ΠΈΡ‚ΡŒΡΡ, Ρ‡Ρ‚ΠΎ ΠΊΠ»Π΅ΠΌΠΌΡ‹ затянуты, Ρ‚Π°ΠΊ ΠΊΠ°ΠΊ это ΠΌΠΎΠΆΠ΅Ρ‚ привСсти ΠΊ ΡΠΌΠ΅Ρ‰Π΅Π½ΠΈΡŽ Π²ΠΎ врСмя процСсса, Ссли ΠΎΠ½ΠΈ Π½Π΅ Π±ΡƒΠ΄ΡƒΡ‚ Ρ‚ΠΎΡ‡Π½ΠΎ сТаты.

    Π¨Π°Π³ 4: Π—Π°ΠΏΠΈΡˆΠΈΡ‚Π΅ значСния

    ПоказанноС ΠΈΠ·ΠΌΠ΅Ρ€Π΅Π½ΠΈΠ΅ Π½Π° Ρ†ΠΈΡ„Ρ€ΠΎΠ²ΠΎΠΌ ΠΌΡƒΠ»ΡŒΡ‚ΠΈΠΌΠ΅Ρ‚Ρ€Π΅ ΠΏΡ€ΠΎΠ²Π΅Ρ€Π΅Π½ΠΎ. Если Π·Π½Π°Ρ‡Π΅Π½ΠΈΠ΅ Смкости ΠΏΠΎΠΏΠ°Π΄Π°Π΅Ρ‚ Π² Π΄ΠΈΠ°ΠΏΠ°Π·ΠΎΠ½ измСрСния, Π½Π° Ρ†ΠΈΡ„Ρ€ΠΎΠ²ΠΎΠΌ экранС отобразится Π·Π½Π°Ρ‡Π΅Π½ΠΈΠ΅ Смкости ΠΌΡƒΠ»ΡŒΡ‚ΠΈΠΌΠ΅Ρ‚Ρ€Π°.Π’Π°ΠΊΠΈΠΌ ΠΎΠ±Ρ€Π°Π·ΠΎΠΌ, Π²Ρ‹ смоТСтС Π½Π°ΠΉΡ‚ΠΈ ΠΏΡ€Π°Π²ΠΈΠ»ΡŒΠ½Ρ‹Π΅ значСния кондСнсатора с ΠΏΠΎΠΌΠΎΡ‰ΡŒΡŽ PCBWay.

    Как ΠΏΡ€ΠΎΠ²Π΅Ρ€ΠΈΡ‚ΡŒ кондСнсатор с ΠΏΠΎΠΌΠΎΡ‰ΡŒΡŽ Π°Π½Π°Π»ΠΎΠ³ΠΎΠ²ΠΎΠ³ΠΎ ΠΌΡƒΠ»ΡŒΡ‚ΠΈΠΌΠ΅Ρ‚Ρ€Π°

    Π”Π°Π²Π°ΠΉΡ‚Π΅ рассмотрим, ΠΊΠ°ΠΊ ΠΏΡ€ΠΎΠ²Π΅Ρ€ΠΈΡ‚ΡŒ кондСнсатор Π°Π½Π°Π»ΠΎΠ³ΠΎΠ²ΠΎΠ³ΠΎ ΠΌΡƒΠ»ΡŒΡ‚ΠΈΠΌΠ΅Ρ‚Ρ€Π°, Π²Ρ‹ΠΏΠΎΠ»Π½ΠΈΠ² ΡΠ»Π΅Π΄ΡƒΡŽΡ‰ΠΈΠ΅ простыС дСйствия. Π’Π½ΠΈΠΌΠ°Ρ‚Π΅Π»ΡŒΠ½ΠΎ выполняйтС ΠΊΠ°ΠΆΠ΄Ρ‹ΠΉ шаг, Ρ‚Π°ΠΊ ΠΊΠ°ΠΊ Π²Ρ‹ ΠΌΠΎΠΆΠ΅Ρ‚Π΅ ΡƒΠ²ΠΈΠ΄Π΅Ρ‚ΡŒ Π»ΠΎΠΆΠ½Ρ‹Π΅ Ρ€Π΅Π·ΡƒΠ»ΡŒΡ‚Π°Ρ‚Ρ‹, Ссли пропуститС ΠΎΠ΄ΠΈΠ½ ΠΈΠ»ΠΈ Π΄Π²Π° шага.

    Π¨Π°Π³ 1: Π’ΠΎΠ·ΡŒΠΌΠΈΡ‚Π΅ разряТСнный кондСнсатор ΠΈ ΠΌΡƒΠ»ΡŒΡ‚ΠΈΠΌΠ΅Ρ‚Ρ€ .

    Π’Ρ‹ Π΄ΠΎΠ»ΠΆΠ½Ρ‹ ΡƒΠ±Π΅Π΄ΠΈΡ‚ΡŒΡΡ, Ρ‡Ρ‚ΠΎ ваш кондСнсатор ΠΏΠΎΠ»Π½ΠΎΡΡ‚ΡŒΡŽ разряТСн ΠΎΡ‚ ΠΏΡ€Π΅Π΄Ρ‹Π΄ΡƒΡ‰Π΅Π³ΠΎ использования.Π’Π΅ΠΏΠ΅Ρ€ΡŒ Π²Ρ‹ ΠΌΠΎΠΆΠ΅Ρ‚Π΅ Π²Π·ΡΡ‚ΡŒ свой AVO-ΠΌΠ΅Ρ‚Ρ€ ΠΈ Π½Π°Ρ‡Π°Ρ‚ΡŒ процСсс измСрСния.

    Π¨Π°Π³ 2: Π’Ρ‹Π±Π΅Ρ€ΠΈΡ‚Π΅ Π°Π½Π°Π»ΠΎΠ³ΠΎΠ²Ρ‹Π΅ значСния

    ΠŸΠΎΡΠΊΠΎΠ»ΡŒΠΊΡƒ Π²Ρ‹ ΠΈΡΠΏΠΎΠ»ΡŒΠ·ΡƒΠ΅Ρ‚Π΅ Π°Π½Π°Π»ΠΎΠ³ΠΎΠ²Ρ‹ΠΉ ΠΈΠ·ΠΌΠ΅Ρ€ΠΈΡ‚Π΅Π»ΡŒ, ΠΎΠ±ΡΠ·Π°Ρ‚Π΅Π»ΡŒΠ½ΠΎ Π²Ρ‹Π±Π΅Ρ€ΠΈΡ‚Π΅ Π°Π½Π°Π»ΠΎΠ³ΠΎΠ²Ρ‹Π΅ ΠΏΠ°Ρ€Π°ΠΌΠ΅Ρ‚Ρ€Ρ‹ с высокими значСниями Ом. Π’Π°ΠΊΠΈΠΌ ΠΎΠ±Ρ€Π°Π·ΠΎΠΌ, Π²Ρ‹ ΠΌΠΎΠΆΠ΅Ρ‚Π΅ Ρ‚Ρ‰Π°Ρ‚Π΅Π»ΡŒΠ½ΠΎ ΡƒΠ±Π΅Π΄ΠΈΡ‚ΡŒΡΡ Π² сопряТСнии ΠΌΡƒΠ»ΡŒΡ‚ΠΈΠΌΠ΅Ρ‚Ρ€Π° с кондСнсатором.

    Π¨Π°Π³ 3: ΠŸΠΎΠ΄ΡΠΎΠ΅Π΄ΠΈΠ½ΠΈΡ‚Π΅ ΠΊΠΎΠ½Ρ†Ρ‹ ΠΊΠ»Π΅ΠΌΠΌ ΠΊ ΠΌΡƒΠ»ΡŒΡ‚ΠΈΠΌΠ΅Ρ‚Ρ€Ρƒ .

    Аккуратно Π²ΠΎΠ·ΡŒΠΌΠΈΡ‚Π΅ Π²Ρ‹Π²ΠΎΠ΄Ρ‹ кондСнсатора ΠΈ присоСдинитС ΠΈΡ… ΠΊ ΠΏΡ€ΠΎΠ²ΠΎΠ΄Π°ΠΌ ΠΌΡƒΠ»ΡŒΡ‚ΠΈΠΌΠ΅Ρ‚Ρ€Π°.Π’Π΅ΠΌ самым Π²Ρ‹ Π²ΠΊΠ»ΡŽΡ‡ΠΈΡ‚Π΅ источник питания ΠΌΠ΅ΠΆΠ΄Ρƒ двумя устройствами. Π§Π΅Ρ€Π΅Π· ΠΏΠ°Ρ€Ρƒ сСкунд Π²Π½ΡƒΡ‚Ρ€ΠΈ устройств ΠΏΡ€ΠΎΠΈΠ·ΠΎΠΉΠ΄Π΅Ρ‚ ΠΎΠ±ΠΌΠ΅Π½ ΠΈΠ½Ρ„ΠΎΡ€ΠΌΠ°Ρ†ΠΈΠ΅ΠΉ.

    Π¨Π°Π³ 4: ΠžΠ±Ρ€Π°Ρ‚ΠΈΡ‚Π΅ Π²Π½ΠΈΠΌΠ°Π½ΠΈΠ΅ Π½Π° показания

    Π’Π½ΠΈΠΌΠ°Ρ‚Π΅Π»ΡŒΠ½ΠΎ Π·Π°ΠΏΠΈΡˆΠΈΡ‚Π΅ Ρ†ΠΈΡ„Ρ€Ρ‹ ΠΈΠ»ΠΈ значСния, ΠΊΠΎΡ‚ΠΎΡ€Ρ‹Π΅ ΠΏΠΎΠΊΠ°ΠΆΠ΅Ρ‚ ваш ΠΌΡƒΠ»ΡŒΡ‚ΠΈΠΌΠ΅Ρ‚Ρ€.

    Π¨Π°Π³ 5: ΠŸΡ€ΠΎΠ²Π΅Ρ€ΡŒΡ‚Π΅, ΠΊΠ°ΠΊΠΎΠΉ Ρƒ вас кондСнсатор .

    Если Ρƒ вас Π΅ΡΡ‚ΡŒ ΠΊΠΎΡ€ΠΎΡ‚ΠΊΠΈΠΉ кондСнсатор, ΠΎΠ½ всСгда Π±ΡƒΠ΄Π΅Ρ‚ ΠΏΠΎΠΊΠ°Π·Ρ‹Π²Π°Ρ‚ΡŒ значСния мСньшСго Π΄ΠΈΠ°ΠΏΠ°Π·ΠΎΠ½Π° сопротивлСния.

    Если Ρƒ вас Π΅ΡΡ‚ΡŒ ΠΎΡ‚ΠΊΡ€Ρ‹Ρ‚Ρ‹Π΅ кондСнсаторы, ΠΎΠ½ΠΈ Π±ΡƒΠ΄ΡƒΡ‚ ΡΡ‚Π°Π±ΠΈΠ»ΡŒΠ½Ρ‹ΠΌΠΈ ΠΈ Π½Π΅ Π±ΡƒΠ΄ΡƒΡ‚ ΠΈΠΌΠ΅Ρ‚ΡŒ Π½ΠΈΠΊΠ°ΠΊΠΈΡ… ΠΏΡ€ΠΈΠ·Π½Π°ΠΊΠΎΠ² ΠΏΡ€ΠΎΠ³ΠΈΠ±Π° Π² стСрТнях.

    Π‘ΠΎ стандартными кондСнсаторами Π²Ρ‹ смоТСтС ΡƒΠ²ΠΈΠ΄Π΅Ρ‚ΡŒ ΠΎΡ‚ΠΊΠ»ΠΎΠ½Π΅Π½ΠΈΠ΅ ΠΌΡƒΠ»ΡŒΡ‚ΠΈΠΌΠ΅Ρ‚Ρ€Π° Π² Π΄ΠΈΠ°ΠΏΠ°Π·ΠΎΠ½Π΅ Π΄ΠΎ бСсконСчности. ΠžΠ±Ρ‹Ρ‡Π½ΠΎ это ΡƒΠΊΠ°Π·Ρ‹Π²Π°Π΅Ρ‚ Π½Π° Ρ‚ΠΎ, Ρ‡Ρ‚ΠΎ ваш кондСнсатор находится Π² Ρ…ΠΎΡ€ΠΎΡˆΠ΅ΠΌ состоянии.

    Как ΠΏΡ€ΠΎΠ²Π΅Ρ€ΠΈΡ‚ΡŒ кондСнсатор с ΠΏΠΎΠΌΠΎΡ‰ΡŒΡŽ Ρ†ΠΈΡ„Ρ€ΠΎΠ²ΠΎΠ³ΠΎ ΠΌΡƒΠ»ΡŒΡ‚ΠΈΠΌΠ΅Ρ‚Ρ€Π°

    Π’ настоящСС врСмя Π½Π°ΠΈΠ±ΠΎΠ»Π΅Π΅ распространСнной Ρ„ΠΎΡ€ΠΌΠΎΠΉ кондСнсаторов ΡΠ²Π»ΡΡŽΡ‚ΡΡ Ρ†ΠΈΡ„Ρ€ΠΎΠ²Ρ‹Π΅ ΠΌΡƒΠ»ΡŒΡ‚ΠΈΠΌΠ΅Ρ‚Ρ€Ρ‹. Π§Ρ‚ΠΎΠ±Ρ‹ ΠΏΡ€ΠΎΠ²Π΅Ρ€ΠΈΡ‚ΡŒ кондСнсатор с ΠΏΠΎΠΌΠΎΡ‰ΡŒΡŽ Ρ†ΠΈΡ„Ρ€ΠΎΠ²ΠΎΠ³ΠΎ ΠΌΡƒΠ»ΡŒΡ‚ΠΈΠΌΠ΅Ρ‚Ρ€Π°, Ρ‚Ρ‰Π°Ρ‚Π΅Π»ΡŒΠ½ΠΎ Π²Ρ‹ΠΏΠΎΠ»Π½ΠΈΡ‚Π΅ шаги, ΡƒΠΊΠ°Π·Π°Π½Π½Ρ‹Π΅ Π½ΠΈΠΆΠ΅. ΠŸΡ€ΠΎΠΏΡƒΡ‰Π΅Π½Π½Ρ‹ΠΉ шаг ΠΈΠ»ΠΈ Π΄Π²Π° появятся Π² Π»ΠΎΠΆΠ½ΠΎΠΌ Ρ€Π΅Π·ΡƒΠ»ΡŒΡ‚Π°Ρ‚Π΅.

    Π¨Π°Π³ 1: Π’ΠΎΠ·ΡŒΠΌΠΈΡ‚Π΅ разряТСнный кондСнсатор ΠΈ ΠΌΡƒΠ»ΡŒΡ‚ΠΈΠΌΠ΅Ρ‚Ρ€

    ΠŸΠΎΠ²Ρ‚ΠΎΡ€ΡΡ Ρ‚Π΅ ΠΆΠ΅ дСйствия, Ρ‡Ρ‚ΠΎ ΠΈ для Π°Π½Π°Π»ΠΎΠ³ΠΎΠ²ΠΎΠ³ΠΎ ΠΌΡƒΠ»ΡŒΡ‚ΠΈΠΌΠ΅Ρ‚Ρ€Π°, Π½Π΅ΠΎΠ±Ρ…ΠΎΠ΄ΠΈΠΌΠΎ Ρ€Π°Π·Ρ€ΡΠ΄ΠΈΡ‚ΡŒ кондСнсатор ΠΈ ΠΌΡƒΠ»ΡŒΡ‚ΠΈΠΌΠ΅Ρ‚Ρ€.Π£Π±Π΅Π΄ΠΈΡ‚Π΅ΡΡŒ, Ρ‡Ρ‚ΠΎ Π² кондСнсаторС Π½Π΅ ΠΎΡΡ‚Π°Π»ΠΎΡΡŒ Π½Π°ΠΊΠΎΠΏΠ»Π΅Π½Π½Ρ‹Ρ… Ρ€Π°Π½Π΅Π΅ зарядов.

    Π¨Π°Π³ 2: УстановитС Π΄ΠΈΠ°ΠΏΠ°Π·ΠΎΠ½ сопротивлСния .

    ΠŸΠΎΡΠΊΠΎΠ»ΡŒΠΊΡƒ это Ρ†ΠΈΡ„Ρ€ΠΎΠ²ΠΎΠΉ ΠΌΡƒΠ»ΡŒΡ‚ΠΈΠΌΠ΅Ρ‚Ρ€, Π° Π½Π΅ Π°Π½Π°Π»ΠΎΠ³ΠΎΠ²Ρ‹ΠΉ, Π²Ρ‹ Π΄ΠΎΠ»ΠΆΠ½Ρ‹ ΡƒΡΡ‚Π°Π½ΠΎΠ²ΠΈΡ‚ΡŒ Π΄ΠΈΠ°ΠΏΠ°Π·ΠΎΠ½ сопротивлСния Π½Π΅ ΠΌΠ΅Π½Π΅Π΅ 1 кОм ΠΈΠ»ΠΈ 1000 Ом. Π’Π°ΠΊΠΈΠΌ ΠΎΠ±Ρ€Π°Π·ΠΎΠΌ, ΠΌΡƒΠ»ΡŒΡ‚ΠΈΠΌΠ΅Ρ‚Ρ€ смоТСт ΠΎΠ±Π½Π°Ρ€ΡƒΠΆΠΈΡ‚ΡŒ ваш Ρ†ΠΈΡ„Ρ€ΠΎΠ²ΠΎΠΉ кондСнсатор.

    Π¨Π°Π³ 3. ΠŸΠΎΠ΄ΡΠΎΠ΅Π΄ΠΈΠ½ΠΈΡ‚Π΅ ΠΌΡƒΠ»ΡŒΡ‚ΠΈΠΌΠ΅Ρ‚Ρ€ ΠΊ кондСнсатору

    К этому ΡˆΠ°Π³Ρƒ Π½ΡƒΠΆΠ½ΠΎ ΠΏΠΎΠ΄ΠΎΠΉΡ‚ΠΈ остороТно. Часто ΠΌΡ‹ Π²ΠΈΠ΄ΠΈΠΌ, ΠΊΠ°ΠΊ ΠΏΠΎΠ»ΡŒΠ·ΠΎΠ²Π°Ρ‚Π΅Π»ΠΈ ΠΏΠΎΠ΄ΠΊΠ»ΡŽΡ‡Π°ΡŽΡ‚ Ρ‚Π΅Ρ€ΠΌΠΈΠ½Π°Π»Ρ‹ Π² спСшкС, ΠΈΠ·-Π·Π° Ρ‡Π΅Π³ΠΎ Ρ‚Π΅Ρ€ΠΌΠΈΠ½Π°Π»Ρ‹ Π»Π΅Π³ΠΊΠΎ ΠΎΡ‚ΠΊΠ»ΡŽΡ‡Π°ΡŽΡ‚ΡΡ.ΠœΡ‹ Π½Π΅ Ρ…ΠΎΡ‚ΠΈΠΌ, Ρ‡Ρ‚ΠΎΠ±Ρ‹ это ΠΏΡ€ΠΎΠΈΠ·ΠΎΡˆΠ»ΠΎ. Π’Π½ΠΈΠΌΠ°Ρ‚Π΅Π»ΡŒΠ½ΠΎ слСдуя ΠΏΡ€ΠΎΡ†Π΅Π΄ΡƒΡ€Π°ΠΌ ΠΏΠΎΠ΄ΠΊΠ»ΡŽΡ‡Π΅Π½ΠΈΡ Ρ‚Π΅Ρ€ΠΌΠΈΠ½Π°Π»Π°, Π²Ρ‹ сэкономитС своС врСмя.

    Π¨Π°Π³ 4: ΠžΠ±Ρ€Π°Ρ‚ΠΈΡ‚Π΅ Π²Π½ΠΈΠΌΠ°Π½ΠΈΠ΅ Π½Π° показания

    Π’Ρ‹ Π΄ΠΎΠ»ΠΆΠ½Ρ‹ Ρ‚Ρ‰Π°Ρ‚Π΅Π»ΡŒΠ½ΠΎ Π·Π°ΠΏΠΈΡΡ‹Π²Π°Ρ‚ΡŒ показания кондСнсатора. ЗначСния Π±ΡƒΠ΄ΡƒΡ‚ Ρ‡Π΅Ρ‚ΠΊΠΎ ΠΎΡ‚ΠΎΠ±Ρ€Π°ΠΆΠ°Ρ‚ΡŒΡΡ Π½Π° Ρ†ΠΈΡ„Ρ€ΠΎΠ²ΠΎΠΌ ΠΌΡƒΠ»ΡŒΡ‚ΠΈΠΌΠ΅Ρ‚Ρ€Π΅.

    Π¨Π°Π³ 5. ΠžΠΏΡ€Π΅Π΄Π΅Π»Π΅Π½ΠΈΠ΅ состояния кондСнсатора

    Π§Π΅Ρ€Π΅Π· Π½Π΅ΠΊΠΎΡ‚ΠΎΡ€ΠΎΠ΅ врСмя ΠΎΠ½ попытаСтся Π²Π΅Ρ€Π½ΡƒΡ‚ΡŒ ΠΎΡ‚ΠΊΡ€Ρ‹Ρ‚ΡƒΡŽ строку ΠΈ ΠΎΡ‚ΠΎΠ±Ρ€Π°Π·ΠΈΡ‚ΡŒ Ρ‚Π΅ ΠΆΠ΅ шаги, Ρ‡Ρ‚ΠΎ ΠΈ Ρ€Π°Π½ΡŒΡˆΠ΅. Если ваш кондСнсатор ΠΏΠΎΠΊΠ°Π·Ρ‹Π²Π°Π΅Ρ‚ значСния, это ΠΎΠ·Π½Π°Ρ‡Π°Π΅Ρ‚, Ρ‡Ρ‚ΠΎ ΠΎΠ½ Π² Ρ…ΠΎΡ€ΠΎΡˆΠ΅ΠΌ состоянии.Однако, Ссли Π·Π½Π°Ρ‡Π΅Π½ΠΈΠΉ Π½Π΅Ρ‚, ваш кондСнсатор разряТСн ΠΈ большС Π½Π΅ Ρ€Π°Π±ΠΎΡ‚Π°Π΅Ρ‚.

    НСкоторыС Ρ„Π°ΠΊΡ‚ΠΎΡ€Ρ‹, Π²Π»ΠΈΡΡŽΡ‰ΠΈΠ΅ Π½Π° ΠΈΠ·ΠΌΠ΅Ρ€Π΅Π½ΠΈΠ΅ Смкости:
    • Π‘Ρ€ΠΎΠΊ слуТбы кондСнсаторов мСньшС, ΠΈ ΠΎΠ½ΠΈ Ρ‚ΠΎΠΆΠ΅ Π²Ρ‹Π·Ρ‹Π²Π°ΡŽΡ‚ нСисправности. Из-Π·Π° ΠΊΠΎΡ€ΠΎΡ‚ΠΊΠΈΡ… Π·Π°ΠΌΡ‹ΠΊΠ°Π½ΠΈΠΉ кондСнсаторы ΠΌΠΎΠ³ΡƒΡ‚ Π±Ρ‹Ρ‚ΡŒ ΠΏΠΎΠ²Ρ€Π΅ΠΆΠ΄Π΅Π½Ρ‹.
    • ΠŸΡ€Π΅Π΄ΠΎΡ…Ρ€Π°Π½ΠΈΡ‚Π΅Π»ΡŒ, ΠΈΡΠΏΠΎΠ»ΡŒΠ·ΡƒΠ΅ΠΌΡ‹ΠΉ Π² Ρ†Π΅ΠΏΠΈ, ΠΌΠΎΠΆΠ΅Ρ‚ ΡΠ³ΠΎΡ€Π΅Ρ‚ΡŒ ΠΏΡ€ΠΈ ΠΊΠΎΡ€ΠΎΡ‚ΠΊΠΎΠΌ Π·Π°ΠΌΡ‹ΠΊΠ°Π½ΠΈΠΈ кондСнсатора. Π­Π»Π΅ΠΌΠ΅Π½Ρ‚Ρ‹ Π² Ρ†Π΅ΠΏΠΈ Π½Π΅ ΠΌΠΎΠ³ΡƒΡ‚ Ρ€Π°Π±ΠΎΡ‚Π°Ρ‚ΡŒ ΠΏΡ€Π°Π²ΠΈΠ»ΡŒΠ½ΠΎ, ΠΊΠΎΠ³Π΄Π° ΠΊΠ»Π΅ΠΌΠΌΡ‹ кондСнсаторов Ρ€Π°Π·ΠΎΠΌΠΊΠ½ΡƒΡ‚Ρ‹.
    • Из-Π·Π° затухания Ρ€Π°Π·Π»ΠΎΠΆΠ΅Π½ΠΈΠ΅ ΠΌΠΎΠΆΠ΅Ρ‚ Ρ‚Π°ΠΊΠΆΠ΅ ΠΈΠ·ΠΌΠ΅Π½ΠΈΡ‚ΡŒ Π·Π½Π°Ρ‡ΠΈΠΌΠΎΡΡ‚ΡŒ Π·Π½Π°Ρ‡Π΅Π½ΠΈΠΉ Смкости. БущСствованиС кондСнсаторов ΠΊΡ€Π°Ρ‚ΠΊΠΎΠ²Ρ€Π΅ΠΌΠ΅Π½Π½ΠΎ ΠΈ всСгда являСтся источником нСисправности.
    • НСисправныС кондСнсаторы ΠΌΠΎΠ³ΡƒΡ‚ ΠΈΠΌΠ΅Ρ‚ΡŒ ΠΎΠ±Ρ€Ρ‹Π² Ρ†Π΅ΠΏΠΈ, ΠΊΠΎΡ€ΠΎΡ‚ΠΊΠΎΠ΅ Π·Π°ΠΌΡ‹ΠΊΠ°Π½ΠΈΠ΅ ΠΈΠ»ΠΈ мСханичСски ΠΈΠ·Π½Π°ΡˆΠΈΠ²Π°Ρ‚ΡŒΡΡ Π²ΠΏΠ»ΠΎΡ‚ΡŒ Π΄ΠΎ Π²Ρ‹Ρ…ΠΎΠ΄Π° ΠΈΠ· строя. ΠŸΡ€Π΅Π΄ΠΎΡ…Ρ€Π°Π½ΠΈΡ‚Π΅Π»ΡŒ ΠΌΠΎΠΆΠ΅Ρ‚ ΠΏΠ΅Ρ€Π΅Π³ΠΎΡ€Π΅Ρ‚ΡŒ ΠΏΡ€ΠΈ ΠΊΠΎΡ€ΠΎΡ‚ΠΊΠΎΠΌ Π·Π°ΠΌΡ‹ΠΊΠ°Π½ΠΈΠΈ рСзистора.
    • Устройство ΠΈΠ»ΠΈ элСмСнты схСмы Π½Π΅ ΠΌΠΎΠ³ΡƒΡ‚ Ρ€Π°Π±ΠΎΡ‚Π°Ρ‚ΡŒ ΠΏΡ€ΠΈ ΠΎΠ±Ρ€Ρ‹Π²Π΅ ΠΈΠ»ΠΈ износС кондСнсатора. Износ ΠΌΠΎΠΆΠ΅Ρ‚ Π΄Π°ΠΆΠ΅ ΠΈΠ·ΠΌΠ΅Π½ΠΈΡ‚ΡŒ Π·Π½Π°Ρ‡Π΅Π½ΠΈΠ΅ Смкости кондСнсатора, Ρ‡Ρ‚ΠΎ ΠΌΠΎΠΆΠ΅Ρ‚ ΡΠΎΠ·Π΄Π°Ρ‚ΡŒ ΠΏΡ€ΠΎΠ±Π»Π΅ΠΌΡ‹.

    Как ΡƒΠ·Π½Π°Ρ‚ΡŒ, нСисправСн Π»ΠΈ кондСнсатор:

    Π’Ρ‹ ΡƒΠ·Π½Π°Π΅Ρ‚Π΅, нСисправСн Π»ΠΈ кондСнсатор, Π²Ρ‹ΠΏΠΎΠ»Π½ΠΈΠ² простоС Π²ΠΈΠ·ΡƒΠ°Π»ΡŒΠ½ΠΎΠ΅ сканированиС. Одним ΠΈΠ· ΠΏΡ€ΠΈΠ·Π½Π°ΠΊΠΎΠ² слабого кондСнсатора являСтся Π²Π·Π΄ΡƒΡ‚ΠΈΠ΅ ΠΈΠ»ΠΈ Π²Π·Π΄ΡƒΡ‚ΠΈΠ΅ свСрху ΠΈΠ»ΠΈ снизу. ΠžΡΠΌΠΎΡ‚Ρ€ΠΈΡ‚Π΅ корпус кондСнсатора ΠΈ ΠΏΠ΅Ρ‡Π°Ρ‚Π½ΡƒΡŽ ΠΏΠ»Π°Ρ‚Ρƒ Π½Π° ΠΏΡ€Π΅Π΄ΠΌΠ΅Ρ‚ обСсцвСчивания ΠΈΠ»ΠΈ Ρ€Π°Π·Ρ€ΡƒΡˆΠ΅Π½ΠΈΡ.

    НаличиС ΠΏΠΎΠ΄Ρ‚Π΅ΠΊΠ°ΡŽΡ‰Π΅Π³ΠΎ элСктролита являСтся Π΅Ρ‰Π΅ ΠΎΠ΄Π½ΠΈΠΌ ΠΏΡ€ΠΈΠ·Π½Π°ΠΊΠΎΠΌ нСисправности кондСнсатора. Если Π²Ρ‹ Π²ΠΈΠ΄ΠΈΡ‚Π΅ всС эти ΠΎΡ‡Π΅Π²ΠΈΠ΄Π½Ρ‹Π΅ ΠΏΡ€ΠΈΠ·Π½Π°ΠΊΠΈ, Π½Π΅ΠΌΠ΅Π΄Π»Π΅Π½Π½ΠΎ Π·Π°ΠΌΠ΅Π½ΠΈΡ‚Π΅ кондСнсаторы.

    Π—Π°ΠΊΠ»ΡŽΡ‡ΠΈΡ‚Π΅Π»ΡŒΠ½Ρ‹Π΅ слова

    ΠŸΠΎΡΡ‚ΠΎΠΌΡƒ ΠΌΡ‹ ΠΎΡ‡Π΅Π½ΡŒ надССмся, Ρ‡Ρ‚ΠΎ ΠΊ ΠΊΠΎΠ½Ρ†Ρƒ этой ΡΡ‚Π°Ρ‚ΡŒΠΈ Π²Ρ‹ ΡƒΠ·Π½Π°Π»ΠΈ ΠΊΠ°ΠΊ ΠΏΡ€ΠΎΠ²Π΅Ρ€ΠΈΡ‚ΡŒ кондСнсатор ΠΌΡƒΠ»ΡŒΡ‚ΠΈΠΌΠ΅Ρ‚Ρ€ΠΎΠΌ Ρ€Π°Π·Π»ΠΈΡ‡Π½Ρ‹ΠΌΠΈ способами ΠΈ ΠΊΠ°ΠΊ ΠΏΡ€ΠΎΠ²Π΅Ρ€ΠΈΡ‚ΡŒ, нСисправСн Π»ΠΈ ваш кондСнсатор ΠΈΠ»ΠΈ Π½Π΅Ρ‚.

    Однако ΠΏΡ€ΠΈ Ρ€Π°Π±ΠΎΡ‚Π΅ с элСктричСскими инструмСнтами ΡΠΎΠ±Π»ΡŽΠ΄Π°ΠΉΡ‚Π΅ ΠΌΠ΅Ρ€Ρ‹ бСзопасности, Ρ‚Π°ΠΊ ΠΊΠ°ΠΊ ΠΎΠ½ΠΈ Π½Π°ΠΊΠ°ΠΏΠ»ΠΈΠ²Π°ΡŽΡ‚ элСктричСство ΠΈ ΠΌΠΎΠ³ΡƒΡ‚ привСсти ΠΊ ΠΏΠΎΡ€Π°ΠΆΠ΅Π½ΠΈΡŽ элСктричСским Ρ‚ΠΎΠΊΠΎΠΌ ΠΏΡ€ΠΈ Π½Π΅ΠΏΡ€Π°Π²ΠΈΠ»ΡŒΠ½ΠΎΠΌ использовании. Π’Ρ‹ Π΄Π°ΠΆΠ΅ ΠΌΠΎΠΆΠ΅Ρ‚Π΅ Π²ΠΎΡΠΏΠΎΠ»ΡŒΠ·ΠΎΠ²Π°Ρ‚ΡŒΡΡ руководством ΠΊ ΠΌΡƒΠ»ΡŒΡ‚ΠΈΠΌΠ΅Ρ‚Ρ€Ρƒ, Ρ‡Ρ‚ΠΎΠ±Ρ‹ ΡƒΠ·Π½Π°Ρ‚ΡŒ большС ΠΎ Π΅Π³ΠΎ функциях.

    РСсурс:

    1. https://www.ifixit.com/Wiki/Troubleshhoting_logic_board_components.

    ΠŸΡ€ΠΎΠ²Π΅Ρ€ΠΊΠ° Ρ„ΡƒΠ½ΠΊΡ†ΠΈΠΈ Смкости эталонного ΠΌΡƒΠ»ΡŒΡ‚ΠΈΠΌΠ΅Ρ‚Ρ€Π° 8588A

    ΠžΠ±Π·ΠΎΡ€

    : Π’ этом ΠΏΡ€ΠΈΠΌΠ΅Ρ‡Π°Π½ΠΈΠΈ ΠΏΠΎ ΠΏΡ€ΠΈΠΌΠ΅Π½Π΅Π½ΠΈΡŽ содСрТатся Ρ€Π΅ΠΊΠΎΠΌΠ΅Π½Π΄Π°Ρ†ΠΈΠΈ для Ρ‚Π΅Ρ…, ΠΊΡ‚ΠΎ Ρ…ΠΎΡ‡Π΅Ρ‚ ΠΏΡ€ΠΎΠ²Π΅Ρ€ΠΈΡ‚ΡŒ Ρ€Π°Π±ΠΎΡ‚Ρƒ Смкостной Ρ„ΡƒΠ½ΠΊΡ†ΠΈΠΈ своСго эталонного ΠΌΡƒΠ»ΡŒΡ‚ΠΈΠΌΠ΅Ρ‚Ρ€Π° 8588A. Π’ Π½Π΅ΠΌ описываСтся мСтодология, исслСдуСтся процСсс измСрСния ΠΈ ΠΎΠΏΠΈΡΡ‹Π²Π°ΡŽΡ‚ΡΡ Π½Π΅ΠΊΠΎΡ‚ΠΎΡ€Ρ‹Π΅ ΠΏΠΎΠ΄Π²ΠΎΠ΄Π½Ρ‹Π΅ ΠΊΠ°ΠΌΠ½ΠΈ, ΠΊΠΎΡ‚ΠΎΡ€Ρ‹Π΅ ΠΌΠΎΠ³ΡƒΡ‚ Π²ΠΎΠ·Π½ΠΈΠΊΠ½ΡƒΡ‚ΡŒ.

    ΠŸΠΎΡΠΊΠΎΠ»ΡŒΠΊΡƒ Π΅ΠΌΠΊΠΎΡΡ‚ΡŒ зависит ΠΎΡ‚ сопротивлСния, Π½ΡƒΠΆΠ½ΠΎ Π»ΠΈ Π²ΠΎΠΎΠ±Ρ‰Π΅ рСгулярно ΠΏΡ€ΠΎΠ²Π΅Ρ€ΡΡ‚ΡŒ Ρ„ΡƒΠ½ΠΊΡ†ΠΈΡŽ Смкости? Π§Ρ‚ΠΎΠ±Ρ‹ ΠΎΡ‚Π²Π΅Ρ‚ΠΈΡ‚ΡŒ Π½Π° этот вопрос, сначала посмотритС, ΠΊΠ°ΠΊ эталонный ΠΌΡƒΠ»ΡŒΡ‚ΠΈΠΌΠ΅Ρ‚Ρ€ 8588A измСряСт Π΅ΠΌΠΊΠΎΡΡ‚ΡŒ.

    ВСстСр 8588A измСряСт Π΅ΠΌΠΊΠΎΡΡ‚ΡŒ с ΠΏΠΎΠΌΠΎΡ‰ΡŒΡŽ ΠΌΠ΅Ρ‚ΠΎΠ΄Π° разряда постоянного Ρ‚ΠΎΠΊΠ°, ΠΊΠΎΡ‚ΠΎΡ€Ρ‹ΠΉ Π² основном Π°Π½Π°Π»ΠΎΠ³ΠΈΡ‡Π΅Π½ ΠΌΠ΅Ρ‚ΠΎΠ΄Ρƒ, ΠΈΡΠΏΠΎΠ»ΡŒΠ·ΡƒΠ΅ΠΌΠΎΠΌΡƒ Ρ€ΡƒΡ‡Π½Ρ‹ΠΌΠΈ ΠΌΡƒΠ»ΡŒΡ‚ΠΈΠΌΠ΅Ρ‚Ρ€Π°ΠΌΠΈ, Π½ΠΎ ΠΈΠΌΠ΅Π΅Ρ‚ Π±ΠΎΠ»Π΅Π΅ Π²Ρ‹ΡΠΎΠΊΡƒΡŽ Ρ‚ΠΎΡ‡Π½ΠΎΡΡ‚ΡŒ. Π’ случаС 8588A нСизвСстный кондСнсатор разряТаСтся Π΄ΠΎ Ρ‚Π΅Ρ… ΠΏΠΎΡ€, ΠΏΠΎΠΊΠ° ΠΎΠ½ Π½Π΅ станСт Π±Π»ΠΈΠ·ΠΊΠΈΠΌ ΠΊ Π½ΡƒΠ»ΡŽ, Π° Π·Π°Ρ‚Π΅ΠΌ заряТаСтся извСстным постоянным Ρ‚ΠΎΠΊΠΎΠΌ; это ΠΏΡ€ΠΈΠ²ΠΎΠ΄ΠΈΡ‚ ΠΊ ΠΎΡ‚Ρ€ΠΈΡ†Π°Ρ‚Π΅Π»ΡŒΠ½ΠΎΠΉ Π»ΠΈΠ½Π΅ΠΉΠ½ΠΎΠΉ Ρ€Π°ΠΌΠΏΠ΅, которая возвращаСтся ΠΊ Π½ΡƒΠ»ΡŽ Π² ΠΊΠΎΠ½Ρ†Π΅ ΠΊΠ°ΠΆΠ΄ΠΎΠ³ΠΎ Ρ†ΠΈΠΊΠ»Π° измСрСния.Π˜Π·ΠΌΠ΅Ρ€ΡΡŽΡ‚ΡΡ напряТСния, Π±Π»ΠΈΠ·ΠΊΠΈΠ΅ ΠΊ Π½Π°Ρ‡Π°Π»Ρƒ ΠΈ ΠΊΠΎΠ½Ρ†Ρƒ Ρ€Π°ΠΌΠΏΡ‹, ΠΈ Π΅ΠΌΠΊΠΎΡΡ‚ΡŒ рассчитываСтся ΠΏΠΎ Ρ„ΠΎΡ€ΠΌΡƒΠ»Π΅ C = I.ẟt/ẟv, Π³Π΄Π΅ C β€” Π΅ΠΌΠΊΠΎΡΡ‚ΡŒ Π² Ρ„Π°Ρ€Π°Π΄Π°Ρ…, I β€” Ρ‚ΠΎΠΊ Π² Π°ΠΌΠΏΠ΅Ρ€Π°Ρ…, Π° v β€” ΠΈΠ·ΠΌΠ΅Π½Π΅Π½ΠΈΠ΅ напряТСния Π·Π° ΠΈΠ½Ρ‚Π΅Ρ€Π²Π°Π» Π²Ρ€Π΅ΠΌΠ΅Π½ΠΈ. Ρ‚.

    Ѐункция сопротивлСния обСспСчиваСт Π²ΠΎΠ·ΠΌΠΎΠΆΠ½ΠΎΡΡ‚ΡŒ измСрСния постоянного Ρ‚ΠΎΠΊΠ° ΠΈ напряТСния ΠΈ ΠΈΡΠΏΠΎΠ»ΡŒΠ·ΡƒΠ΅Ρ‚ Ρ‚Π΅ ΠΆΠ΅ константы ΠΊΠ°Π»ΠΈΠ±Ρ€ΠΎΠ²ΠΊΠΈ для напряТСния ΠΈ Ρ‚ΠΎΠΊΠ°, ΠΊΠΎΡ‚ΠΎΡ€Ρ‹Π΅ Π±Ρ‹Π»ΠΈ ΠΎΠΏΡ€Π΅Π΄Π΅Π»Π΅Π½Ρ‹ ΠΏΡ€ΠΈ Ρ€Π΅Π³ΡƒΠ»ΠΈΡ€ΠΎΠ²ΠΊΠ΅ сопротивлСния, Π·Π° ΠΈΡΠΊΠ»ΡŽΡ‡Π΅Π½ΠΈΠ΅ΠΌ Π΄ΠΈΠ°ΠΏΠ°Π·ΠΎΠ½Π° 1 Π½Π€, ΠΊΠΎΡ‚ΠΎΡ€Ρ‹ΠΉ настраиваСтся с использованиСм внСшнСго эталонного кондСнсатора ΠΈΠ·-Π·Π° слоТности опрСдСлСния характСристик. Π²Π½ΡƒΡ‚Ρ€Π΅Π½Π½ΠΈΠ΅ Ρ‚ΠΎΠΊΠΈ смСщСния.

    На Ρ„ΡƒΠ½ΠΊΡ†ΠΈΡŽ Смкости Ρ‚Π°ΠΊΠΆΠ΅ Π²Π»ΠΈΡΡŽΡ‚ ошибки синхронизации. Π’Π½ΡƒΡ‚Ρ€Π΅Π½Π½Π΅ 8588A ΠΈΠΌΠ΅Π΅Ρ‚ ΠΎΠ΄ΠΈΠ½ Π³Π»Π°Π²Π½Ρ‹ΠΉ Ρ‚Π°ΠΊΡ‚ΠΎΠ²Ρ‹ΠΉ Π³Π΅Π½Π΅Ρ€Π°Ρ‚ΠΎΡ€, поэтому эти ошибки синхронизации ΠΌΠΎΠΆΠ½ΠΎ ΠΎΡ†Π΅Π½ΠΈΡ‚ΡŒ ΠΏΡƒΡ‚Π΅ΠΌ измСрСния счСтчика частоты. БпСцификация часов составляСт 1 ΠΌΠΊΠ“Ρ†/Π“Ρ†, Ρ‡Ρ‚ΠΎ Π½Π° ΠΌΠ½ΠΎΠ³ΠΎ порядков Π»ΡƒΡ‡ΡˆΠ΅, Ρ‡Π΅ΠΌ спСцификация Π½Π°ΠΈΠ»ΡƒΡ‡ΡˆΠ΅Π³ΠΎ Π΄ΠΈΠ°ΠΏΠ°Π·ΠΎΠ½Π° Смкости, поэтому Π»ΡŽΠ±Ρ‹Π΅ ошибки синхронизации ΠΌΠΎΠΆΠ½ΠΎ эффСктивно ΠΈΠ³Π½ΠΎΡ€ΠΈΡ€ΠΎΠ²Π°Ρ‚ΡŒ.

    ВСстСр 8588A измСряСт Π΅ΠΌΠΊΠΎΡΡ‚ΡŒ Π΄Π²ΡƒΡ… Π²Ρ‹Π²ΠΎΠ΄ΠΎΠ² с использованиСм ΠΌΠ΅Ρ‚ΠΎΠ΄Π° разряда постоянным Ρ‚ΠΎΠΊΠΎΠΌ

    Из этого описания Π²ΠΈΠ΄Π½ΠΎ, Ρ‡Ρ‚ΠΎ ΠΏΡ€ΠΎΠΈΠ·Π²ΠΎΠ΄ΠΈΡ‚Π΅Π»ΡŒΠ½ΠΎΡΡ‚ΡŒ Π² Π΄ΠΈΠ°ΠΏΠ°Π·ΠΎΠ½Π΅ 10 Π½Π€ ΠΈ Π²Ρ‹ΡˆΠ΅ ΠΌΠΎΠΆΠ½ΠΎ ΠΊΠΎΠ½Ρ‚Ρ€ΠΎΠ»ΠΈΡ€ΠΎΠ²Π°Ρ‚ΡŒ ΠΏΡƒΡ‚Π΅ΠΌ ΠΎΡ†Π΅Π½ΠΊΠΈ ΡΠΎΠΎΡ‚Π²Π΅Ρ‚ΡΡ‚Π²ΡƒΡŽΡ‰Π΅Π³ΠΎ Π΄ΠΈΠ°ΠΏΠ°Π·ΠΎΠ½Π° сопротивлСний.Для ΠΌΠ½ΠΎΠ³ΠΈΡ… Π»Π°Π±ΠΎΡ€Π°Ρ‚ΠΎΡ€ΠΈΠΉ это ΠΌΠΎΠΆΠ΅Ρ‚ Π±Ρ‹Ρ‚ΡŒ ΠΏΡ€Π΅Π΄ΠΏΠΎΡ‡Ρ‚ΠΈΡ‚Π΅Π»ΡŒΠ½Ρ‹ΠΌ Π²Π°Ρ€ΠΈΠ°Π½Ρ‚ΠΎΠΌ ΠΈΠ·-Π·Π° отсутствия подходящих прослСТиваСмых стандартов. Π’ΠΎΠ·ΠΌΠΎΠΆΠ½ΠΎ, Ρ‡Ρ‚ΠΎ Π½Π° ΠΈΠ·ΠΌΠ΅Ρ€Π΅Π½ΠΈΠ΅ Смкости ΠΌΠΎΠ³ΡƒΡ‚ ΠΏΠΎΠ²Π»ΠΈΡΡ‚ΡŒ Π²Π½ΡƒΡ‚Ρ€Π΅Π½Π½ΠΈΠ΅ физичСскиС измСнСния, ΠΊΠΎΡ‚ΠΎΡ€Ρ‹Π΅ Π½Π΅ ΠΏΡ€ΠΎΡΠ²Π»ΡΡŽΡ‚ΡΡ ΠΊΠ°ΠΊ ΠΈΠ·ΠΌΠ΅Π½Π΅Π½ΠΈΠ΅ сопротивлСния; ΠΎΠ΄Π½Π°ΠΊΠΎ любоС ΠΈΠ·ΠΌΠ΅Π½Π΅Π½ΠΈΠ΅ ΠΎΠΊΠ°ΠΆΠ΅Ρ‚ большСС влияниС Π½Π° Π±ΠΎΠ»Π΅Π΅ Π½ΠΈΠ·ΠΊΠΈΠ΅ Π΄ΠΈΠ°ΠΏΠ°Π·ΠΎΠ½Ρ‹ Смкости, ΠΊΠΎΡ‚ΠΎΡ€Ρ‹Π΅ ΠΌΠΎΠΆΠ½ΠΎ ΠΎΡ†Π΅Π½ΠΈΡ‚ΡŒ ΠΏΡƒΡ‚Π΅ΠΌ сравнСния с общСдоступными эталонными кондСнсаторами. Π›ΡŽΠ±ΠΎΠ΅ ΠΈΠ·ΠΌΠ΅Π½Π΅Π½ΠΈΠ΅, наблюдаСмоС Π² этих Π½ΠΈΠΆΠ½ΠΈΡ… Π΄ΠΈΠ°ΠΏΠ°Π·ΠΎΠ½Π°Ρ…, ΠΎΡ‡Π΅Π½ΡŒ быстро станСт Π½Π΅Π·Π½Π°Ρ‡ΠΈΡ‚Π΅Π»ΡŒΠ½Ρ‹ΠΌ ΠΏΠΎ ΠΌΠ΅Ρ€Π΅ Ρ‚ΠΎΠ³ΠΎ, ΠΊΠ°ΠΊ Π²Ρ‹ ΠΏΠΎΠ΄Π½ΠΈΠΌΠ°Π΅Ρ‚Π΅ΡΡŒ Π²Π²Π΅Ρ€Ρ….

    Π§Ρ‚ΠΎΠ±Ρ‹ ΠΎΡ‚Π²Π΅Ρ‚ΠΈΡ‚ΡŒ Π½Π° вопрос; Π²Π°ΠΌ Π½ΡƒΠΆΠ½ΠΎ ΠΎΡ‚Π΄Π΅Π»ΡŒΠ½ΠΎ ΠΏΡ€ΠΎΠ²Π΅Ρ€ΠΈΡ‚ΡŒ Ρ„ΡƒΠ½ΠΊΡ†ΠΈΡŽ Смкости? Π΄Π°.

    Для Π΄ΠΈΠ°ΠΏΠ°Π·ΠΎΠ½Π° 1 Π½Π€, ΠΏΠΎΡΠΊΠΎΠ»ΡŒΠΊΡƒ Π΅Π³ΠΎ ΠΊΠ°Π»ΠΈΠ±Ρ€ΠΎΠ²ΠΊΠ° Π½Π΅ зависит ΠΎΡ‚ ΠΈΡΠΏΠΎΠ»ΡŒΠ·ΡƒΠ΅ΠΌΠΎΠ³ΠΎ Π΄ΠΈΠ°ΠΏΠ°Π·ΠΎΠ½Π° сопротивлСний, Π½ΠΎ Π½Π΅ΠΎΠ±ΡΠ·Π°Ρ‚Π΅Π»ΡŒΠ½Π° для Π΄ΠΈΠ°ΠΏΠ°Π·ΠΎΠ½Π° 10 Π½Π€ ΠΈ Π²Ρ‹ΡˆΠ΅. Π­Ρ‚ΠΎ связано с Ρ‚Π΅ΠΌ, Ρ‡Ρ‚ΠΎ характСристики этих Π΄ΠΈΠ°ΠΏΠ°Π·ΠΎΠ½ΠΎΠ² ΠΌΠΎΠΆΠ½ΠΎ ΠΊΠΎΠ½Ρ‚Ρ€ΠΎΠ»ΠΈΡ€ΠΎΠ²Π°Ρ‚ΡŒ, наблюдая Π·Π° ΠΈΠ·ΠΌΠ΅Π½Π΅Π½ΠΈΠ΅ΠΌ ΡΠΎΠΎΡ‚Π²Π΅Ρ‚ΡΡ‚Π²ΡƒΡŽΡ‰Π΅Π³ΠΎ Π΄ΠΈΠ°ΠΏΠ°Π·ΠΎΠ½Π° сопротивлСния, ΡƒΠΊΠ°Π·Π°Π½Π½ΠΎΠ³ΠΎ Π² Ρ‚Π°Π±Π»ΠΈΡ†Π΅ 1.

    Π”ΠΈΠ°ΠΏΠ°Π·ΠΎΠ½ Смкости Π˜ΡΠΏΠΎΠ»ΡŒΠ·ΡƒΠ΅ΠΌΡ‹ΠΉ Π΄ΠΈΠ°ΠΏΠ°Π·ΠΎΠ½ сопротивлСния
    1 нЀ 100 МОм, Lo I
    10 нЀ 10 МОм, Lo I
    100 нЀ 1 МОм, Lo I
    1 мкЀ 100 кОм, Lo I
    10 мкЀ 10 кОм
    100 мкЀ 1 кОм
    ΠΎΡ‚ 1 ΠΌΠ€ Π΄ΠΎ 100 ΠΌΠ€ 100 Ом
    ΠΎΡ‚ 1 ΠΌΠ€ Π΄ΠΎ 100 ΠΌΠ€ Lo I 1 кОм

    Π’Π°Π±Π»ΠΈΡ†Π° 1.ΠžΡ‚ΠΎΠ±Ρ€Π°ΠΆΠ΅Π½ΠΈΠ΅ Π΄ΠΈΠ°ΠΏΠ°Π·ΠΎΠ½Π° сопротивлСния Смкости

    ИспользованиС эталонных кондСнсаторов для ΠΏΡ€ΠΎΠ²Π΅Ρ€ΠΊΠΈ 8588A

    Π—Π½Π°Ρ‡Π΅Π½ΠΈΠ΅ Ρ‚ΠΈΠΏΠΈΡ‡Π½ΠΎΠ³ΠΎ эталонного кондСнсатора, ΠΎΠ±Ρ‹Ρ‡Π½ΠΎ ΠΈΡΠΏΠΎΠ»ΡŒΠ·ΡƒΠ΅ΠΌΠΎΠ³ΠΎ Π² ΠΊΠ°Π»ΠΈΠ±Ρ€ΠΎΠ²ΠΎΡ‡Π½Ρ‹Ρ… лабораториях, зависит ΠΎΡ‚ частоты ΠΈ Π΅Π³ΠΎ ΠΊΠΎΠ½Ρ„ΠΈΠ³ΡƒΡ€Π°Ρ†ΠΈΠΈ Π² Π²ΠΈΠ΄Π΅ устройства с 2 ΠΈΠ»ΠΈ 3 Π²Ρ‹Π²ΠΎΠ΄Π°ΠΌΠΈ. МодСль 8588A измСряСт Π΅ΠΌΠΊΠΎΡΡ‚ΡŒ 2 Π²Ρ‹Π²ΠΎΠ΄ΠΎΠ², ΠΈΡΠΏΠΎΠ»ΡŒΠ·ΡƒΡ описанный Π²Ρ‹ΡˆΠ΅ ΠΌΠ΅Ρ‚ΠΎΠ΄ разряда постоянным Ρ‚ΠΎΠΊΠΎΠΌ. ΠŸΠΎΡΡ‚ΠΎΠΌΡƒ Π½Π°ΠΌ Π½Π΅ΠΎΠ±Ρ…ΠΎΠ΄ΠΈΠΌΠΎ Π·Π½Π°Ρ‚ΡŒ, ΠΊΠ°ΠΊΠΎΠ²Π° эффСктивная тСстовая частота 8588A, ΠΈ Π΄Π²ΡƒΡ…ΠΏΠΎΠ»ΡŽΡΠ½ΠΎΠ΅ Π·Π½Π°Ρ‡Π΅Π½ΠΈΠ΅ эталонного кондСнсатора Π½Π° этой частотС.

    НСт прямой эквивалСнтности ΠΌΠ΅ΠΆΠ΄Ρƒ Π»ΠΈΠ½Π΅ΠΉΠ½ΠΎ ΠΈΠ·ΠΌΠ΅Π½ΡΡŽΡ‰ΠΈΠΌΡΡ сигналом 8588A ΠΈ частотой ΡΠΈΠ½ΡƒΡΠΎΠΈΠ΄Π°Π»ΡŒΠ½ΠΎΠ³ΠΎ сигнала. Однако Ρ…ΠΎΡ€ΠΎΡˆΠ΅Π΅ ΠΏΡ€ΠΈΠ±Π»ΠΈΠΆΠ΅Π½ΠΈΠ΅ ΠΌΠΎΠΆΠ½ΠΎ ΠΏΠΎΠ»ΡƒΡ‡ΠΈΡ‚ΡŒ, приравняв ΡΠΊΠΎΡ€ΠΎΡΡ‚ΡŒ нарастания ΠΏΠΈΠ»ΠΎΠΎΠ±Ρ€Π°Π·Π½ΠΎΠ³ΠΎ сигнала, Π³Π΅Π½Π΅Ρ€ΠΈΡ€ΡƒΠ΅ΠΌΠΎΠ³ΠΎ 8588A, ΠΊ максимальной скорости нарастания ΡΠΈΠ½ΡƒΡΠΎΠΈΠ΄Π°Π»ΡŒΠ½ΠΎΠ³ΠΎ сигнала, ΠΈΡΠΏΠΎΠ»ΡŒΠ·ΡƒΡ ΡΠΎΠΎΡ‚Π½ΠΎΡˆΠ΅Π½ΠΈΠ΅: v/s = 2,Ο€f.f.Vpk, Π³Π΄Π΅ v/s β€” это ΡΠΊΠΎΡ€ΠΎΡΡ‚ΡŒ нарастания Π² Π²ΠΎΠ»ΡŒΡ‚Π°Ρ… Π² сСкунду, f β€” частота Π² Π“Ρ†, Π° Vpk β€” ΠΏΠΈΠΊΠΎΠ²ΠΎΠ΅ напряТСниС. Π’ Ρ‚Π°Π±Π»ΠΈΡ†Π΅ 2 пСрСчислСны эквивалСнтныС тСстовыС частоты, ΠΎΠΏΡ€Π΅Π΄Π΅Π»Π΅Π½Π½Ρ‹Π΅ с использованиСм этого ΡΠΎΠΎΡ‚Π½ΠΎΡˆΠ΅Π½ΠΈΡ. ΠžΠ±Ρ€Π°Ρ‚ΠΈΡ‚Π΅ Π²Π½ΠΈΠΌΠ°Π½ΠΈΠ΅, Ρ‡Ρ‚ΠΎ частота мСняСтся Π½Π΅ Ρ‚ΠΎΠ»ΡŒΠΊΠΎ Π² зависимости ΠΎΡ‚ Π΄ΠΈΠ°ΠΏΠ°Π·ΠΎΠ½Π°, Π½ΠΎ ΠΈ ΠΎΡ‚ Ρ‚ΠΎΠ³ΠΎ, Π³Π΄Π΅ Π·Π½Π°Ρ‡Π΅Π½ΠΈΠ΅ находится Π² ΠΏΡ€Π΅Π΄Π΅Π»Π°Ρ… Π΄ΠΈΠ°ΠΏΠ°Π·ΠΎΠ½Π°.

    8588А сСрия ΠŸΡ€ΠΈΠΌΠ΅Π½ΡΠ΅ΠΌΠΎΠ΅ Π·Π½Π°Ρ‡Π΅Π½ΠΈΠ΅ % ΠΎΡ‚ Π΄ΠΈΠ°ΠΏΠ°Π·ΠΎΠ½Π° ЭквивалСнтная тСстовая частота
    ΠΎΡ‚ 1 Π½Π€ Π΄ΠΎ 1 ΠΌΠ€
    Β 
    20% 5,63 Π“Ρ†
    100% 1,13 Π“Ρ†
    200% 0,56 Π“Ρ†
    10 ΠΌΠ€
    Β 
    20% 0,68 Π“Ρ†
    100% 0,14 Π“Ρ†
    200% 0.079 Π“Ρ†
    100 ΠΌΠ€
    Β 
    20% 0,056 Π“Ρ†
    50% 0,023 Π“Ρ†
    100% 0,011 Π“Ρ†
    1 ΠΌΠ€ Lo I
    Β 
    20% 0,56 Π“Ρ†
    100% 0,11 Π“Ρ†
    200% 0,056 Π“Ρ†
    10 ΠΌΠ€ Lo I
    Β 
    20% 0,056 Π“Ρ†
    100% 0.011 Π“Ρ†
    200% 0,005 6 Π“Ρ†
    100 ΠΌΠ€ Lo I
    Β 
    20% 0,005 6 Π“Ρ†
    50% 0,002 3 Π“Ρ†
    100% 0,001 1 Π“Ρ†

    Π’Π°Π±Π»ΠΈΡ†Π° 2. ЭффСктивная тСстовая частота 8588A

    ΠœΠ°Π»ΠΎΠ²Π΅Ρ€ΠΎΡΡ‚Π½ΠΎ, Ρ‡Ρ‚ΠΎ ΠΏΡ€ΠΎΡΠ»Π΅ΠΆΠΈΠ²Π°Π΅ΠΌΠΎΡΡ‚ΡŒ Π±ΡƒΠ΄Π΅Ρ‚ доступна Π½Π° частотах, пСрСчислСнных Π² Ρ‚Π°Π±Π»ΠΈΡ†Π΅, поэтому Π½Π΅ΠΎΠ±Ρ…ΠΎΠ΄ΠΈΠΌΠΎ ΠΈΠ·ΠΌΠ΅Ρ€ΠΈΡ‚ΡŒ эталонный кондСнсатор Π½Π° Π½Π΅ΡΠΊΠΎΠ»ΡŒΠΊΠΈΡ… частотах, максимально сниТая ΠΈΡ…, Π° Π·Π½Π°Ρ‡Π΅Π½ΠΈΠ΅ ΡΠΊΡΡ‚Ρ€Π°ΠΏΠΎΠ»ΠΈΡ€ΠΎΠ²Π°Ρ‚ΡŒ.

    К ΡΡ‡Π°ΡΡ‚ΡŒΡŽ, Π΄ΠΈΠ°ΠΏΠ°Π·ΠΎΠ½ 1 Π½Π€ ΠΏΡ€ΠΎΠ²Π΅Ρ€ΠΈΡ‚ΡŒ Π½Π°ΠΌΠ½ΠΎΠ³ΠΎ ΠΏΡ€ΠΎΡ‰Π΅. ΠŸΡ€ΠΈ этом Π·Π½Π°Ρ‡Π΅Π½ΠΈΠΈ доступны кондСнсаторы с Π²ΠΎΠ·Π΄ΡƒΡˆΠ½Ρ‹ΠΌ диэлСктриком, ΠΊΠΎΡ‚ΠΎΡ€Ρ‹Π΅ ΠΈΠΌΠ΅ΡŽΡ‚ Π½Π΅Π·Π½Π°Ρ‡ΠΈΡ‚Π΅Π»ΡŒΠ½ΡƒΡŽ Π·Π°Π²ΠΈΡΠΈΠΌΠΎΡΡ‚ΡŒ ΠΎΡ‚ частоты; это позволяСт ΠΎΡ…Π°Ρ€Π°ΠΊΡ‚Π΅Ρ€ΠΈΠ·ΠΎΠ²Π°Ρ‚ΡŒ ΠΈΡ… Π½Π° частотС 1 ΠΊΠ“Ρ† ΠΈ ΠΈΡΠΏΠΎΠ»ΡŒΠ·ΠΎΠ²Π°Ρ‚ΡŒ Π½Π° этих Π½ΠΈΠ·ΠΊΠΈΡ… частотах. ОсновноС сообраТСниС с этими кондСнсаторами Π·Π°ΠΊΠ»ΡŽΡ‡Π°Π΅Ρ‚ΡΡ Π² Ρ‚ΠΎΠΌ, Ρ‡Ρ‚ΠΎ ΠΎΠ½ΠΈ, ΠΊΠ°ΠΊ ΠΏΡ€Π°Π²ΠΈΠ»ΠΎ, ΡΠ²Π»ΡΡŽΡ‚ΡΡ Ρ‚Ρ€Π΅Ρ…Π²Ρ‹Π²ΠΎΠ΄Π½Ρ‹ΠΌΠΈ устройствами. Π‘Π»Π΅Π΄ΠΎΠ²Π°Ρ‚Π΅Π»ΡŒΠ½ΠΎ, ΠΈΡ… Π½ΡƒΠΆΠ½ΠΎ Π±ΡƒΠ΄Π΅Ρ‚ ΠΌΠΎΠ΄ΠΈΡ„ΠΈΡ†ΠΈΡ€ΠΎΠ²Π°Ρ‚ΡŒ, установив Π·Π°ΠΊΠΎΡ€Π°Ρ‡ΠΈΠ²Π°ΡŽΡ‰ΠΈΠΉ Ρ€Π°Π·ΡŠΠ΅ΠΌ Π½Π° ΠΈΡ… Π½ΠΈΠΆΠ½ΠΈΠΉ Π²Ρ‹Π²ΠΎΠ΄, ΠΊΠΎΡ‚ΠΎΡ€Ρ‹ΠΉ соСдиняСт Π΅Π³ΠΎ с корпусом, Π° Π·Π°Ρ‚Π΅ΠΌ ΠΈΠ·ΠΌΠ΅Ρ€ΠΈΡ‚ΡŒ ΠΊΠ°ΠΊ 2-ΠΊΠΎΠ½Ρ‚Π°ΠΊΡ‚Π½ΠΎΠ΅ устройство.

    ΠΠ»ΡŒΡ‚Π΅Ρ€Π½Π°Ρ‚ΠΈΠ²Π½Ρ‹ΠΉ ΠΏΠΎΠ΄Ρ…ΠΎΠ΄

    Π’ ΠΊΠΎΠΌΠΏΠ°Π½ΠΈΠΈ Fluke ΠΈΡΠΏΠΎΠ»ΡŒΠ·ΡƒΠ΅Ρ‚ΡΡ нСсколько ΠΈΠ½ΠΎΠΉ ΠΏΠΎΠ΄Ρ…ΠΎΠ΄ ΠΈΠ·-Π·Π° большого количСства ΠΏΡ€ΠΈΠ±ΠΎΡ€ΠΎΠ², проходящих Ρ‡Π΅Ρ€Π΅Π· Π»Π°Π±ΠΎΡ€Π°Ρ‚ΠΎΡ€ΠΈΡŽ. ΠœΡ‹ ΠΈΡΠΏΠΎΠ»ΡŒΠ·ΡƒΠ΅ΠΌ ΠΎΡ…Π°Ρ€Π°ΠΊΡ‚Π΅Ρ€ΠΈΠ·ΠΎΠ²Π°Π½Π½Ρ‹ΠΉ ΡƒΠ½ΠΈΠ²Π΅Ρ€ΡΠ°Π»ΡŒΠ½Ρ‹ΠΉ ΠΊΠ°Π»ΠΈΠ±Ρ€Π°Ρ‚ΠΎΡ€ 5522A, ΠΊΠΎΡ‚ΠΎΡ€Ρ‹ΠΉ Π±Ρ‹Π» ΠΎΡ…Π°Ρ€Π°ΠΊΡ‚Π΅Ρ€ΠΈΠ·ΠΎΠ²Π°Π½ с использованиСм эталона ΠΏΠ΅Ρ€Π΅Π΄Π°Ρ‡ΠΈ 8588A ΠΈ Ρ€Π°Π½Π΅Π΅ описанного кондСнсатора Π΅ΠΌΠΊΠΎΡΡ‚ΡŒΡŽ 1 Π½Π€.

    8588A ΠΈ 5222A Ρ€Π°Π±ΠΎΡ‚Π°ΡŽΡ‚ вмСстС, Ρ‡Ρ‚ΠΎΠ±Ρ‹ ΡΠΎΠ·Π΄Π°Ρ‚ΡŒ Π°Π»ΡŒΡ‚Π΅Ρ€Π½Π°Ρ‚ΠΈΠ²Π½Ρ‹ΠΉ ΠΏΠΎΠ΄Ρ…ΠΎΠ΄

    ΠŸΡ€ΠΎΡΠ»Π΅ΠΆΠΈΠ²Π°Π΅ΠΌΠΎΡΡ‚ΡŒ Смкости для эталона ΠΏΠ΅Ρ€Π΅Π΄Π°Ρ‡ΠΈ основана Π½Π° напряТСнии, Ρ‚ΠΎΠΊΠ΅ ΠΈ Π²Ρ€Π΅ΠΌΠ΅Π½ΠΈ, ΠΊΠΎΡ‚ΠΎΡ€Ρ‹Π΅ Π±Ρ‹Π»ΠΈ ΠΏΡ€ΠΎΠ²Π΅Ρ€Π΅Π½Ρ‹ ΠΏΠΎ ΡΡ€Π°Π²Π½Π΅Π½ΠΈΡŽ с эталонными кондСнсаторами ΠΈ Π°Π²Ρ‚ΠΎΠ½ΠΎΠΌΠ½ΠΎΠΉ систСмой измСрСния, которая Ρ‚Π°ΠΊΠΆΠ΅ основана Π½Π° напряТСнии, Ρ‚ΠΎΠΊΠ΅ ΠΈ Π²Ρ€Π΅ΠΌΠ΅Π½ΠΈ.Π­Ρ‚Π° автономная ΠΈΠ·ΠΌΠ΅Ρ€ΠΈΡ‚Π΅Π»ΡŒΠ½Π°Ρ систСма ΠΈΡΠΏΠΎΠ»ΡŒΠ·ΡƒΠ΅Ρ‚ΡΡ ΠΊΠΎΠΌΠΏΠ°Π½ΠΈΠ΅ΠΉ Fluke с Π½Π°Ρ‡Π°Π»Π° 90-Ρ… Π³ΠΎΠ΄ΠΎΠ² ΠΈ Π²ΠΊΠ»ΡŽΡ‡Π΅Π½Π° Π² список Π°ΠΊΠΊΡ€Π΅Π΄ΠΈΡ‚Π°Ρ†ΠΈΠΈ UKAS.

    ΠŸΡ€ΠΎΠ΄ΠΎΠ»ΠΆΠ°ΠΉΡ‚Π΅ ΡƒΡ‡ΠΈΡ‚ΡŒΡΡ

    ΠŸΠΎΡΠΌΠΎΡ‚Ρ€Π΅Ρ‚ΡŒ ΡΠΎΠΏΡƒΡ‚ΡΡ‚Π²ΡƒΡŽΡ‰ΠΈΠ΅ Ρ‚ΠΎΠ²Π°Ρ€Ρ‹

    ΠŸΠΎΠ»ΡƒΡ‡ΠΈΡ‚ΡŒ ΠΏΠΎΠΌΠΎΡ‰ΡŒ

    Как ΠΏΡ€ΠΎΠ²Π΅Ρ€ΠΈΡ‚ΡŒ кондСнсатор ΠΌΡƒΠ»ΡŒΡ‚ΠΈΠΌΠ΅Ρ‚Ρ€ΠΎΠΌ? (ΠŸΠΎΠ»Π΅Π·Π½Ρ‹Π΅ ΠΏΡ€ΠΈΠΌΠ΅Ρ€Ρ‹)

    Π—Π½Π°Ρ‡Π΅Π½ΠΈΠ΅ кондСнсаторов Π² Π½Π°ΡˆΠΈΡ… повсСднСвных Π±Ρ‹Ρ‚ΠΎΠ²Ρ‹Ρ… ΠΏΡ€ΠΈΠ±ΠΎΡ€Π°Ρ… слишком Π²Π΅Π»ΠΈΠΊΠΎ, Ρ‡Ρ‚ΠΎΠ±Ρ‹ Π΅Π³ΠΎ ΠΈΠ³Π½ΠΎΡ€ΠΈΡ€ΠΎΠ²Π°Ρ‚ΡŒ. Основной ΠΎΡΠΎΠ±Π΅Π½Π½ΠΎΡΡ‚ΡŒΡŽ кондСнсатора являСтся Π½Π°ΠΊΠΎΠΏΠ»Π΅Π½ΠΈΠ΅ элСктричСского заряда, ΠΈ ΠΎΠ½ ΠΈΠ³Ρ€Π°Π΅Ρ‚ Ρ€Π΅ΡˆΠ°ΡŽΡ‰ΡƒΡŽ Ρ€ΠΎΠ»ΡŒ Π² зарядкС ΠΈ разрядкС устройства.

    Он ΠΈΠ³Ρ€Π°Π΅Ρ‚ ΠΆΠΈΠ·Π½Π΅Π½Π½ΠΎ Π²Π°ΠΆΠ½ΡƒΡŽ Ρ€ΠΎΠ»ΡŒ Π² запускС устройств ΠΎΡ‚ внСшнСго источника, высвобоТдая заряд Π² Ρ†Π΅ΠΏΡŒ с ΠΏΠΎΠΌΠΎΡ‰ΡŒΡŽ пластины, ΡƒΠ΄Π΅Ρ€ΠΆΠΈΠ²Π°ΡŽΡ‰Π΅ΠΉ Ρ‚ΠΎΠΊ. НаиболСС распространСнными Ρ„Π°ΠΊΡ‚ΠΎΡ€Π°ΠΌΠΈ, Π²Ρ‹Π·Ρ‹Π²Π°ΡŽΡ‰ΠΈΠΌΠΈ ΠΏΠΎΠ²Ρ€Π΅ΠΆΠ΄Π΅Π½ΠΈΠ΅ кондСнсатора, ΡΠ²Π»ΡΡŽΡ‚ΡΡ высокоС напряТСниС, Ρ‚Π΅ΠΏΠ»ΠΎ, Π²Π»Π°ΠΆΠ½ΠΎΡΡ‚ΡŒ, химичСскоС загрязнСниС ΠΈ Π²Π»Π°ΠΆΠ½ΠΎΡΡ‚ΡŒ.

    Одной ΠΈΠ· основных ΠΏΡ€ΠΈΡ‡ΠΈΠ½ элСктричСских ΠΈ элСктронных ΠΏΠΎΠ»ΠΎΠΌΠΎΠΊ ΡΠ²Π»ΡΡŽΡ‚ΡΡ слабыС кондСнсаторы. Они Π΄ΠΎΠ»ΠΆΠ½Ρ‹ Π±Ρ‹Ρ‚ΡŒ ΠΏΡ€ΠΎΠ²Π΅Ρ€Π΅Π½Ρ‹ воврСмя, Ρ‡Ρ‚ΠΎΠ±Ρ‹ ΠΈΠ·Π±Π΅ΠΆΠ°Ρ‚ΡŒ ΠΊΠ°ΠΊΠΈΡ…-Π»ΠΈΠ±ΠΎ элСктричСских ΠΈΠ»ΠΈ элСктронных ΠΏΠΎΠ»ΠΎΠΌΠΎΠΊ Π² Π±ΡƒΠ΄ΡƒΡ‰Π΅ΠΌ. ΠœΡƒΠ»ΡŒΡ‚ΠΈΠΌΠ΅Ρ‚Ρ€ β€” это ΠΏΡ€ΠΈΠ±ΠΎΡ€ для поиска ΠΈ устранСния нСисправностСй, ΠΊΠΎΡ‚ΠΎΡ€Ρ‹ΠΉ ΠΈΡΠΏΠΎΠ»ΡŒΠ·ΡƒΠ΅Ρ‚ΡΡ для выявлСния слабых кондСнсаторов.

    ΠœΠ΅Ρ‚ΠΎΠ΄ 1: использованиС ΠΌΡƒΠ»ΡŒΡ‚ΠΈΠΌΠ΅Ρ‚Ρ€Π° с настройкой Смкости

    Π¦ΠΈΡ„Ρ€ΠΎΠ²ΠΎΠΉ ΠΌΡƒΠ»ΡŒΡ‚ΠΈΠΌΠ΅Ρ‚Ρ€ с настройкой Смкости β€” ΠΈΠ΄Π΅Π°Π»ΡŒΠ½Ρ‹ΠΉ Π²Ρ‹Π±ΠΎΡ€ для провСдСния этого тСста, ΠΏΠΎΡΠΊΠΎΠ»ΡŒΠΊΡƒ ΠΎΠ½ Π΄Π°Π΅Ρ‚ самыС быстрыС ΠΈ Ρ‚ΠΎΡ‡Π½Ρ‹Π΅ Ρ€Π΅Π·ΡƒΠ»ΡŒΡ‚Π°Ρ‚Ρ‹.

    Π­Ρ„Ρ„Π΅ΠΊΡ‚ кондСнсатора извСстСн ΠΊΠ°ΠΊ Β«Π•ΠΌΠΊΠΎΡΡ‚ΡŒΒ», Π° Π΅Π΄ΠΈΠ½ΠΈΡ†Π΅ΠΉ измСрСния Смкости являСтся Β«Π€Π°Ρ€Π°Π΄Ρ‹Β». Π‘Π»Π΅Π΄ΠΎΠ²Π°Ρ‚Π΅Π»ΡŒΠ½ΠΎ, ΠΌΡƒΠ»ΡŒΡ‚ΠΈΠΌΠ΅Ρ‚Ρ€ ΠΌΠΎΠΆΠ΅Ρ‚ ΠΈΠ·ΠΌΠ΅Ρ€ΡΡ‚ΡŒ Смкости ΠΊΠ°ΠΊ Π½ΠΈΠΆΠ΅ Π½Π°Π½ΠΎΡ„Π°Ρ€Π°Π΄, Ρ‚Π°ΠΊ ΠΈ Π²Ρ‹ΡˆΠ΅ ΠΌΠΈΠΊΡ€ΠΎΡ„Π°Ρ€Π°Π΄.

    • ΠžΡ‚ΡΠΎΠ΅Π΄ΠΈΠ½ΠΈΡ‚Π΅ кондСнсатор: Β Π§Ρ‚ΠΎΠ±Ρ‹ ΠΏΡ€ΠΎΠ²Π΅Ρ€ΠΈΡ‚ΡŒ кондСнсатор с ΠΏΠΎΠΌΠΎΡ‰ΡŒΡŽ ΠΌΡƒΠ»ΡŒΡ‚ΠΈΠΌΠ΅Ρ‚Ρ€Π°, ΠΏΠ΅Ρ€Π²ΠΎΠ΅, Ρ‡Ρ‚ΠΎ Π²Π°ΠΌ Π½ΡƒΠΆΠ½ΠΎ ΡΠ΄Π΅Π»Π°Ρ‚ΡŒ, это ΠΎΡ‚ΡΠΎΠ΅Π΄ΠΈΠ½ΠΈΡ‚ΡŒ кондСнсатор ΠΎΡ‚ ΠΏΠ΅Ρ‡Π°Ρ‚Π½ΠΎΠΉ ΠΏΠ»Π°Ρ‚Ρ‹.
    • ΠŸΠΎΠ΄ΠΊΠ»ΡŽΡ‡Π΅Π½ΠΈΠ΅ ΠΊ рСзистору: Π‘Π»Π΅Π΄ΡƒΡŽΡ‰ΠΈΠΌ шагом являСтся полная разрядка кондСнсатора ΠΏΡƒΡ‚Π΅ΠΌ ΠΏΠΎΠ΄ΠΊΠ»ΡŽΡ‡Π΅Π½ΠΈΡ ΠΊ свСтодиоду ΠΈΠ»ΠΈ ΠΌΠΎΡ‰Π½ΠΎΠΌΡƒ рСзистору.
    • НоминальноС напряТСниС: Β Π”Π°Π»Π΅Π΅ слСдуСт Π·Π°ΠΏΠΈΡΠ°Ρ‚ΡŒ Π΅ΠΌΠΊΠΎΡΡ‚ΡŒ ΠΈ номинальноС напряТСниС, ΡƒΠΊΠ°Π·Π°Π½Π½Ρ‹Π΅ Π½Π° Π·Π°Π΄Π½Π΅ΠΉ сторонС кондСнсатора.
    • ΠŸΠΎΠ΄ΡΠΎΠ΅Π΄ΠΈΠ½ΠΈΡ‚Π΅ Ρ€ΡƒΡ‡ΠΊΡƒ: Β Π˜ΡΠΏΠΎΠ»ΡŒΠ·ΡƒΠΉΡ‚Π΅ Ρ€ΡƒΡ‡ΠΊΡƒ Π½Π° Ρ†ΠΈΡ„Ρ€ΠΎΠ²ΠΎΠΌ ΠΌΡƒΠ»ΡŒΡ‚ΠΈΠΌΠ΅Ρ‚Ρ€Π΅ , Ρ‡Ρ‚ΠΎΠ±Ρ‹ Π½Π°ΡΡ‚Ρ€ΠΎΠΈΡ‚ΡŒ Π΅Π³ΠΎ Π½Π° ΠΏΠ°Ρ€Π°ΠΌΠ΅Ρ‚Ρ€Ρ‹ Смкости, Π° Π·Π°Ρ‚Π΅ΠΌ ΠΏΡ€ΠΈΠΊΡ€Π΅ΠΏΠΈΡ‚Π΅ Ρ‰ΡƒΠΏΡ‹ ΠΊ ΠΊΠ»Π΅ΠΌΠΌΠ°ΠΌ кондСнсатора. На элСктролитичСском кондСнсаторС соСдинитС ΠΎΡ‚Ρ€ΠΈΡ†Π°Ρ‚Π΅Π»ΡŒΠ½ΡƒΡŽ ΠΊΠ»Π΅ΠΌΠΌΡƒ с Ρ‡Π΅Ρ€Π½Ρ‹ΠΌ Ρ‰ΡƒΠΏΠΎΠΌ, Π° ΠΏΠΎΠ»ΠΎΠΆΠΈΡ‚Π΅Π»ΡŒΠ½ΡƒΡŽ ΠΊΠ»Π΅ΠΌΠΌΡƒ с красным, Ρ‡Ρ‚ΠΎΠ±Ρ‹ ΠΏΠΎΠ»ΡƒΡ‡ΠΈΡ‚ΡŒ Ρ‚ΠΎΡ‡Π½Ρ‹Π΅ показания.ΠšΠ»Π΅ΠΌΠΌΡ‹ ΠΌΠΎΠ³ΡƒΡ‚ Π±Ρ‹Ρ‚ΡŒ ΠΏΠΎΠ΄ΠΊΠ»ΡŽΡ‡Π΅Π½Ρ‹ Π»ΡŽΠ±Ρ‹ΠΌ способом для нСэлСктролитичСских кондСнсаторов.
    • ΠŸΡ€ΠΎΠ²Π΅Ρ€ΡŒΡ‚Π΅ показания:  Если всС эти шаги Π²Ρ‹ΠΏΠΎΠ»Π½Π΅Π½Ρ‹ ΠΏΡ€Π°Π²ΠΈΠ»ΡŒΠ½ΠΎ, ΠΏΡ€ΠΎΠ²Π΅Ρ€ΡŒΡ‚Π΅ показания Π½Π° экранС ΠΌΡƒΠ»ΡŒΡ‚ΠΈΠΌΠ΅Ρ‚Ρ€Π° ΠΈ сравнитС ΠΈΡ… с Π·Π°Π΄Π°Π½Π½Ρ‹ΠΌΠΈ значСниями.

    НСбольшая Ρ€Π°Π·Π½ΠΈΡ†Π° Π² показаниях ΠΈ ΡƒΠΊΠ°Π·Π°Π½Π½ΠΎΠΌ Π½ΠΎΠΌΠΈΠ½Π°Π»Π΅ допустима, Ρ‚Π°ΠΊ ΠΊΠ°ΠΊ элСктролитичСскиС кондСнсаторы ΠΈΠΌΠ΅ΡŽΡ‚ Ρ‚Π΅Π½Π΄Π΅Π½Ρ†ΠΈΡŽ ΠΊ Π²Ρ‹ΡΡ‹Ρ…Π°Π½ΠΈΡŽ, Π½ΠΎ Π·Π½Π°Ρ‡ΠΈΡ‚Π΅Π»ΡŒΠ½Π°Ρ Ρ€Π°Π·Π½ΠΈΡ†Π° Π΄ΠΎΠ»ΠΆΠ½Π° ΡƒΠΊΠ°Π·Ρ‹Π²Π°Ρ‚ΡŒ Π½Π° Ρ‚ΠΎ, Ρ‡Ρ‚ΠΎ кондСнсатор нСисправСн ΠΈ являСтся ΠΏΠΎΡ‚Π΅Π½Ρ†ΠΈΠ°Π»ΡŒΠ½ΠΎΠΉ ΠΏΡ€ΠΈΡ‡ΠΈΠ½ΠΎΠΉ ΠΏΡ€ΠΎΠ±Π»Π΅ΠΌ с вашим устройством.

    Бпособ 2. ИспользованиС ΠΌΡƒΠ»ΡŒΡ‚ΠΈΠΌΠ΅Ρ‚Ρ€Π° Π±Π΅Π· настройки Смкости

    НСкоторыС ΠΌΡƒΠ»ΡŒΡ‚ΠΈΠΌΠ΅Ρ‚Ρ€Ρ‹ ΠΌΠΎΠ³ΡƒΡ‚ Π½Π΅ ΠΈΠΌΠ΅Ρ‚ΡŒ настройки Смкости, Π½ΠΎ ΠΈΡ… ΠΌΠΎΠΆΠ½ΠΎ ΠΈΡΠΏΠΎΠ»ΡŒΠ·ΠΎΠ²Π°Ρ‚ΡŒ для ΠΏΡ€ΠΎΠ²Π΅Ρ€ΠΊΠΈ кондСнсаторов.

    • Разрядка кондСнсатора:  Разрядка кондСнсатора Ρ‚Π°ΠΊΠΆΠ΅ являСтся ΠΏΠ΅Ρ€Π²Ρ‹ΠΌ шагом Π² этом Ρ‚ΠΈΠΏΠ΅ ΠΌΡƒΠ»ΡŒΡ‚ΠΈΠΌΠ΅Ρ‚Ρ€Π°.
    • ΠŸΠΎΠ΄ΠΊΠ»ΡŽΡ‡Π΅Π½ΠΈΠ΅ ΠΊ ΠΎΠΌΠ°ΠΌ: Β Π’ ΠΎΡ‚Π»ΠΈΡ‡ΠΈΠ΅ ΠΎΡ‚ ΠΏΡ€Π΅Π΄Ρ‹Π΄ΡƒΡ‰Π΅Π³ΠΎ ΠΌΠ΅Ρ‚ΠΎΠ΄Π°, Π½Π° этот Ρ€Π°Π· ΠΌΡƒΠ»ΡŒΡ‚ΠΈΠΌΠ΅Ρ‚Ρ€ Π΄ΠΎΠ»ΠΆΠ΅Π½ Π±Ρ‹Ρ‚ΡŒ настроСн Π½Π° ΠΎΠΌΡ‹ для измСрСния сопротивлСния, Π° настройка Π΄ΠΎΠ»ΠΆΠ½Π° Π±Ρ‹Ρ‚ΡŒ ΠΎΡ‚Ρ€Π΅Π³ΡƒΠ»ΠΈΡ€ΠΎΠ²Π°Π½Π° для измСрСния Π² Π²Π΅Ρ€Ρ…Π½Π΅ΠΌ Π΄ΠΈΠ°ΠΏΠ°Π·ΠΎΠ½Π΅.
    • Π‘ΠΎΠ΅Π΄ΠΈΠ½Π΅Π½ΠΈΠ΅ с элСктролитом: Β ΠŸΠΎΠ΄ΠΊΠ»ΡŽΡ‡ΠΈΡ‚Π΅ ΠΏΠΎΠ»ΠΎΠΆΠΈΡ‚Π΅Π»ΡŒΠ½ΡƒΡŽ ΠΊΠ»Π΅ΠΌΠΌΡƒ ΠΊ красному Ρ‰ΡƒΠΏΡƒ, Π° ΠΎΡ‚Ρ€ΠΈΡ†Π°Ρ‚Π΅Π»ΡŒΠ½ΡƒΡŽ ΠΊΠ»Π΅ΠΌΠΌΡƒ ΠΊ Ρ‡Π΅Ρ€Π½ΠΎΠΌΡƒ Ρ‰ΡƒΠΏΡƒ элСктролитичСского кондСнсатора. Π’Ρ‹ ΠΌΠΎΠΆΠ΅Ρ‚Π΅ ΠΏΠΎΠ΄ΠΊΠ»ΡŽΡ‡ΠΈΡ‚ΡŒ Π΄Π°Ρ‚Ρ‡ΠΈΠΊΠΈ Π»ΡŽΠ±Ρ‹ΠΌ способом, Ссли Ρƒ вас Π΅ΡΡ‚ΡŒ нСэлСктролитичСский кондСнсатор.
    • ΠŸΡ€ΠΎΠ²Π΅Ρ€ΡŒΡ‚Π΅ ΠΏΠΎΠΊΠ°Π·Π°Π½ΠΈΠ΅:  Быстро Π·Π°ΠΏΠΈΡˆΠΈΡ‚Π΅ ΠΏΠΎΠΊΠ°Π·Π°Π½ΠΈΠ΅ сопротивлСния, ΠΎΡ‚ΠΎΠ±Ρ€Π°ΠΆΠ°Π΅ΠΌΠΎΠ΅ Π½Π° экранС, ΠΏΡ€Π΅ΠΆΠ΄Π΅ Ρ‡Π΅ΠΌ ΠΎΠ½ΠΎ измСнится Π½Π° сопротивлСниС Ρ€Π°Π·ΠΎΠΌΠΊΠ½ΡƒΡ‚ΠΎΠΉ Ρ†Π΅ΠΏΠΈ, Ρ€Π°Π²Π½ΠΎΠ΅ бСсконСчности.
    • ΠžΡ‚ΠΊΠ»ΡŽΡ‡Π΅Π½ΠΈΠ΅ ΠΎΡ‚ кондСнсатора:  НаконСц, отсоСдинитС Ρ‰ΡƒΠΏΡ‹ ΠΎΡ‚ кондСнсаторов ΠΈ ΠΏΠΎΠ²Ρ‚ΠΎΡ€ΠΈΡ‚Π΅ процСсс нСсколько Ρ€Π°Π·.Если ΠΊΠ°ΠΆΠ΄Ρ‹ΠΉ тСст ΠΏΠΎΠΊΠ°Π·Ρ‹Π²Π°Π΅Ρ‚ Ρ€Π°Π·Π½Ρ‹Π΅ показания сопротивлСния, этого Π΄ΠΎΠ»ΠΆΠ½ΠΎ Π±Ρ‹Ρ‚ΡŒ достаточно, Ρ‡Ρ‚ΠΎΠ±Ρ‹ Π΄ΠΎΠΊΠ°Π·Π°Ρ‚ΡŒ, Ρ‡Ρ‚ΠΎ кондСнсатор Ρ€Π°Π±ΠΎΡ‚Π°Π΅Ρ‚ ΠΏΡ€Π°Π²ΠΈΠ»ΡŒΠ½ΠΎ, Π½ΠΎ Ссли ΠΎΠ½ ΠΊΠ°ΠΆΠ΄Ρ‹ΠΉ Ρ€Π°Π· ΠΏΠΎΠΊΠ°Π·Ρ‹Π²Π°Π΅Ρ‚ ΠΎΠ΄Π½ΠΈ ΠΈ Ρ‚Π΅ ΠΆΠ΅ Ρ€Π΅Π·ΡƒΠ»ΡŒΡ‚Π°Ρ‚Ρ‹, ваш кондСнсатор ΠΏΠΎΠ²Ρ€Π΅ΠΆΠ΄Π΅Π½.

    Как ΠΏΡ€ΠΎΠ²Π΅Ρ€ΠΈΡ‚ΡŒ кондСнсатор с ΠΏΠΎΠΌΠΎΡ‰ΡŒΡŽ ΠΌΡƒΠ»ΡŒΡ‚ΠΈΠΌΠ΅Ρ‚Ρ€Π° Π½Π° Ρ†Π΅Π½Ρ‚Ρ€Π°Π»ΡŒΠ½Ρ‹Ρ… ΠΊΠΎΠ½Π΄ΠΈΡ†ΠΈΠΎΠ½Π΅Ρ€Π°Ρ…?

    Π’ установках кондиционирования Π²ΠΎΠ·Π΄ΡƒΡ…Π° ΠΈΠ»ΠΈ ΠžΠ’Πš Π² основном ΠΈΡΠΏΠΎΠ»ΡŒΠ·ΡƒΡŽΡ‚ΡΡ кондСнсаторы Π΄Π²ΡƒΡ… Ρ‚ΠΈΠΏΠΎΠ², Π° ΠΈΠΌΠ΅Π½Π½ΠΎ Β«Ρ€Π°Π±ΠΎΡ‡ΠΈΠ΅ кондСнсаторы» ΠΈ «пусковыС кондСнсаторы».

    Π Π°Π±ΠΎΡ‡ΠΈΠ΅ кондСнсаторы: Β«Π Π°Π±ΠΎΡ‡ΠΈΠ΅ кондСнсаторы» ΠΈΡΠΏΠΎΠ»ΡŒΠ·ΡƒΡŽΡ‚ΡΡ Π² двигатСлях вСнтиляторов ΠΈ компрСссорах.

    ΠŸΡƒΡΠΊΠΎΠ²Ρ‹Π΅ кондСнсаторы: Β«ΠŸΡƒΡΠΊΠΎΠ²Ρ‹Π΅ кондСнсаторы» ΠΈΡΠΏΠΎΠ»ΡŒΠ·ΡƒΡŽΡ‚ΡΡ Π² ΠΊΠΎΠ½Π΄ΠΈΡ†ΠΈΠΎΠ½Π΅Ρ€Π°Ρ… ΠΈ Ρ‚Π΅ΠΏΠ»ΠΎΠ²Ρ‹Ρ… насосах.

    Для ΠΏΡ€ΠΎΠ²Π΅Ρ€ΠΊΠΈ этих Ρ‚ΠΈΠΏΠΎΠ² кондСнсаторов Π½Π΅ΠΎΠ±Ρ…ΠΎΠ΄ΠΈΠΌΠΎ ΠΎΡ‚ΠΊΠ»ΡŽΡ‡ΠΈΡ‚ΡŒ ΠΏΠΈΡ‚Π°Π½ΠΈΠ΅ ΠΈ ΠΎΡ‚ΡΠΎΠ΅Π΄ΠΈΠ½ΠΈΡ‚ΡŒ ΠΊΠ»Π΅ΠΌΠΌΡ‹ с ΠΏΠΎΠΌΠΎΡ‰ΡŒΡŽ ΠΎΡ‚Π²Π΅Ρ€Ρ‚ΠΊΠΈ. ПослС этого слСдуСт ΠΎΡ‚ΡΠΎΠ΅Π΄ΠΈΠ½ΠΈΡ‚ΡŒ ΠΏΡ€ΠΎΠ²ΠΎΠ΄Π° Π½Π° кондСнсаторах ΠΈ ΠΏΠΎΠ΄ΠΊΠ»ΡŽΡ‡ΠΈΡ‚ΡŒ Ρ‰ΡƒΠΏΡ‹ ΠΌΡƒΠ»ΡŒΡ‚ΠΈΠΌΠ΅Ρ‚Ρ€Π°, Ρ‡Ρ‚ΠΎΠ±Ρ‹ ΠΏΠΎΠ»ΡƒΡ‡ΠΈΡ‚ΡŒ ΠΏΠΎΠΊΠ°Π·Π°Π½ΠΈΠ΅ Π½Π° экранС ΠΌΡƒΠ»ΡŒΡ‚ΠΈΠΌΠ΅Ρ‚Ρ€Π°.

    Как ΠΏΡ€ΠΎΠ²Π΅Ρ€ΠΈΡ‚ΡŒ ΠΌΠΈΠΊΡ€ΠΎΠ²ΠΎΠ»Π½ΠΎΠ²Ρ‹ΠΉ кондСнсатор с ΠΏΠΎΠΌΠΎΡ‰ΡŒΡŽ ΠΌΡƒΠ»ΡŒΡ‚ΠΈΠΌΠ΅Ρ‚Ρ€Π°?

    ΠœΠΈΠΊΡ€ΠΎΠ²ΠΎΠ»Π½Ρ‹ Ρ‚Π°ΠΊΠΆΠ΅ ΠΏΠΈΡ‚Π°ΡŽΡ‚ΡΡ ΠΎΡ‚ кондСнсаторов. Если Π²Ρ‹ ΡΠΎΠ±ΠΈΡ€Π°Π΅Ρ‚Π΅ΡΡŒ ΠΏΡ€ΠΎΠ²Π΅Ρ€ΠΈΡ‚ΡŒ кондСнсатор ΠΌΠΈΠΊΡ€ΠΎΠ²ΠΎΠ»Π½ΠΎΠ²ΠΊΠΈ, Π²ΠΎ-ΠΏΠ΅Ρ€Π²Ρ‹Ρ…, Π²Π°ΠΌ Π½ΡƒΠΆΠ½ΠΎ ΠΎΡ‚ΠΊΠ»ΡŽΡ‡ΠΈΡ‚ΡŒ ΠΏΠΈΡ‚Π°Π½ΠΈΠ΅.Π—Π°Ρ‚Π΅ΠΌ снимитС ΠΊΡ€Ρ‹ΡˆΠΊΡƒ ΠΌΠΈΠΊΡ€ΠΎΠ²ΠΎΠ»Π½ΠΎΠ²ΠΎΠΉ ΠΏΠ΅Ρ‡ΠΈ ΠΈ ΠΎΠ±Ρ€Π°Ρ‚ΠΈΡ‚Π΅ Π²Π½ΠΈΠΌΠ°Π½ΠΈΠ΅ Π½Π° ΠΏΡ€ΠΎΠ²ΠΎΠ΄, ΠΏΠΎΠ΄ΠΊΠ»ΡŽΡ‡Π΅Π½Π½Ρ‹ΠΉ ΠΊ ΠΊΠ»Π΅ΠΌΠΌΠ°ΠΌ кондСнсатора.

    ΠŸΡ€ΠΎΠ²Π΅Ρ€ΡŒΡ‚Π΅, Π½Π΅Ρ‚ Π»ΠΈ Π² вашСй ΠΌΠΈΠΊΡ€ΠΎΠ²ΠΎΠ»Π½ΠΎΠ²ΠΎΠΉ ΠΏΠ΅Ρ‡ΠΈ ΠΏΡ€ΠΎΠ΄ΡƒΠ²ΠΎΡ‡Π½ΠΎΠ³ΠΎ рСзистора; Ссли ΠΎΠ½ установлСн, Π΅Π³ΠΎ Π½Π΅ΠΎΠ±Ρ…ΠΎΠ΄ΠΈΠΌΠΎ ΡƒΠ΄Π°Π»ΠΈΡ‚ΡŒ ΠΏΠ΅Ρ€Π΅Π΄ тСстированиСм. Π’Π΅ΠΏΠ΅Ρ€ΡŒ помСститС ΠΊΠ°ΠΆΠ΄Ρ‹ΠΉ Ρ‰ΡƒΠΏ ΠΌΡƒΠ»ΡŒΡ‚ΠΈΠΌΠ΅Ρ‚Ρ€Π° Π½Π° ΠΊΠ°ΠΆΠ΄ΡƒΡŽ ΠΊΠ»Π΅ΠΌΠΌΡƒ кондСнсатора ΠΈ Π·Π°ΠΏΠΈΡˆΠΈΡ‚Π΅ показания, Π° Π·Π°Ρ‚Π΅ΠΌ помСняйтС мСстами Ρ‰ΡƒΠΏΡ‹ Ρ‚Π°ΠΊ, Ρ‡Ρ‚ΠΎΠ±Ρ‹ ΠΊΠ°ΠΆΠ΄Ρ‹ΠΉ ΠΈΠ· Π½ΠΈΡ… касался Π΄Ρ€ΡƒΠ³ΠΎΠ³ΠΎ Π²Ρ‹Π²ΠΎΠ΄Π°.

    alexxlab

    Π”ΠΎΠ±Π°Π²ΠΈΡ‚ΡŒ ΠΊΠΎΠΌΠΌΠ΅Π½Ρ‚Π°Ρ€ΠΈΠΉ

    Π’Π°Ρˆ адрСс email Π½Π΅ Π±ΡƒΠ΄Π΅Ρ‚ ΠΎΠΏΡƒΠ±Π»ΠΈΠΊΠΎΠ²Π°Π½. ΠžΠ±ΡΠ·Π°Ρ‚Π΅Π»ΡŒΠ½Ρ‹Π΅ поля ΠΏΠΎΠΌΠ΅Ρ‡Π΅Π½Ρ‹ *